CONSTI 2 Case Digest PDF

You might also like

Download as pdf or txt
Download as pdf or txt
You are on page 1of 172

CONSTITUTIONAL LAW II b.) Yes.

In its plain and ordinary meaning, the term


PROFESSOR: Atty. Thomas Kristino B. Caharian patrimony pertains to heritage. 35 When the Constitution
speaks of national patrimony, it refers not only to the
DIGESTS COMPILATION BY 1-WIGMORE natural resources of the Philippines, but also to the
I. DUE PROCESS AND EQUAL PROTECTION AS LIMITATION cultural heritage of the Filipinos.
ON POLICE POWER, EMINENT DOMAIN AND TAXATION

FUNDAMENTAL PRINCIPLES ON CONSTITUTIONAL LAW AND


THE BILL OF RIGHTS c.) Yes. 51% of the equity of the MHC comes within the
purview of the constitutional shelter for it comprises the
majority and controlling stock, so that anyone who
Manila Prince Hotel vs. GSIS
G.R. No. 122156 (February 3, 1997) acquires or owns the 51% will have actual control and
management of the hotel. In this instance, 51% of the
MHC cannot be disassociated from the hotel and the land
FACTS: on which the hotel edifice stands.
The controversy arose when respondent GSIS decided to
sell through public bidding 30% to 51% of the outstanding
shares of Manila Hotel. Only two (2) bidders participated: d.) Yes. Since petitioner has already matched the bid
petitioner Manila Prince Hotel Corporation, a Filipino price tendered by the foreign firm, respondent GSIS is left
corporation, which offered to buy 51% shares at with no alternative but to award to petitioner the shares of
P41.58/share, and a Malaysian firm, at P44.00/share. MH accordance not only with the bidding guidelines and
Pending the declaration of the winning bidder, petitioner procedures but with the Constitution as well. The refusal
matched the bid price of P44.00 per share tendered by of respondent GSIS to execute the corresponding
the Malaysian Firm which respondent GSIS refused to documents with petitioner after the latter has matched the
accept. The petitioner posits that since Manila Hotel is bid of the Malaysian firm clearly constitutes grave abuse
part of the national patrimony, petitioner should be of discretion.
preferred after it has matched the bid offer of the
Malaysian firm invoking Sec. 10, second par., Art. XII, of
the 1987 Constitution. Hence, GSIS(respondent) is ordered to accept the
matching bid of petitioner and execute the necessary
clearances for the purchase of the subject 51% MHC
ISSUE/S: shares.

a.) WON self-executing provision Reasoning: The Constitution is the fundamental,


paramount and supreme law of the nation, it is deemed
b.) Granting that this provision is self-executing, WON written in every statute and contract.
Manila Hotel falls under the term national patrimony.

c.) Granting that the Manila Hotel forms part of the Pamatong vs. Comelec
national patrimony, WON selling mere 51% shares and (G.R. No. 161872, April 13, 2004)
not the land itself can be considered part of national
patrimony. FACTS:
d.) WON GSIS committed grave abuse of discretion.

The COMELEC declared petitioner and thirty-five (35)


RULING: others nuisance candidates who could not wage a
nationwide campaign. Petitioner seeks to reverse the
a.) Yes. Sec. 10, second par., Art. XII of the of the 1987 resolutions which were allegedly rendered in violation of
Constitution is self-executing which needs no further his right to "equal access to opportunities for public
guidelines or implementing laws or rules for its service" under Section 26, Article II of the 1987
enforcement. It is per se judicially enforceable The Constitution.
Constitution mandates that qualified Filipinos shall be
preferred. And when our Constitution declares that a right
exists in certain specified circumstances an action may be ISSUE/S:
maintained to enforce such right notwithstanding the
absence of any legislation on the subject. Where there is
a right there is a remedy. Ubi jus ibi remedium.
WON the constitutional provision ensuring "equal access
to opportunities for public office" grants a constitutional
right to run for or hold public office
RULING: c.) WON the petitioner can invoke the equal protection
clause guaranty

No. What is recognized is merely a privilege subject to


limitations imposed by law. Also, the "equal access" RULING:
provision is a subsumed part of Article II of the
Constitution. The provisions under the Article are
generally considered not self-executing. a.) Yes. It would fall under Article 282 of the Labor Code.
As earlier noted, the privilege of equal access to A reading of the weight standards of PAL would lead to
opportunities to public office may be subjected to no other conclusion than that they constitute a continuing
limitations. Some valid limitations specifically on the qualification of an employee in order to keep the job.
privilege to seek elective office are found in the Tersely put, an employee may be dismissed the moment
provisions9 of the Omnibus Election Code on "Nuisance he is unable to comply with his ideal weight as prescribed
Candidates" and outlined instances wherein the by the weight standards.
COMELEC may motu proprio refuse to give due course b.) Yes. Though generally, employment in particular jobs
to or cancel a Certificate of Candidacy. may not be limited to persons of a particular sex, religion,
As long as the limitations apply to everybody equally or national origin. However, if the employer can show that
without discrimination, the equal access clause is not sex, religion, or national origin is an actual qualification for
violated. performing the job, the qualification is then considered
valid and referred to as bona fide occupational
Reasoning: There is a need to limit the number of qualification (BFOQ).
candidates especially in the case of candidates for
national positions because the election process becomes Applying the “Meiorin Test” in determining whether an
a mockery even if those who cannot clearly wage a employment policy is justified, the following must concur:
national campaign are allowed to run. Their names would (1) the employer must show that it adopted the standard
have to be printed in the Certified List of Candidates, for a purpose rationally connected to the performance of
Voters Information Sheet and the Official Ballots. These the job;64 (2) the employer must establish that the
would entail additional costs to the government xxx[I]t standard is reasonably necessary to the accomplishment
serves no practical purpose to allow those candidates to of that work-related purpose; and (3) the employer must
continue if they cannot wage a decent campaign enough establish that the standard is reasonably necessary in
to project the prospect of winning, no matter how slim. order to accomplish the legitimate work-related purpose.

It would be then a senseless sacrifice on the part of the The weight standards of PAL are reasonable. The most
State. important activity of the cabin crew is to care for the safety
of passengers and the evacuation of the aircraft when an
emergency occurs. Passenger safety goes to the core of
the job of a cabin attendant. Truly, airlines need cabin
Yrasuegui vs. PAL
(G.R. No. 168081, October 17, 2008)
attendants who have the necessary strength to open
emergency doors, the agility to attend to passengers in
cramped working conditions, and the stamina to withstand
FACTS: grueling flight schedules among other reasons.

c.) No. In the absence of governmental interference, the


liberties guaranteed by the Constitution cannot be
On June 15, 1993, petitioner was formally informed by invoked.87 Put differently, the Bill of Rights is not meant
PAL that due to his inability to attain his ideal weight, "and to be invoked against acts of private individuals. Private
considering the utmost leniency" extended to him "which actions, no matter how egregious, cannot violate the
spanned a period covering a total of almost five (5) years," equal protection guarantee.
his services were considered terminated "effective
immediately."11

ISSUE/S:

a.) WON the obesity of petitioner is a ground for dismissal


under the Labor Code.

b.) WON the dismissal of petitioner can be predicated on


the bona fide occupational disqualification defense.
DUE PROCESS IN GENERAL power to define crimes and prescribe their corresponding
penalties is legislative in nature and inherent in the
People vs. Siton sovereign power of the state to maintain social order as
an aspect of police power. The legislature however, must
inform the citizen with reasonable precision what acts it
FACTS: intends to prohibit so that he may have a certain
In the case at bar the affidavit of the arresting police understandable rule of conduct and know what acts it is
officer, lucidly shows that there was a prior surveillance his duty to avoid. This requirement has come to be known
conducted in view of the reports that vagrants and as the void-for-vagueness doctrine which states that “a
prostitutes proliferate in the place where the respondents statute which either forbids or requires the doing of an act
(among other women) were wandering and in the wee in terms so vague that men of common intelligence must
hours of night and soliciting male customer. Hence, necessarily guess at its meaning and differ as to its
respondents Evangeline Siton and Krystel Kate Sagarano application, violates the first essential of due process of
were charged with vagrancy pursuant to Article 202 (2) of law.”
the Revised Penal. In turn, the respondents filed separate
Motions to Quash on the ground that Article 202 (2) is
unconstitutional for being vague and overbroad. b.) No. Article 202 (2) does not violate the equal protection
clause; neither does it discriminate against the poor and
the unemployed. Offenders of public order laws are
ISSUE/S: punished not for their status, as for being poor or
unemployed, but for conducting themselves under such
circumstances as to endanger the public peace or cause
alarm and apprehension in the community. Being poor or
a.) WON the definition of the crime of vagrancy under unemployed is not a license or a justification to act
Article 202 (2) is unconstitutional for being vague. indecently or to engage in immoral conduct.

b.) WON Article 202 (2) violated the equal protection Vagrancy must not be so lightly treated as to be
clause under the Constitution because it discriminates considered constitutionally offensive. It is a public order
against the poor and unemployed, thus permitting an crime which punishes persons for conducting themselves,
arbitrary and unreasonable classification. at a certain place and time which orderly society finds
unusual, under such conditions that are repugnant and
outrageous to the common standards and norms of
RULING: decency and morality in a just, civilized and ordered
society, as would engender a justifiable concern for the
safety and well-being of members of the community.
a.) No. The trial court in its assailed ruling relied on the
underlying principles in Papachristou vs. City of
Jacksonville case: First, that the assailed ordinance fails DLSU vs. CA
to give a person of ordinary intelligence fair notice that his
contemplated conduct is forbidden by a statute and;
FACTS:
Second, it encourages or promotes opportunities for the
application of discriminatory law enforcement. However, The private respondents were involved in mauling
the aforementioned principles do not apply in the case at incidents that were a result of a fraternity war. On May 3,
bar since: The first principle finds no application here 1995, the DLSU-CSB Joint Discipline Board issued a
because under our legal system, ignorance of the law Resolution finding private respondents guilty. They were
excuses no one from compliance therewith and; Second, meted the supreme penalty of automatic expulsion,
the fear exhibited by the respondents, that unfettered pursuant to CHED Order No. 4.
discretion is placed in the hands of the police to make an
arrest or search, is assuaged by the constitutional
requirement of probable cause, which is one less than
ISSUE/S:
certainty or proof, but more than suspicion or possibility.
The requirement of probable cause provides an
acceptable limit on police or executive authority that may
otherwise be abused in relation to the search or arrest of WON private respondents accorded due process of law?
persons found to be violating Article 202 (2). Furthermore,
it may be note worthy that the Jacksonville ordinance was
declared unconstitutional for provisions that are not found
in the questioned Vagrancy law in the case at bar. The
RULING: Surigao Electric vs. ERC

FACTS:
1. Yes. Private respondents were accorded due process
of law.
On March 19, 2007, the ERC issued its assailed Order,
mandating that the discounts earned by SURNECO from
In administrative cases, such as investigations of students its power supplier should be deducted from the
found violating school discipline, "[t]here are minimum computation of the power cost upon ascertaining that the
standards which must be met before to satisfy the Purchased Power Adjustment (PPA) of SURNECO
demands of procedural due process and these are: that resulted to an over-recovery amounting to
(1) the students must be informed in writing of the nature PhP18,188,794.
and cause of any accusation against them; (2) they shall
have the right to answer the charges against them and
with the assistance if counsel, if desired; (3) they shall be
ISSUE:
informed of the evidence against them; (4) they shall have
the right to adduce evidence in their own behalf; and (5)
the evidence must be duly considered by the investigating
committee or official designated by the school authorities WON SURNECO was deprived of the opportunity to be
to hear and decide the case. heard in ordering it to refund alleged over-recoveries
arrived at by the ERC.

In the case at bar, the Private respondents were duly


informed in writing of the charges against them by the RULING:
DLSU-CSB Joint Discipline Board. They were given the
opportunity to answer the charges against them as they,
in fact, submitted their respective answers. NO. In directing SURNECO to refund its over-recoveries
based on PPA policies, which only ensured that the PPA
mechanism remains a purely cost-recovery mechanism
They were also informed of the evidence presented and not a revenue-generating scheme for the electric
against them as they attended all the hearings before the cooperatives, the ERC merely exercised its authority to
Board. Moreover, private respondents were given the regulate and approve the rates imposed by the electric
right to adduce evidence on their behalf and they did. cooperatives on their consumers. The ERC simply
Lastly, the Discipline Board considered all the pieces of performed its mandate to protect the public interest
evidence submitted to it by all the parties before rendering imbued in those rates.
its resolution in Discipline Case No. 9495-3-25121.

It is beyond cavil that the State, in the exercise of police


The essence of due process is simply an opportunity to power, can regulate the rates imposed by a public utility
be heard, or as applied to administrative proceedings, an such as SURNECO.— The regulation of rates to be
opportunity to explain one's side or an opportunity to seek charged by public utilities is founded upon the police
reconsideration of the action or ruling complained of. powers of the State and statutes prescribing rules for the
control and regulation of public utilities are a valid
exercise thereof. When private property is used for a
public purpose and is affected with public interest, it
A formal trial-type hearing is not, at all times and in all
ceases to be juris privati only and becomes subject to
instances, essential to due process – it is enough that the
regulation. The regulation is to promote the common
parties are given a fair and reasonable opportunity to
good. Submission to regulation may be withdrawn by the
explain their respective sides of the controversy and to
owner by discontinuing use; but as long as use of the
present supporting evidence on which a fair decision can
property is continued, the same is subject to public
be based. "To be heard" does not only mean presentation
regulation.
of testimonial evidence in court – one may also be heard
through pleadings and where the opportunity to be heard
through pleadings is accorded, there is no denial of due
process. We likewise differ from SURNECO’s stance that it was
denied due process when the ERC issued its questioned
Orders. Administrative due process simply requires an
opportunity to explain one’s side or to seek
reconsideration of the action or ruling complained of. It
means being given the opportunity to be heard before
judgment, and for this purpose, a formal trial-type hearing
is not even essential. It is enough that the parties are In the present case, the dismal absence of the first two
given a fair and reasonable chance to demonstrate their requisites, which are the most essential, renders the
respective positions and to present evidence in support discussion of the last two superfluous.
thereof.

Petitioners lack locus standi. For a concerned party to be


Verily, the PPA confirmation necessitated a review of the allowed to raise a constitutional question, it must show
electric cooperatives’ monthly documentary submissions that (1) it has personally suffered some actual or
to substantiate their PPA charges. The cooperatives were threatened injury as a result of the allegedly illegal
duly informed of the need for other required supporting conduct of the government, (2) the injury is fairly traceable
documents and were allowed to submit them accordingly. to the challenged action, and (3) the injury is likely to be
In fact, hearings were conducted. Moreover, the ERC redressed by a favorable action.
conducted exit conferences with the electric cooperatives’
representatives, SURNECO included, to discuss
preliminary figures and to double-check these figures for Petitioners in this case have not presented any personal
inaccuracies, if there were any. In addition, after the stake in the outcome of the controversy. None of them
issuance of the ERC Orders, the electric cooperatives faces any charge under RA 9372. Petitioner-
were allowed to file their respective motions for organizations assert locus standi on the basis of being
reconsideration. It cannot claimed, therefore, that suspected "communist fronts" by the government,
SURNECO was denied due process. especially the military; whereas individual petitioners
invariably invoke the "transcendental importance"
doctrine and their status as citizens and taxpayers.
Southern Hemisphere vs. Anti-Terrorism

While jurisprudence holds that transcendental public


FACTS:
importance dispenses with the requirement that petitioner
has experienced or is in actual danger of suffering direct
and personal injury, this does not apply in cases involving
Before the Court are six petitions challenging the the constitutionality of penal legislation.
constitutionality of Republic Act No. 9372 (RA 9372), "An
Act to Secure the State and Protect our People from
Terrorism," otherwise known as the Human Security Act
b.) NO. Petitioners fail to present an actual case or
of 2007.
controversy By constitutional fiat, judicial power operates
only when there is an actual case or controversy.

ISSUE/S:
Herein petitioners have failed to show that the challenged
provisions of RA 9372 forbid constitutionally protected
a.) WON petitioners possess locus standi conduct or activity that they seek to do. No demonstrable
threat has been established, much less a real and existing
b.) WON there is an actual case or controversy
one.
c.) WON RA9372 Regulates speech so as to permit a
facial analysis of its validity
c.) NO. The rule established in our jurisdiction is; only
statutes on free speech, religious freedom, and other
RULING: fundamental rights may be facially challenged. Under no
case may ordinary penal statutes be subjected to a facial
challenge.
a.) YES. It must be noted that in constitutional limitations,
the power of judicial review is limited by four exacting
requisites, viz: (a) there must be an actual case or A facial challenge is allowed to be made to a vague
controversy; (b) petitioners must possess locus standi; (c) statute and to one which is overbroad because of
the question of constitutionality must be raised at the possible"chilling effect" upon protected speech. The
earliest opportunity; and (d) the issue of constitutionality theory is that "[w]hen statutes regulate or proscribe
must be the lis mota of the case. speech and no readily apparent construction suggests
itself as a vehicle for rehabilitating the statutes in a single
prosecution, the transcendent value to all society of From the definition of the crime of terrorism in the earlier
constitutionally protected expression is deemed to justify cited Section 3 of RA 9372, the following elements may
allowing attacks on overly broad statutes with no be culled: (1) the offender commits an act punishable
requirement that the person making the attack under any of the cited provisions of the Revised Penal
demonstrate that his own conduct could not be regulated Code, or under any of the enumerated special penal laws;
by a statute drawn with narrow specificity." (2) the commission of the predicate crime sows and
creates a condition of widespread and extraordinary fear
and panic among the populace; and (3) the offender is
This rationale does not apply to penal statutes. Criminal actuated by the desire to coerce the government to give
statutes have general in terrorem effect resulting from in to an unlawful demand.
their very existence, and, if facial challenge is allowed for
this reason alone, the State may well be prevented from
enacting laws against socially harmful conduct. In the In insisting on a facial challenge on the invocation that the
area of criminal law, the law cannot take chances as in law penalizes speech, petitioners contend that the
the area of free speech. element of "unlawful demand" in the definition of terrorism
must necessarily be transmitted through some form of
expression protected by the free speech clause. The
VAGUENESS VS. OVERBREADTH: argument does not persuade. What the law seeks to
penalize is conduct, not speech.

A statute or act suffers from the defect of vagueness when


it lacks comprehensible standards that men of common Hence petitions are dismissed.
intelligence must necessarily guess at its meaning and
differ as to its application. It is repugnant to the
Constitution in two respects: (1) it violates due process for DUE PROCESS AND POLICE POWER
failure to accord persons, especially the parties targeted
by it, fair notice of the conduct to avoid; and (2) it leaves
law enforcers unbridled discretion in carrying out its White Light Corporation vs. City of Manila
provisions and becomes an arbitrary flexing of the
Government muscle. The overbreadth doctrine, FACTS:
meanwhile, decrees that a governmental purpose to On December 3, 1992, City Mayor Alfredo S. Lim signed
control or prevent activities constitutionally subject to into law the Ordinance prohibiting short time admission in
state regulations may not be achieved by means which hotels, motels, lodging houses, pension houses and
sweep unnecessarily broadly and thereby invade the area similar establishments in the City of Manila.
of protected freedoms. As distinguished from the
vagueness doctrine, the overbreadth doctrine assumes Petitioners herein are operators of drive-in-hotels and
that individuals will understand what a statute prohibits motels in Manila which are directly affected by the said
and will accordingly refrain from that behavior, even ordinance.
though some of it is protected.
Petitioners argued that the Ordinance is unconstitutional
FACIAL CHALLENGE VS. AS-APPLIED CHALLENGE: and void since it violates the right to privacy and the
freedom of movement; it is an invalid exercise of police
power; and it is an unreasonable and oppressive
Distinguished from an as-applied challenge which interference in their business.
considers only extant facts affecting real litigants, a facial
invalidation is an examination of the entire law,
pinpointing its flaws and defects, not only on the basis of ISSUE/S:
its actual operation to the parties, but also on the
assumption or prediction that its very existence may a.) WON the petitioners’ has legal standing
cause others not before the court to refrain from
constitutionally protected speech or activities.
b.) WON the petitioners have the requisite standing to
plead for protection of their patrons' equal protection
There is no merit in the claim that RA 9372 regulates rights.
speech so as to permit a facial analysis of its validity

c.) WON the ordinance is unconstitutional


RULING:

a.) Yes. Petitioners in this case are owners of The test of a valid ordinance is well established. For an
establishments offering "wash-up" rates and their ordinance to be valid, it must not only be within the
business is being unlawfully interfered with by the corporate powers of the local government unit to enact
Ordinance. Standing or locus standi is the ability of a party and pass according to the procedure prescribed by law, it
to demonstrate to the court sufficient connection to and must also conform to the following substantive
harm from the law or action challenged to support party's requirements: (1) must not contravene the Constitution or
participation in the case. any statute; (2) must not be unfair or oppressive; (3) must
not be partial or discriminatory; (4) must not prohibit but
may regulate trade; (5) must be general and consistent
b.) Yes. The requirement of standing is a core component with public policy; and (6) must not be unreasonable.
of the judicial system derived directly from the
Constitution. In this jurisdiction, the extancy of "a direct
and personal interest" presents the most obvious cause The Ordinance prohibits two specific and distinct business
for a petitioner's standing. Nonetheless, the general rules practices, namely wash rate admissions and renting out a
on standing admit of several exceptions such as the room more than twice a day.
overbreadth doctrine, taxpayer suits, third party standing
and, especially in the Philippines, the doctrine of
transcendental importance. The primary constitutional question that confronts us is
one of due process. The due process guaranty has
traditionally been interpreted as imposing two related but
For this particular set of facts, the concept of third party distinct restrictions on government, "procedural due
standing as an exception and the overbreadth doctrine process" and "substantive due process." Procedural due
are appropriate. In Powers v. Ohio, the United States process refers to the procedures that the government
Supreme Court wrote that: "We have recognized the right must follow before it deprives a person of life, liberty, or
of litigants to bring actions on behalf of third parties, property However, if due process were confined solely to
provided three important criteria are satisfied: the litigant its procedural aspects, there would arise absurd situation
must have suffered an ‘injury-in-fact,’ thus giving him or of arbitrary government action, provided the proper
her a "sufficiently concrete interest" in the outcome of the formalities are followed. Substantive due process then
issue in dispute; the litigant must have a close relation to completes the protection envisioned by the due process
the third party; and there must exist some hindrance to clause. It inquires whether the government has sufficient
the third party's ability to protect his or her own interests. justification for depriving a person of life, liberty, or
“Herein, it is clear that the business interests of the property.
petitioners are likewise injured by the Ordinance. They
rely on the patronage of their customers for their
continued viability which appears to be threatened by the The general test of the validity of an ordinance on
enforcement of the Ordinance. substantive due process grounds is best tested when
assessed with the evolved footnote 4 test laid down by the
U.S. Supreme Court in U.S. v. Carolene Products, to wit;
Assuming arguendo that petitioners do not have a
relationship with their patrons for the former to assert the
rights of the latter, the overbreadth doctrine comes into First, the rational basis examination - laws or ordinances
play. In overbreadth analysis, challengers to government are upheld if they rationally further a legitimate
action are in effect permitted to raise the rights of third governmental interest.
parties. Generally applied to statutes infringing on the
freedom of speech, the overbreadth doctrine applies
when a statute needlessly restrains even constitutionally
Second, the intermediate review - governmental interest
guaranteed rights. In this case, the petitioners claim that
is extensively examined and the availability of less
the Ordinance makes a sweeping intrusion into the right
restrictive measures is considered.
to liberty of their clients. We can see that based on the
allegations in the petition, the Ordinance suffers from
overbreadth. It is thus recognized that the petitioners have
a right to assert the constitutional rights of their clients to Third, applying strict scrutiny- the focus is on the presence
patronize their establishments for a "wash-rate" time of compelling, rather than substantial, governmental
frame. interest and on the absence of less restrictive means for
achieving that interest.

c.) Yes. The said ordinance is unconstitutional.


If we were to take the myopic view that an Ordinance Lacking a concurrence of these requisites, the police
should be analyzed strictly as to its effect only on the measure shall be struck down as an arbitrary intrusion
petitioners at bar, then it would seem that the only into private rights.
restraint imposed by the law which we are capacitated to
act upon is the injury to property sustained by the
petitioners, an injury that would warrant the application of Based on the foregoing facts, Ordinance No. 7774 is
the most deferential standard – the rational basis test. Yet hereby declared UNCONSTITUTIONAL.
as earlier stated, we recognize the capacity of the
petitioners to invoke as well the constitutional rights of
their patrons – those persons who would be deprived of
Office of the Solicitor vs. City of Manila
availing short time access or wash-up rates to the lodging
establishments in question.
FACTS:

Respondents Ayala Land, Robinsons, and Shangri-la


Viewed cynically, one might say that the infringed rights maintain and operate shopping malls in various locations
of these customers were are trivial since they seem shorn
in Metro Manila. The shopping malls operated or leased
of political consequence. Still, the Bill of Rights does not
out by respondents have parking facilities for all kinds of
shelter gravitas alone. Indeed, it is those "trivial" yet
motor vehicles, they collect parking fees from the persons
fundamental freedoms – which the people reflexively
making use of their parking facilities, regardless of
exercise any day without the impairing awareness of their
whether said persons are mall patrons or not.
constitutional consequence – that accurately reflect the
degree of liberty enjoyed by the people.

In 1999, the Senate Committees on Trade and Commerce


and on Justice and Human Rights conducted a joint
It cannot be denied that the primary animus behind the
investigation on which the following purpose is included:
ordinance is the curtailment of sexual behavior. Whether
(1) to inquire into the legality of the prevalent practice of
or not this depiction of a mise-en-scene of vice is
shopping malls of charging parking fees. The Committees
accurate, it cannot be denied that legitimate sexual
found that the collection of parking fees by shopping malls
behavior among willing married or consenting single
is contrary to the National Building Code and is therefor
adults which is constitutionally protected will be curtailed [sic] illegal.
as well. We cannot discount other legitimate activities
which the Ordinance would proscribe or impair. There are
very legitimate uses for a wash rate or renting the room
out for more than twice a day. E.g. Entire families are Respondent SM Prime thereafter received information
known to choose pass the time in a motel or hotel whilst enjoining respondent SM Prime and similar
the power is momentarily out in their homes. establishments from collecting parking fees, and to
impose upon said establishments penal sanctions under
Presidential Decree No. 1096.
That the Ordinance prevents the lawful uses of a wash
rate depriving patrons of a product and the petitioners of
lucrative business ties in with another constitutional ISSUE:
requisite for the legitimacy of the Ordinance as a police Whether or not compelling the respondent to provide
power measure. It must appear that the interests of the parking spaces in their malls for the use of their patrons
public generally, as distinguished from those of a or the public in general, free of charge constitutes an
particular class, require an interference with private rights unlawful taking of property without just Compensation?
and the means must be reasonably necessary for the
accomplishment of the purpose and not unduly
oppressive of private rights. It must also be evident that
no other alternative for the accomplishment of the RULING:
purpose less intrusive of private rights can work. More YES. The Court finds that in totally prohibiting
importantly, a reasonable relation must exist between the respondents from collecting parking fees from the public
purposes of the measure and the means employed for its for the use of the mall parking facilities, the State would
accomplishment, for even under the guise of protecting be acting beyond the bounds of police power.
the public interest, personal rights and those pertaining to
private property will not be permitted to be arbitrarily
invaded.
Police power is the power of promoting the public welfare
by restraining and regulating the use of liberty and
property. It is usually exerted in order to merely regulate
the use and enjoyment of the property of the owner. The because they were not furnished with ROEs. They further
power to regulate, however, does not include the power contend that the sanction of closure that the MB might
to prohibit. A fortiori, the power to regulate does not impose upon the receipt of ROE will result in irreparable
include the power to confiscate. Police power does not damage to them as well as to the public.
involve the taking or confiscation of property, with the
exception of a few cases where there is a necessity to ISSUE: W/N due process has been violated by SED due
to its failure to send copies of the ROEs to the respondent
confiscate private property in order to destroy it for the
banks.
purpose of protecting peace and order and of promoting
the general welfare; for instance, the confiscation of an
illegally possessed article, such as opium and firearms.

RULING:
Although in the present case, title to and/or possession of
the parking facilities remain/s with respondents, the No, The respondent banks have failed to show that they
prohibition against their collection of parking fees from the are entitled to copies of the ROEs. They can point to no
provision of law, no section in the procedures of the BSP
public, for the use of said facilities, is already tantamount
that shows that the BSP is required to give them copies
to a taking or confiscation of their properties. The State is of the ROEs. The respondent banks cannot claim a
not only requiring that respondents devote a portion of the violation of their right to due process if they are not
latter’s properties for use as parking spaces, but is also provided with copies of the ROEs. The same ROEs are
mandating that they give the public access to said parking based on the lists of findings/exceptions containing the
spaces for free. Such is already an excessive intrusion deficiencies found by the SED examiners when they
into the property rights of respondents. Not only are they examined the books of the respondent banks. As found
being deprived of the right to use a portion of their by the RTC, these lists of findings/exceptions were
properties as they wish, they are further prohibited from furnished to the officers or representatives of the
profiting from its use or even just recovering therefrom the respondent banks, and the respondent banks were
expenses for the maintenance and operation of the required to comment and to undertake remedial
required parking facilities. measures stated in said lists. Despite these instructions,
respondent banks failed to comply with the SED’s
directive.

BSP MB VS. Antonio Valenzuela Respondent banks are already aware of what is required
G.R. No. 184778 (October 2, 2009) of them by the BSP, and cannot claim violation of their
right to due process simply because they are not
FACTS: furnished with copies of the ROEs.
In September of 2007 the Supervision and Examination The "close now, hear later" doctrine has already been
Department (SED) of BSP conducted examinations of the justified as a measure for the protection of the public
books of the following banks: Rural Bank of Parañaque, interest. Swift action is called for on the part of the BSP
Inc. (RBPI), Rural Bank of San Jose (Batangas), Inc., when it finds that a bank is in dire straits. Unless adequate
Rural Bank of Carmen (Cebu), Inc., Pilipino Rural Bank, and determined efforts are taken by the government
Inc., Philippine Countryside Rural Bank, Inc., Rural Bank against distressed and mismanaged banks, public faith in
of Calatagan (Batangas), Inc. (now Dynamic Rural Bank), the banking system is certain to deteriorate to the
Rural Bank of Darbci, Inc., Rural Bank of Kananga prejudice of the national economy itself, not to mention
(Leyte), Inc. (now First Interstate Rural Bank), Rural Bank the losses suffered by the bank depositors, creditors, and
de Bisayas Minglanilla (now Bank of East Asia), and San stockholders, who all deserve the protection of the
Pablo City Development Bank, Inc. government.
After the examinations, exit conferences were held with WHEREFORE, the petition is hereby GRANTED. The
the officers or representatives of the banks wherein the assailed CA Decision is REVERSED
SED examiners provided them with copies of Lists of
Findings/Exceptions containing the deficiencies
discovered during the examinations. The banks were then
required to comment and to undertake the remedial
measures stated in the list within 30 days. The banks
requested for extension to comply with the remedial
measures and noted that none of them had received the
Report of Examination (ROE) which finalizes the audit
findings.

The respondent banks then filed before the RTC an action


to nullify the ROE and issuance of restraining order
contending that their right to due process was violated
ROXAS & CO vs DAMBA-NFSW DUE PROCESS AND EMINENT DOMAIN

FACTS: SOLGEN VS AYALA

Petitioner voluntarily offered to sell its Hacienda Palico to


FACTS:
DAR pursuant to CARL. Subsequently, it withdraw its
offer believing said hacienda was already converted as The respondents, Ayala Land, Robinsons, and Shangri-la
non-agricultural land, not covered by CARL, under maintain and operate shopping malls while SM Prime
PP1520 declaring the municipalities of Maragondon and constructs and leases building structures.
ternate in Cavite province and the municipality of
Nasugbu as a TOURIST ZONE and for OTHER The shopping malls that respondents operate have
PURPOSE. parking facilities in which they collect parking fees from
the persons making use of their facilities.
There were proceeding in the DAR to acquire these
properties covered by CARL, which was contested by the In May 2000, a joint investigation conducted by the
Roxas & Co claiming that they were not notified of the Senate Committees on Trade and Commerce and on
proceedings before the DAR, Roxas & Co were arguing Justice and Human Rights issued Senate Committee
that they were not afforded due process.Petitioners Report No. 225 in which they find that the collection of
allegation of lack of due process goes into the validity of parking fees by herein respondents are contrary to the
the acquisition proceedings themselves. National Building Code. The Code merely requires malls
to provide parking spaces, without specifying whether it is
ISSUE: free or not.
WON due process was violated. On 4 October 2000, the OSG, herein petitioner, filed a
RULING: petition to the RTC of Makati praying that the practice of
respondents in charging parking fees is violative of the
YES. The procedure in the sending of these notices is National Building Code and its Implementing Rules and
important to comply with the requisites of due process Regulations.
especially when the owner, as in this case, is a juridical
The RTC ruled in favour of the respondents. Upon appeal,
entity.
CA denied the petition and likewise denied the motion for
The importance of the first notice, i.e., the Notice of reconsideration.
Coverage and the letter of invitation to the conference,
and its actual conduct cannot be understated. They are
steps designed to comply with the requirements of ISSUE:
administrative due process.
Whether or not the act of OSG is a valid exercise of the
The implementation of the CARL is an exercise of the power of eminent domain.
States police power and the power of eminent domain. To
the extent that the CARL prescribes retention limits to the
landowners, there is an exercise of police power for the
RULING:
regulation of private property in accordance with the
Constitution.But where, to carry out such regulation, the No. Eminent domain enables the State to forcibly acquire
owners are deprived of lands they own in excess of the private lands intended for public use upon payment of just
maximum area allowed, there is also a taking under the compensation to the owner.
power of eminent domain. The taking contemplated is not
a mere limitation of the use of the land. What is required Although in the present case, title to and/or possession of
is the surrender of the title to and physical possession of the parking facilities remain/s with respondents, the
the said excess and all beneficial rights accruing to the prohibition against their collection of parking fees from the
owner in favor of the farmer beneficiary. public, for the use of said facilities, is already tantamount
to a taking or confiscation of their properties. The State is
The Bill of Rights provides that [n]o person shall be not only requiring that respondents devote a portion of the
deprived of life, liberty or property without due process of latter’s properties for use as parking spaces, but is also
law. The CARL was not intended to take away property mandating that they give the public access to said parking
without due process of law. The exercise of the power of spaces for free. Such is already an excessive intrusion
eminent domain requires that due process be observed in into the property rights of respondents. Not only are they
the taking of private property. being deprived of the right to use a portion of their
properties as they wish, they are further prohibited from
its use or even just recovering therefrom the expenses for
the maintenance and operation of the required parking
facilities.
In conclusion, the total prohibition against the collection Cebu City then filed before the RTC a Motion to Dissolve,
by respondents of parking fees from persons who use the Quash or Recall the Writ of Garnishment, contending that
mall parking facilities has no basis in the National Building the account mentioned in Ordinance No. 1519 is not an
Code or its IRR. The State also cannot impose the same existing bank account and the garnishment of Cebu City’s
prohibition by generally invoking police power, since said bank account with Philippine Postal Bank is illegal,
prohibition amounts to a taking of respondents’ property because government funds and properties may not be
without payment of just compensation. seized under writ of execution or garnishment to satisfy
such judgement, on obvious reason of public policy. RTC
denied the said motion and likewise denied its Motion for
ORTEGA VS CITY OF CEBU Reconsideration.

FACTS:
On appeal, CA affirmed RTC’s denial of Cebu City’s
The spouses Ortega are the registered owners of a parcel Omnibus Motion to Modify Judgement and to be allowed
of land. On May 23, 1994, the Sangguniang Panglungsod to withdraw from the Expropriation Proceedings.
of Cebu City enacted City Ordinance No. 1519, giving
authority to the City Mayor to expropriate 1⁄2 portion of
such land, and appropriating for that purpose the amount ISSUE/S:
of P3,284,400.00. The amount will be charged against
Account No. 8- 93-310, Continuing Appropriation,
Account No. 101-8918-334.
a.) Whether or not the CA erred in affirming the RTC’s
denial of Cebu City’s Omnibus Motion to Modify Judgment
and to be Allowed to Withdraw from the Expropriation
Pursuant to said ordinance, Cebu City filed a Complaint Proceedings.
for Eminent Domain before the RTC against the Spouses
Ortega. RTC then declared that [Cebu City] "has the
lawful right to take the property subject of the instant case,
b.) Whether the deposit of Cebu City with the Philippine
for public use or purpose described in the complaint upon
Postal Bank, appropriated for a different purpose by its
payment of just compensation."
Sangguniang Panglungsod, can be subject to
garnishment as payment for the expropriated lot covered
by City Ordinance No. 1519.
Based on the recommendation of the appointed
Commissioners, RTC issued another order, ordering
Cebu City to pay the spouses the sum of P31,416,000.00
RULING:
as just compensation for the expropriated land.
a.) No. Section 4, Rule 67 of the Rules of Court on
Expropriation speaks of 2 stages:
RTC’s decision became final and executory after Cebu
City failed to perfect an appeal on time. RTC then issued
another order stating that Ordinance No. 1519 a. Determination of the authority of the plaintiff to exercise
appropriating the sum of P3,284,400.00 for payment of the power of eminent domain and the propriety of its
the subject lot is chargeable to Account No. 101-8918-334 exercise in the context of the facts involved in the suit.
and is now subject for execution or garnishment for it is
no longer exempted from execution.
b. Determination by the court of the just compensation for
the property sought to be taken.
Cebu City filed an Omnibus Motion to Stay Execution,
Modification of Judgment and Withdrawal of the Case,
contending that the price set by the RTC as just An order of expropriation denotes the end of the first stage
compensation to be paid to the Spouses is way beyond of expropriation. Its end then paves the way for the
the reach of its intended beneficiaries for its socialized second stage—the determination of just compensation,
housing program. The motion was denied by the RTC. and, ultimately, payment. An order of expropriation puts
Cebu City’s Motion for Reconsideration was likewise an end to any ambiguity regarding the right of the
denied. petitioner to condemn the respondents’ properties.
By virtue of the said order, a Notice of Garnishment was Because an order of expropriation merely determines the
served to Philippine Postal Bank. authority to exercise the power of eminent domain and the
propriety of such exercise, its issuance does not hinge on compel its Sangguniang Panglungsod to enact an
the payment of just compensation. After all, there would appropriation ordinance for the satisfaction of the
be no point in determining just compensation if, in the first Spouses Ortega’s claim.
place, the plaintiff’s right to expropriate the property was
not first clearly established.
It is a settled rule that government funds and properties
may not be seized under writs of execution or
Conversely, as is evident from the foregoing, an order by garnishment to satisfy judgments, based on obvious
the trial court fixing just compensation does not affect a consideration of public policy. Disbursements of public
prior order of expropriation. As applied to the case at bar, funds must be covered by the corresponding
Cebu City can no longer ask for modification of the appropriation as required by law. The functions and public
judgment, much less, withdraw its complaint, after it failed services rendered by the State cannot be allowed to be
to appeal even the first stage of the expropriation paralyzed or disrupted by the diversion of public funds
proceedings. from their legitimate and specific objects, as appropriated
by law.

It is well-settled in jurisprudence that the determination of


just compensation is a judicial prerogative. In Export EUSEBIO VS LUIS
Processing Zone Authority v. Dulay, we declared:
FACTS:

The determination of "just compensation" in eminent Respondents are the registered owners of a parcel of land
domain cases is a judicial function. The executive which was taken by the City of Pasig sometime in 1980
department or the legislature may make the initial and was used as a municipal road.
determinations but when a party claims a violation of the
guarantee in the Bill of Rights that private property may
not be taken for public use without just compensation, no On February 1, 1983, the Sanggunian of Pasig City
statute, decree, or executive order can mandate that its passed Resolution No. 15 authorizing payments to
own determination shall prevail over the court’s findings. respondents for said parcel of land. However, the
Much less can the courts be precluded from looking into Appraisal Committee assessed the value of the land only
the "just-ness" of the decreed compensation. at P150/sqm. In a letter dated June 26, 1995, respondents
requested the Appraisal Commiitte to consider
P2,000/sqm as the value of their land.
Likewise, in the recent cases of National Power
Corporation v. dela Cruz and Forfom Development
Corporation v. Philippine National Railways, we Respondent’s counsel sent a letter to Mayor Eusebio,
emphasized the primacy of judicial prerogative in the demanding the amount of P5,000.00/sqm as just
ascertainment of just compensation as aided by the compensation for respondents property. In his reply,
appointed commissioners, to wit: Mayor Eusebio said that the City of Pasig cannot pay
them more than the amount set by the Appraisal
Committee.
Though the ascertainment of just compensation is a
judicial prerogative, the appointment of commissioners to
ascertain just compensation for the property sought to be Thus, on October 8, 1996, respondents’ filed a complaint
taken is a mandatory requirement in expropriation cases. before the RTC praying that the property be returned to
While it is true that the findings of commissioners may be them with payment of reasonable rental for 16 years of
disregarded and the trial court may substitute its own use, or in the event said property can no longer be
estimate of the value, it may only do so for valid reasons; returned, that petitioners be ordered to pay just
that is, where the commissioners have applied illegal compensation.
principles to the evidence submitted to them, where they
have disregarded a clear preponderance of evidence, or
where the amount allowed is either grossly inadequate or
RTC ruled in favor of the respondents. Upon appeal, CA
excessive. Thus, "trial with the aid of the commissioners
affirmed the decision of the RTC and denied the
is a substantial right that may not be done away with petitioners motion for reconsideration.
capriciously or for no reason at all.”

b.) No. The proper remedy of [the Spouses Ortega] is to


file a mandamus case against Cebu City in order to
ISSUE:

Whether or not property taken without the benefit of With regard to the time as to when just compensation
expropriation proceeding required by law in the taking of should be fixed, it is settled jurisprudence that where
private property for public use can be regained. property was taken without the benefit of expropriation
proceedings, and its owner files an action for recovery of
possession thereof before the commencement of
RULING: expropriation proceedings, it is the value of the property
at the time of taking that is controlling.
Citing the case of Republic of the Philippines vs Court of
Appeals, the Court emphasized that where private
property is taken by the Government for public use In this case, the trial court should have fixed just
without first acquiring title thereto either through compensation for the property at its value as of the time
expropriation or negotiated sale, the owners action to of taking in 1980, but there is nothing on record showing
recover the land or the value thereof does not prescribe. the value of the property at that time. The trial court,
The Court went on to remind government agencies not to therefore, clearly erred when it based its valuation for the
exercise the power of eminent domain with wanton subject land on the price paid for properties in the same
disregard for property rights as Section 9, Article III of the location, taken by the city government only sometime in
Constitution provides that private property shall not be the year 1994.
taken for public use without just compensation.

City of Iloilo vs. Contreras and Javellana


In Forform Development Corporation vs Philippine (G.R. No. 168967)
National Railways, the court declared that recovery of
possession of the property by the landowner can no
longer be allowed on the grounds of estoppel and, more FACTS:
importantly, of public policy which imposes upon the Petitioner filed a Complaint for eminent domain against
public utility the obligation to continue its services to the private respondent Elpidio T. Javellana and Southern
public. The non-filing of the case for expropriation will not Negros Development Bank, the latter as mortgagee which
necessarily lead to the return of the property to the sought to expropriate two parcels of land registered in
landowner. What is left to the landowner is the right of Javellana’s name to be used as a school site for Lapaz
compensation. It is settled that non-payment of just High School. Petitioner alleged that the Subject Property
compensation does not entitle the private landowners to was declared for tax purposes to have a value of P60.00
recover possession of their expropriated lot. Herein per square meter, or a total value of P43,560.00.
respondents also failed to question the taking of their Javellana also claimed that the true fair market value of
property for a long period of time (from 1980 until the early his property was no less than P220.00 per square meter.
1990s). Petitioner was able to take physical possession of the
The prevailing doctrine on judicial determination of just properties sometime in the middle of 1985. Private
compensation is that set forth in Forfom. Therein, the respondent thus demanded his just compensation as well
Court ruled that even if there are no expropriation as interest. Private respondent alleged that since he had
proceedings instituted to determine just compensation, not been compensated for the Subject Property,
the trial court is still mandated to act in accordance with petitioner’s possession was illegal, and he was entitled to
the procedure provided for in Section 5, Rule 67 of the recovery of possession of his lots. Private respondent
1997 Rules of Civil Procedure, requiring the appointment could only demand for the payment of just compensation.
of not more than three competent and disinterested
commissioners to ascertain and report to the court the just
compensation for the subject property. The Court ISSUE:
reiterated its ruling in National Power Corporation v. Dela
Cruz that trial with the aid of commissioners is a Whether or not just compensation should be based on the
substantial right that may not be done away with fair market value of a property at the time of the filing of
capriciously or for no reason at all. It was also complaint.
emphasized therein that although ascertainment of just
compensation is a judicial prerogative, the commissioners
findings may only be disregarded or substituted with the RULING:
trial courts own estimation of the propertys value only if
When the taking of the property sought to be expropriated
the commissioners have applied illegal principles to the
coincides with the commencement of the expropriation
evidence submitted to them, where they have disregarded
proceedings, or takes place subsequent to the filing of the
a clear preponderance of evidence, or where the amount
complaint for eminent domain, the just compensation
allowed is either grossly inadequate or excessive.
should be determined as of the date of the filing of the proceedings for the same parcels of land. Rule 67,
complaint. Just compensation is to be determined “as of Section 1 of the Rules of Court allows filing of
the date of filing of the complaint”. The reckoning date expropriation even when “the title to any property sought
should be in 2004 because of the clear injustice to the to be condemned appears to be the Republic, although
private respondent who all these years has been deprived occupied by private individuals.”
of the beneficial use of his properties. City of Iloilo is held
liable for damages for taking private respondent’s
property without payment of just compensation.

c.) Elementary is the rule that prescription does not run


RP vs. MANGOTARA against the State. The indefeasibility of a Title over land
previously public is not a bar to an investigation by the
Director of Lands as to how such title has been acquired,
FACTS: if the purpose of such investigation is to determine w/n
fraud had been committed in securing such title in order
Dona Demetria owns two parcels of land and has titles to
that the appropriate action for reversion may be filed by
prove ownership. 83 years later authenticity of said
the government.
ownership was still in doubt and the government filed a
case of expropriation against occupants of the land (not
really the owners, just renters). The occupants questioned EQUAL PROTECTION
the expropriation case contending that expropriation
cases can only be filed against owners, not mere Dumlao vs. COMELEC
occupants, of the land. The government also filed a case (G.R. No. L-52245)
of Reversion against Dona Demetria’s heirs contending
two parcels of land were fraudulently acquired. The
FACTS:
owners of the land questioned the case for Reversion
contending that, since they filed a case for Reconstitution Patricio Dumlao, is a former Governor of Nueva Vizcaya,
of Titles in 1914 and they still have said titles at present, who has filed his certificate of candidacy for said position
the government has no right to the land. of Governor in the forthcoming elections of January 30,
1980. Petitioner Dumlao specifically questions the
constitutionality of section 4 of Batas Pambansa Blg. 52
ISSUE/S: as discriminatory and contrary to the equal protection and
due process guarantees of the Constitution which
a.) WON owners of parcels of land are indispensable provides that “….Any retired elective provincial city or
parties to an expropriation proceeding? municipal official who has received payment of the
retirement benefits to which he is entitled under the law
and who shall have been 65 years of age at the
b.) WON the government erred in filing both a case for commencement of the term of office to which he seeks to
expropriation and a case for Reversion against the be elected shall not be qualified to run for the same
occupants and the owners of the land? elective local office from which he has retired.” He
likewise alleges that the provision is directed insidiously
against him, and is based on “purely arbitrary grounds,
c.) WON, in filing the case for Reversion, the government therefore, class legislation.
is barred by res judicata and prescription?

ISSUE:
RULING: Whether or Not Sec. 4 of BP.52 is unconstitutional being
contrary to the equal protection and due process rights.

a.) No. The defendants in an expropriation case are not


limited to the owners of the property condemned. They RULING:
include all other persons, owning, occupying or claiming
to own the property. When property is taken by eminent No. The guarantee of equal protection is subject to
domain, the owner is not an indispensable party to the rational classification based on reasonable and real
proceeding. differentiations. In the present case, employees 65 years
of age have been classified differently from younger
employees. The former are subject to compulsory
b.) No. The Republic is not engaging in contradictions retirement while the latter are not.
when it instituted both expropriation/reversion
2010 Decision Section 4(a) of COMELEC Resolution No.
8678 is not violative of the equal protection clause.
In respect of election to provincial, city, or municipal
positions, to require that candidates should not be more
than 65 years of age at the time they assume office, if
applicable to everyone, might or might not be a There is substantial distinction. Elective officials are
reasonable classification although, as the Solicitor elected by his constituents, if they are deemed resigned,
General has intimated, a good policy of the law should be the constituents will be affected. On the other hand, in the
to promote the emergence of younger blood in our case of appointive officials, they do not have ordinary
political elective echelons. succession, thus, there will be vacancy during
resignation.

The equal protection clause does not forbid all legal


classification. What is proscribed is a classification which The dichotomized treatment of appointive and elective
is arbitrary and unreasonable. There is reason to officials is therefore germane to the purposes of the law.
disqualify a 65 year old elective official who is trying to run For the law was made not merely to preserve the integrity,
for office because there is the “need for new blood to efficiency, and discipline of the public service; the
assume relevance”. When an official has retired he has Legislature, whose wisdom is outside the rubric of judicial
already declared himself tired and unavailable for the scrutiny, also thought it wise to balance this with the
same government work. competing, yet equally compelling, interest of deferring to
the sovereign will.

The first paragraph of section 4 of Batas Pambansa


Bilang 52 is valid.
Ang Ladlad vs. COMELEC
GR No. 190582
Quinto vs. COMELEC
(G. R. No. 189698)
FACTS:

FACTS: Ang LADLAD is an organization who identifies


themselves as LGBT that applied for registration with the
R.A. 8676 provides that all elective officials are not COMELEC as party-list but the application was
deemed resigned upon filing of their certificate of denied/dismissed the petition (1) on moral grounds, (2)
candidacy, however, appointive officials are considered nuisance. The COMELEC also cited verses/provisions
deemed resigned upon filing of the COC’s. from the Bible and the Qur’an, that they advocate sexual
immorality among other things. It would expose our youth
to an environment that does not conform to the teachings
ISSUE: of our faith.

Whether or not Section 4 (a) of COMELEC Resolution No. Ang LADLAD is questioning that the basis of the
8678 is violative of the equal protection clause. COMELEC to discredit, deny or dismiss the petition was
baseless. And also there was a violation of equal
protection according to LADLAD. One of the rulings of the
COMELEC here was that the LGBT sector is not
RULING:
enumerated in the constitution and RA 7941 nor is it
Yes. Persons holding appointive positions as ipso facto related to any of the sectors enumerated in that law.
resigned upon filing of CoCs, but not considering resigned
The Court said that the ruling of the COMELEC was
all other civil servants, specifically elective ones, the law
wrong because the listing under that law is not exclusive.
duly discriminates against the first class (appointive
Meaning if in analogous circumstances they may fall
officials). Applying the four requisites of valid
under that listing as enumerated and explicitly ruled in
classification, the Court finds that treatment of persons
Ang Bagong Bayani vs. COMELEC. The enumeration of
holding appointive officers as opposed to those holding
the marginalized sectors is not exclusive.
elective ones is not germane to the purposes of law
(Requisite No. 2). There is no valid justification to treat The Court said that the invocation of the Bible and Qur’an
appointive officials differently from the elective ones. The contravenes Article 3 Section 5 of the constitution on
classification simply fails to meet the test that it should be neutrality. No law shall be made respecting an
germane to the purposes of law. establishment of religion, or prohibiting the free exercise
thereof. Meaning all government issuances must be
neutral. There must be a secular purpose of the religious [EMRASON] paid the fees, dues and licenses needed to
purpose. proceed with property development.

On June15. 1988, Republic Act No. 6657, otherwise


known as the Comprehensive Agrarian Reform Law or
ISSUE: CARL, took effect, ushering in a new process of land
Whether or not denying the application of Ang Ladlad is classification, acquisition and distribution. Secretary
violative of the equal protection clause. Benjamin Leong sent out the first of four batches of
notices of acquisition to EMARSON.EMRASON] filed with
the Department of Agrarian Reform Adjudication Board
(DARAB),Region IV, Pasig, Metro Manila, separate
RULING:
petitions to nullify the first three sets of the above notices.
Yes. COMELEC made an unwarranted and Legal Division of DAR rendered a decision declaring as
impermissible classification not justified by the null and void all the notices of acquisitions. At this
circumstances of the case. According to COMELEC, the juncture, the DAR had already prepared Certificates of
majority of Filipinos considers homosexual conduct as Land Ownership Award (CLOAs) to distribute the subject
immoral and unacceptable. Such reason is sufficient to property to farmer-beneficiaries. However, on appeal, the
disqualify the petitioner. Homosexuals are a class in writ of preliminary injunction issued by the Court of
themselves for the purposes of equal protection clause. Appeals enjoined the release of the CLOAs The Court of
Moral disapproval of an unpopular minority is not a Appeals allowed the intervention of Buklod because -the
legitimate state interest that is sufficient to satisfy the latter's participation was "not being in any way prejudicial
rational basis review under the equal protection clause. to the interest of the original parties, nor will such
LGBT has the same interest in participating in the party- intervention change the factual legal complexion of the
list system. Laws of general application should apply with case." The Court of Appeals further observed that the
equal force to LGBTs. subject property has never been devoted to any
agricultural activity. thus, CA granted the petition.

BUKLOD NANG MAGBUBUKID SA LUPAING RAMOS, INC.


vs E. M. RAMOS AND SONS, INC. ISSUE: W/n the subject property could be placed under
G.R. No. 131481 (March 16 , 2011) the CARP.

FACTS:
RULING:

Several parcels of unirrigated land (303.38545 hectares)


which from part of a larger expanse with an area of 372 No, Section 4, Chapter II of the CARL, as amended,24
hectares situated at Barangay Langkaan, Dasmarinas, particularly defines the coverage of the CARP, to wit:
Cavite was originally owned by Manila Golf and Country
SEC. 4. Scope. - The Comprehensive Agrarian Reform
Club, he property was acquired by the [herein respondent
Law of 1988 shall cover, regardless of tenurial
EMRASON] in 1965 for the purpose of developing the
arrangement and commodity produced, all public and
same into a residential subdivision known as "Traveller's
private agricultural lands as provided in Proclamation
Life Homes".
No. 131 and Executive Order No. 229, including other
In 1971, the Municipal Council of Dasmarinas, Cavite, lands of the public domain suitable for
acting pursuant to Republic Act (R.A.) No. 2264, agriculture: Provided, That landholdings of landowners
otherwise known as the "Local Autonomy Act", enacted with a total area of five (5) hectares and below shall not
Municipal Ordinance No. 1, hereinafter referred to as be covered for acquisition and distribution to qualified
Ordinance No. 1, entitled "An Ordinance Providing beneficiaries.
Subdivision Regulation and Providing Penalties for
More specifically, the following lands are covered by the
Violation Thereof." In May, 1972 E.M. Ramos and Sons,
CARP:
Inc., applied for an authority to convert and development
its aforementioned 372-hectare property into a residential
(a) All alienable and disposable lands of the public domain
subdivision, ataching to the application detailed
devoted to or suitable for agriculture. No reclassification
development plans and development proposals from
of forest or mineral lands to agricultural lands shall be
Bancom Development Corporation and San Miguel
undertaken after the approval of this Act until Congress,
Corporation. Acting thereon the Municipal Council of
taking into account ecological, developmental and equity
Dasmarinas, Cavite passed on July 9, 1972 Municipal
considerations, shall have determined by law, the
Ordinance No. 29-A (Ordinance "No. 29-A, for brevity),
specific limits of the public domain;
approving [EMRASON's] application. Subsequently,
(b) All lands of the public domain in excess of the specific Conversion is the act of changing the current use of a
limits as determined by Congress in the preceding piece of agricultural land into some other use as approved
paragraph; by the Department of Agrarian Reform. Reclassification,
on the other hand, is the act of specifying how agricultural
(c) All other lands owned by the Government devoted to lands shall be utilized for non-agricultural uses such as
or suitable for agriculture; and residential, industrial, commercial, as embodied in the
land use plan, subject to the requirements and procedure
(d) All private lands devoted to or suitable for for land use conversion, x x x. (Italics supplied.)
agriculture regardless of the agricultural products
raised or that can be raised thereon.
Reclassification also includes the reversion of non-
Section 3(c), Chapter I of the CARL further narrows down agricultural lands to agricultural use.
the definition of agricultural land that is subject to CARP
to "land devoted to agricultural activity as defined in this Under the present Local Government Code, it is clear that
Act and not classified as mineral, forest, residential, the authority to reclassify agricultural lands primarily
commercial or industrial land." resides in the sanggunian of the city or municipality.

The CARL took effect on June 15, 1988. To be exempt Resolution No. 29-A is a valid ordinance, which, upon its
from the CARP, the subject property should have already approval on July 9, 1972, immediately effected the zoning
been reclassified as residential prior to said date. and reclassifying of the subject property for residential
use. It need not comply with any of the requirements or
Zoning is governmental regulation of the uses of land and conditions which DAR and Buklod are insisting upon.
buildings according to districts or zones. It is
comprehensive where it is governed by a single plan for DAR and Buklod aver that Resolution No. 29-A was not
the entire municipality and prevails throughout the reviewed and approved by the NPC, in violation of the line
municipality in accordance with that plan. It is partial or in Section 3 of the Local Autonomy Act of 1959, stating
limited where it is applicable only to a certain part of the that "[c]ities and municipalities may, however, consult the
municipality or to certain uses. Fire limits, height districts National Planning Commission on matters pertaining to
and building regulations are forms of partial or limited planning and zoning." Consideration must be given,
zoning or use regulation that are antecedents of modern however, to the use of the word "may" in the said
comprehensive zoning, (pp. 11-12.) sentence. Where the provision reads "may," this word
shows that it is not mandatory but discretionary. It is an
The term "zoning," ordinarily used with the connotation of auxiliary verb indicating liberty, opportunity, permission
comprehensive or general zoning, refers to governmental and possibility. The use of the word "may" in a statute
regulation of the uses of land and buildings according to denotes that it is directory in nature and generally
districts or zones. This regulation must and does utilize permissive only. The "plain meaning rule" or verba legis
classification of uses within districts as well as in statutory construction is thus applicable in this case.
classification of districts, inasmuch as it manifestly is Where the words of a statute are clear, plain, and free
impossible to deal specifically with each of the from ambiguity, it must be given its literal meaning and
innumerable uses made of land and buildings. applied without attempted interpretation. Since
Accordingly, (zoning has been defined as the confining of consultation with the NPC was merely discretionary, then
certain classes of buildings and uses to certain localities, there were only two mandatory requirements for a valid
areas, districts or zones.) It has been stated that zoning zoning or subdivision ordinance or regulation under
is the regulation by districts of building development and Section 3 of the Local Autonomy Act of 1959, namely,
uses of property, and that the term "zoning" is not only that (1) the ordinance or regulation be adopted by the city
capable of this definition but has acquired a technical and or municipal board or council; and (2) it be approved by
artificial meaning in accordance therewith. (Zoning is the the city or municipal mayor, both of which were complied
separation of the municipality into districts and the with byl Resolution No. 29-A.
regulation of buildings and structures within the districts
so created, in accordance with their construction, and
nature and extent of their use. It is a dedication of districts
delimited to particular uses designed to subserve the
general welfare.) Numerous other definitions of zoning
more or less in accordance with these have been given in
the cases, (pp. 27-28.)

The concept that concerns this Court in the instant cases


is the reclassification of agricultural lands. In Alarcon v.
Court of Appeals, the Court had the occasion to define
and distinguish reclassification from conversion as
follows:
Trillanes vs. Pimentel Biraogo vs. Philippine Truth Commission
(G.R. No. 179817) (G.R. No. 192935)

FACTS: FACTS:

Petitioner Trillanes IV is on trial for coup d’état in relation EO No. 1 was issued by President Aquino to investigate
to the “Oakwood Incident.” In the 2007 elections, he won reported cases of graft and corruption of previous
a seat in the Senate with a six-year term commencing at administration. The petitioners assail EO No. 1 is violative
noon on June 30, 2007. Petitioner now asks the Court that of the constitutional safeguard because it does not apply
he be allowed to attend all official functions of the Senate, equally to all members of the same class such that the
alleging mainly that his case is distinct from that of intent of singling out the "previous administration" as its
Jalosjos as his case is still pending resolution whereas sole object makes the PTC an "adventure in partisan
that in the Jalosjos case, there was already conviction. He hostility." To be accorded with validity, the commission
asserts that he continues to enjoy civil and political rights must also cover reports of graft and corruption in virtually
since the presumption of innocence is still in his favor. all administrations previous to that of former President
Petitioner also illustrates that Jalosjos was charged with Arroyo.
crimes involving moral turpitude, whereas he is indicted
for coup d'etat which is regarded as a "political offense."
ISSUE:

ISSUE: Whether or not the Truth Commission is violative of the


equal protection clause.
Whether or not petitioner may be allowed to attend the
Senate sessions.
RULING:

RULING: YES. Executive Order No. 1 should be struck down as


violative of the equal protection clause. For a
No. The distinctions cited by petitioner were not elemental classification to meet the requirements of constitutionality,
in the pronouncement in Jalosjos that election to it must include or embrace all persons who naturally
Congress is not a reasonable classification in criminal law belong to the class. The clear mandate of the envisioned
enforcement as the functions and duties of the office are truth commission is to investigate and find out the truth
not substantial distinctions which lift one from the class of “concerning the reported cases of graft and corruption
prisoners interrupted in their freedom and restricted in during the previous administration” only. The previous
liberty of movement. The Constitution provides: administration was picked out was deliberate and
intentional as can be gleaned from the fact that it was
underscored at least three times in the assailed executive
All persons, except those charged with offenses order. The Arroyo administration is but just a member of
punishable by reclusion perpetua when evidence of guilt a class, that is, a class of past administrations. It is not a
is strong, shall, before conviction, be bailable by sufficient class of its own. Executive Order No. 1 suffers from
sureties, or be released on recognizance as may be arbitrary classification.
provided by law. The right to bail shall not be impaired
even when the privilege of the writ of habeas corpus is
suspended. Excessive bail shall not be required.

The cited provisions apply equally to rape and coup d'etat


cases, both being punishable by reclusion perpetua.
Within the class of offenses covered by the stated range
of imposable penalties, there is clearly no distinction as to
the political complexion of or moral turpitude involved in
the crime charged.
II. DUE PROCESS AND EQUAL PROTECTION AS LIMITATION ON coercion. The burden of proving, by clear and positive
POLICE POWER, EMINENT DOMAIN AND TAXATION
testimony, that the necessary consent was obtained and
REQUIREMENTS FOR SEARCH WARRANTS that it was freely and voluntarily given lies with the State.

IN RE MORALES ACQUIESCENCE IN THE LOSS OF FUNDAMENTAL


RIGHTS IS NOT TO BE PRESUMED AND COURTS
INDULGE EVERY REASONABLE PRESUMPTION
FACTS: AGAINST WAIVER OF FUNDAMENTAL
OCA received two unsigned and undated letters from CONSTITUTIONAL RIGHTS. To constitute a valid
anonymous who claim alleging Atty. Morales, was consent or waiver of the constitutional guarantee against
consuming his working hours filing and attending to obtrusive searches, it must be shown that (1) the right
personal cases, such as administrative cases against exists; (2) that the person involved had knowledge, either
employees in his old sala, using office supplies, actual or constructive, of the existence of such right; and
equipment and utilities. Because of the said letter, DCA (3) the said person had an actual intention to relinquish
Dela Cruz together with four NBI agents conducted a spot the right.
investigation. The team was able to access the personal
computer of Atty. Morales and printed two documents
stored in its hard drive, and thereby seized and took In this case, what is missing is a showing that Atty.
Morales's computer to the custody of the OCA Morales had an actual intention to relinquish his right.
While he may have agreed to the opening of his personal
computer and the printing of files therefrom, in the
ISSUE: presence of DCA Dela Cruz, his staff and some NBI
agents during the March 16, 2005 spot investigation, it is
Whether or not the pleadings found in Atty. Morales's also of record that Atty. Morales immediately filed an
personal computer admissible in the present administrative case against said persons questioning the
administrative case against him? validity of the investigation, specifically invoking his
constitutional right against unreasonable search and
seizure.
RULING:

No. Enshrined in our Constitution is the inviolable right of While Atty. Morales may have fallen short of the exacting
the people to be secure in their persons and properties standards required of every court employee,
against unreasonable searches and seizures, which is unfortunately, the Court cannot use the evidence obtained
provided for under Section 2, Article III thereof. The from his personal computer against him for it violated his
exclusionary rule under Section 3(2), Article III of the constitutional right.
Constitution also bars the admission of evidence obtained
in violation of such right. The fact that the present case is
administrative in nature does not render the above
PEOPLE V. NUNEZ
principle inoperative. As expounded in Zulueta v. Court of
Appeals, any violation of the aforestated constitutional
right renders the evidence obtained inadmissible for any FACTS:
purpose in any proceeding.
With a summon, Brgy. Captain and Tanod of Brgy. San
Antonio assisted Laguna Police Detectives in
coordination with the Los Banos Police Station and IID
DCA Dela Cruz in his report claims that that they were
Mobile Force in a search operation in the house of Raul
able to obtain the subject pleadings with the consent of
R. Nuez based on reports of drug possession. Upon
Atty. Morales. The Court finds however that such
arriving at appellants house, Mundin called on appellant
allegation on his part, even with a similar allegation from
to come out. Thereafter, Commanding Officer
one of his staff, is not sufficient to make the present case
Pagkalinawan showed Nuez the warrant. Several shabu
fall under the category of a valid warrantless search.
paraphernalia were found. Following the search, SPO1
Ilagan issued a Receipt for Property Seized and a
Certification of Orderly Search which appellant signed.
Consent to a search is not to be lightly inferred and must RTC convicted appellant for a violation of R.A. 6425 as
be shown by clear and convincing evidence. It must be amended which was affirmed by the CA. Appellant assails
voluntary in order to validate an otherwise illegal search; the validity of the search warrant as it did not indicate his
that is, the consent must be unequivocal, specific, exact address but only the barangay and street of his
intelligently given and uncontaminated by any duress or residence. He maintains that none of the occupants
witnessed the search as they were all kept in the living DEL ROSARIO V. DONATO
room. Finally, appellant questions why the prosecution did
not call the barangay officials as witnesses to shed light
FACTS:
on the details of the search.
Del Rosarios alleged that NBI agents entered by force in
their premises to search for fake Marlboro Cigarettes
ISSUE: pursuant to a Search Warrant unlawfully obtained by RTC
of Angeles. Their search yielded no fake Marlboro
W/N there was an irregularity in the search conducted. cigarettes. Subsequently, petitioner initiated a 50 million
suit for damages. As the case was elevated before CA, it
held that the petitioner’s complaint before the RTC failed
RULING: to state a cause of action against NBI. Del Rosarios allege
is that respondents NBI agents used an unlawfully
YES. Turning to the objects which may be confiscated obtained search warrant against them, evidenced by the
during the search, Section 3, Rule 126 of the Rules of fact that, contrary to the sworn statements used to get
Court is pertinent: SEC. 3. Personal property to be seized. such warrant, the NBI agents found no fake Marlboro
A search warrant may be issued for the search and cigarettes in petitioner Alexander del Rosarios premises.
seizure of personal property: (a) Subject of the offense;
(b) Stolen or embezzled and other proceeds, or fruits of
the offense; or (c) Used or intended to be used as the
means of committing an offense. As a rule, only the
personal properties described in the search warrant may ISSUE:
be seized by the authorities.[23] In the case at bar, Search
Warrant No. 42[24] specifically authorized the taking of W/N CA correctly rule that the complaint of the Del
methamphetamine hydrochloride (shabu) and Rosarios did not state a cause of action against NBI
paraphernalia(s) only. By the principle of ejusdem agents.
generis, where a statute describes things of a particular
class or kind accompanied by words of a generic
character, the generic word will usually be limited to things RULING:
of a similar nature with those particularly enumerated,
unless there be something in the context of the statement YES. A JUDICIALLY ORDERED SEARCH THAT FAILS
which would repel such inference. Thus, we are here TO YIELD THE DESCRIBED ILLICIT ARTICLE DOES
constrained to point out an irregularity in the search NOT OF ITSELF RENDER THE COURTS ORDER
conducted. Certainly, the ladys wallet, cash, grinder, UNLAWFUL. The Del Rosarios did not allege that
camera, component, speakers, electric planer, jigsaw, respondents NBI agents violated their right by fabricating
electric tester, saws, hammer, drill, and bolo were not testimonies to convince the RTC of Angeles City to issue
encompassed by the word paraphernalia as they bear no the search warrant. Their allegation that the NBI agents
relation to the use or manufacture of drugs. In seizing the used an unlawfully obtained search warrant is a mere
said items then, the police officers exercised their own conclusion of law. Allegations of bad faith, malice, and
discretion and determined for themselves which items in other related words without ultimate facts to support the
appellants residence they believed were proceeds of the same are mere conclusions of law. As such, it exposes
crime or means of committing the offense. This is, in our the complaint to a motion to dismiss on ground of failure
view, absolutely impermissible. to state a cause of action.

The purpose of the constitutional requirement that the


articles to be seized be particularly described in the
warrant is to limit the things to be taken to those, and only
those particularly described in the search warrant -- to
leave the officers of the law with no discretion regarding
what articles they should seize. A search warrant is not a
sweeping authority empowering a raiding party to
undertake a fishing expedition to confiscate any and all
kinds of evidence or articles relating to a crime.
Accordingly, the objects taken which were not specified in
the search warrant should be restored to appellant.
PEOPLE V. TUAN prudent man to believe that an offense has been
committed, and the objects in connection with the offense
sought to be seized are in the place sought to be
FACTS:
searched. Substantial basis exists in this case. Judge
Two separate Information were filed before the RTC Cortes found probable cause for the issuance of the
against accused- appellant for illegal possession of Search Warrant for accused-appellants residence after
marijuana and illegal possession of firearm. Accused- said judges personal examination of SPO2 Fernandez,
appellant insists that the items allegedly seized from her
house are inadmissible as evidence because the Search
Warrant issued for her house was invalid for failing to CONSTITUTIONAL REQUIREMENT OF
comply with the constitutional and statutory requirements. DEFINITENESS.A description of the place to be searched
is sufficient if the officer serving the warrant can, with
reasonable effort, ascertain and identify the place
She alleged that the following defects which made said intended and distinguish it from other places in the
Search Warrant void: (1) the informants made community. A designation or description that points out
misrepresentation of facts in the Application for Search the place to be searched to the exclusion of all others, and
Warrant filed with the MTCC; (2) MTCC Judge failed to on inquiry unerringly leads the peace officers to it,
consider the informants admission that they themselves satisfies the constitutional requirement of definiteness. In
were selling marijuana; and (3) the Search Warrant failed the case at bar, the address and description of the place
to particularly describe the place to be searched because to be searched in the Search Warrant was specific
the house was a two-storey building composed of several enough. There was only one house located at the stated
rooms. address, which was accused-appellants residence,
consisting of a structure with two floors and composed of
several rooms.
ISSUE:

W/N the search warrant is valid? PEOPLE V. MAMARIL

FACTS:
RULING:
OLIVE RUBIO MAMARIL was convicted due to
Yes. The validity of the issuance of a search warrant rests
possession of dangerous drugs in violation of Section 11,
upon the following factors: (1) it must be issued upon
Article II of the Comprehensive Dangerous Drugs Act of
probable cause; (2) the probable cause must be
2002.
determined by the judge himself and not by the applicant
or any other person; (3) in the determination of probable Accused-appellant alleged that the issued search warrant
cause, the judge must examine, under oath or affirmation, was not based on probable cause. It relied heavily on its
the complainant and such witnesses as the latter may argument that SPO4 Gotidoc, as the applicant of the
produce; and (4) the warrant issued must particularly search warrant, did not testify on facts personally known
describe the place to be searched and persons or things to him but simply relied on stories that the accused-
to be seized. appellant was peddling illegal drugs. W/N there is a
probable cause in issuing the search warrant? Yes.
PROBABLE CAUSE means such facts and
In this case, (2) and (3) for a validly issued search warrant circumstances which would lead a reasonable discreet
were complied with, i.e., personal determination of and prudent man to believe that an offense has been
probable cause by Judge Cortes; and examination, under committed and that the objects sought in connection with
oath or affirmation, of SPO2 Fernandez and the two the offense are in the place sought to be searched.
informants by Judge Cortes.

Based on the records, the Court is convinced that the


and (4) i.e., existence of probable cause; and particular questioned search warrant was based on a probable
description of the place to be searched and things to be cause. A portion of the direct testimony of SPO4 Gotidoc
seized is to be determined. A MAGISTRATE’S where he said that there were many persons who were
DETERMINATION OF PROBABLE CAUSE FOR THE going to her place and they’ve been hearing news that
ISSUANCE OF A SEARCH WARRANT MUST HAVE she is selling prohibited drugs and some of them were
SUBSTANTIAL BASIS FOR THAT DETERMINATION. even identified. Furthermore, the authorities already
Substantial basis means that the questions of the conducted surveillance prior to the application for search
examining judge brought out such facts and warrant.
circumstances as would lead a reasonably discreet and
ISSUE:

Whether or not there was a valid issuance of a search Hence the case was dismissed. Now the petitioner, before
warrant. Supreme Court contended that he filed information for
qualified theft based on the same subject matter of the
dismissed robbery and would like to use the item seized
YES. Section 6, Rule 126 of the Rules on Criminal in the previously conducted search for the new
Procedure provides that: information of qualified theft.

If the judge is satisfied of the existence of facts upon


which the application is based or that there is probable
cause to believe that they exist, he shall issue the warrant,
which must be substantially in the form prescribed by ISSUE:
these Rules. (Emphasis supplied)
Whether or not the items seized in the previously
conducted search warrant issued by the court for robbery
be included and used for the filing of for an information for
THERE IS NO GENERAL FORMULA OR FIXED RULE qualified theft.
FOR THE DETERMINATION OF PROBABLE CAUSE
since the same must be decided in light of the conditions
obtaining in given situations and its existence depends to
a large degree upon the findings or opinion of the judge RULING:
conducting the examination. NO. Contrary to petitioners contention, he cannot use the
items seized as evidence in any other offense except in
that in which the subject search warrants were issued.
JUDICIAL FUNCTION’S PRESUMPTION OF Section 4, Rule 126 of the Revised Rules of Court
REGULARITY It is presumed that a judicial function has provides:
been regularly performed, absent a showing to the
contrary. A magistrates determination of a probable cause
for the issuance of a search warrant is paid with great Section 4. Requisites for issuing search warrant. A search
deference by a reviewing court, as long as there was warrant shall not issue except upon probable cause in
substantial basis for that determination. connection with one specific offense to be determined
personally by the judge after examination under oath or
affirmation of the complainant and the witnesses he may
The defenses reliance of the quoted testimony of the produce, and particularly describing the place to be
police officer alone, without any other evidence to show searched and things to be seized which may be anywhere
that there was indeed lack of personal knowledge, is in the Philippines.
insufficient to overturn the finding of the trial court. The
accused-appellant, having failed to present substantial
rebuttal evidence to defeat the presumption of regularity Thus, a search warrant may be issued only if there is
of duty of the issuing judge, will not be sustained by this probable cause in connection with only one specific
Court. offense alleged in an application on the basis of the
applicants personal knowledge and his or her witnesses.
Petitioner cannot, therefore, utilize the evidence seized by
SY TAN vs SY TIONG GUE virtue of the search warrants issued in connection with the
case of Robbery in a separate case of Qualified Theft,
even if both cases emanated from the same incident.
FACTS:

Romer Sy Tan (PETITIONER) filed a criminal case of


robbery against respondents Tiong Gue, et al. The Moreover, considering that the withdrawal of the
Respondents moved for the withdrawal of the information Information was based on the findings of the CA, as
which was subsequently granted by the RTC on the affirmed by this Court, that there was no probable cause
ground that the essential elements of robbery i.e., to indict respondents for the crime of Robbery absent the
unlawful taking with intent to gain, with force and essential element of unlawful taking, which is likewise an
intimidation, were absent as decided upon by the Court of essential element for the crime of Qualified Theft, all
Appeals on an prior complaint. Thus, there was lack of offenses which are necessarily included in the crime of
probable cause, warranting the withdrawal of the Robbery can no longer be filed, much more, prosper.
Information.
TY v DE JEMIL NO. The ownership of the seized branded LPG cylinders,
allegedly owned by Omni customers as petitioners
adamantly profess, is of no consequence.
FACTS:

Petitioners are stockholders of Omni Gas Corporation


("Omni"). (JGAC Law Offices sent a letter to the NBI The law does not require that the property to be seized
requesting, on behalf of their clients Shellane Dealers should be owned by the person against whom the search
Association, Inc., Petron Gasul Dealers Association, Inc., warrants is directed. Ownership, therefore, is of no
and Totalgaz Dealers Association, Inc., for the consequence, and it is sufficient that the person against
surveillance, investigation, and apprehension of persons whom the warrant is directed has control or possession of
or establishments in Pasig City that are engaged in the property sought to be seized.
alleged illegal trading of petroleum products and
underfilling of branded LPG cylinders in violation of Batas
Pambansa Blg. (BP) 33, as amended by Presidential
Petitioners cannot deny that the seized LPG cylinders
Decree No. (PD) 1865.
were in the possession of Omni, found as they were inside
the Omni compound.
VALID INSTANCES OF WARRANTLESS SEARCHES AND
NBI Agents Marvin De Jemil and Edgardo Kawada
SEIZURES
conducted surveillance operations on Omni. On 15 April
2004, the NBI Agents carried out a test-buy. Using eight A. SEARCH OF MOVING VEHICLES
branded LPG cylinders from Shell, Petron and Total, they
went to Omni for refilling. Omni refilled the cylinders. The
NBI agents paid more than P1500. Inspector Noel Navio ANIAG v. COMELEC
found that the LPG cylinders were without LPG valve
seals and one of the cylinders was actually underfilled.
FACTS: In preparation for the synchronized national and
local elections scheduled on 11 May 1992, the
Commission on Elections (COMELEC) issued on 11
Agent De Jemil obtained a search warrant from Pasig
December 1991 Resolution No. 2323 otherwise referred
RTC branch 167. The NBI seized several items from
to as the "Gun Ban,".
Omni's premises. Subsequently, Agent De Jemil filed his
Complaint-Affidavit before the DOJ. Assistant City
Prosecutor of Pasig found probable cause for violation of
BP 33. This was later approved by Chief State Prosecutor On 10 January 1992, pursuant to the "Gun Ban," Mr.
Jovencito Zuno. Serapio P. Taccad, Sergeant-at-Arms, House of
Representatives, wrote petitioner who was then
Congressman of the 1st District of Bulacan requesting the
return of the two (2) firearms issued to him by the House
The CA found strong probable violation of refilling of
of Representatives. Upon being advised of the request on
another companys or firms cylinders without such
13 January 1992 by his staff, petitioner immediately
companys or firms written authorization under Sec. 3 (c)
instructed his driver, Ernesto Arellano, to pick up the
of BP 33, as amended. The CA relied on the affidavits of
firearms from petitioner's house at Valle Verde and return
Agents De Jemil and Kawada, the certifications from
them to Congress.
various LPG producers that Omni is not authorized to refill
their branded LPG cylinders.

Meanwhile, at about five o'clock in the afternoon of the


same day, the Philippine National Police (PNP) headed
Petitioners contend that they were not the owners of the
by Senior Superintendent Danilo Cordero set up a
seized LPG cylinders
checkpoint outside the Batasan Complex some twenty
(20) meters away from its entrance. About thirty minutes
later, the policemen manning the outpost flagged down
ISSUE: the car driven by Arellano as it approached the
checkpoint. They searched the car and found the firearms
Whether or not the issue on ownership of confiscated are
neatly packed in their gun cases and placed in a bag in
material to the determination of probable cause and to the
the trunk of the car. Arellano was then apprehended and
validity of the search warrant?
detained. He explained that he was ordered by petitioner
to get the firearms from the house and return them to
Sergeant-at-Arms Taccad of the House of
RULING: Representatives.
evidence for the prosecution was clearly violative of Secs.
2 and 3, par. (2), Art. III, of the Constitution. Petitioner
The police referred Arellano's case to the Office of the City further maintains that he was neither impleaded as party
Prosecutor for inquest. The City Prosecutor ordered the respondent in the preliminary investigation before the
release of Arellano after finding the latter's sworn Office of the City Prosecutor nor included in the charge
explanation meritorious. sheet. Consequently, making him a respondent in the
criminal information would violate his constitutional right
to due process.
Upon the invitation of the City Prosecutor, the petitioner
to shed light on the circumstances mentioned in
Arellano's sworn explanation. Petitioner not only COMELEC claims that petitioner is charged with violation
appeared at the preliminary investigation to confirm of Sec. 261, par. (q), in relation to Sec. 263, of B.P. Blg.
Arellano's statement but also wrote the City Prosecutor 881 which provides that "the principals, accomplices and
urging him to exonerate Arellano. He explained that accessories, as defined in the Revised Penal Code, shall
Arellano did not violate the firearms ban as he in fact was be criminally liable for election offenses." It points out that
complying with it when apprehended by returning the it was upon petitioner's instruction that Arellano brought
firearms to Congress; and, that he was petitioner's driver, the firearms in question outside petitioner's residence,
not a security officer nor a bodyguard. submitting that his right to be heard was not violated as
he was invited by the City Prosecutor to explain the
circumstances regarding Arellano's possession of the
City Prosecutor issued a resolution ordering the dismissal firearms. Petitioner also filed a sworn written explanation
of the case against Arellano. However, upon about the incident. Finally, COMELEC claims that
recommendation of the Law Department, COMELEC violation of the "Gun Ban" is mala prohibita, hence, the
issued resolution directing the filing of the information intention of the offender is immaterial.
against the petitioner and Arellano for violation of Sec.
261, par. (q), of B.P. Blg. 881 otherwise known as the
Omnibus Election Code, in relation to Sec. 32 of R.A. No. As a rule, a valid search must be authorized by a search
7166;7 and petitioner to show cause why he should not warrant duly issued by an appropriate authority. However,
be disqualified from running for an elective position, this is not absolute. Aside from a search incident to a
pursuant to COMELEC Resolution No. 2327, in relation to lawful arrest, a warrantless search had been upheld in
Sec. 32, 33 and 35 of R.A. 7166, and Sec. 52, par. (c), of cases of moving vehicles and the seizure of evidence in
B.P. Blg. 881. Petitioner moved for reconsideration. plain view,17 as well as the search conducted at police or
However, this was denied by the COMELEC. Hence, this military checkpoints which we declared are not illegal per
petition, se, and stressed that the warrantless search is not
violative of the Constitution for as long as the vehicle is
neither searched nor its occupants subjected to a body
ISSUE: W/N he can be validly prosecuted for instructing search, and the inspection of the vehicle is merely limited
his driver to return to the Sergeant-at-Arms of the House to a visual search.
of Representatives the two firearms issued to him on the
basis of the evidence gathered from the warrantless
search of his car. Petitioner contends that the guns were not tucked in
Arellano's waist nor placed within his reach, and that they
were neatly packed in gun cases and placed inside a bag
RULING: No. The warrantless search conducted by the at the back of the car. Significantly, COMELEC did not
Philippine National Police on 13 January 1992 is declared rebut this claim. The records do not show that the manner
illegal and the firearms seized during the warrantless by which the package was bundled led the PNP to
search cannot be used as evidence in any proceeding suspect that it contained firearms. There was no mention
against petitioner. either of any report regarding any nervous, suspicious or
unnatural reaction from Arellano when the car was
stopped and searched. Given these circumstances and
Petitioner strongly protests against the manner by which relying on its visual observation, the PNP could not
the PNP conducted the search. According to him, without thoroughly search the car lawfully as well as the package
a warrant and without informing the driver of his without violating the constitutional injunction.
fundamental rights the policemen searched his car. The
firearms were not tucked in the waist nor within the
immediate reach of Arellano but were neatly packed in An extensive search without warrant could only be
their gun cases and wrapped in a bag kept in the trunk of resorted to if the officers conducting the search had
the car. Thus, the search of his car that yielded the reasonable or probable cause to believe before the
search that either the motorist was a law offender or that any, could not be more than a mere passive conformity
they would find the instrumentality or evidence pertaining on Arellano's part to the search, and "consent" given
to the commission of a crime in the vehicle to be under intimidating or coercive circumstances is no
searched. The existence of probable cause justifying the consent within the purview of the constitutional guaranty.
warrantless search is determined by the facts of each
case. Thus, we upheld the validity of the warrantless
search in situations where the smell of marijuana EPIE v. ULAT-MARREDO
emanated from a plastic bag owned by the accused, or
where the accused was acting suspiciously, and
attempted to flee. FACTS: Accused Ulat was charged with violation of
Section 68 of Presidential Decree No. 705; that without
any authority of law or without any license or permit
granted by the Department of Environment and Natural
In the case at bench, we find that the checkpoint was set
Resources (DENR), and with intent of gain and without
up twenty (20) meters from the entrance to the Batasan
the knowledge and consent of the owner thereof, he
Complex to enforce Resolution No. 2327. There was no
possess and transport 870 bd. ft. of Benguet Pine lumber
evidence to show that the policemen were impelled to do
having a total market value of ₱24,360.00 belonging to
so because of a confidential report leading them to
the REPUBLIC OF THE PHILIPPINES, to the damage
reasonably believe that certain motorists matching the
and prejudice of the GOVERNMENT in the actual sum
description furnished by their informant were engaged in
aforesaid.
gunrunning, transporting firearms or in organizing special
strike forces. Nor, as adverted to earlier, was there any
indication from the package or behavior of Arellano that
could have triggered the suspicion of the policemen. The evidence for the prosecution shows that at around
Absent such justifying circumstances specifically pointing 2:30 p.m. of September 6, 1998, SPO2 Alberto Ngina of
to the culpability of petitioner and Arellano, the search the Philippine National Police (PNP) Tublay Station
could not be valid. The action then of the policemen received an information from a confidential agent that a
unreasonably intruded into petitioner's privacy and the jeepney with Plate No. AYB 117 at Km. 96, Atok, Benguet
security of his property, in violation of Sec. 2, Art. III, of was loaded with Benguet pine lumber. SPO2 Ngina
the Constitution. Consequently, the firearms obtained in immediately relayed the information to SPO4 Rentao
violation of petitioner's right against warrantless search Quitoriano and SPO1 Domingo Pulig. They then swiftly
cannot be admitted for any purpose in any proceeding. established a checkpoint in Acop, Tublay, Benguet. At
around 4:00 p.m. of the same day, the PNP operatives
spotted the jeepney heading toward La Trinidad. They
flagged it down but it did not stop. Hence, they chased the
It may be argued that the seeming acquiescence of
vehicle up to Shilan, La Trinidad where it finally halted.
Arellano to the search constitutes an implied waiver of
The police saw five persons inside the jeepney then
petitioner's right to question the reasonableness of the
loaded with assorted vegetables, like womboc3 and chili.
search of the vehicle and the seizure of the firearms. The
When SPO4 Quitoriano lifted a womboc, he found some
facts show that PNP installed the checkpoint at about five
pieces of lumber under it. The driver and his companions
o'clock in the afternoon of 13 January 1992. The search
admitted they have no permit to transport the lumber. The
was made soon thereafter, or thirty minutes later. It was
police immediately arrested and investigated petitioners,
not shown that news of impending checkpoints without
Marso Insiong Dumpit, Armando Palasi, and Ben Arinos.
necessarily giving their locations, and the reason for the
Only petitioners were charged with violation of Section 68
same have been announced in the media to forewarn the
of the Revised Forestry Code.
citizens. Nor did the informal checkpoint that afternoon
carry signs informing the public of the purpose of its After the prosecution presented its evidence, petitioners,
operation. As a result, motorists passing that place did not through counsel, filed a "Motion to Suppress Evidence of
have any inkling whatsoever about the reason behind the the Prosecution" on the ground that the pieces of Benguet
instant exercise. With the authorities in control to stop and pine lumber were illegally seized.
search passing vehicles, the motorists did not have any
choice but to submit to the PNP's scrutiny. Otherwise, any This was denied by the RTC. CA affirmed the decision.
attempt to turnabout albeit innocent would raise suspicion Hence, this petition.
and provide probable cause for the police to arrest the
motorist and to conduct an extensive search of his
vehicle. In the face of fourteen (14) armed policemen ISSUE: W/N police officers have a probable cause to
conducting the operation,29 driver Arellano being alone believe that the subject vehicle was loaded with illegal
and a mere employee of petitioner could not have cargo and that, therefore, it can be stopped and searched
marshalled the strength and the courage to protest without a warrant.
against the extensive search conducted in the vehicle. In
such scenario, the "implied acquiescence," if there was
RULING: Yes. There is no hard and fast rule or fixed formula in
determining probable cause for its determination varies
according to the facts of each case.
In this jurisdiction, the fundamental law of the land
recognizes and protects the right of a person to privacy
against unreasonable intrusions by the agents of the Here, the search involved a moving vehicle, an instance
State. This right to undisturbed privacy is guaranteed by where a warrantless search and seizure may be
Section 2, Article III of the Constitution which provides: conducted by peace officers. The only issue we should
The right of the people to be secure in their persons, determine is whether there was probable cause to justify
houses, papers and effects against unreasonable such warrantless search and seizure.
searches and seizures of whatever nature and for any
purpose shall be inviolable, and no search warrant or
warrant of arrest shall issue except upon probable cause We recall that at around 2:30 p.m. of September 6, 1998,
to be determined personally by the judge after a confidential informer disclosed to SPO2 Ngina that a
examination under oath or affirmation of the complainant passenger jeepney with Plate No. AYB 117 loaded with
and the witnesses he may produce, and particularly Benguet pine lumber was at Km. 96, Atok, Benguet. The
describing the place to be searched and the persons or lumber was covered with assorted vegetables. A PNP
things to be seized. roadblock was then placed in Acop, Tublay, Benguet to
intercept the jeepney. At around 4:00 p.m. of that same
day, the police spotted the vehicle. They flagged it down
Section 3(2), also of Article III, provides that any evidence but it did not stop, forcing the police to chase it until it
obtained in violation of the above provision shall be reached Shilan, La Trinidad. A search of the vehicle
inadmissible for any purpose in any proceeding. disclosed several pieces of Benguet pine lumber.
Petitioners could not produce the required DENR permit
to cut and transport the same.
Hence, as a general rule, a search and seizure must be
carried through with judicial warrant, otherwise, such
search and seizure constitutes derogation of a In People v. Vinecarao,10 we ruled that where a vehicle
constitutional right. sped away after noticing a checkpoint and even after
having been flagged down by police officers, in an
apparent attempt to dissuade the police from proceeding
The above rule, however, is not devoid of exceptions. In with their inspection, there exists probable cause to justify
People v. Sarap,7 we listed the exceptions where search a reasonable belief on the part of the law enforcers that
and seizure may be conducted without warrant, thus: the persons on board said vehicle were officers of the law
or that the vehicle contained objects which were
(1) search incident to a lawful arrest; instruments of some offense. This ruling squarely applies
to the present case. Verily, the Court of Appeals did not
(2) search of a moving motor vehicle;
err in holding that respondent judge did not commit grave
(3) search in violation of customs laws; abuse of discretion amounting to lack or excess of
jurisdiction when she ruled that the warrantless search is
(4) seizure of the evidence in plain view; valid and that the lumber seized is admissible in evidence
against petitioners.
(5) search when the accused himself waives his right
against unreasonable searches and seizures;

(6) stop and frisk; and

(7) exigent and emergency circumstances.

The only requirement in these exceptions is the presence


of probable cause. Probable cause is the existence of
such facts and circumstances which would lead a
reasonable, discreet, and prudent man to believe that an
offense has been committed and that the objects sought
in connection with the offense are in the place to be
searched.8 In People v. Aruta,9 we ruled that in
warrantless searches, probable cause must only be
based on reasonable ground of suspicion or belief that a
crime has been committed or is about to be committed.
PEOPLE v. TUASON his address and the name of his employer. After two days,
he was allegedly forced to admit that he was in fact the
owner of the Gemini car as well as of the shabu and the
FACTS: Accused Tuazon not being lawfully authorized to
gun recovered from said vehicle. He learned later on that
possess any regulated drug have in his possession,
he was charged with violations of Republic Act No. 6425
custody and control seven (7) heat-sealed transparent
for illegal possession of shabu and Presidential Decree
plastic bags each containing 97.92 grams, 95.46 grams,
No. 1866 for illegal possession of firearm. The latter case
40.47 grams, 5.36 grams, 5.41 grams, 2.95 grams and
was eventually dismissed. At the end of his direct
3.17 grams for a total weight of 250.74 grams of white
examination, appellant reiterated that he should not have
crystalline substance, which after the corresponding
been the one charged with illegal possession of shabu,
laboratory examination conducted gave positive result to
but Reyes who was driving the Gemini car.
the test for methylamphetamine hydrochloride also known
as "shabu" a regulated drug, in violation of the Section 16,
Article III, R.A. 6425.
RTC found the accused guilty and imposed upon
appellant the penalty of reclusion perpetua and to pay a
fine of P500,000. Upon appeal, contends that at the time
The prosecution's version of the case relied heavily on the
of his warrantless arrest, he was merely driving within
testimony of PO3 Glenon Bueno (PO3 Bueno) who
Marville Subdivision. He had not committed, was not
testified that in the morning of 7 March 1999, the Antipolo
committing, and was not about to commit any crime which
City Police Station received through telephone, a
could have justified his apprehension. He goes on to
confidential information that a Gemini car bearing plate
argue that even if he had waived the issue regarding the
number PFC 4116 would deliver an unspecified amount
validity of his arrest by his failure to raise the matter before
of shabu in Marville Subdivision, Antipolo City. Acting on
entering his plea, such waiver did not affect the
said tip, Antipolo City Chief of Police Major Rene
unlawfulness of the search and seizure conducted by the
Quintana dispatched a team of policemen to the area to
police. Appellant claims that as the confidential informant
conduct a surveillance. When the team arrived in Marville
had been cooperating with the police for three weeks prior
Subdivision, they saw the said Gemini car and
to his arrest, the authorities were already informed of his
immediately flagged it down. The driver of the car pulled
identity and his alleged illegal activities. They should have
to a stop and opened a window of said vehicle giving the
conducted a prior surveillance and then sought a search
policemen the opportunity to identify themselves as
warrant from the court. Absent said warrant, the shabu
members of the Antipolo City Police Station. It was then
seized from him should be excluded from evidence. CA
that PO1 Manuel Padlan (PO1 Padlan) saw a gun tucked
affirmed RTC’s decision. In sustaining the trial court, the
on appellant's waist. PO1 Padlan inquired about the gun
Court of Appeals found PO3 Bueno's testimony to be
and appellant allegedly replied it did not belong to him nor
"clear and unequivocal"14 and should therefore prevail
could he produce any pertinent document relating to said
over appellant's defense of denial.15 The Court of
firearm. This prompted PO3 Bueno to order appellant to
Appeals likewise brushed aside appellant's contention
get down from the car. As soon as appellant stepped
that he was a victim of frame-up as this defense has been
down from the vehicle, PO3 Bueno saw five plastic
viewed with disfavor and has become a standard line of
sachets on the driver's seat, the contents of which
defense in most prosecutions arising from violations of the
appellant allegedly admitted to be shabu. Appellant was
Dangerous Drugs Act.16 It also took note of appellant's
thereafter immediately brought to the police station.
failure to give any credible reason why the police singled
him out considering that they were strangers to one
another prior to the date of the incident.17
Appellant’s account of events: On 6 March 1999, he
reported for work at six o'clock in the evening. Later that
night, unidentified men walked up to him. One of these
Hence, this petition.
men asked him regarding the ownership of the car parked
outside the bar. He allegedly accompanied the men
outside so he could confirm the identity of the owner of
the car that the men were inquiring about. Thereupon, the ISSUE: W/N the warrantless search is valid.
men pointed to him a green colored Isuzu Gemini car
which according to him was driven by his employer,
Reyes. After revealing this information to the unidentified RULING: Yes.
men, the latter purportedly pointed guns at him and
ordered him to board an owner-type jeepney. The men
allegedly asked him regarding the whereabouts of Reyes No less than our Constitution recognizes the right of the
and threatened to include him in whatever trouble Reyes people to be secure in their persons, houses, papers and
was in. A few hours passed and he was then brought to effects against unreasonable searches and seizures. This
the police headquarters where he was asked regarding
right is encapsulated in Article III, Section 2 of the
Constitution which states:
Nevertheless, the exception from securing a search
warrant when it comes to moving vehicles does not give
the police authorities unbridled discretion to conduct a
SEC. 2. The right of the people to be secure in their warrantless search of an automobile. To do so would
persons, houses, papers, and effects against render the aforementioned constitutional stipulations
unreasonable searches and seizures of whatever nature inutile and expose the citizenry to indiscriminate police
and for any purpose shall be inviolable, and no search distrust which could amount to outright harassment.
warrant or warrant of arrest shall issue except upon Surely, the policy consideration behind the exemption of
probable cause to be determined personally by the judge search of moving vehicles does not encompass such
after examination under oath or affirmation of the arbitrariness on the part of the police authorities. In
complainant and the witnesses he may produce, and recognition of the possible abuse, jurisprudence dictates
particularly describing the place to be searched and the that at all times, it is required that probable cause exist in
persons or things to be seized. order to justify the warrantless search of a vehicle.33

Complementing this provision is the so-called When a vehicle is flagged down and subjected to an
exclusionary rule embodied in Section 3(2) of the same extensive search, such a warrantless search has been
article ' held to be valid as long as the officers conducting the
search have reasonable or probable cause to believe
prior to the search that they would find the instrumentality
(2) Any evidence obtained in violation of this or the or evidence pertaining to a crime, in the vehicle to be
preceding section shall be inadmissible for any purpose searched.35
in any proceeding.

In this case, we hold that the police had probable cause


It is recognized, however, that these constitutional to effect the warrantless search of the Gemini car driven
provisions against warrantless searches and seizures by appellant. A confidential informer tipped them off that
admit of certain exceptions, as follows: (1) warrantless said car was going to deliver shabu at Marville
search incidental to a lawful arrest recognized under Subdivision. Pursuing said lead, the Antipolo City police
Section 12, Rule 126 of the Rules of Court and by sent a team to Marville Subdivision to monitor said
prevailing jurisprudence; (2) seizure of evidence in plain vehicle. The information provided by the informer turned
view; (3) search of a moving vehicle; (4) consented out to be correct as, indeed, the Gemini car was spotted
warrantless search; (5) customs search; (6) stop and frisk; in the place where it was said to be bringing shabu. When
and (7) exigent and emergency circumstances.31 they stopped the car, they saw a gun tucked in appellant's
waist. Appellant did not have any document to support his
possession of said firearm which all the more
In the case of People v. Lo Ho Wing,32 this Court had the strengthened the police's suspicion. After he was told to
occasion to elucidate on the rationale for the exemption step out of the car, they found on the driver's seat plastic
of searches of moving vehicles from the requirement of sachets containing white powdery substance. These
search warrant, thus: circumstances, taken together, are sufficient to establish
probable cause for the warrantless search of the Gemini
car and the eventual admission into evidence of the
plastic packets against appellant.
[T]he rules governing search and seizure have over the
years been steadily liberalized whenever a moving
vehicle is the object of the search on the basis of
practicality. This is so considering that before a warrant In any case, appellant failed to timely object to the
could be obtained, the place, things and persons to be admissibility of the evidence against him on the ground
searched must be described to the satisfaction of the that the same was obtained through a warrantless search.
issuing judge - a requirement which borders on the His failure amounts to a waiver of the objection on the
impossible in the case of smuggling effected by the use legality of the search and the admissibility of the evidence
of a moving vehicle that can transport contraband from obtained by the police. It was only proper for the trial court
one place to another with impunity. We might add that a to admit said evidence.36
warrantless search of a moving vehicle is justified on the
ground that "it is not practicable to secure a warrant
because the vehicle can be quickly moved out of the Appellant Bernardo Tuazon y Nicolas guilty beyond
locality or jurisdiction in which the warrant must be reasonable doubt of violation of Section 16, Article III of
sought." Republic Act No. 6425, as amended.
PEOPLE v. MARIACOS showed that the stuff found in the bags all tested positive
for marijuana, a dangerous drug.
FACTS: Accused-appellant Belen Mariacos was charged Accused’s account of events: While the jeepney was still
in an Information, dated November 7, 2005 of violating at the terminal waiting for passengers, one Bennie Lao-
Section 5, Article II of Republic Act [No.] 9165; that he ang ("Lao-ang"), her neighbor, requested her to carry a
transport, deliver 7,030.3, (sic) grams of dried marijuana few bags which had been loaded on top of the jeepney.
fruiting tops without the necessary permit or authority from At first, accused-appellant refused, but she was
the proper government agency or office. persuaded later when she was told that she would only be
carrying the bags. When they reached the poblacion, Lao-
ang handed accused-appellant and her companion, Lani
San Gabriel Police Station of San Gabriel, La Union, Herbacio, the bags, and then Lao-ang suddenly ran away.
conducted a checkpoint near the police station at the A few moments later, PO2 Pallayoc was upon them,
poblacion to intercept a suspected transportation of arresting them. Without explanation, they were brought to
marijuana from Barangay Balbalayang, San Gabriel, La the police station. When they were at the police station,
Union. The group at the checkpoint was composed of Lani Herbacio disappeared. It was also at the police
PO2 Lunes B. Pallayoc ("PO2 Pallayoc"), the Chief of station that accused-appellant discovered the true
Police, and other policemen. When the checkpoint did not contents of the bags which she was asked to carry. She
yield any suspect or marijuana, the Chief of Police maintained that she was not the owner of the bags and
instructed PO2 Pallayoc to proceed to Barangay that she did not know what were contained in the bags.
Balbalayang to conduct surveillance operation (sic).

RTC found the accused Mariacos guilty as charged and


At dawn on October 27, 2005, in Barangay Balbalayang, sentenced to suffer the penalty of life imprisonment and
PO2 Pallayoc met with a secret agent of the Barangay to pay a fine of ₱500,000.00.
Intelligence Network who informed him that a baggage of
Appellant appealed her conviction to the CA. She argued
marijuana had been loaded on a passenger jeepney that
that the trial court erred in considering the evidence of the
was about to leave for the poblacion. The agent
prosecution despite its inadmissibility.5 She claimed that
mentioned three (3) bags and one (1) blue plastic bag.
her right against an unreasonable search was flagrantly
Further, the agent described a backpack bag with an
violated by Police Officer (PO)2 Pallayoc when the latter
"O.K." marking. PO2 Pallayoc then boarded the said
searched the bag, assuming it was hers, without a search
jeepney and positioned himself on top thereof. While the
warrant and with no permission from her. She averred that
vehicle was in motion, he found the black backpack with
PO2 Pallayoc’s purpose for apprehending her was to
an "O.K." marking and peeked inside its contents. PO2
verify if the bag she was carrying was the same one he
Pallayoc found bricks of marijuana wrapped in
had illegally searched earlier. Moreover, appellant
newspapers. He then asked the other passengers on top
contended that there was no probable cause for her
of the jeepney about the owner of the bag, but no one
arrest.
knew.
On the other hand, the People, through the Office of the
When the jeepney reached the poblacion, PO2 Pallayoc
Solicitor General (OSG), argued that the warrantless
alighted together with the other passengers.
arrest of appellant and the warrantless seizure of
Unfortunately, he did not notice who took the black
marijuana were valid and legal,8 justified as a search of a
backpack from atop the jeepney. He only realized a few
moving vehicle. It averred that PO2 Pallayoc had
moments later that the said bag and three (3) other bags,
reasonable ground to believe that appellant had
including a blue plastic bag, were already being carried
committed the crime of delivering dangerous drugs based
away by two (2) women. He caught up with the women
on reliable information from their agent, which was
and introduced himself as a policeman. He told them that
confirmed when he peeked into the bags and smelled the
they were under arrest, but one of the women got away.
distinctive odor of marijuana.9 The OSG also argued that
PO2 Pallayoc brought the woman, who was later appellant was now estopped from questioning the
identified as herein accused-appellant Belen Mariacos, illegality of her arrest since she voluntarily entered a plea
and the bags to the police station. The bags were opened of "not guilty" upon arraignment and participated in the
and three (3) bricks of marijuana wrapped in newspaper, trial and presented her evidence.
two (2) round bundles of marijuana, and two (2) bricks of
CA dismissed appellant’s appeal and affirmed the RTC
marijuana fruiting tops, all wrapped in a newspaper, were
decision in toto. The CA ruled that appellant was caught
recovered.
in flagrante delicto of "carrying and conveying" the bag
Thereafter, the investigators marked, inventoried and that contained the illegal drugs, and thus held that
forwarded the confiscated marijuana to the crime appellant’s warrantless arrest was valid. The appellate
laboratory for examination. The laboratory examination court ratiocinated:
It must be stressed that PO2 Pallayoc had earlier (b) the evidence was inadvertently discovered by the
ascertained the contents of the bags when he was aboard police who had the right to be where they are;
the jeep. He saw the bricks of marijuana wrapped in
newspaper. That said marijuana was on board the (c) the evidence must be immediately apparent[;] and;
jeepney to be delivered to a specified destination was (d) "plain view" justified mere seizure of evidence without
already unlawful. PO2 Pallayoc needed only to see for further search.
himself to whom those bags belonged. So, when he saw
accused-appellant carrying the bags, PO2 Pallayoc was 3. Search of a moving vehicle. Highly regulated by the
within his lawful duty to make a warrantless arrest of government, the vehicle's inherent mobility reduces
accused-appellant. expectation of privacy especially when its transit in public
thoroughfares furnishes a highly reasonable suspicion
Firstly, this Court opines that the invocation of Section 2, amounting to probable cause that the occupant
Article III of the Constitution is misplaced. At the time, committed a criminal activity;
when PO2 Pallayoc looked into the contents of the
suspicious bags, there was no identified owner. He asked 4. Consented warrantless search;
the other passengers atop the jeepney but no one knew
5. Customs search;
who owned the bags. Thus, there could be no violation of
the right when no one was entitled thereto at that time. 6. Stop and Frisk; and
Secondly, the facts of the case show the urgency of the 7. Exigent and Emergency Circumstances.14
situation. The local police has been trying to intercept the
transport of the illegal drugs for more than a day, to no
avail. Thus, when PO2 Pallayoc was tipped by the secret
Both the trial court and the CA anchored their respective
agent of the Barangay Intelligence Network, PO2
decisions on the fact that the search was conducted on a
Pallayoc had no other recourse than to verify as promptly
moving vehicle to justify the validity of the search.
as possible the tip and check the contents of the bags.
With regard to the search of moving vehicles, this had
Thirdly, x x x the search was conducted in a moving
been justified on the ground that the mobility of motor
vehicle. Time and again, a search of a moving vehicle has
vehicles makes it possible for the vehicle to be searched
been justified on the ground that the mobility of motor
to move out of the locality or jurisdiction in which the
vehicles makes it possible for the vehicle to move out of
warrant must be sought.
the locality or jurisdiction in which the warrant must be
sought. Thus, under the facts, PO2 Pallayoc could not be This in no way, however, gives the police officers
expected to secure a search warrant in order to check the unlimited discretion to conduct warrantless searches of
contents of the bags which were loaded on top of the automobiles in the absence of probable cause. When a
moving jeepney. Otherwise, a search warrant would have vehicle is stopped and subjected to an extensive search,
been of no use because the motor vehicle had already left such a warrantless search has been held to be valid only
the locality.13 as long as the officers conducting the search have
reasonable or probable cause to believe before the
Hence, this petition.
search that they will find the instrumentality or evidence
pertaining to a crime, in the vehicle to be searched.

ISSUE: W/N the warrantless search is valid. The essential requisite of probable cause must be
satisfied before a warrantless search and seizure can be
lawfully conducted.17 Without probable cause, the
articles seized cannot be admitted in evidence against the
RULING: Yes.
person arrested.18
Law and jurisprudence have laid down the instances
Probable cause is defined as a reasonable ground of
when a warrantless search is valid. These are:
suspicion supported by circumstances sufficiently strong
1. Warrantless search incidental to a lawful arrest in themselves to induce a cautious man to believe that the
recognized under Section 12 [now Section 13], Rule 126 person accused is guilty of the offense charged. It refers
of the Rules of Court and by prevailing jurisprudence; to the existence of such facts and circumstances that can
lead a reasonably discreet and prudent man to believe
2. Seizure of evidence in "plain view," the elements of that an offense has been committed, and that the items,
which are: articles or objects sought in connection with said offense
(a) a prior valid intrusion based on the valid warrantless or subject to seizure and destruction by law are in the
arrest in which the police are legally present in the pursuit place to be searched.19
of their official duties; The grounds of suspicion are reasonable when, in the
absence of actual belief of the arresting officers, the
suspicion that the person to be arrested is probably guilty the police has probable cause to make the arrest at the
of committing the offense is based on actual facts, i.e., outset of the search.25
supported by circumstances sufficiently strong in
themselves to create the probable cause of guilt of the Given that the search was valid, appellant’s arrest based
person to be arrested. A reasonable suspicion therefore on that search is also valid.
must be founded on probable cause, coupled with good In her defense, appellant averred that the packages she
faith on the part of the peace officers making the arrest.20 was carrying did not belong to her but to a neighbor who
It is readily apparent that the search in this case is valid. had asked her to carry the same for him. This contention,
The vehicle that carried the contraband or prohibited however, is of no consequence.
drugs was about to leave. PO2 Pallayoc had to make a When an accused is charged with illegal possession or
quick decision and act fast. It would be unreasonable to transportation of prohibited drugs, the ownership thereof
require him to procure a warrant before conducting the is immaterial. Consequently, proof of ownership of the
search under the circumstances. Time was of the confiscated marijuana is not necessary.26
essence in this case. The searching officer had no time to
obtain a warrant. Indeed, he only had enough time to Appellant’s alleged lack of knowledge does not constitute
board the vehicle before the same left for its destination. a valid defense. Lack of criminal intent and good faith are
not exempting circumstances where the crime charged is
This Court has also, time and again, upheld as valid a malum prohibitum, as in this case.27 Mere possession
warrantless search incident to a lawful arrest. Thus, and/or delivery of a prohibited drug, without legal
Section 13, Rule 126 of the Rules of Court provides: authority, is punishable under the Dangerous Drugs
Act.28

SEC. 13. Search incident to lawful arrest.—A person


lawfully arrested may be searched for dangerous Anti-narcotics laws, like anti-gambling laws, are
weapons or anything which may have been used or regulatory statutes. They are rules of convenience
constitute proof in the commission of an offense without a designed to secure a more orderly regulation of the affairs
search warrant.23 of society, and their violation gives rise to crimes mala
For this rule to apply, it is imperative that there be a prior prohibita. Laws defining crimes mala prohibita condemn
valid arrest. Although, generally, a warrant is necessary behavior directed not against particular individuals, but
for a valid arrest, the Rules of Court provides the against public order.
exceptions therefor, to wit: It is admitted that there were no photographs taken of the
SEC. 5. Arrest without warrant; when lawful.—A peace drugs seized, that appellant was not accompanied by
officer or a private person may, without a warrant, arrest counsel, and that no representative from the media and
a person: the DOJ were present. However, this Court has already
previously held that non-compliance with Section 21 is not
(a) When, in his presence, the person to be arrested has fatal and will not render an accused’s arrest illegal, or
committed, is actually committing, or is attempting to make the items seized inadmissible. What is of utmost
commit an offense; importance is the preservation of the integrity and
evidentiary value of the seized items.37
(b) When an offense has just been committed and he has
probable cause to believe based on personal knowledge Based on the testimony of PO2 Pallayoc, after appellant’s
of facts or circumstances that the person to be arrested arrest, she was immediately brought to the police station
has committed it; and where she stayed while waiting for the Mayor. It was the
Mayor who opened the packages, revealing the illegal
(c) When the person to be arrested is a prisoner who has drugs, which were thereafter marked and sent to the
escaped from a penal establishment or place where he is police crime laboratory the following day. Contrary to
serving final judgment or is temporarily confined while his appellant’s claim, the prosecution’s evidence establishes
case is pending, or has escaped while being transferred the chain of custody from the time of appellant’s arrest
from one confinement to another. until the prohibited drugs were tested at the police crime
In cases falling under paragraphs (a) and (b) above, the laboratory.
person arrested without a warrant shall be forthwith While it is true that the arresting officer failed to state
delivered to the nearest police station or jail and shall be explicitly the justifiable ground for non-compliance with
proceeded against in accordance with section 7 of Rule Section 21, this does not necessarily mean that
112.24 appellant’s arrest was illegal or that the items seized are
Be that as it may, we have held that a search substantially inadmissible. The justifiable ground will remain unknown
contemporaneous with an arrest can precede the arrest if because appellant did not question the custody and
disposition of the items taken from her during the trial.38
Even assuming that the police officers failed to abide by
Section 21, appellant should have raised this issue before Thus, an Information was filed before the RTC against
the trial court. She could have moved for the quashal of petitioner, charging him with violation of Section 16,
the information at the first instance. But she did not. Article III of R.A. 6425, as amended.
Hence, she is deemed to have waived any objection on
the matter.

Further, the actions of the police officers, in relation to the According to the petitioner, the small structure, 20 meters
procedural rules on the chain of custody, enjoyed the away from his house where they found the confiscated
presumption of regularity in the performance of official items, was owned by his older brother and was used as a
functions. Courts accord credence and full faith to the storage place by his father.
testimonies of police authorities, as they are presumed to
be performing their duties regularly, absent any
convincing proof to the contrary.39 RTC found him guilty of violating Section 16, Article III,
Republic Act No. 6425, as amended. Aggrieved,
In sum, the prosecution successfully established petitioner appealed his case with the CA, but the latter
appellant’s guilt. Thus, her conviction must be affirmed. affirmed the decision of the RTC in toto and the appeal is
DISMISSED.

B. SEARCH INCIDENT TO A VALID ARREST

Hence, this petition.

DEL CASTILLO v. PEOPLE

ISSUE: W/N accused herein is guilty of the crime


FACTS: Pursuant to a confidential information that charged.
petitioner was engaged in selling shabu, police officers
headed by SPO3 Bienvenido Masnayon, after conducting
surveillance and test-buy operation at the house of RULING: No.
petitioner, secured a search warrant from the RTC and
served the search warrant to petitioner. Upon arrival,
somebody shouted "raid," which prompted them to
As to the validity of the search warrant - Petitioner insists
immediately disembark from the jeep they were riding and
that there was no probable cause to issue the search
went directly to petitioner's house and cordoned it. The
warrant, considering that SPO1 Reynaldo Matillano, the
structure of the petitioner's residence is a two-storey
police officer who applied for it, had no personal
house and the petitioner was staying in the second floor.
knowledge of the alleged illegal sale of drugs during a
When they went upstairs, they met petitioner's wife and
test-buy operation conducted prior to the application of
informed her that they will implement the search warrant.
the same search warrant. The OSG, however, maintains
But before they can search the area, SPO3 Masnayon
that the petitioner, aside from failing to file the necessary
claimed that he saw petitioner run towards a small
motion to quash the search warrant pursuant to Section
structure, a nipa hut, in front of his house. Masnayon
14, Rule 127 of the Revised Rules on Criminal Procedure,
chased him but to no avail, because he and his men were
did not introduce clear and convincing evidence to show
not familiar with the entrances and exits of the place.
that Masnayon was conscious of the falsity of his
assertion or representation.

They all went back to the residence of the petitioner and


closely guarded the place where the subject ran for cover.
As to the admissibility of the four (4) packs of shabu
In the presence of the barangay tanod, Nelson
seized inside the shop of petitioner - Petitioner asserts
Gonzalado, and the elder sister of petitioner named Dolly
that the nipa hut located about 20 meters away from his
del Castillo, searched the house of petitioner including the
house is no longer within the "permissible area" that may
nipa hut where the petitioner allegedly ran for cover. His
be searched by the police officers due to the distance and
men who searched the residence of the petitioner found
that the search warrant did not include the same nipa hut
nothing, but one of the barangay tanods was able to
as one of the places to be searched. The OSG, on the
confiscate from the nipa hut several articles, including four
other hand, argues that the constitutional guaranty
(4) plastic packs containing white crystalline substance.
against unreasonable searches and seizure is applicable
Consequently, the articles that were confiscated were
only against government authorities and not to private
sent to the PNP Crime Laboratory for examination. The
individuals such as the barangay tanod who found the
contents of the four (4) heat- sealed transparent plastic
folded paper containing packs of shabu inside the nipa
packs were subjected to laboratory examination, the
hut.
result of which proved positive for the presence of
methamphetamine hydrochloride, or shabu.
As to CA’s decision in finding petitioner guilty of the
charge - petitioner claims that the CA erred in finding him
guilty beyond reasonable doubt of illegal possession of With regard to the second argument of petitioner, it must
prohibited drugs, because he could not be presumed to be remembered that the warrant issued must particularly
be in possession of the same just because they were describe the place to be searched and persons or things
found inside the nipa hut. Nevertheless, the OSG to be seized in order for it to be valid. A designation or
dismissed the argument of the petitioner, stating that, description that points out the place to be searched to the
when prohibited and regulated drugs are found in a house exclusion of all others, and on inquiry unerringly leads the
or other building belonging to and occupied by a particular peace officers to it, satisfies the constitutional requirement
person, the presumption arises that such person is in of definiteness.19 In the present case, Search Warrant
possession of such drugs in violation of law, and the fact No. 570-9-1197-2420 specifically designates or describes
of finding the same is sufficient to convict. the residence of the petitioner as the place to be
searched. Incidentally, the items were seized by a
barangay tanod in a nipa hut, 20 meters away from the
residence of the petitioner. The confiscated items, having
This Court finds no merit on the first argument of been found in a place other than the one described in the
petitioner. search warrant, can be considered as fruits of an invalid
warrantless search, the presentation of which as an
evidence is a violation of petitioner's constitutional
The requisites for the issuance of a search warrant are: guaranty against unreasonable searches and seizure.
The OSG argues that, assuming that the items seized
(1) probable cause is present;
were found in another place not designated in the search
(2) such probable cause must be determined personally warrant, the same items should still be admissible as
by the judge; evidence because the one who discovered them was a
barangay tanod who is a private individual, the
(3) the judge must examine, in writing and under oath or constitutional guaranty against unreasonable searches
affirmation, the complainant and the witnesses he or she and seizure being applicable only against government
may produce; authorities. The contention is devoid of merit.
(4) the applicant and the witnesses testify on the facts
personally known to them; and
It was testified to during trial by the police officers who
(5) the warrant specifically describes the place to be effected the search warrant that they asked the
searched and the things to be seized. assistance of the barangay tanods. Having been
established that the assistance of the barangay tanods
was sought by the police authorities who effected the
A finding of probable cause needs only to rest on searched warrant, the same barangay tanods therefore
evidence showing that, more likely than not, a crime has acted as agents of persons in authority. Thus, the search
been committed and that it was committed by the conducted was unreasonable and the confiscated items
accused. Probable cause demands more than bare are inadmissible in evidence. Assuming ex gratia
suspicion; it requires less than evidence which would argumenti that the barangay tanod who found the
justify conviction.14 The judge, in determining probable confiscated items is considered a private individual, thus,
cause, is to consider the totality of the circumstances making the same items admissible in evidence,
made known to him and not by a fixed and rigid formula,15 petitioner's third argument that the prosecution failed to
and must employ a flexible, totality of the circumstances establish constructive possession of the regulated drugs
standard.16 The existence depends to a large degree seized, would still be meritorious.
upon the finding or opinion of the judge conducting the
examination. This Court, therefore, is in no position to
disturb the factual findings of the judge which led to the It must be put into emphasis that this present case is
issuance of the search warrant. A magistrate's about the violation of Section 16 of R.A. 6425. In every
determination of probable cause for the issuance of a prosecution for the illegal possession of shabu, the
search warrant is paid great deference by a reviewing following essential elements must be established:
court, as long as there was substantial basis for that
determination.17 Substantial basis means that the (a) the accused is found in possession of a regulated
questions of the examining judge brought out such facts drug;
and circumstances as would lead a reasonably discreet
(b) the person is not authorized by law or by duly
and prudent man to believe that an offense has been
constituted authorities; and
committed, and the objects in connection with the offense
sought to be seized are in the place sought to be (c) the accused has knowledge that the said drug is a
searched. regulated drug.
While it is not necessary that the property to be searched to take out the contents of the pocket of his jacket as the
or seized should be owned by the person against whom latter may have a weapon inside it; that the accused
the search warrant is issued, there must be sufficient obliged and slowly put out the contents of the pocket of
showing that the property is under appellant’s control or his jacket which was a nickel-like tin or metal container
possession.29 The CA, in its Decision, referred to the about two (2) to three (3) inches in size, including two (2)
possession of regulated drugs by the petitioner as a cellphones, one (1) pair of scissors and one (1) Swiss
constructive one. Constructive possession exists when knife; that upon seeing the said container, he asked the
the drug is under the dominion and control of the accused accused to open it; that after the accused opened the
or when he has the right to exercise dominion and control container, he noticed a cartoon cover and something
over the place where it is found.30 The records are void beneath it; and that upon his instruction, the accused
of any evidence to show that petitioner owns the nipa hut spilled out the contents of the container on the table which
in question nor was it established that he used the said turned out to be four (4) plastic sachets, the two (2) of
structure as a shop. The RTC, as well as the CA, merely which were empty while the other two (2) contained
presumed that petitioner used the said structure due to suspected shabu.
the presence of electrical materials, the petitioner being
an electrician by profession.
During trial, petitioner testified for himself and raised the
defense of planting of evidence and extortion. RTC
The prosecution must prove that the petitioner had convicted petitioner of illegal possession of dangerous
knowledge of the existence and presence of the drugs in drugs5 committed on 10 March 2003. It found the
the place under his control and dominion and the prosecution evidence sufficient to show that he had been
character of the drugs.35 With the prosecution's failure to lawfully arrested for a traffic violation and then subjected
prove that the nipa hut was under petitioner's control and to a valid search, which led to the discovery on his person
dominion, there casts a reasonable doubt as to his guilt. of two plastic sachets later found to contain shabu. The
In considering a criminal case, it is critical to start with the RTC also found his defense of frame-up and extortion to
law's own starting perspective on the status of the be weak, self-serving and unsubstantiated. RODEL LUZ
accused - in all criminal prosecutions, he is presumed y ONG GUILTY beyond reasonable doubt for the crime of
innocent of the charge laid unless the contrary is proven violation of Section 11, Article II of Republic Act No. 9165.
beyond reasonable doubt.36 Proof beyond reasonable
doubt, or that quantum of proof sufficient to produce a
moral certainty that would convince and satisfy the Upon review, the CA affirmed the RTC’s Decision. Hence,
conscience of those who act in judgment, is indispensable this petition.
to overcome the constitutional presumption of innocence.

Petitioner claims that there was no lawful search and


Petitioner Ruben del Castillo is ACQUITTED on seizure, because there was no lawful arrest. He claims
reasonable doubt. that the finding that there was a lawful arrest was
erroneous, since he was not even issued a citation ticket
or charged with violation of the city ordinance. Even
LUZ v. PEOPLE assuming there was a valid arrest, he claims that he had
never consented to the search conducted upon him.
FACTS: PO2 Emmanuel L. Alteza, who was then
assigned at the Sub-Station 1 of the Naga City Police
Station as a traffic enforcer, substantially testified that on On the other hand, finding that petitioner had been
March 10, 2003 at around 3:00 o’clock in the morning, he lawfully arrested, the RTC held thus: The accused himself
saw the accused, who was coming from the direction of admitted that he was not wearing a helmet at the time
Panganiban Drive and going to Diversion Road, Naga when he was flagged down by the said police officers,
City, driving a motorcycle without a helmet; that this albeit he had a helmet in his possession. Obviously, there
prompted him to flag down the accused for violating a is legal basis on the part of the apprehending officers to
municipal ordinance which requires all motorcycle drivers flag down and arrest the accused because the latter was
to wear helmet (sic) while driving said motor vehicle; that actually committing a crime in their presence, that is, a
he invited the accused to come inside their sub-station violation of City Ordinance No. 98-012. In other words, the
since the place where he flagged down the accused is accused, being caught in flagrante delicto violating the
almost in front of the said sub-station; that while he and said Ordinance, he could therefore be lawfully stopped or
SPO1 Rayford Brillante were issuing a citation ticket for arrested by the apprehending officers.
violation of municipal ordinance, he noticed that the
accused was uneasy and kept on getting something from
his jacket; that he was alerted and so, he told the accused
ISSUE: W/N there was a valid arrest that may allow the
public officer to conduct a warrantless search and seizure.
SECTION 7. Procedure in Flagging Down or Accosting
Vehicles While in Mobile Car. This rule is a general
concept and will not apply in hot pursuit operations. The
RULING: No. mobile car crew shall undertake the following, when
applicable: x x x

We find the Petition to be impressed with merit, but not for


the particular reasons alleged. m. If it concerns traffic violations, immediately issue a
Traffic Citation Ticket (TCT) or Traffic Violation Report
(TVR). Never indulge in prolonged, unnecessary
First - There was no valid arrest of petitioner. When he conversation or argument with the driver or any of the
was flagged down for committing a traffic violation, he was vehicle’s occupants;
not, ipso facto and solely for this reason, arrested.

At the time that he was waiting for PO3 Alteza to write his
Arrest is the taking of a person into custody in order that citation ticket, petitioner could not be said to have been
he or she may be bound to answer for the commission of "under arrest." There was no intention on the part of PO3
an offense.10 It is effected by an actual restraint of the Alteza to arrest him, deprive him of his liberty, or take him
person to be arrested or by that person’s voluntary into custody. Prior to the issuance of the ticket, the period
submission to the custody of the one making the arrest. during which petitioner was at the police station may be
Neither the application of actual force, manual touching of characterized merely as waiting time. In fact, as found by
the body, or physical restraint, nor a formal declaration of the trial court, PO3 Alteza himself testified that the only
arrest, is required. It is enough that there be an intention reason they went to the police sub-station was that
on the part of one of the parties to arrest the other, and petitioner had been flagged down "almost in front" of that
that there be an intent on the part of the other to submit, place. Hence, it was only for the sake of convenience that
under the belief and impression that submission is they were waiting there. There was no intention to take
necessary. petitioner into custody.

Under R.A. 4136, or the Land Transportation and Traffic However, we decline to accord talismanic power to the
Code, the general procedure for dealing with a traffic phrase in the Miranda opinion emphasized by
violation is not the arrest of the offender, but the respondent. Two features of an ordinary traffic stop
confiscation of the driver’s license of the latter: mitigate the danger that a person questioned will be
induced "to speak where he would not otherwise do so
freely," Miranda v. Arizona, 384 U. S., at 467. First,
SECTION 29. Confiscation of Driver's License. — Law detention of a motorist pursuant to a traffic stop is
enforcement and peace officers of other agencies duly presumptively temporary and brief. Second,
deputized by the Director shall, in apprehending a driver circumstances associated with the typical traffic stop are
for any violation of this Act or any regulations issued not such that the motorist feels completely at the mercy of
pursuant thereto, or of local traffic rules and regulations the police. The usual traffic stop is more analogous to a
not contrary to any provisions of this Act, confiscate the so-called "Terry stop," see Terry v. Ohio, 392 U. S. 1
license of the driver concerned and issue a receipt (1968), than to a formal arrest. x x x The comparatively
prescribed and issued by the Bureau therefor which shall nonthreatening character of detentions of this sort
authorize the driver to operate a motor vehicle for a period explains the absence of any suggestion in our opinions
not exceeding seventy-two hours from the time and date that Terry stops are subject to the dictates of Miranda.
of issue of said receipt. The period so fixed in the receipt The similarly noncoercive aspect of ordinary traffic stops
shall not be extended, and shall become invalid prompts us to hold that persons temporarily detained
thereafter. Failure of the driver to settle his case within pursuant to such stops are not "in custody" for the
fifteen days from the date of apprehension will be a purposes of Miranda.
ground for the suspension and/or revocation of his
license.
It also appears that, according to City Ordinance No. 98-
012, which was violated by petitioner, the failure to wear
Similarly, the Philippine National Police (PNP) Operations a crash helmet while riding a motorcycle is penalized by
Manual12 provides the following procedure for flagging a fine only. Under the Rules of Court, a warrant of arrest
down vehicles during the conduct of checkpoints: need not be issued if the information or charge was filed
for an offense penalized by a fine only. It may be stated
as a corollary that neither can a warrantless arrest be It must be noted that the evidence seized, although
made for such an offense. alleged to be inadvertently discovered, was not in "plain
view." It was actually concealed inside a metal container
inside petitioner’s pocket. Clearly, the evidence was not
This ruling does not imply that there can be no arrest for immediately apparent.16
a traffic violation. Certainly, when there is an intent on the
part of the police officer to deprive the motorist of liberty,
or to take the latter into custody, the former may be Neither was there a consented warrantless search.
deemed to have arrested the motorist. In this case, Consent to a search is not to be lightly inferred, but shown
however, the officer’s issuance (or intent to issue) a traffic by clear and convincing evidence.17 It must be voluntary
citation ticket negates the possibility of an arrest for the in order to validate an otherwise illegal search; that is, the
same violation. consent must be unequivocal, specific, intelligently given
and uncontaminated by any duress or coercion. While the
prosecution claims that petitioner acceded to the
Even if one were to work under the assumption that instruction of PO3 Alteza, this alleged accession does not
petitioner was deemed "arrested" upon being flagged suffice to prove valid and intelligent consent. In fact, the
down for a traffic violation and while awaiting the issuance RTC found that petitioner was merely "told" to take out the
of his ticket, then the requirements for a valid arrest were contents of his pocket.18
not complied with.

Whether consent to the search was in fact voluntary is a


This Court has held that at the time a person is arrested, question of fact to be determined from the totality of all the
it shall be the duty of the arresting officer to inform the circumstances. Relevant to this determination are the
latter of the reason for the arrest and must show that following characteristics of the person giving consent and
person the warrant of arrest, if any. Persons shall be the environment in which consent is given:
informed of their constitutional rights to remain silent and (1) the age of the defendant;
to counsel, and that any statement they might make could
be used against them.14 It may also be noted that in this (2) whether the defendant was in a public or a secluded
case, these constitutional requirements were complied location;
with by the police officers only after petitioner had been
arrested for illegal possession of dangerous drugs. (3) whether the defendant objected to the search or
passively looked on;

(4) the education and intelligence of the defendant;


Second - There being no valid arrest, the warrantless
search that resulted from it was likewise illegal. (5) the presence of coercive police procedures;

(6) the defendant’s belief that no incriminating evidence


would be found;
The following are the instances when a warrantless
search is allowed: (7) the nature of the police questioning;

(i) a warrantless search incidental to a lawful arrest; (8) the environment in which the questioning took place;
and
(ii) search of evidence in "plain view;"
(9) the possibly vulnerable subjective state of the person
(iii) search of a moving vehicle; consenting.

(iv) consented warrantless search;

(v) customs search; It is the State that has the burden of proving, by clear and
positive testimony, that the necessary consent was
(vi) a "stop and frisk" search; and obtained, and was freely and voluntarily given.19 In this
(vii) exigent and emergency circumstances.15 case, all that was alleged was that petitioner was alone at
the police station at three in the morning, accompanied by
several police officers. These circumstances weigh
heavily against a finding of valid consent to a warrantless
None of the above-mentioned instances, especially a
search.
search incident to a lawful arrest, are applicable to this
case.

Neither does the search qualify under the "stop and frisk"
rule. While the rule normally applies when a police officer
observes suspicious or unusual conduct, which may lead AMBRE v. PEOPLE
him to believe that a criminal act may be afoot, the stop
and frisk is merely a limited protective search of outer
FACTS: Accused Ambre, Castro and Mendoza were
clothing for weapons.20
charged with illegal possession of drug paraphernalia and
illegal use of methylamphetamine hydrochloride,
otherwise known as shabu. Both Castro and Mendoza
In Robinson, supra, we noted the two historical rationales pleaded guilty to both charges. Ambre, on the other hand,
for the "search incident to arrest" exception: entered a plea of not guilty to the charges.7 Trial on the
merits ensued.
(1) the need to disarm the suspect in order to take him
into custody, and

(2) the need to preserve evidence for later use at trial. x x From the testimonies of prosecution witnesses, the
x But neither of these underlying rationales for the search Caloocan Police Station Anti-Illegal Drug-Special
incident to arrest exception is sufficient to justify the Operation Unit conducted a buy-bust operation pursuant
search in the present case. to a tip from a police informant that a certain Abdulah
Sultan (Sultan) and his wife Ina Aderp (Aderp) were
engaged in the selling of dangerous drugs at a residential
We have recognized that the first rationale—officer compound in Caloocan City; that the buy-bust operation
safety—is "‘both legitimate and weighty,’" x x x The threat resulted in the arrest of Aderp and a certain Moctar
to officer safety from issuing a traffic citation, however, is Tagoranao (Tagoranao); that Sultan ran away from the
a good deal less than in the case of a custodial arrest. In scene of the entrapment operation and PO3 Moran, PO2
Robinson, we stated that a custodial arrest involves Masi and PO1 Mateo, pursued him; that in the course of
"danger to an officer" because of "the extended exposure the chase, Sultan led the said police officers to his house;
which follows the taking of a suspect into custody and that inside the house, the police operatives found Ambre,
transporting him to the police station." 414 U. S., at 234- Castro and Mendoza having a pot session; that Ambre, in
235. We recognized that "[t]he danger to the police officer particular, was caught sniffing what was suspected to be
flows from the fact of the arrest, and its attendant shabu in a rolled up aluminum foil; and that PO3 Moran
proximity, stress, and uncertainty, and not from the ran after Sultan while PO2 Masi and PO1 Mateo arrested
grounds for arrest." Id., at 234, n. 5. A routine traffic stop, Ambre, Castro and Mendoza for illegal use of shabu. The
on the other hand, is a relatively brief encounter and "is items confiscated from the three were marked and,
more analogous to a so-called ‘Terry stop’ . . . than to a thereafter, submitted for laboratory examination, which
formal arrest." Berkemer v. McCarty, 468 U. S. 420, 439 later on turned out to be positive for the presence of
(1984). See also Cupp v. Murphy, 412 U. S. 291, 296 shabu.
(1973) ("Where there is no formal arrest . . . a person
might well be less hostile to the police and less likely to
take conspicuous, immediate steps to destroy However, Ambre vehemently denied the charges against
incriminating evidence"). her. Through the testimonies of Ambre, Mendoza and Lily
Rosete (Rosete), Ambre was inside the residential
The foregoing considered, petitioner must be acquitted.
compound in Caloocan to buy malong; that when they
While he may have failed to object to the illegality of his
failed to buy malong, Rosete and Buban left her inside the
arrest at the earliest opportunity, a waiver of an illegal
residential compound to look for other vendors; that ten
warrantless arrest does not, however, mean a waiver of
minutes later, the policemen barged inside the compound
the inadmissibility of evidence seized during the illegal
and arrested her.
warrantless arrest. The subject items seized during the
illegal arrest are inadmissible.25 The drugs are the very
corpus delicti of the crime of illegal possession of
dangerous drugs. Thus, their inadmissibility precludes RTC rendered its decision declaring that the prosecution
conviction and calls for the acquittal of the accused.26 was able to establish with certitude the guilt of Ambre for
illegal use of methylamphetamine hydrochloride or
violation of Section 15, Article II of R.A. No. 9165. The
RTC, however, acquitted her of the crime of violation of
Petitioner Rodel Luz y Ong is hereby ACQUITTED and
Section 12, Article II of R.A. No. 9165 for failure of the
ordered immediately released from detention, unless his
prosecution to prove with particularity the drug
continued confinement is warranted by some other cause
paraphernalia found in her possession.
or ground.

On appeal, CA affirmed the RTC’s decision. Hence, this


petition.
ISSUES: (a) When, in his presence, the person to be arrested has
committed, is actually committing, or is attempting to
commit an offense;
1.) Whether the warrantless arrest of Ambre and the
search of her person was valid; and
(b) When an offense has in fact just been committed, and
2.) Whether the items seized are inadmissible in he has personal knowledge of facts indicating that the
evidence. person to be arrested has committed it; and

Essentially, Ambre insists that the warrantless arrest and (c) When the person to be arrested is a prisoner who
search made against her were illegal because no offense escaped from a penal establishment or place where he is
was being committed at the time and the police operatives serving final judgment or temporarily confined while his
were not authorized by a judicial order to enter the case is pending, or has escaped while being transferred
dwelling of Sultan. She argues that the alleged "hot from one confinement to another. (Emphasis supplied)
pursuit" on Sultan which ended in the latter's house,
where she, Mendoza and Castro were supposedly found
having a pot session, was more imaginary than real. In
this regard, Ambre cites the April 29, 2005 Resolution of In arrest in flagrante delicto, the accused is apprehended
the Prosecutor's Office of Caloocan City dismissing the at the very moment he is committing or attempting to
case against Aderp and Sultan for insufficiency of commit or has just committed an offense in the presence
evidence because the April 20, 2005 buy-bust operation of the arresting officer. Clearly, to constitute a valid in
was highly suspicious and doubtful. She posits that the flagrante delicto arrest, two requisites must concur:
items allegedly seized from her were inadmissible in (1) the person to be arrested must execute an overt act
evidence being fruits of a poisonous tree. She claims that indicating that he has just committed, is actually
the omission of the apprehending team to observe the committing, or is attempting to commit a crime; and
procedure outlined in R.A. No. 9165 for the seizure of
evidence in drugs cases significantly impairs the (2) such overt act is done in the presence or within the
prosecution s case. Lastly, Ambre maintains that she was view of the arresting officer.
not subjected to a confirmatory test and, hence, the
imposition of the penalty of six months rehabilitation was
not justified. In the case at bench, there is no gainsaying that Ambre
was caught by the police officers in the act of using shabu
and, thus, can be lawfully arrested without a warrant. PO1
RULING: Yes, the warrantless arrest was valid, hence, Mateo positively identified Ambre sniffing suspected
the items seized were admissible. mbre stands. shabu from an aluminum foil being held by Castro.17
Ambre, however, made much of the fact that there was no
prior valid intrusion in the residence of Sultan. The
The conviction of A argument is specious.

One of the recognized exception established by Suffice it to state that prior justification for intrusion or prior
jurisprudence is search incident to a lawful arrest.15 In lawful intrusion is not an element of an arrest in flagrante
this exception, the law requires that a lawful arrest must delicto. Thus, even granting arguendo that the
precede the search of a person and his belongings. As a apprehending officers had no legal right to be present in
rule, an arrest is considered legitimate if effected with a the dwelling of Sultan, it would not render unlawful the
valid warrant of arrest. Section 5, Rule 113 of the Rules arrest of Ambre, who was seen sniffing shabu with Castro
of Criminal Procedure, however, recognizes permissible and Mendoza in a pot session by the police officers.
warrantless arrests: Accordingly, PO2 Masi and PO1 Mateo were not only
authorized but were also duty-bound to arrest Ambre
together with Castro and Mendoza for illegal use of
methamphetamine hydrochloride in violation of Section
"Sec. 5. Arrest without warrant; when lawful. -- A peace 15, Article II of R.A. No. 9165. Moreover, the Court holds
officer or a private person may, without a warrant, arrest that Ambre is deemed to have waived her objections to
a person: her arrest for not raising them before entering her plea.
Considering that the warrantless arrest of Ambre was any. Hence, SPO1 Requejo confiscated Abenes’ firearm,
valid, the subsequent search and seizure done on her which was later identified as a Norinco .45 caliber pistol
person was likewise lawful. After all, a legitimate bearing Serial No. 906347, including its magazine
warrantless arrest necessarily cloaks the arresting police containing seven live ammunitions.
officer with authority to validly search and seize from the
offender (1) dangerous weapons, and (2) those that may
be used as proof of the commission of an offense. Upon the indorsement of Abenes to a certain SPO2
Further, the physical evidence corroborates the Benvienido Albon for further investigation, it was found
testimonies of the prosecution witnesses that Ambre, out Abenes is not a registered nor a licensed firearm
together with Castro and Mendoza, were illegally using holder, through the certification issued by Firearms and
shabu. The urine samples taken from them were found Explosives License Processing Section of the PNP.
positive for the presence of shabu

In his defense, accused-appellant tried to establish that


the firearm did not belong to and was not recovered from
C. WHEN THINGS SEIZED ARE WITH PLAIN VIEW OF A him; that the firearm was recovered by the policemen from
SEARCHING PARTY the floor of the vehicle inside a clutch bag which was
allegedly left by an unidentified person who hitched a ride
somewhere along the national highway of Tawagan Norte
ABENES v. CA Zamboanga Del Sur and alighted near the Mabuhay
Bazaar in Pagadian City.

FACTS: Three days prior to the May 11, 1998 national


and local elections, the Philippine National Police (PNP)
RTC convicted petitioner on the following charges:
of Pagadian City, through its Company Commander Major
Pedronisto Quano, created a team composed of seven 1. ILLEGAL POSSESSION OF HIGH POWERED
policemen with a directive to establish and man a FIREARM & ITS AMMUNITIONS (Violation of P.D. No.
checkpoint in Barangay Danlugan at said city, for the 1866, as amended by R.A. No. 8294)
purpose of enforcing the Gun Ban which was then being
implemented by the COMELEC. SPO3 Cipriano Q.
Pascua was the designated team leader. Vehicles
2. Election Offense in violation of Sec. 261 (9)3 , BP 881
passing through the road block were required by the team
(OMNIBUS ELECTION CODE), vis-à-vis COMELEC
to stop and their occupants were then politely requested
RESOLUTION # 1958 (GUN BAN)
to alight in order to allow routine inspection and checking
of their vehicles. Motorists who refused the request were
not forced to do so.
In addition thereto, herein accused is disqualified to hold
any public office and deprived [of] the right of suffrage
At about 10:30 in the morning of the same day, a red
Tamaraw FX trying to pass through the check point was
stopped by the team and directed to park at the side of The petitioner appealed to the CA claiming that the
the road. As the occupants within the vehicle could not be checkpoint was not shown to have been legally set up,
seen through its tinted windows, SPO1 Eliezer Requejo, and/or that the frisking of the petitioner who was ordered
a member of the team, knocked on the vehicle’s window to alight from the Tamaraw FX, along with his companions
and requested the occupants to step down for a routine in the vehicle, violated his constitutional right against
inspection. The eight occupants, which included the unlawful search and seizure. CA affirmed RTC’s decision
accused-appellant Rodolfo Abenes who is the Barangay with modification as to the penalty. The CA found that with
Chairman of Tawagan Norte, Labangan, Zamboanga Del respect to the admissibility of the firearm as evidence, the
Sur, alighted from the vehicle. At this juncture, SPO1 prosecution witnesses convincingly established that the
Requejo and SPO3 Pascua noticed that a holstered .45 caliber pistol, tucked into the right waist of the
firearm was tucked at the right waist of Abenes. The petitioner when he alighted from the vehicle, was readily
firearm was readily visible to the policemen; it was not visible, and, therefore, could be seized without a search
covered by the shirt worn by Abenes. Abenes was then warrant under the "plain view" doctrine.
asked by SPO3 Pascua whether he had a license and
authority to carry the firearm, and whether his possession
was exempted from the Gun Ban being enforced by the Hence, this petition.
COMELEC. Accused answered in the affirmative. The
policemen then demanded for the pertinent documents to
be shown to support Abenes’ claim. He could not show
ISSUE: W/N the warrantless search is valid by virtue of were merely stopping cars they deemed suspicious, such
the plain view doctrine. as those whose windows are heavily tinted just to see if
the passengers thereof were carrying guns. At best they
would merely direct their flashlights inside the cars they
RULING: Yes. would stop, without opening the car’s doors or subjecting
its passengers to a body search. There is nothing
discriminatory in this as this is what the situation
demands.17 (Emphasis supplied)
The appeal is partly meritorious. The Court reverses the
CA’s finding of his conviction in Criminal Case No. 4559-
98 (illegal possession of firearms).
Thus, the Court agrees with the Solicitor General that
petitioner’s reliance on Aniag is misplaced.
As to the validity of checkpoint, petitioner insists that the
prosecution should have produced the mission order
constituting the checkpoint, and invokes Aniag, Jr. v. In the instant case, the firearm was seized from the
Comelec,15 where the Court purportedly held that petitioner when in plain view, the policemen saw it tucked
firearms seized from a motor vehicle without a warrant are into his waist uncovered by his shirt.
inadmissible because there was no indication that would
trigger any suspicion from the policemen nor any other
circumstance showing probable cause. In the present Under the plain view doctrine, objects falling in the "plain
case, the production of the mission order is not necessary view" of an officer who has a right to be in the position to
in view of the fact that the checkpoint was established have that view are subject to seizure and may be
three days before the May 11, 1998 elections; and, the presented as evidence.18 The "plain view" doctrine
circumstances under which the policemen found the gun applies when the following requisites concur:
warranted its seizure without a warrant.
(a) the law enforcement officer in search of the evidence
has a prior justification for an intrusion or is in a position
from which he can view a particular area;
This Court has ruled that not all checkpoints are illegal.
Those which are warranted by the exigencies of public (b) the discovery of the evidence in plain view is
order and are conducted in a way least intrusive to inadvertent; and
motorists are allowed. For, admittedly, routine
checkpoints do intrude, to a certain extent, on motorists’ (c) it is immediately apparent to the officer that the item
right to "free passage without interruption," but it cannot he observes may be evidence of a crime, contraband or
be denied that, as a rule, it involves only a brief detention otherwise subject to seizure.19
of travelers during which the vehicle’s occupants are
required to answer a brief question or two. For as long as
the vehicle is neither searched nor its occupants All the foregoing requirements are present in the instant
subjected to a body search, and the inspection of the case. The law enforcement officers lawfully made an
vehicle is limited to a visual search, said routine checks initial intrusion because of the enforcement of the Gun
cannot be regarded as violative of an individual’s right Ban and were properly in a position from which they
against unreasonable search. In fact, these routine particularly viewed the area. In the course of such lawful
checks, when conducted in a fixed area, are even less intrusion, the policemen came inadvertently across a
intrusive. piece of evidence incriminating the petitioner where they
saw the gun tucked into his [right] waist. The gun was in
plain view and discovered inadvertently when the
The checkpoint herein conducted was in pursuance of the petitioner alighted from the vehicle.
gun ban enforced by the COMELEC. The COMELEC
would be hard put to implement the ban if its deputized
agents were limited to a visual search of pedestrians. It However, the Court must underscore that the prosecution
would also defeat the purpose for which such ban was failed to satisfactorily prove the negative allegation in the
instituted. Those who intend to bring a gun during said Information that the petitioner possessed no license or
period would know that they only need a car to be able to permit to bear the subject firearm.
easily perpetrate their malicious designs.

It is a well-entrenched rule "that in crimes involving illegal


The facts adduced do not constitute a ground for a possession of firearm, the prosecution has the burden of
violation of the constitutional rights of the accused against proving the elements thereof, viz: the existence of the
illegal search and seizure. PO3 Suba admitted that they subject firearm, and the fact that the accused who owned
or possessed the firearm does not have the sealed transparent plastic sachet containing white
corresponding license or permit to possess the same." substance. While PO1 Cruz was not sure what the plastic
sachet contained, he became suspicious when petitioner
started acting strangely as he began to approach her. He
Witness for the prosecution SPO4 Gilbert C. Senados then introduced himself as a police officer to petitioner
admitted that his records were outdated, i.e., that his and inquired about the plastic sachet she was placing
Master List of holders of firearms only covered licenses inside her cigarette case. Instead of replying, however,
up to 1994; that it was possible for the petitioner to acquire petitioner attempted to flee to her house nearby but was
a license after 1994; and that he issued the Certification, timely restrained by PO1 Cruzin who then requested her
dated May 18, 1998, stating that the petitioner carried no to take out the transparent plastic sachet from the
license or permit to possess the guns because he was cigarette case. After apprising petitioner of her
ordered to do so by his superiors. There is no evidence constitutional rights, PO1 Cruzin confiscated the plastic
that between 1994 and May 8, 1998, the date the crime sachet7 on which he marked her initials "SRE." With the
was allegedly committed, no license was issued to seized item, petitioner was brought for investigation to a
petitioner. Thus, for failure of the prosecution to prove Pasay City Police Station.
beyond reasonable doubt that petitioner was carrying a
firearm without prior authority, license or permit, the latter
must be exculpated from criminal liability under P.D. No. However, petitioner has her own account of events stating
1866, as amended. that at around 1:00 to 2:00 p.m. of the date in question,
while she was sick and resting at home, several
policemen in civilian garb with guns tucked in their waists
With respect to the charge of violating Section 261(q) of barged in and asked her whether she knew one named
B.P. Blg. 881, as amended, otherwise known as the "Ryan" who they claimed was a notorious snatcher
Omnibus Election Code, the Court is constrained to affirm operating in the area, to which she replied in the negative.
the conviction of the petitioner, since the prosecution The police officers then forced her to go with them to the
successfully discharged its burden of proof. Pasay City Police Station-SOG office where she was
detained. While she was under detention, the police
officers were toying with a wallet which they claimed
contained shabu and recovered from her. In fine,
Petitioner Rodolfo Abenes Y Gacutan is ACQUITTED
petitioner claimed that the evidence against her was
from the charge of illegal possession of firearm under P.D.
"planted," stemming from an all too obvious attempt by
No. 1866, as amended, for failure of the prosecution to
the police officers to extort money from her and her family.
prove his guilt beyond unreasonable doubt. With respect
to Criminal Case No. 4563-98 (Violation of the provisions
of Gun Ban), the assailed Decision of the Court of
Appeals is AFFIRMED with MODIFICATIONS as to RTC found the petitioner guilty of illegal possession of
penalties. Methylamphetamine Hydrochloride or shabu. Susan
Esquillo y Romines GUILTY beyond reasonable doubt of
the crime of Violation of par. 3 of Section 11, Article II of
R. A. 9165, otherwise known as the Comprehensive
ESQUILLO v. PEOPLE
Dangerous Drugs Act of 2002.

FACTS: Accused Esquillo was charged with violation of


Section 11, Article II of Republic Act (R.A.) No. 9165 (the On appeal, appellant questioned as illegal her arrest
Comprehensive Dangerous Drugs Act of 2002) – without warrant to thus render any evidence obtained on
possession of methamphetamine hydrochloride or shabu, the occasion thereof inadmissible. CA, affirming RTC’s
when she has found to be in possession, custody and decision, citing People v. Chua, held that the police
control 0.1224 gram of shabu. officers had probable cause to search petitioner under the
"stop-and-frisk" concept, a recognized exception to the
general rule prohibiting warrantless searches.
On the basis of an informant’s tip, PO1 Cruzin, together
with PO2 Angel Aguas (PO2 Aguas), proceeded at
around 4:00 p.m. on December 10, 2002 to Bayanihan In her present petition to the SC, petitioner assails the
St., Malibay, Pasay City to conduct surveillance on the appellate court’s application of the "stop-and-frisk"
activities of an alleged notorious snatcher operating in the principle in light of PO1 Cruzin’s failure to justify his
area known only as "Ryan." As PO1 Cruzin alighted from suspicion that a crime was being committed, he having
the private vehicle that brought him and PO2 Aguas to the merely noticed her placing something inside a cigarette
target area, he glanced in the direction of petitioner who case which could hardly be deemed suspicious. To
was standing three meters away and seen placing inside petitioner, such legal principle could only be invoked if
a yellow cigarette case what appeared to be a small heat-
there were overt acts constituting unusual conduct that contraband) concealed about him. It should therefore be
would arouse the suspicion. emphasized that a search and seizure should precede the
arrest for this principle to apply.

ISSUE: W/N the warrantless search, and subsequent


arrest, are valid. What is, therefore, essential is that a genuine reason must
exist, in light of the police officer’s experience and
surrounding conditions, to warrant the belief that the
RULING: Yes. person who manifests unusual suspicious conduct has
weapons or contraband concealed about him. Such a
"stop-and-frisk" practice serves a dual purpose: (1) the
general interest of effective crime prevention and
Appellant’s conviction stands.
detection, which underlies the recognition that a police
officer may, under appropriate circumstances and in an
appropriate manner, approach a person for purposes of
Petitioner did not question early on her warrantless arrest investigating possible criminal behavior even without
– before her arraignment. Neither did she take steps to probable cause; and (2) the more pressing interest of
quash the Information on such ground. Verily, she raised safety and self-preservation which permit the police
the issue of warrantless arrest – as well as the officer to take steps to assure himself that the person with
inadmissibility of evidence acquired on the occasion whom he deals is not armed with a deadly weapon that
thereof– for the first time only on appeal before the could unexpectedly and fatally be used against the police
appellate court.18 By such omissions, she is deemed to officer.23
have waived any objections on the legality of her arrest.

From these standards, the Court finds that the questioned


PLAIN VIEW act of the police officers constituted a valid "stop-and-
frisk" operation. The search/seizure of the suspected
Be that as it may, the circumstances under which
shabu initially noticed in petitioner’s possession - later
petitioner was arrested indeed engender the belief that a
voluntarily exhibited24 to the police operative - was
search on her was warranted. Recall that the police
undertaken after she was interrogated on what she placed
officers were on a surveillance operation as part of their
inside a cigarette case, and after PO1 Cruzin introduced
law enforcement efforts. When PO1 Cruzin saw petitioner
himself to petitioner as a police officer. And, at the time of
placing a plastic sachet containing white crystalline
her arrest, petitioner was exhibiting suspicious behavior
substance into her cigarette case, it was in his plain view.
and in fact attempted to flee after the police officer had
Given his training as a law enforcement officer, it was
identified himself.
instinctive on his part to be drawn to curiosity and to
approach her. That petitioner reacted by attempting to flee
after he introduced himself as a police officer and inquired
about the contents of the plastic sachet all the more It has not escaped the Court’s attention that petitioner
pricked his curiosity. seeks exculpation by adopting two completely
inconsistent or incompatible lines of defense. On one
hand, she argues that the "stop-and-frisk" search upon
her person and personal effects was unjustified as it
STOP-AND-FRISK
constituted a warrantless search in violation of the
Elucidating on what includes "stop-and-frisk" operation Constitution. In the same breadth, however, she denies
and how it is to be carried out, the Court in People v. culpability by holding fast to her version that she was at
Chua22 RULING: home resting on the date in question and had been
forcibly dragged out of the house by the police operatives
and brought to the police station, for no apparent reason
than to try and extort money from her.
. . . the act of a police officer to stop a citizen on the street,
interrogate him, and pat him for weapon(s) or contraband.
The police officer should properly introduce himself and
make initial inquiries, approach and restrain a person who
manifests unusual and suspicious conduct, in order to
check the latter’s outer clothing for possibly concealed
weapons. The apprehending police officer must have a
genuine reason, in accordance with the police officer’s
experience and the surrounding conditions, to warrant the
belief that the person to be held has weapons (or
PEOPLE v. MARTINEZ RULING: No.

FACTS: Accused were found to be sniffing and


possessing dangerous drugs (shabu residues) contained After an assiduous assessment of the evidentiary records,
in empty plastic sachets and rolled aluminum foil, during the Court finds that the prosecution failed to prove the
a party, or at a social gathering or meeting, or in the guilt of the accused. The principal reasons are
proximate company of at least two (2) person[s].
1] that the evidence against the accused are inadmissible;
and

According to the prosecution, that on September 2, 2006, 2] that granting the same to be admissible, the chain of
at around 12:45 o’clock in the afternoon, a concerned custody has not been duly established.
citizen entered the precinct and reported that a pot
session was going on in the house of accused Rafael
Gonzales (Gonzales). Upon receipt of the report, PO1 Indeed, the accused is estopped from assailing the
Azardon, PO1 Alejandro Dela Cruz (PO1 Dela Cruz), and legality of his arrest if he fails to raise such issue before
members of the Special Weapons and Tactics (SWAT) arraignment.5 However, this waiver is limited only to the
team hied to Trinidad Subdivision, Dagupan City. Upon arrest. The legality of an arrest affects only the jurisdiction
inquiry from people in the area, the house of Gonzales of the court over the person of the accused. A waiver of
was located. As the police officers entered the gate of the an illegal warrantless arrest does not carry with it a waiver
house, they saw accused Orlando Doria (Doria) coming of the inadmissibility of evidence seized during the illegal
out of the side door and immediately arrested him. Inside warrantless arrest.
the house, they saw accused Gonzales, Arnold Martinez
(A. Martinez), Edgar Dizon (Dizon), and Rezin Martinez
(R. Martinez) in a room. The four were surprised by the This case would appear to fall under either a warrantless
presence of the police. In front of them were open plastic search incidental to a lawful arrest or a plain view search,
sachets (containing shabu residue), pieces of rolled used both of which require a lawful arrest in order to be
aluminum foil and pieces of used aluminum foil. The considered valid exceptions to the constitutional
accused were arrested and brought to the police precinct. guarantee. Rule 113 of the Revised Rules of Criminal
Procedure provides for the circumstances under which a
warrantless arrest is lawful. Thus:
The defense, through its witnesses, accused A. Martinez,
Dizon, and R. Martinez, claimed that in the morning of
September 2, 2006, the three of them were along Arellano Sec. 5. Arrest without warrant; when lawful. – A peace
Street in Trinidad Subdivision, Dagupan City, to meet with
officer or a private person may, without a warrant, arrest
a certain Apper who bumped the passenger jeep of R. a person:
Martinez and who was to give the materials for the
painting of said jeep. As they were going around the
subdivision looking for Apper, they saw Gonzales in front
of his house and asked him if he noticed a person pass (a) When, in his presence, the person to be arrested has
by. While they were talking, Doria arrived. It was then that committed, is actually committing, or is attempting to
five to seven policemen emerged and apprehended them. commit an offense;
They were handcuffed and brought to the police station in
Perez, Dagupan City, where they were incarcerated and
charged with sniffing shabu. (b) When an offense has just been committed and he has
probable cause to believe based on personal knowledge
of facts or circumstances that the person to be arrested
RTC found the petitioners guilty f the crime of Possession has committed it; and
of Dangerous Drugs During Parties, Social Gatherings or
Meetings defined and penalized under Section 13 in
relation to Section 11, Article II of Republic Act 9165. (c) When the person to be arrested is a prisoner who has
Case against Doria was dismissed on a demurrer to escaped from a penal establishment or place where he is
evidence. This was affirmed by the CA. Hence, this serving final judgment or is temporarily confined while his
petition. case is pending, or has escaped while being transferred
from one confinement to another.

ISSUE: W/N there was a valid warrantless search.


A review of the facts reveal that the arrest of the accused
was illegal and the subject items were confiscated as an
incident thereof. According to the testimony of PO1 exigent and emergency circumstances, for the evidence
Azardon and his Joint Affidavit13 with PO1 Dela Cruz, at hand is bereft of any such showing.
they proceeded to, and entered, the house of accused
Gonzales based solely on the report of a concerned
citizen that a pot session was going on in said house. On the contrary, it indicates that the apprehending officers
should have conducted first a surveillance considering
that the identities and address of the suspected culprits
Although this Court has ruled in several dangerous drugs were already ascertained. After conducting the
cases16 that tipped information is sufficient probable surveillance and determining the existence of probable
cause to effect a warrantless search,17 such rulings cause for arresting accused-appellants, they should have
cannot be applied in the case at bench because said secured a search warrant prior to effecting a valid arrest
cases involve either a buy-bust operation or drugs in and seizure. The arrest being illegal ab initio, the
transit, basically, circumstances other than the sole tip of accompanying search was likewise illegal. Every
an informer as basis for the arrest. None of these drug evidence thus obtained during the illegal search cannot
cases involve police officers entering a house without be used against accused-appellants; hence, their
warrant to effect arrest and seizure based solely on an acquittal must follow in faithful obeisance to the
informer’s tip. fundamental law.19

In People v. Bolasa, an anonymous caller tipped off the It has been held that personal knowledge of facts in
police that a man and a woman were repacking prohibited arrests without warrant must be based upon probable
drugs at a certain house. The police immediately cause, which means an actual belief or reasonable
proceeded to the house of the suspects. They walked grounds of suspicion. The grounds of suspicion are
towards the house accompanied by their informer. When reasonable when the suspicion, that the person to be
they reached the house, they peeped inside through a arrested is probably guilty of committing an offense, is
small window and saw a man and woman repacking based on actual facts, that is, supported by circumstances
marijuana. They then entered the house, introduced sufficiently strong in themselves to create the probable
themselves as police officers, confiscated the drug cause of guilt of the person to be arrested. 20
paraphernalia, and arrested the suspects. This Court
ruled:
As to paragraph (a) of Section 5 of Rule 113, the arresting
officers had no personal knowledge that at the time of the
The manner by which accused-appellants were arrest, accused had just committed, were committing, or
apprehended does not fall under any of the above- were about to commit a crime, as they had no probable
enumerated categories. Perforce, their arrest is illegal. cause to enter the house of accused Rafael Gonzales in
First, the arresting officers had no personal knowledge order to arrest them. As to paragraph (b), the arresting
that at the time of their arrest, accused-appellants had just officers had no personal knowledge of facts and
committed, were committing, or were about to commit a circumstances that would lead them to believe that the
crime. Second, the arresting officers had no personal accused had just committed an offense. As admitted in
knowledge that a crime was committed nor did they have the testimony of PO1 Azardon, the tip originated from a
any reasonable ground to believe that accused- concerned citizen who himself had no personal
appellants committed it. Third, accused-appellants were knowledge of the information that was reported to the
not prisoners who have escaped from a penal police.
establishment.

Neither can it be said that the subject items were seized


Neither can it be said that the objects were seized in plain in plain view. The elements of plainview are:
view. First, there was no valid intrusion. As already
discussed, accused-appellants were illegally arrested. (a) a prior valid intrusion based on the valid warrantless
Second, the evidence, i.e., the tea bags later on found to arrest in which the police are legally present in the pursuit
contain marijuana, was not inadvertently discovered. The of their official duties;
police officers intentionally peeped first through the (b) the evidence was inadvertently discovered by the
window before they saw and ascertained the activities of police who have the right to be where they are;
accused-appellants inside the room. In like manner, the
search cannot be categorized as a search of a moving (c) the evidence must be immediately apparent; and,
vehicle, a consented warrantless search, a customs
(d) "plain view" justified mere seizure of evidence without
search, or a stop and frisk; it cannot even fall under
further search.22
and trial on the merits a decision was rendered on
October 8, 1987 finding petitioner guilty of the offense
The evidence was not inadvertently discovered as the charged.
police officers intentionally entered the house with no prior
surveillance or investigation before they discovered the CA affirmed RTC’s decision in toto. Hence, this petition.
accused with the subject items.

ISSUE: W/N there was a valid warrantless search and


The apprehending officers should have first conducted a arrest.
surveillance considering that the identity and address of
one of the accused were already ascertained. After
conducting the surveillance and determining the RULING: Yes.
existence of probable cause, then a search warrant
should have been secured prior to effecting arrest and
seizure. The arrest being illegal, the ensuing search as a
Section 5, Rule 113 of the 1985 Rules on Criminal
result thereof is likewise illegal. Evidence procured on the
Procedure provides as follows:
occasion of an unreasonable search and seizure is
deemed tainted for being the proverbial fruit of a
poisonous tree and should be excluded.23 The subject
items seized during the illegal arrest are thus SEC. 5. Arrest without warrant; when lawful — A peace
inadmissible. The drug, being the very corpus delicti of the officer or a private person may, without a warrant, arrest
crime of illegal possession of dangerous drugs, its a person:
inadmissibility thus precludes conviction, and calls for the
(a) When in his presence, the person to be arrested has
acquittal of the accused.
committed is actually committing, or is attempting to
D. STOP AND FRISK commit an offense;

(b) When an offense has in fact just been committed, and


he has personal knowledge of facts indicating that the
POSADAS v. CA
person to be arrested has committed it; and

(c) When the person to be arrested is a prisoner who has


FACTS: On October 16, 1986 at about 10:00 o'clock in
escaped from a penal establishment or place where he is
the morning Pat. Ursicio Ungab and Pat. Umbra Umpar,
serving final judgment or temporarily confined while his
both members of the Integrated National Police (INP) of
case is pending, or has escaped while being transferred
the Davao Metrodiscom assigned with the Intelligence
from one confinement to another.
Task Force, were conducting a surveillance along
Magallanes Street, Davao City. While they were within the In cases falling under paragraphs (a) and (b) hereof, the
premises of the Rizal Memorial Colleges they spotted person arrested without a warrant shall be forthwith
petitioner (minor) carrying a "buri" bag and they noticed delivered to the nearest police station or jail, and he shall
him to be acting suspiciously. be proceeded against in accordance with Rule 112,
Section 7. (6a, 17a)
They approached the petitioner and identified themselves
as members of the INP. Petitioner attempted to flee but The Solicitor General argues that when the two policemen
his attempt to get away was thwarted by the two approached the petitioner, he was actually committing or
notwithstanding his resistance. had just committed the offense of illegal possession of
firearms and ammunitions in the presence of the police
They then checked the "buri" bag of the petitioner where
officers and consequently the search and seizure of the
they found one (1) caliber .38 Smith & Wesson revolver
contraband was incidental to the lawful arrest in
with Serial No. 770196 1 two (2) rounds of live
accordance with Section 12, Rule 126 of the 1985 Rules
ammunition for a .38 caliber gun 2 a smoke (tear gas)
on Criminal Procedure. We disagree.
grenade,3 and two (2) live ammunitions for a .22 caliber
gun. 4 They brought the petitioner to the police station for At the time the peace officers in this case identified
further investigation. In the course of the same, the themselves and apprehended the petitioner as he
petitioner was asked to show the necessary license or attempted to flee they did not know that he had
authority to possess firearms and ammunitions found in committed, or was actually committing the offense of
his possession but he failed to do so. He was then taken illegal possession of firearms and ammunitions. They just
to the Davao Metrodiscom office and the prohibited suspected that he was hiding something in the buri bag.
articles recovered from him were indorsed to M/Sgt. Didoy They did now know what its contents were. The said
the officer then on duty. He was prosecuted for illegal circumstances did not justify an arrest without a warrant.
possession of firearms and ammunitions in the Regional
Trial Court of Davao City wherein after a plea of not guilty
However, there are many instances where a warrant and PEOPLE v. MENGOTE
seizure can be effected without necessarily being
preceded by an arrest, foremost of which is the "stop and
FACTS: Accused-appellant Rogelio Mengote was
search" without a search warrant at military or police
convicted of illegal possession of firearms on the strength
checkpoints, the constitutionality or validity of which has
mainly of the stolen pistol found on his person at the
been upheld by this Court in Valmonte vs. de Villa, 7 as
moment of his warrantless arrest. In this appeal, he
follows:
pleads that the weapon was not admissible as evidence
Not all searches and seizures are prohibited. Those which against him because it had been illegally seized and was
are reasonable are not forbidden. A reasonable search is therefore the fruit of the poisonous tree. The Government
not to be determined by any fixed formula but is to be disagrees. It insists that the revolver was validly received
resolved according to the facts of each case. in evidence by the trial judge because its seizure was
incidental to an arrest that was doubtless lawful even if
Between the inherent right of the state to protect its admittedly without warrant.
existence and promote public welfare and an individual's
right against a warrantless search which is however
reasonably conducted, the former should prevail.
The incident occurred shortly before noon of August 8,
1987, after the Western Police District received a
telephone call from an informer that there were three
Thus, as between a warrantless search and seizure suspicious-looking persons at the corner of Juan Luna
conducted at military or police checkpoints and the search and North Bay Boulevard in Tondo, Manila. A surveillance
thereat in the case at bar, there is no question that, team of plainclothesmen was forthwith dispatched to the
indeed, the latter is more reasonable considering that place. There they saw two men "looking from side to side,"
unlike in the former, it was effected on the basis of a one of whom was holding his abdomen. They approached
probable cause. The probable cause is that when the these persons and identified themselves as policemen,
petitioner acted suspiciously and attempted to flee with whereupon the two tried to run away but were unable to
the buri bag there was a probable cause that he was escape because the other lawmen had surrounded them.
concealing something illegal in the bag and it was the right The suspects were then searched. One of them, who
and duty of the police officers to inspect the same. turned out to be the accused-appellant, was found with a
.38 caliber Smith and Wesson revolver with six live bullets
in the chamber. His companion, later identified as Nicanor
It is too much indeed to require the police officers to Morellos, had a fan knife secreted in his front right pants
search the bag in the possession of the petitioner only pocket. The weapons were taken from them. Mengote
after they shall have obtained a search warrant for the and Morellos were then turned over to police
purpose. Such an exercise may prove to be useless, futile headquarters for investigation by the Intelligence Division.
and much too late.

Besides the police officers, one other witness presented


What constitutes a reasonable or unreasonable search or by the prosecution was Rigoberto Danganan, who
seizure becomes purely a judicial question, determinable identified the subject weapon as among the articles stolen
from the uniqueness of the circumstances involved, from him during the robbery in his house in Malabon on
including the purpose of the search or seizure, the June 13, 1987. He pointed to Mengote as one of the
presence or absence of probable cause, the manner in robbers. For his part, Mengote made no effort to prove
which the search and seizure was made, the place or that he owned the firearm or that he was licensed to
thing searched and the character of the articles procured. possess it and claimed instead that the weapon had been
(People vs. CFI of Rizal) "Planted" on him at the time of his arrest. 3

The United States Supreme Court held that "a police It is submitted in the Appellant's Brief that the revolver
officer may in appropriate circumstances and in an should not have been admitted in evidence because of its
appropriate manner approach a person for the purpose of illegal seizure. no warrant therefor having been previously
investigating possible criminal behaviour even though obtained. Neither could it have been seized as an incident
there is no probable cause to make an arrest." In such a of a lawful arrest because the arrest of Mengote was itself
situation, it is reasonable for an officer rather than simply unlawful, having been also effected without a warrant.
to shrug his shoulder and allow a crime to occur, to stop The defense also contends that the testimony regarding
a suspicious individual briefly in order to determine his the alleged robbery in Danganan's house was irrelevant
identity or maintain the status quo while obtaining more and should also have been disregarded by the trial court.
information. (Terry v. Ohio)
Solicitor General maintains that the arrest and search of policemen themselves testified that they were dispatched
Mengote and the seizure of the revolver from him were to that place only because of the telephone call from the
lawful under Rule 113, Section 5, of the Rules of Court. informer that there were "suspicious-looking" persons in
that vicinity who were about to commit a robbery at North
Bay Boulevard. The caller did not explain why he thought
the men looked suspicious nor did he elaborate on the
impending crime.
ISSUE: W/N there was a valid warrantless arrest and
seizure.
Par. (b) is no less applicable because its no less stringent
requirements have also not been satisfied. The
RULING: No. prosecution has not shown that at the time of Mengote's
arrest an offense had in fact just been committed and that
the arresting officers had personal knowledge of facts
We have carefully examined the wording of this Rule and indicating that Mengote had committed it. All they had was
cannot see how we can agree with the prosecution. hearsay information from the telephone caller, and about
a crime that had yet to be committed.

Par. (c) of Section 5 is obviously inapplicable as Mengote


was not an escapee from a penal institution when he was The truth is that they did not know then what offense, if at
arrested. We therefore confine ourselves to determining all, had been committed and neither were they aware of
the lawfulness of his arrest under either Par. (a) or Par. the participation therein of the accused-appellant. It was
(b) of this section. only later, after Danganan had appeared at the Police
headquarters, that they learned of the robbery in his
house and of Mengote's supposed involvement therein. 8
As for the illegal possession of the firearm found on
Par. (a) requires that the person be arrested (1) after he
Mengote's person, the policemen discovered this only
has committed or while he is actually committing or is at
after he had been searched and the investigation
least attempting to commit an offense, (2) in the presence
conducted later revealed that he was not its owners nor
of the arresting officer.
was he licensed to possess it.

These requirements have not been established in the


Before these events, the Peace officers had no
case at bar. At the time of the arrest in question, the
knowledge even of Mengote' identity, let alone the fact (or
accused-appellant was merely "looking from side to side"
suspicion) that he was unlawfully carrying a firearm or that
and "holding his abdomen," according to the arresting
he was involved in the robbery of Danganan's house.
officers themselves. There was apparently no offense that
had just been committed or was being actually committed
or at least being attempted by Mengote in their presence.
MANALILI v. CA

The Solicitor General submits that the actual existence of FACTS: At about 2:10 o'clock in the afternoon of April 11,
an offense was not necessary as long as Mengote's acts 1988, policemen from the Anti-Narcotics Unit of the
"created a reasonable suspicion on the part of the Kalookan City Police Station were conducting a
arresting officers and induced in them the belief that an surveillance along A. Mabini street, Kalookan City, in front
offense had been committed and that the accused- of the Kalookan City Cemetery. The surveillance was
appellant had committed it." The question is, What being made because of information that drug addicts were
offense? What offense could possibly have been roaming the area in front of the Kalookan City Cemetery.
suggested by a person "looking from side to side" and They then chanced upon a male person in front of the
"holding his abdomen" and in a place not exactly cemetery who appeared high on drugs. The male person
forsaken? was observed to have reddish eyes and to be walking in
a swaying manner. When this male person tried to avoid
the policemen, the latter approached him and introduced
On the other hand, there could have been a number of themselves as police officers. The policemen then asked
reasons, all of them innocent, why his eyes were darting the male person what he was holding in his hands. The
from side to side and be was holding his abdomen. If they male person tried to resist. Pat Romeo Espiritu asked the
excited suspicion in the minds of the arresting officers, as male person if he could see what said male person had in
the prosecution suggests, it has nevertheless not been his hands. The latter showed the wallet and allowed Pat.
shown what their suspicion was all about. In fact, the Romeo Espiritu to examine the same. Pat. Espiritu took
the wallet and examined it. He found suspected crushed inadmissibility of the marijuana leaves was waived
marijuana residue inside. He kept the wallet and its because petitioner never raised this issue in the
marijuana contents. The male person was then brought to proceedings below nor did he object to their admissibility
the Anti-Narcotics Unit of the Kalookan City Police in evidence. He adds that, even assuming arguendo that
Headquarters and was turned over to Cpl. Wilfredo there was no waiver, the search was legal because it was
Tamondong for investigation. Pat. Espiritu also turned incidental to a warrantless arrest under Section 5 (a), Rule
over to Cpl. Tamondong the confiscated wallet and its 113 of the Rules of Court.
suspected marijuana contents. The man turned out to be
the accused ALAIN MANALILI y DIZON.
We disagree with petitioner and hold that the search was
valid, being akin to a stop-and-frisk.
However, according to the defense, at about 2:00 o'clock
in the afternoon of April 11, 1988, the accused ALAIN
MANALILI was aboard a tricycle at A. Mabini street near In the case at hand, Patrolman Espiritu and his
the Kalookan City Cemetery on the way to his boarding companions observed during their surveillance that
house. Three policemen ordered the driver of the tricycle appellant had red eyes and was wobbling like a drunk
to stop because the tricycle driver and his lone passenger along the Caloocan City Cemetery, which according to
were under the influence of marijuana. The policemen police information was a popular hangout of drug addicts.
brought the accused and the tricycle driver inside the Ford From his experience as a member of the Anti-Narcotics
Fiera which the policemen were riding in. The policemen Unit of the Caloocan City Police, such suspicious
then bodily searched the accused and the tricycle driver. behavior was characteristic of drug addicts who were
At this point, the accused asked the policemen why he "high." The policemen therefore had sufficient reason to
was being searched and the policemen replied that he stop petitioner to investigate if he was actually high on
(accused) was carrying marijuana. However, nothing was drugs. During such investigation, they found marijuana in
found on the persons of the accused and the driver. Upon petitioner's possession
arrival at the police station, the accused was asked to
remove his pants in the presence of said neighbor and
another companion. However, nothing was found, except
Furthermore, we concur with the Solicitor General's
for some dirt and dust. This prompted the companion of
contention that petitioner effectively waived the
the neighbor of the accused to tell the policemen to
inadmissibility of any evidence illegally obtained when he
release the accused. That the accused was told by the
failed to raise this issue or to object thereto during the trial.
policemen to call his parents in order to "settle" the case
A valid waiver of a right, more particularly of the
and that the accused was brought in the office of an
constitutional right against unreasonable search, requires
inquest fiscal and there he was told that no marijuana was
the concurrence of the following requirements: (1) the
found on his person but the Fiscal told the accused not to
right to be waived existed;
say anything.
(2) the person waiving it had knowledge, actual or
constructive, thereof; and
The trial court convicted petitioner of illegal possession of
(3) he or she had an actual intention to relinquish the right.
marijuana residue largely on the strength of the arresting
officers' testimony. This was affirmed by the CA. Hence,
this petition.
Otherwise, the Courts will indulge every reasonable
presumption against waiver of fundamental safeguards
and will not deduce acquiescence from the failure to
ISSUE: W/N there was a valid warrantless search.
exercise this elementary right. In the present case,
however, petitioner is deemed to have waived such right
for his failure to raise its violation before the trial court.
RULING: Yes.

Second ISSUE: Assessment of Evidence


First ISSUE: Admissibility of the Evidence Seized During
a Stop-and-Frisk Petitioner also contends that the two arresting officers'
testimony contained "polluted, irreconcilable and
Petitioner protests the admission of the marijuana leaves unexplained" contradictions which did not support
found in his possession, contending that they were petitioner's conviction.
products of an illegal search. The Solicitor General, in his
Comment dated July 5, 1994, which was adopted as
memorandum for respondent, counters that the
We disagree. Time and again, this Court has ruled that sachet7 on which he marked her initials "SRE." With the
the trial court's assessment of the credibility of witnesses, seized item, petitioner was brought for investigation to a
particularly when affirmed by the Court of Appeals as in Pasay City Police Station.
this case, is accorded great weight and respect, since it
had the opportunity to observe their demeanor and
deportment as they testified before it. Moreover, However, petitioner has her own account of events stating
assuming that there were inconsistencies in the that at around 1:00 to 2:00 p.m. of the date in question,
prosecution’s testimonies, we do not find them substantial while she was sick and resting at home, several
enough to impair the essential veracity of their narration. policemen in civilian garb with guns tucked in their waists
barged in and asked her whether she knew one named
"Ryan" who they claimed was a notorious snatcher
However, we find that, aside from the presumption of operating in the area, to which she replied in the negative.
regularity in the performance of duty, the bestowal of full The police officers then forced her to go with them to the
credence on Pat. Espiritu's testimony is justified by Pasay City Police Station-SOG office where she was
tangible evidence on record. Despite Pat. Lumabas' detained. While she was under detention, the police
contradictory testimony, that of Espiritu is supported by officers were toying with a wallet which they claimed
the Joint Affidavit29 signed by both arresting policemen. contained shabu and recovered from her. In fine,
The question of whether the marijuana was found inside petitioner claimed that the evidence against her was
petitioner's wallet or inside a plastic bag is immaterial, "planted," stemming from an all too obvious attempt by
considering that petitioner did not deny possession of said the police officers to extort money from her and her family.
substance. Failure to present the wallet in evidence did
not negate that marijuana was found in petitioner's
possession. This shows that such contradiction is minor RTC found the petitioner guilty of illegal possession of
and does not destroy Espiritu's credibility. Methylamphetamine Hydrochloride or shabu. Susan
Esquillo y Romines GUILTY beyond reasonable doubt of
the crime of Violation of par. 3 of Section 11, Article II of
ESQUILLO v. PEOPLE R. A. 9165, otherwise known as the Comprehensive
Dangerous Drugs Act of 2002.
FACTS: Accused Esquillo was charged with violation of
Section 11, Article II of Republic Act (R.A.) No. 9165 (the
Comprehensive Dangerous Drugs Act of 2002) – On appeal, appellant questioned as illegal her arrest
possession of methamphetamine hydrochloride or shabu, without warrant to thus render any evidence obtained on
when she has found to be in possession, custody and the occasion thereof inadmissible. CA, affirming RTC’s
control 0.1224 gram of shabu. decision, citing People v. Chua, held that the police
officers had probable cause to search petitioner under the
"stop-and-frisk" concept, a recognized exception to the
general rule prohibiting warrantless searches.
On the basis of an informant’s tip, PO1 Cruzin, together
with PO2 Angel Aguas (PO2 Aguas), proceeded at
around 4:00 p.m. on December 10, 2002 to Bayanihan
St., Malibay, Pasay City to conduct surveillance on the In her present petition to the SC, petitioner assails the
activities of an alleged notorious snatcher operating in the appellate court’s application of the "stop-and-frisk"
area known only as "Ryan." As PO1 Cruzin alighted from principle in light of PO1 Cruzin’s failure to justify his
the private vehicle that brought him and PO2 Aguas to the suspicion that a crime was being committed, he having
target area, he glanced in the direction of petitioner who merely noticed her placing something inside a cigarette
was standing three meters away and seen placing inside case which could hardly be deemed suspicious. To
a yellow cigarette case what appeared to be a small heat- petitioner, such legal principle could only be invoked if
sealed transparent plastic sachet containing white there were overt acts constituting unusual conduct that
substance. While PO1 Cruz was not sure what the plastic would arouse the suspicion.
sachet contained, he became suspicious when petitioner
started acting strangely as he began to approach her. He
then introduced himself as a police officer to petitioner ISSUE: W/N the warrantless search, and subsequent
and inquired about the plastic sachet she was placing arrest, are valid.
inside her cigarette case. Instead of replying, however,
petitioner attempted to flee to her house nearby but was
timely restrained by PO1 Cruzin who then requested her
to take out the transparent plastic sachet from the
cigarette case. After apprising petitioner of her
constitutional rights, PO1 Cruzin confiscated the plastic
RULING: Yes. person who manifests unusual suspicious conduct has
weapons or contraband concealed about him. Such a
"stop-and-frisk" practice serves a dual purpose: (1) the
Appellant’s conviction stands. general interest of effective crime prevention and
detection, which underlies the recognition that a police
officer may, under appropriate circumstances and in an
appropriate manner, approach a person for purposes of
Petitioner did not question early on her warrantless arrest
investigating possible criminal behavior even without
– before her arraignment. Neither did she take steps to
probable cause; and (2) the more pressing interest of
quash the Information on such ground. Verily, she raised
safety and self-preservation which permit the police
the issue of warrantless arrest – as well as the
officer to take steps to assure himself that the person with
inadmissibility of evidence acquired on the occasion
whom he deals is not armed with a deadly weapon that
thereof– for the first time only on appeal before the
could unexpectedly and fatally be used against the police
appellate court.18 By such omissions, she is deemed to
officer.23
have waived any objections on the legality of her arrest.

From these standards, the Court finds that the questioned


PLAIN VIEW
act of the police officers constituted a valid "stop-and-
Be that as it may, the circumstances under which frisk" operation. The search/seizure of the suspected
petitioner was arrested indeed engender the belief that a shabu initially noticed in petitioner’s possession - later
search on her was warranted. Recall that the police voluntarily exhibited24 to the police operative - was
officers were on a surveillance operation as part of their undertaken after she was interrogated on what she placed
law enforcement efforts. When PO1 Cruzin saw petitioner inside a cigarette case, and after PO1 Cruzin introduced
placing a plastic sachet containing white crystalline himself to petitioner as a police officer. And, at the time of
substance into her cigarette case, it was in his plain view. her arrest, petitioner was exhibiting suspicious behavior
Given his training as a law enforcement officer, it was and in fact attempted to flee after the police officer had
instinctive on his part to be drawn to curiosity and to identified himself.
approach her. That petitioner reacted by attempting to flee
after he introduced himself as a police officer and inquired
about the contents of the plastic sachet all the more It has not escaped the Court’s attention that petitioner
pricked his curiosity. seeks exculpation by adopting two completely
inconsistent or incompatible lines of defense. On one
hand, she argues that the "stop-and-frisk" search upon
STOP-AND-FRISK her person and personal effects was unjustified as it
constituted a warrantless search in violation of the
Elucidating on what includes "stop-and-frisk" operation Constitution. In the same breadth, however, she denies
and how it is to be carried out, the Court in People v. Chua culpability by holding fast to her version that she was at
RULING: home resting on the date in question and had been
forcibly dragged out of the house by the police operatives
and brought to the police station, for no apparent reason
. . . the act of a police officer to stop a citizen on the street, than to try and extort money from her.
interrogate him, and pat him for weapon(s) or contraband.
The police officer should properly introduce himself and
make initial inquiries, approach and restrain a person who
manifests unusual and suspicious conduct, in order to
check the latter’s outer clothing for possibly concealed
weapons. The apprehending police officer must have a
genuine reason, in accordance with the police officer’s
experience and the surrounding conditions, to warrant the
belief that the person to be held has weapons (or
contraband) concealed about him. It should therefore be
emphasized that a search and seizure should precede the
arrest for this principle to apply.

What is, therefore, essential is that a genuine reason must


exist, in light of the police officer’s experience and
surrounding conditions, to warrant the belief that the
E. EXPRESS WAIVER medicines and religious pamphlets was found in the
master's bedroom.
VEROY v. LAYAGUE

Fiscal Ponferrada recommended the filing of an


FACTS: Petitioners who were presently residing in
information against herein petitioners for Violation of
Quezon City, were the owners of the house in Catalunan
Presidential Decree No. 1866 (Illegal Possession of
Grande, the upkeep of which was left to two (2)
Firearms and Ammunitions in Furtherance of Rebellion)
houseboys. While the Veroys had the keys to the interior
of the house, only the key to the kitchen, where the circuit
breakers were located, was entrusted to Edna Soguilon
to give her access in case of an emergency. Hence, since Petitioners question the admissibility in evidence of the
1988, the key to the master's bedroom as well as the keys articles seized in violation of their constitutional right
to the children's rooms were retained by herein against unreasonable search and seizure
Petitioners so that neither Edna Soguilon nor the
caretakers could enter the house.
ISSUE: W/N the seized firearms considered as
admissible evidence.
On April 12, 1990, Capt. Reynaldo Obrero of the Talomo
Patrol Station, PC/INP, acting upon a directive issued by
Metrodiscom Commander Col. Franco Calida, raided the RULING: No.
house of herein petitioners in Davao City on information
that the said residence was being used as a safehouse of
rebel soldiers. They were able to enter the yard with the Petitioners aver that while they concede that Capt. Obrero
help of the caretakers but did not enter the house since
had permission from Ma. Luisa Veroy to break open the
the owner was not present and they did not have a search
door of their residence, it was merely for the purpose of
warrant. Petitioner Ma. Luisa was contacted by telephone ascertaining thereat the presence of the alleged "rebel"
in her Quezon City residence by Capt. Obrero to ask soldiers. The permission did not include any authority to
permission to search the house in Davao City as it was conduct a room to room search once inside the house.
reportedly being used as a hideout and recruitment center The items taken were, therefore, products of an illegal
of rebel soldiers. Petitioner Ma. Luisa Veroy responded search, violative of their constitutional rights As such, they
that she is flying to Davao City to witness the search but are inadmissible in evidence against them.
relented if the search would not be conducted in the
presence of Major Ernesto Macasaet, an officer of the
PC/INP, Davao City and a long time family friend of the
Veroys. The authority given by Ma. Luisa Veroy was The Constitution guarantees the right of the people to be
relayed by Capt. Obrero to Major Macasaet who secure in their persons, houses, papers and effects
answered that Ma. Luisa Veroy has called him twice by against unreasonable searches and seizures (Article III,
telephone on the matter and that the permission was Section 2 of the 1987 Constitution). However, the rule that
given on the condition that the search be conducted in his searches and seizures must be supported by a valid
presence. warrant is not an absolute one. Among the recognized
exceptions thereto are: (1) a search incidental to an
arrest; (2) a search of a moving vehicle; and (3) seizure
of evidence in plain view (People v. Lo Ho Wing, G.R. No.
Capt. Obrero and Major Macasaet then entered the 88017, January 21, 1991 [193 SCRA 122]).
children's room and conducted the search. Capt. Obrero
recovered a .45 cal. handgun with a magazine containing
seven (7) live bullets in a black clutch bag inside an
unlocked drawer. Three (3) half-full jute sacks containing None of these exceptions pertains to the case at bar. The
printed materials of RAM-SFP (samples of which were reason for searching the house of herein petitioners is that
attached as Annexes "H" and "H-1" of the petition) (Rollo, it was reportedly being used as a hideout and recruitment
pp. 49-55) were also found in the children's room. A center for rebel soldiers. While Capt. Obrero was able to
search of the children's recreation and study area enter the compound, he did not enter the house because
revealed a big travelling bag containing assorted polo he did not have a search warrant and the owners were
shirts, men's brief, two (2) pieces polo barong and short not present. This shows that he himself recognized the
sleeve striped gray polo. sweat shirt, two (2) pairs men's need for a search warrant, hence, he did not persist in
socks, a towel made in U.S.A., one blanket, a small black entering the house but rather contacted the Veroys to
bag, Gandhi brand, containing a book entitled "Islamic seek permission to enter the same. Permission was
Revolution Future Path of the Nation", a road map of the indeed granted by Ma. Luisa Veroy to enter the house but
Philippines, a telescope, a plastic bag containing assorted only to ascertain the presence of rebel soldiers. Under the
circumstances it is undeniable that the police officers had Din and Inocencio, could be located. From there, they saw
ample time to procure a search warrant but did not. and approached two (2) persons along the National
Highway, introducing themselves as police officers. Din
was carrying a light blue plastic bag. When asked, Din
Undeniably, the offense of illegal possession of firearms disclosed that the bag belonged to Nuevas. Fami then
is malum prohibitum but it does not follow that the subject took the bag and upon inspection found inside it
thereof is necessarily illegal per se. Motive is immaterial "marijuana packed in newspaper and wrapped therein."8
in mala prohibita but the subjects of this kind of offense After confiscating the items, Fami and Cabling brought
may not be summarily seized simply because they are Nuevas, Din and Inocencio to the police office at Purok III
prohibited. A search warrant is still necessary. Hence, the for proper documentation.
rule having been violated and no exception being
applicable, the articles seized were confiscated illegally
and are therefore protected by the exclusionary principle. On cross-examination, Cabling testified that the arrest of
They cannot be used as evidence against the petitioners Nuevas was the result of a tip from Fami’s informant,
in the criminal action against them for illegal possession conceding though that the name of Nuevas was not
of firearms. (Roan v. Gonzales, 145 SCRA 689-690 included in the list of persons under surveillance. Fami
[1986]). Besides, assuming that there was indeed a then relayed the tip to Cabling.13 Cabling restated that
search warrant, still in mala prohibita, while there is no Nuevas had voluntarily submitted the plastic bag he was
need of criminal intent, there must be knowledge that the holding and that after Nuevas had been informed of the
same existed. Without the knowledge or voluntariness violation of law attributed to him, he admitted his
there is no crime. willingness to cooperate and point to his other cohorts.14
When Fami and Cabling proceeded to the identified
Criminal case against the petitioners for illegal possession location of Nuevas’s cohorts, they chanced upon Din and
of firearms is DISMISSED Inocencio along the road. Din was holding a bag while
Inocencio was looking into its contents.15 Cabling
averred that Din voluntarily handed the plastic bag he was
PEOPLE v. NUEVAS holding to the police officers.

FACTS: PO3 Teofilo B. Fami (Fami) testified that in the


morning of 27 September 1997, he and SPO3 Cesar B. Nuevas, Din, and Inocencio provided different testimonies
Cabling (Cabling) conducted a stationary surveillance and as to their whereabouts. All three were found guilty as
monitoring of illegal drug trafficking along Perimeter charged and the judgment of conviction was elevated to
Street, Barangay Pag-asa, Olongapo City. They had the Court for automatic review.
received information that a certain male person, more or
less 5’4" in height, 25 to 30 years old, with a tattoo mark
on the upper right hand, and usually wearing a sando and Upon appeal, CA affirmed the RTC’s decision. The
maong pants, would make a delivery of marijuana dried appellate court stated that the search in the instant case
leaves. While stationed thereat, they saw a male person is exempted from the requirement of a judicial warrant as
who fit the description, carrying a plastic bag, later appellants themselves waived their right against
identified as Jesus Nuevas (Nuevas), alight from a motor unreasonable searches and seizures. According to the
vehicle. They accosted Nuevas and informed him that appellate court, both Cabling and Fami testified that Din
they are police officers. Fami asked Nuevas where he voluntarily surrendered the bag. Appellants never
was going. Nuevas answered arrogantly but afterwards, presented evidence to rebut the same. Thus, in the instant
calmed down. Nuevas and Fami conversed in the Waray case, the exclusionary rule does not apply.
dialect. Nuevas informed him that there were other stuff
in the possession of a certain Vangie, an associate, and
two other male persons. Later on, Nuevas voluntarily
Hence, this petition.
pointed to the police officers a plastic bag which, when
opened, contained marijuana dried leaves and bricks
wrapped in a blue cloth. Shortly, in his bid to escape
charges, Nuevas disclosed where the two (2) other male ISSUE: W/N the acts of the accused constitute an
persons would make the delivery of marijuana weighing express waiver.
more or less five (5) kilos.

Fami and Cabling, together with Nuevas, then proceeded


to Purok 12, Old Cabalan, Olongapo City, which
according to Nuevas was where his two (2) companions,
RULING: Nuevas – Yes; Din and Inocencio - No The courts below anchor appellants’ conviction on the
ground that the searches and seizure conducted in the
instant case based on a tip from an informant fall under
Our Constitution states that a search and seizure must be one of the exceptions as Nuevas, Din and Inocencio all
carried through or with a judicial warrant; otherwise, such allegedly voluntarily surrendered the plastic bags
search and seizure becomes "unreasonable" and any containing marijuana to the police officers.38
evidence obtained therefrom is inadmissible for any
purpose in any proceeding.35 The constitutional
proscription, however, is not absolute but admits of We differ.
exceptions, namely:

First, the Court holds that the searches and seizures


1. Warrantless search incidental to a lawful arrest. (Sec. conducted do not fall under the first exception,
12, Rule 126 of the Rules of Court and prevailing warrantless searches incidental to lawful arrests.
jurisprudence);

A search incidental to a lawful arrest is sanctioned by the


2. Search of evidence in "plain view." The elements are: Rules of Court.39 Recent jurisprudence holds that the
(a) a prior valid intrusion based on the valid warrantless arrest must precede the search; the process cannot be
arrest in which the police are legally present in the pursuit reversed as in this case where the search preceded the
of their official duties; (b) the evidence was inadvertently arrest. Nevertheless, a search substantially
discovered by the police who have the right to be where contemporaneous with an arrest can precede the arrest if
they are; (c) the evidence must be immediately apparent; the police have probable cause to make the arrest at the
(d) "plain view" justified mere seizure of evidence without outset of the search. 40
further search;

In this case, Nuevas, Din and Inocencio were not


3. Search of a moving vehicle. Highly regulated by the committing a crime in the presence of the police officers.
government, the vehicle’s inherent mobility reduces Moreover, police officers Fami and Cabling did not have
expectation of privacy especially when its transit in public personal knowledge of the facts indicating that the
thoroughfares furnishes a highly reasonable suspicion persons to be arrested had committed an offense. The
amounting to probable cause that the occupant searches conducted on the plastic bag then cannot be
committed a criminal activity; said to be merely incidental to a lawful arrest. Reliable
information alone is not sufficient to justify a warrantless
arrest under Section 5(a), Rule 113. The rule requires, in
4. Consented warrantless search; addition, that the accused perform some overt act that
would indicate that he "has committed, is actually
committing, or is attempting to commit an offense."41
5. Customs search;

Secondly, neither could the searches be justified under


the plain view doctrine.
6. Stop and Frisk; and

An object is in plain view if it is plainly exposed to sight.


7. Exigent and emergency circumstances.36
Where the object seized was inside a closed package, the
object itself is not in plain view and therefore cannot be
seized without a warrant. However, if the package
In the instances where a warrant is not necessary to effect proclaims its contents, whether by its distinctive
a valid search or seizure, or when the latter cannot be configuration, its transparency, or if its contents are
performed except without a warrant, what constitutes a obvious to an observer, then the contents are in plain view
reasonable or unreasonable search or seizure is purely a and may be seized. In other words, if the package is such
judicial question, determinable from the uniqueness of the that an experienced observer could infer from its
circumstances involved, including the purpose of the appearance that it contains the prohibited article, then the
search or seizure, the presence or absence of probable article is deemed in plain view. It must be immediately
cause, the manner in which the search and seizure was apparent to the police that the items that they observe
made, the place or thing searched and the character of may be evidence of a crime, contraband or otherwise
the articles procured.37 subject to seizure.42
have affirmed Nuevas’s conviction had he not withdrawn
his appeal.
Records show that the dried marijuana leaves were inside
the plastic bags that Nuevas and Din were carrying and
were not readily apparent or transparent to the police
officers. In Nuevas’s case, the dried marijuana leaves However, with respect to the search conducted in the
found inside the plastic bag were wrapped inside a blue case of Din, the Court finds that no such consent had
cloth.43 In Din’s case, the marijuana found upon actually been given.
inspection of the plastic bag was "packed in newspaper
and wrapped therein."44 It cannot be therefore said the
items were in plain view which could have justified mere Jurisprudence requires that in case of consented
seizure of the articles without further search.45 searches or waiver of the constitutional guarantee against
obtrusive searches, it is fundamental that to constitute a
waiver, it must first appear that (1) the right exists; (2) the
On the other hand, the Court finds that the search person involved had knowledge, either actual or
conducted in Nuevas’s case was made with his consent. constructive, of the existence of such right; and (3) the
In Din’s case, there was none. said person had an actual intention to relinquish the right.

Indeed, the constitutional immunity against unreasonable It is worthy to note that in cases where the Court upheld
searches and seizures is a personal right which may be the validity of consented search, the police authorities
waived. However, it must be seen that the consent to the expressly asked, in no uncertain terms, for the consent of
search was voluntary in order to validate an otherwise the accused to be searched. And the consent of the
illegal detention and search, i.e., the consent was accused was established by clear and positive proof.
unequivocal, specific, and intelligently given,
uncontaminated by any duress or coercion. The consent
to a search is not to be lightly inferred, but must be shown Neither can Din’s silence at the time be construed as an
by clear and convincing evidence. The question whether implied acquiescence to the warrantless search. In
a consent to a search was in fact voluntary is a question People v. Burgos,54 the Court aptly ruled:
of fact to be determined from the totality of all the
circumstances. Relevant to this determination are the
following characteristics of the person giving consent and x x x As the constitutional guaranty is not dependent upon
the environment in which consent is given: (1) the age of any affirmative act of the citizen, the courts do not place
the defendant; (2) whether he was in a public or secluded the citizen in the position of either contesting an officer’s
location; (3) whether he objected to the search or authority by force, or waiving his constitutional rights; but
passively looked on; (4) the education and intelligence of instead they hold that a peaceful submission to a search
the defendant; (5) the presence of coercive police or seizure is not a consent or an invitation thereto, but is
procedures; (6) the defendant's belief that no merely a demonstration of regard for the supremacy of the
incriminating evidence will be found; (7) the nature of the law.55
police questioning; (8) the environment in which the
questioning took place; and (9) the possibly vulnerable
subjective state of the person consenting. It is the State
Without the dried marijuana leaves as evidence, Din’s
which has the burden of proving, by clear and positive
conviction cannot be sustained based on the remaining
testimony, that the necessary consent was obtained and
evidence. The Court has repeatedly declared that the
that it was freely and voluntarily given.46
conviction of the accused must rest not on the weakness
of the defense but on the strength of the
prosecution.1awphi1.net56 As such, Din deserves an
In Nuevas’s case, the Court is convinced that he indeed acquittal.
voluntarily surrendered the incriminating bag to the police
officers.
Inspite of any alleged waiver, the dried marijuana leaves
cannot be admitted in evidence against the appellants,
It can be seen that in his desperate attempt to exculpate Din more specifically, as they were seized during a
himself from any criminal liability, Nuevas cooperated with warrantless search which was not lawful. A waiver of an
the police, gave them the plastic bag and even revealed illegal warrantless arrest does not also mean a waiver of
his ‘associates,’ offering himself as an informant. His the inadmissibility of evidence seized during an illegal
actuations were consistent with the lamentable human warrantless arrest.
inclination to find excuses, blame others and save oneself
even at the cost of others’ lives. Thus, the Court would
Turning to Inocencio’s case, the Court likewise finds that with their bags in their patrol car. During the investigation,
he was wrongly convicted of the crime charged. it was discovered that each of the three black travelling
Inocencio’s supposed possession of the dried marijuana bags confiscated from the three accused contained
leaves was sought to be shown through his act of looking eleven bricks of marijuana.
into the plastic bag that Din was carrying.58 Taking a look
at an object, more so in this case peeping into a bag while
held by another, is not the same as taking possession RTC found the accused-appellants guilty as charged. The
thereof. MFR was also denied. CA affirmed RTC’s decision.
Hence, this petition.

Appellants Reynaldo Din y Gonzaga and Fernando


Inocencio y Abadeos are hereby ACQUITTED. ISSUE: W/N the accused-appellants actuations constitute
of waiver (and thus resulted to a valid warrantless search
and arrest).
PEOPLE v. DEQUINA

FACTS: Accused-appellants were charged with violation RULING: Yes.


of Republic Act No. 6425, otherwise known as the
Dangerous Drugs Act of 1972, as amended by Republic
Act No. 7659. That they sell, or offer for sale, deliver or Accused-appellants assail their conviction, asserting that
transport marijuana dried flowering tops with total weight their arrests were illegal.1âwphi1 They were not doing
of thirty two thousand nine hundred ninety five (32,995) anything illegal that would have justified their warrantless
grams which is a prohibited drug. arrest, much less a warrantless search of their persons
and belongings. A search made without a warrant cannot
be justified as an incident of arrest unless the arrest itself
Police Officer III Wilfredo Masanggue testified that at was lawful. Accused-appellants insist that the description
about 6:00 a.m., of September 29, 1999, he and SPO1 of the persons who were transporting marijuana relayed
Anthony Blanco were instructed by their superior, Chief by the Chief of Police to the apprehending officers, PO3
Inspector Romulo Sapitula to proceed at the corner of Masanggue and SPO1 Blanco, was so general that it
Juan Luna and Raxabago Sts., Tondo, Manila, where, could not be sufficient ground for the apprehension of
according to the report given by the informant, three accused-appellants.
persons – a male and two female[s] would be coming from
Baguio City to deliver unknown quantity of marijuana. At
around 9:00 a.m., they noticed a taxi cab coming from The People counters that accused-appellants’ arrests
Yuseco St. heading towards the direction of the pier. At a were lawful as they were then actually committing a crime.
certain point along Raxabago Street, about a hundred Since accused-appellants were lawfully arrested, the
meters away from the position of their patrol car the taxi resulting warrantless search of their persons and
stopped. From it emerged three passengers – a man and belongings was also valid. In addition, accused-
two women – each one of them carrying a black travelling appellants did not refute that they were indeed
bag. As the trio fitted the descriptions given to them by transporting prohibited drugs when they were arrested
Inspector Sapitula, they intently watched and monitored and, instead, alleged as defenses that Dequina acted
their movements. About one or two minutes later, as the under the impulse of uncontrollable fear, and Jundoc and
trio started walking towards the western portion of Jingabo were merely accommodating a trusted childhood
Raxabago St., they drove and trailed them. As the patrol friend.
car got closer behind them, [Dequina] noticed its
presence. She started walking in a more hurried pace
("parang walkathon") as if she wanted to run away After a thorough review of the records, we find that the
("parang patakbo"). SPO1 Blanco alighted from the car judgment of the RTC, as affirmed by the Court of Appeals,
and chased [Dequina] while PO3 Masanggue, who was was supported by the evidence on record. The People
behind the wheels also alighted and restrained [Jundoc] was able to discharge the burden of proving the accused-
and [Jingabo]. While thus trying to get away, [Dequina] appellants’ guilt beyond reasonable doubt.
dropped the bag she was carrying. As a result, the
zipper of the bag gave way. Bundles of dried leaves
wrapped in transparent plastic bags case into view.
Suspecting the stuffs to be marijuana, they further There is no question that the warrantless arrest of
inspected the other two bags in the possession of accused-appellants and the warrantless seizure of the
[Jingabo] and [Jundoc] and found out that they had the marijuana were valid and legal.
same contents. They boarded the three accused, along
Settled is the rule that no arrest, search or seizure can be legally justified, and the following warrantless search of
made without a valid warrant issued by a competent their traveling bags was allowable as incidental to their
judicial authority. The Constitution guarantees the right of lawful arrest.
the people to be secure in their persons, houses, papers
and effects against unreasonable searches and
seizures.13 It further decrees that any evidence obtained Besides, accused-appellants did not raise any protest
in violation of said right shall be inadmissible for any when they, together with their bags containing marijuana,
purpose in any proceeding.14 were brought to the police station for investigation and
subsequent prosecution. In People v. Fernandez,17 we
ruled that:
Nevertheless, the constitutional proscription against
warrantless searches and seizures admits of certain legal
and judicial exceptions, as follows: (1) warrantless search When one voluntarily submits to a search or consents to
incidental to a lawful arrest recognized under Section 12, have it made of his person or premises, he is precluded
Rule 126 of the Rules of Court and by prevailing from later complaining thereof. x x x. The right to be
jurisprudence; (2) seizure of evidence in plain view; (3) secure from unreasonable search may, like every right,
search of a moving vehicle; (4) consented warrantless be waived and such waiver may be made either expressly
search; (5) customs search; (6) stop and frisk; and (7) or impliedly.18
exigent and emergency circumstances.15

In order to exonerate herself from criminal liability,


On the other hand, Section 5, Rule 113 of the Rules of Dequina contends that she transported the marijuana
Court provides that a lawful arrest without a warrant may under the compulsion of an irresistible fear. Jundoc and
be made by a peace officer or a private person under the Jingabo, on the other hand, claim that they went along to
following circumstances: accommodate Dequina, a trusted childhood friend.

(a) When, in his presence, the person to be arrested has We are unconvinced.
committed, is actually committing, or is attempting to
commit an offense;
A person who acts under the compulsion of an irresistible
force, like one who acts under the impulse of an
(b) When an offense has just been committed, and he has uncontrollable fear of equal or greater injury, is exempt
probable cause to believe based on personal knowledge from criminal liability because he does not act with
of facts or circumstances that the person to be arrested freedom. Actus me invito factus non est meus actus. An
has committed it; and act done by me against my will is not my act. The force
contemplated must be so formidable as to reduce the
actor to a mere instrument who acts not only without will
(c) When the person to be arrested is a prisoner who has but against his will. The duress, force, fear or intimidation
escaped from a penal establishment or place where he is must be present, imminent and impending, and of such
serving final judgment or is temporarily confined while his nature as to induce a well-grounded apprehension of
case is pending, or has escaped while being transferred death or serious bodily harm if the act be done. A threat
from one confinement to another. of future injury is not enough. The compulsion must be of
such a character as to leave no opportunity for the
accused for escape or self-defense in equal combat.19
"Transport" as used under the Dangerous Drugs Act is Here, Dequina’s version of events that culminated with
defined to mean "to carry or convey from one place to her and Jundoc and Jingabo’s arrests on September 29,
another."16 The evidence in this case shows that at the 1999 is implausible. Equally far-fetched is Jundoc and
time of their arrest, accused-appellants were caught in Jingabo’s assertion of blind trust in Dequina and total
flagrante carrying/transporting dried marijuana leaves in ignorance of the transportation of marijuana.
their traveling bags. PO3 Masanggue and SPO1 Blanco
need not even open Dequina’s traveling bag to determine
its content because when the latter noticed the police Accused-appellants guilty of the crime of illegal transport
officers’ presence, she walked briskly away and in her of marijuana
hurry, accidentally dropped her traveling bag, causing the
F. SEARCH OF WAREHOUSE IN VIOLATION OF CUSTOMS
zipper to open and exposed the dried marijuana bricks AND TARIFF CODE OR TO ENFORCE CUSTOMS LAWS
therein. Since a crime was then actually being committed
by the accused-appellants, their warrantless arrest was
BOC v. OGARIO RTC held that the Warrant of Seizure and Detention
issued by the Bureau of Customs cannot divest this court
of jurisdiction since its issuance is without legal basis as
FACTS: On December 9, 1998, Felipe A. Bartolome,
it was anchored merely on suspicion that the items in
District Collector of Customs of Cebu, issued a Warrant
question were imported or smuggled. It is very clear that
of Seizure and Detention1 of 25,000 bags of rice, bearing
the defendants are bereft of any evidence to prove that
the name of "SNOWMAN, Milled in Palawan" shipped on
the goods were indeed imported or smuggled, which is
board the M/V "Alberto," which was then docked at Pier 6
why the plaintiffs have very vigorously protested against
in Cebu City. The warrant was issued on the basis of the
the seizure of cargoes by the defendants. In fact, as
report of the Economic Intelligence and Investigation
revealed by defendants counsel, the Warrant of Seizure
Bureau (EIIB), Region VII that the rice had been illegally
and Detention was issued merely to shift the burden of
imported. The report stated that the rice was landed in
proof to the shippers or owners of the goods to prove that
Palawan by a foreign vessel and then placed in sacks
the bags of rice were not imported or smuggled. However,
marked "SNOWMAN, Milled in Palawan." It was then
the court feels this is unfair because the settled rule is that
shipped to Cebu City on board the vessel M/V "Alberto."
he who alleges must prove the same. Besides, at this time
when our economy is not good, it would be a [dis]service
to the nation to use the strong arm of the law to make
On December 10, 1998, respondent Mark Montelibano, things hard or difficult for the businessmen. The 25,000
the consignee of the sacks of rice, and his buyer, bags of rice were ordered returned to respondents upon
respondent Elson Ogario, filed a complaint for injunction the posting by them of an P8,000,000.00 bond.
in the RTC alleging that the defendants rushed to the port
with long arms commanding the plaintiffs laborer[s] to
stopped [sic] the unloading of the same from the vessel
CA affirmed the decision of the RTC.
named M/V Alberto alleging that the said rice was
smuggled without even proof that the same were [sic]
purchased from a particular country; that by the mere
suspicion of the defendants that the goods were In the forfeiture proceedings before the Collector of
smuggled from abroad, they immediately put on hold the Customs of Cebu, it was decreed that the 25,000 sacks
release of the goods from the ship and at the same time of rice be released to the owners. Since this decision
they jointly barred unloading and loading activities of the involves the release of some of the articles subject matter
plaintiffs laborers of the herein-mentioned rice. The of herein case which is considered adverse to the
plaintiffs then presented all the pertinent and necessary government, the same is hereby elevated to the
documents to all of the defendants but the latter refused Commissioner of Customs for automatic review pursuant
to believe that the same is from Palawan because their to Republic Act 7651. The District Collector of Customs
minds are closed due to some reason or another [while] found "strong reliable, and convincing evidence" that the
the plaintiffs believed that the same is merely an act of 25,000 bags of rice were smuggled. Said evidence
harassment. That the acts of the defendants in stopping consisted of certifications by the Philippine Coast Guard,
the loading and unloading activities of the plaintiffs the Philippine Ports Authority, and the Arrastre
laborers [have] no basis in law and in fact; thus, unlawful Stevedoring Office in Palawan that M/V "Alberto" had
and illegal. A mere suspicion which is not coupled with never docked in Palawan since November, 1998; a
any proof or evidence to that effect is [a] matter which the certification by Officer-in-Charge Elenita Ganelo of the
law prohibits. That a Warrant of Seizure and detention National Food Authority (NFA) Palawan that her signature
issued by the Collector of Custom[s] dated December 9, in NFA Grains Permit Control No. 00986, attesting that the
1998 be quashed because the defendants act of seizing 25,000 bags of rice originated from Palawan, was forged;
and detaining the herein-mentioned sacks of rice are and the result of the laboratory analysis of a sample of the
illegal. The continuing act of detaining the herein- subject rice by the International Rice Research Institute
mentioned sacks of rice will lead to the deterioration of the (IRRI) stating that the sample "does not compare with any
same. That no public auction sale of the same should be of our IRRI released varieties."
conducted by the Bureau of Custom[s] or any government
agenc[y].
Respondent Montelibano did not take part in the
proceedings before the District Collector of Customs
Petitioners Bureau of Customs (BOC), Port of Cebu3 and despite due notice sent to his counsel because he refused
the EIIB, as well as the Philippine Navy and Coast Guard, to recognize the validity of the forfeiture proceedings.
sought the dismissal of the complaint on the ground that
the RTC had no jurisdiction, but their motions were
denied. Hence, this petition.
ISSUE: W/N RTC is vested with jurisdiction over seizure jurisdiction through seizure/forfeiture proceedings."11
ad forfeiture of dutiable goods (in this case are sacks of They overlook the fact, however, that under the law, the
rice). question of whether probable cause exists for the seizure
of the subject sacks of rice is not for the Regional Trial
Court to determine. The customs authorities do not have
RULING: No. to prove to the satisfaction of the court that the articles on
board a vessel were imported from abroad or are intended
to be shipped abroad before they may exercise the power
to effect customs searches, seizures, or arrests provided
In Jao v. Court of Appeals,10 this Court, reiterating its
by law and continue with the administrative hearings.12
ruling in a long line of cases, said:
As the Court held in Ponce Enrile v.
Vinuya:13cräläwvirtualibräry

There is no question that Regional Trial Courts are devoid


of any competence to pass upon the validity or regularity
The governmental agency concerned, the Bureau of
of seizure and forfeiture proceedings conducted by the
Customs, is vested with exclusive authority. Even if it be
Bureau of Customs and to enjoin or otherwise interfere
assumed that in the exercise of such exclusive
with these proceedings. The Collector of Customs sitting
competence a taint of illegality may be correctly imputed,
in seizure and forfeiture proceedings has exclusive
the most that can be said is that under certain
jurisdiction to hear and determine all questions touching
circumstances the grave abuse of discretion conferred
on the seizure and forfeiture of dutiable goods. The
may oust it of such jurisdiction. It does not mean however
Regional Trial Courts are precluded from assuming
that correspondingly a court of first instance is vested with
cognizance over such matters even through petitions of
competence when clearly in the light of the above
certiorari, prohibition or mandamus.
decisions the law has not seen fit to do so. The
proceeding before the Collector of Customs is not final.
An appeal lies to the Commissioner of Customs and
It is likewise well-settled that the provisions of the Tariff thereafter to the Court of Tax Appeals. It may even reach
and Customs Code and that of Republic Act No. 1125, as this Court through the appropriate petition for review. The
amended, otherwise known as "An Act Creating the Court proper ventilation of the legal issues raised is thus
of Tax Appeals," specify the proper fora and procedure for indicated. Certainly a court of first instance is not therein
the ventilation of any legal objections or issues raised included. It is devoid of jurisdiction.
concerning these proceedings. Thus, actions of the
Collector of Customs are appealable to the Commissioner
of Customs, whose decision, in turn, is subject to the
Court categorically declared inn v. Natividad (213 SCRA
exclusive appellate jurisdiction of the Court of Tax
734, 742 [1992]) that "[b]y express provision of law, amply
Appeals and from there to the Court of Appeals.
supported by well-settled jurisprudence, the Collector of
Customs has exclusive jurisdiction over seizure and
forfeiture proceedings, and regular courts cannot interfere
The rule that Regional Trial Courts have no review powers with his exercise thereof or stifle or put it to naught."
over such proceedings is anchored upon the policy of
placing no unnecessary hindrance on the governments
drive, not only to prevent smuggling and other frauds
The decision, dated April 15, 1999, of the Court of
upon Customs, but more importantly, to render effective
Appeals is REVERSED and Civil Case No. CEB-23077 in
and efficient the collection of import and export duties due
the Regional Trial Court, Branch 5, Cebu City is
the State, which enables the government to carry out the
DISMISSED.
functions it has been instituted to perform.

SO ORDERED.
Even if the seizure by the Collector of Customs were
illegal, which has yet to be proven, we have said that such
act does not deprive the Bureau of Customs of jurisdiction
thereon.

Respondents cite the statement of the Court of Appeals


that regular courts still retain jurisdiction "where, as in this
case, for lack of probable cause, there is serious doubt as
to the propriety of placing the articles under Customs
RIETA v. PEOPLE Hall. He learned that Boy will haul household appliances
from Divisoria. They boarded a jeep driven by Boy and
they proceeded to Cartimar, Pasay City. While they were
FACTS: The accused fraudulently import or bring into the
along Roxas Boulevard near the Daily Express Building,
Philippines or assist in so doing contrary to law, three
two (2) vehicles intercepted them and ordered them to
hundred five (305) cases of assorted brands of blue seal
pull-over. The passengers of the said vehicles introduced
cigarettes which are foreign articles valued at
themselves as Metrocom soldiers, and ordered them to
P513,663.47 including duties and taxes, and/or buy, sell,
alight and to raise their hands while poking guns at them.
transport or assist and facilitate the buying, selling and
They were ordered to l[ie down] flat on their belly on the
transporting of the above-named foreign articles after
pavement and were bodily frisked and searched. The
importation knowing the same to have been imported
Metrocom soldiers did not find anything from their bodies.
contrary to law which was found in the possession of said
Thereafter, they (Rieta, Rimorin and Gonzalo) were
accused and under their control which articles said
ordered by the Metrocom soldiers to transfer to a jeep.
accused fully well knew have not been properly declared
While they were aboard the jeep, he overheard from the
and that the duties and specific taxes thereon have not
Metrocom soldiers that their driver was able to escape.
been paid to the proper authorities in violation of said Sec.
Likewise, they were also informed by the Metrocom
3601 of the Tariff and Customs Code of the Philippines,
soldiers that the cargo truck was loaded with blue seal
as amended by Presidential Decree No. 34, in relation to
cigarettes. The cargo truck was not opened in their
Sec. 3602 of said Code and Sec. 184 of the National
presence, nor were the contents thereof shown to them
Internal Revenue Code."7
upon their apprehension.

Col. Panfilo Lacson, the[n] Chief of the Police Intelligence


RTC found the petitioner guilty. This was affirmed by the
Branch of the Metrocom Intelligence and Security Group
CA. Hence, this petition.
(MISG for brevity), received information that certain
syndicated groups were engaged in smuggling activities
somewhere in Port Area, Manila. It was further revealed
that the activities [were being] done at nighttime and the ISSUE: W/N there was a valid warrantless search.
smuggled goods in a delivery panel and delivery truck
[were] being escorted by some police and military
personnel. At around 5 minutes before 4:00 o'clock that RULING: Yes.
morning, a green cargo truck with Plate No. T-SY-167
came out from the 2nd COSAC Detachment followed and
escorted closely by a light brown Toyota Corona car with Petitioner contends that his arrest by virtue of Arrest
Plate No. GR-433 and with 4 men on board. At that time, Search and Seizure Order (ASSO) No. 4754 was invalid,
Lt. Col. Panfilo Lacson had no information whatsoever as the law upon which it was predicated -- General Order
about the car, so he gave an order by radio to his men to No. 60, issued by then President Ferdinand E. Marcos --
intercept only the cargo truck. The cargo truck was was subsequently declared by the Court, in Tañada v.
intercepted. Col. Lacson noticed that the Toyota car Tuvera,33 to have no force and effect. Thus, he asserts,
following the cargo truck suddenly made a sharp U-turn any evidence obtained pursuant thereto is inadmissible in
towards the North, unlike the cargo truck [that] was going evidence.
south. Almost by impulse, Col. Lacson's car also made a
U-turn and gave chase to the speeding Toyota car. The
chase lasted for less than 5 minutes until said car made a
stop along Bonifacio Drive, at the foot of Del Pan Bridge. We do not agree. In Tañada, the Court addressed the
Col. Lacson and his men searched the car and they found possible effects of its declaration of the invalidity of
several firearms, particularly: three (3) .45 cal. Pistols and various presidential issuances. Discussing therein how
one (1) armalite M-16 rifle. When the cargo truck with such a declaration might affect acts done on a
Plate No. T-SY-167 was searched, 305 cases of blue seal presumption of their validity, the Court said:
or untaxed cigarettes were found inside. The cargo truck
driver known only as 'Boy' was able to escape while the
other passengers or riders of said truck were "x x x. In similar situations in the past this Court had taken
apprehended, namely: Police Sgt. Arturo Rimorin of the pragmatic and realistic course set forth in Chicot
Pasay City Police Force, Pat. Felicisimo Rieta of Kawit County Drainage District vs. Baxter Bank to wit:
Police Force, and Gonzalo Vargas, a civilian.

'The courts below have proceeded on the theory that the


According to the petitioner, in the early morning of Act of Congress, having been found to be
October 15, 1979 he met Boy in front of the Kawit Town unconstitutional, was not a law; that it was inoperative,
conferring no rights and imposing no duties, and hence has personally determined the existence of probable
affording no basis for the challenged decree. x x x It is cause.37
quite clear, however, that such broad statements as to the
effect of a determination of unconstitutionality must be
taken with qualifications. The actual existence of a Under the Tariff and Customs Code, a search, seizure
statute, prior to [the determination of its invalidity], is an and arrest may be made even without a warrant for
operative fact and may have consequences which cannot purposes of enforcing customs and tariff laws. Without
justly be ignored. The past cannot always be erased by a mention of the need to priorly obtain a judicial warrant, the
new judicial declaration. The effect of the subsequent Code specifically allows police authorities to enter, pass
ruling as to invalidity may have to be considered in various through or search any land, enclosure, warehouse, store
aspects –with respect to particular conduct, private and or building that is not a dwelling house; and also to
official. Questions of rights claimed to have become inspect, search and examine any vessel or aircraft and
vested, of status, of prior determinations deemed to have any trunk, package, box or envelope or any person on
finality and acted upon accordingly, of public policy in the board; or to stop and search and examine any vehicle,
light of the nature both of the statute and of its previous beast or person suspected of holding or conveying any
application, demand examination. These questions are dutiable or prohibited article introduced into the
among the most difficult of those which have engaged the Philippines contrary to law.38
attention of courts, state and federal, and it is manifest
from numerous decisions that an all-inclusive statement
of a principle of absolute retroactive invalidity cannot be
SALVADOR v. PEOPLE
justified.'

FACTS: On the wee hours of June 4, 1994, Aurelio


xxxx xx xxx Mandin, Danilo Santos and petitioner Tomas Salvador,
then aircraft mechanics employed by the Philippine Air
Lines (PAL) and assigned at the Ninoy Aquino
International Airport (NAIA) and Manila Domestic Airport,
"Similarly, the implementation/enforcement of
were nabbed by intelligence operatives of the Philippine
presidential decrees prior to their publication in the Official
Air Force (PAF) for possessing thirteen (13) packets
Gazette is 'an operative fact which may have
containing assorted smuggled watches and jewelries
consequences which cannot be justly ignored. The past
valued at more than half a million pesos. They were
cannot always be erased by a new judicial declaration x x
charged before the RTC with violation of Section 3601 of
x that an all-inclusive
the Tariff and Customs Code.

statement of a principle of absolute retroactive invalidity


198 pieces of means watches…………… ₱187,110.00
cannot be justified.'"34

76 pieces of men’s diving watches……… 8,640.00


The Chicot doctrine cited in Tañada advocates that, prior
to the nullification of a statute, there is an imperative
necessity of taking into account its actual existence as an
operative fact negating the acceptance of "a principle of 32 pieces of ladies watches……………… 11,600.00
absolute retroactive invalidity." Whatever was done while
the legislative or the executive act was in operation should
be duly recognized and presumed to be valid in all 1600 grams of assorted jewelry…………. 322,000.00
respects.35 The ASSO that was issued in 1979 under
General Order No. 60 -- long before our Decision in
Tañada and the arrest of petitioner -- is an operative fact with a total market value of ₱537,500.00
that can no longer be disturbed or simply ignored.

Prosecution FACTS: Special Mission Group from the PAF


Furthermore, the search and seizure of goods, suspected Special Operations Squadron conducted routine
to have been introduced into the country in violation of surveillance operations at the Manila Domestic Airport to
customs laws, is one of the seven doctrinally accepted check on reports of alleged drug trafficking and smuggling
exceptions36 to the constitutional provision. Such being facilitated by certain PAL personnel. Major
provision mandates that no search or seizure shall be Pagcaliuangan then ordered Sgts. Teves and Ople to
made except by virtue of a warrant issued by a judge who keep close watch on the second airplane parked inside
the Domestic Airport terminal. This aircraft is an Airbus
300 with tail number RPC-3001. At 12:15 a.m. the People v. Burgos,9 he maintains that at the time he and
following day (June 4), Sgt. Teves reported that the three his co-accused were stopped by the PAF law enforces,
(3) persons who earlier boarded the Airbus 300 had they were unaware that a crime was being committed.
disembarked with their abdominal areas bulging. They Accordingly, the law enforcers were actually engaged in
then boarded an airplane tow truck with its lights off. The a fishing expedition in violation of his Constitutional right
PAF surveillance team promptly boarded their vehicles against unlawful search and seizure. Thus, the seized
and followed the aircraft tow truck. At the Lima Gate of the items should not have been admitted in evidence against
Domestic Airport, the team blocked and stopped the tow him.
truck. Sgt. Teves then got off, identified himself and asked
the four (4) persons on board to alight. They were later
identified as Tomas Salvador, petitioner, Aurelio Mandin, The Office of the Solicitor General (OSG) counters that
Danilo Santos and Napoleon Clamor, the driver of the tow under the factual circumstances of the case at bar, there
truck. Sgt. Teves approached Aurelio Mandin. He noticed was sufficient probable cause for the PAF surveillance
that Mandin’s uniform was partly open, showing a girdle. team to stop and search petitioner and his companions.
While Sgt. Teves was reaching for the girdle, a package They boarded the parked Air Bus 300 PAL plane at the
wrapped in brown packaging tape fell. Suspecting that the time when there were no other PAL personnel working
package contained smuggled items, Sgt. Teves yelled to therein. They stayed inside the plane for some time and
his teammates, "Positive!" Thereupon, the rest of the surprisingly, came out with bulging waists. They then
team surrounded petitioner and his two co-accused who stopped and looked around and made apparent signals.
surrendered without a fight. The team searched their All these acts were sufficient to engender a reasonable
bodies and found that the three were wearing girdles suspicion that petitioner and his colleagues were up to
beneath their uniforms, all containing packets wrapped in something illegal. Moreover, the search and seizure was
packaging tape. Mandin yielded five (5) packets, while conducted in connection with the enforcement of customs
petitioner and Santos had four (4) each. The team law when the petitioner and his co-accused were riding a
confiscated the packets and brought all the accused to the motor vehicle. In addition, the search was conducted at
PAFSECOM Office. the vicinity of Lima Gate of the Manila Domestic Airport
which, like every gate in the airport perimeter, has a
checkpoint. Finally, the petitioner and his companions
At around 8:00 o’clock the following morning, Emilen agreed to the search after one of them was caught with a
Balatbat, an examiner of the Bureau of Customs, arrived suspicious-looking packet. Under these circumstances,
at the PAFSECOM Office. She opened one of the packets the search and seizure is legal and the seized items are
and on seeing that it contained dutiable goods, she admissible in evidence.
proceeded to weigh the thirteen (13) packets seized from
the accused. She then prepared an inventory of the items
seized and listed the weight of the packets.4 Thereafter, We agree with the OSG.
she brought the seized packets to the In-Board Section,
Bureau of Customs, Airport Office where their contents
were identified and appraised. The Bureau of Customs
As a rule, the Bill of Rights prohibits intrusions by the law
found 248 pieces of assorted watches and fourteen karat
enforcers to a person’s body, personal effects or
(14K) gold jewelries.
residence, unless the same are conducted pursuant to a
valid search warrant issued in compliance with the
procedure mandated by the Constitution and the Rules of
All these allegations were denied by the accused stating Court.
that they were being framed up by the military.

Our jurisprudence provides for privileged areas where


RTC found the accused guilty of the charge. CA affirmed searches and seizures may lawfully be effected sans a
the decision. Hence, this petition. search warrant. These recognized exceptions include: (1)
search of moving vehicles; (2) search in plain view; (3)
customs searches; (4) waiver or consented searches; (5)
ISSUE: W/N there was a valid warrantless search. stop-and-frisk situations; and (6) search incidental to a
lawful arrest.10

RULING: Yes.
Here, it should be noted that during the incident in
question, the special mission of the PAF operatives was
Petitioner contends that the warrantless search and to conduct a surveillance operation to verify reports of
seizure conducted by the PAF operatives is illegal. Citing drug trafficking and smuggling by certain PAL personnel
in the vicinity of the airport. In other words, the search G. EXIGENCY
made by the PAF team on petitioner and his co-accused
was in the nature of a customs search. As such, the team PEOPLE v. DE GRACIA
properly effected the search and seizure without a search
warrant since it exercised police authority under the
customs law.11 FACTS: The incidents involved in this case took place at
the height of the coup d' etat staged in December, 1989
by ultra-rightist elements headed by the Reform the
Armed Forces Movement-Soldiers of the Filipino People
In Papa vs. Mago12 involving a customs search, we held
(RAM-SFP) against the Government. At that time, various
that law enforcers who are tasked to effect the
government establishments and military camps in Metro
enforcement of the customs and tariff laws are authorized
Manila were being bombarded by the rightist group with
to search and seize, without a search warrant, any article,
their "tora-tora" planes. At around midnight of November
cargo or other movable property when there is reasonable
30, 1989, the 4th Marine Battalion of the Philippine
cause to suspect that the said items have been introduced
Marines occupied Villamor Air Base, while the Scout
into the Philippines in violation of the tariff and customs
Rangers took over the Headquarters of the Philippine
law. They may likewise conduct a warrantless search of
Army, the Army Operations Center, and Channel 4, the
any vehicle or person suspected of holding or conveying
government television station. Also, some elements of the
the said articles, as in the case at bar.
Philippine Army coming from Fort Magsaysay occupied
the Greenhills Shopping Center in San Juan, Metro
Manila. Accused-appellant Rolando de Gracia was
In short, Mago clearly recognizes the power of the State charged in two separate informations for illegal
to foil any fraudulent schemes resorted to by importers possession of ammunition and explosives in furtherance
who evade payment of customs duties. The of rebellion, and for attempted homicide. In Criminal Case
Government’s policy to combat the serious malady of No. Q-90-11755, Rolando de Gracia, Chito Henson and
smuggling cannot be reduced to futility and impotence on several John Does whose true names and identities have
the ground that dutiable articles on which the duty has not not as yet been ascertained, were charged with the crime
been paid are entitled to the same Constitutional of illegal possession of ammunition and explosives in
protection as an individual’s private papers and effects. furtherance of rebellion, penalized under Section 1,
Here, we see no reason not to apply this State policy paragraph 3, of Presidential Decree No. 1866.
which we have continued to affirm.

Five (5) bundles of C-4 or dynamites


Moreover, we recall that at the time of the search,
petitioner and his co-accused were on board a moving Six (6) cartoons of M-16 ammunition at 20 each
PAL aircraft tow truck. As stated earlier, the search of a
One hundred (100) bottles of MOLOTOV bombs
moving vehicle is recognized in this jurisdiction as a valid
exception to the requirement for a search warrant. Such
exception is easy to understand. A search warrant may
readily be obtained when the search is made in a store, In Criminal Case No. Q-90-11756, Rolando de Gracia,
dwelling house or other immobile structure. But it is Chito Henson, Lamberto Bicus, Rodolfo Tor and several
impracticable to obtain a warrant when the search is John Does were charged with attempted homicide
conducted in a mobile ship, aircraft or other motor vehicle allegedly committed on December 1, 1989 in Quezon City
since they can quickly be moved out of the locality or upon the person of Crispin Sagario who was shot and hit
jurisdiction where the warrant must be sought.14 Verily, on the right thigh.
we rule that the Court of Appeals committed no reversible
error in holding that the articles involved in the instant
controversy were validly seized by the authorities even Appellant was convicted for illegal possession of firearms
without a search warrant, hence, admissible in evidence in furtherance of rebellion, but was acquitted of attempted
against petitioner and his co-accused. homicide.

The records show that in the early morning of December


1, 1989, Maj. Efren Soria of the Intelligence Division,
National Capital Region Defense Command, was on
board a brown Toyota car conducting a surveillance of the
Eurocar Sales Office. The surveillance, which actually
started on the night of November 30, 1989 at around
10:00 P.M., was conducted pursuant to an intelligence
report received by the division that said establishment surveillance team was fired at by a group of men coming
was being occupied by elements of the RAM-SFP as a from the Eurocar building. When the military operatives
communication command post. Sagario was tasked to raided the place, the occupants thereof refused to open
park the car arounf 10-15 meters away from the building. the door despite requests for them to do so, thereby
A group of five men disengaged themselves from the compelling the former to break into the office. 17 The
crowd and walked towards the car of the surveillance Eurocar Sales Office is obviously not a gun store and it is
team. At that moment, Maj. Soria, who was then seated definitely not an armory or arsenal which are the usual
in front, saw the approaching group and immediately depositories for explosives and ammunition. It is primarily
ordered Sgt. Sagario to start the car and leave the area. and solely engaged in the sale of automobiles. The
As they passed by the group, then only six meters away, presence of an unusual quantity of high-powered firearms
the latter pointed to them, drew their guns and fired at the and explosives could not be justifiably or even colorably
team, which attack resulted in the wounding of Sgt. explained. In addition, there was general chaos and
Sagario on the right thigh. Nobody in the surveillance disorder at that time because of simultaneous and intense
team was able to retaliate because they sought cover firing within the vicinity of the office and in the nearby
inside the car and they were afraid that civilians or Camp Aguinaldo which was under attack by rebel forces.
bystanders might be caught in the cross-fire. As a 18 The courts in the surrounding areas were obviously
consequence, at around 6:30 A.M. of December 5, 1989, closed and, for that matter, the building and houses
a searching team raided the Eurocar Sales Office. They therein were deserted.
were able to find and confiscate six cartons of M-16
ammunition, five bundles of C-4 dynamites, M-shells of Under the foregoing circumstances, it is our considered
different calibers, and "molotov" bombs inside one of the opinion that the instant case falls under one of the
rooms belonging to a certain Col. Matillano which is exceptions to the prohibition against a warrantless
located at the right portion of the building. As a result of search. In the first place, the military operatives, taking
the raid, the team arrested appellant, as well as Soprieso into account the facts obtaining in this case, had
Verbo and Roberto Jimena who were janitors at the reasonable ground to believe that a crime was being
Eurocar building. No search warrant was secured by the committed. There was consequently more than sufficient
raiding team because, according to them, at that time probable cause to warrant their action. Furthermore,
there was so much disorder considering that the nearby under the situation then prevailing, the raiding team had
Camp Aguinaldo was being mopped up by the rebel no opportunity to apply for and secure a search warrant
forces and there was simultaneous firing within the vicinity from the courts. The trial judge himself manifested that on
of the Eurocar office, aside from the fact that the courts December 5, 1989 when the raid was conducted, his court
were consequently closed. was closed. 19 Under such urgency and exigency of the
moment, a search warrant could lawfully be dispensed
with.

Accused De Gracia denied all the allegations against him In addition, we find the principle enunciated in Umil, et al.,
implying that his arrest was maliciously made due to the vs. Ramos,
prior conflict between Col. Matillano and the prosecution
witness, and the fact that he is working for Matillano. et al., 21 applicable, by analogy, to the present case:

RTC found De Gracia not guilty of attempted homicide but Absence of a judicial warrant is no legal impediment to
guilty of the offense of illegal possession of firearms in arresting or capturing persons committing overt acts of
furtherance of rebellion. violence against government forces, or any other milder
acts but really in pursuance of the rebellious movement.
That judgment of conviction is now challenged before us The arrest or capture is thus impelled by the exigencies
in this appeal. of the situation that involves the very survival of society
and its government and duly constituted authorities. If
killing and other acts of violence against the rebels find
ISSUE: W/N there was a valid search and seizure. justification in the exigencies of armed hostilities which
(are) of the essence of waging a rebellion or insurrection,
most assuredly so in case of invasion, merely seizing their
persons and detaining them while any of these
RULING: Yes.
contingencies continues cannot be less justified.

It is admitted that the military operatives who raided the


RTC decision hereby affirmed.
Eurocar Sales Office were not armed with a search
warrant at that time. 15 The raid was actually precipitated
by intelligence reports that said office was being used as
headquarters by the RAM. 16 Prior to the raid, there was
a surveillance conducted on the premises wherein the
H. SEARCH AND SEIZURE BY PRIVATE PERSONS should be held inadmissible in evidence (Sec. 3 (2), Art.
III).

PEOPLE v. MARTI
ISSUE: W/N the contention of the appellant is tenable.
FACTS: Appellant Marti and his common-law wife,
Shirley Reyes, went to the booth of the "Manila Packing
and Export Forwarders". Anita Reyes (the proprietress RULING: No.
and no relation to Shirley Reyes) attended to them. The
appellant informed Anita Reyes that he was sending the
packages to a friend in Zurich, Switzerland. Anita Reyes It must be noted, however, that in all those cases adverted
then asked the appellant if she could examine and inspect to, the evidence so obtained were invariably procured by
the packages. Appellant, however, refused, assuring her the State acting through the medium of its law enforcers
that the packages simply contained books, cigars, and or other authorized government agencies.
gloves and were gifts to his friend in Zurich. In view of
appellant's representation, Anita Reyes no longer insisted
on inspecting the packages.
In the case at bar, it assumes a peculiar character since
the evidence sought to be excluded was primarily
discovered and obtained by a private person, acting in a
Before delivery of appellant's box to the Bureau of private capacity and without the intervention and
Customs and/or Bureau of Posts, Mr. Job Reyes participation of State authorities. Under the
(proprietor) and husband of Anita (Reyes), following circumstances, can accused/appellant validly claim that
standard operating procedure, opened the boxes for final his constitutional right against unreasonable searches
inspection. When he opened appellant's box, a peculiar and seizure has been violated? Stated otherwise, may an
odor emitted therefrom. His curiousity aroused, he act of a private individual, allegedly in violation of
squeezed one of the bundles allegedly containing gloves appellant's constitutional rights, be invoked against the
and felt dried leaves inside. Opening one of the bundles, State?
he pulled out a cellophane wrapper protruding from the
opening of one of the gloves. He made an opening on one
of the cellophane wrappers and took several grams of the
We hold in the negative. In the absence of governmental
contents thereof. Job Reyes forthwith prepared a letter
interference, the liberties guaranteed by the Constitution
reporting the shipment to the NBI and requesting a
cannot be invoked against the State.
laboratory examination of the samples he extracted from
the cellophane wrapper. Job Reyes brought out the box
in which appellant's packages were placed and, in the
presence of the NBI agents, opened the top flaps, In the case of Bernas v. US (373 F.2d 517 (1967). The
removed the styro-foam and took out the cellophane Court there said:
wrappers from inside the gloves. Dried marijuana leaves
were found to have been contained inside the cellophane
wrappers. The package which allegedly contained books The search of which appellant complains, however, was
was likewise opened by Job Reyes. He discovered that made by a private citizen — the owner of a motel in which
the package contained bricks or cake-like dried marijuana appellant stayed overnight and in which he left behind a
leaves. The package which allegedly contained travel case containing the evidence*** complained of. The
tabacalera cigars was also opened. It turned out that dried search was made on the motel owner's own initiative.
marijuana leaves were neatly stocked underneath the Because of it, he became suspicious, called the local
cigars. police, informed them of the bag's contents, and made it
available to the authorities.

An Information was filed against appellant for violation of


RA 6425, otherwise known as the Dangerous Drugs Act. The fourth amendment and the case law applying it do not
After trial, the court a quo rendered the assailed decision. require exclusion of evidence obtained through a search
by a private citizen. Rather, the amendment only
proscribes governmental action."
On appeal, appellant contends that the evidence subject
of the imputed offense had been obtained in violation of
his constitutional rights against unreasonable search and The contraband in the case at bar having come into
seizure and privacy of communication (Sec. 2 and 3, Art. possession of the Government without the latter
III, Constitution) and therefore argues that the same transgressing appellant's rights against unreasonable
search and seizure, the Court sees no cogent reason why
the same should not be admitted against him in the
prosecution of the offense charged. First, the general reflections. The protection of
fundamental liberties in the essence of constitutional
democracy. Protection against whom? Protection against
the state. The Bill of Rights governs the relationship
Appellant, however, would like this court to believe that between the individual and the state. Its concern is not the
NBI agents made an illegal search and seizure of the relation between individuals, between a private individual
evidence later on used in prosecuting the case which and other individuals. What the Bill of Rights does is to
resulted in his conviction. declare some forbidden zones in the private sphere
inaccessible to any power holder. (Sponsorship Speech
of Commissioner Bernas , Record of the Constitutional
The postulate advanced by accused/appellant needs to Commission, Vol. 1, p. 674; July 17, 1986; Emphasis
be clarified in two days. In both instances, the argument supplied)
stands to fall on its own weight, or the lack of it.

The constitutional proscription against unlawful searches


First, the factual considerations of the case at bar readily and seizures therefore applies as a restraint directed only
foreclose the proposition that NBI agents conducted an against the government and its agencies tasked with the
illegal search and seizure of the prohibited merchandise. enforcement of the law. Thus, it could only be invoked
Records of the case clearly indicate that it was Mr. Job against the State to whom the restraint against arbitrary
Reyes, the proprietor of the forwarding agency, who made and unreasonable exercise of power is imposed.
search/inspection of the packages. Said inspection was
reasonable and a standard operating procedure on the
part of Mr. Reyes as a precautionary measure before If the search is made upon the request of law enforcers,
delivery of packages to the Bureau of Customs or the a warrant must generally be first secured if it is to pass the
Bureau of Posts. test of constitutionality. However, if the search is made at
the behest or initiative of the proprietor of a private
establishment for its own and private purposes, as in the
It will be recalled that after Reyes opened the box case at bar, and without the intervention of police
containing the illicit cargo, he took samples of the same authorities, the right against unreasonable search and
to the NBI and later summoned the agents to his place of seizure cannot be invoked for only the act of private
business. Thereafter, he opened the parcel containing the individual, not the law enforcers, is involved. In sum, the
rest of the shipment and entrusted the care and custody protection against unreasonable searches and seizures
thereof to the NBI agents. Clearly, the NBI agents made cannot be extended to acts committed by private
no search and seizure, much less an illegal one, contrary individuals so as to bring it within the ambit of alleged
to the postulate of accused/appellant. unlawful intrusion by the government.

Second, the mere presence of the NBI agents did not Appellant argues, however, that since the provisions of
convert the reasonable search effected by Reyes into a the 1935 Constitution has been modified by the present
warrantless search and seizure proscribed by the phraseology found in the 1987 Charter, expressly
Constitution. Merely to observe and look at that which is declaring as inadmissible any evidence obtained in
in plain sight is not a search. Having observed that which violation of the constitutional prohibition against illegal
is open, where no trespass has been committed in aid search and seizure, it matters not whether the evidence
thereof, is not search. Where the contraband articles are was procured by police authorities or private individuals
identified without a trespass on the part of the arresting (Appellant's Brief, p. 8, Rollo, p. 62).
officer, there is not the search that is prohibited by the
constitution.
The argument is untenable. For one thing, the
constitution, in laying down the principles of the
That the Bill of Rights embodied in the Constitution is not government and fundamental liberties of the people, does
meant to be invoked against acts of private individuals not govern relationships between individuals. Moreover, it
finds support in the deliberations of the Constitutional must be emphasized that the modifications introduced in
Commission. True, the liberties guaranteed by the the 1987 Constitution (re: Sec. 2, Art. III) relate to the
fundamental law of the land must always be subject to issuance of either a search warrant or warrant of arrest
protection. But protection against whom? Commissioner vis-a-vis the responsibility of the judge in the issuance
Bernas in his sponsorship speech in the Bill of Rights thereof (See Soliven v. Makasiar, 167 SCRA 393 [1988];
answers the query which he himself posed, as follows: Circular No. 13 [October 1, 1985] and Circular No. 12
[June 30, 1987]. The modifications introduced deviate in her "talagang ganyan, bukas." It appears that the amount
no manner as to whom the restriction or inhibition against in question (P640.00) had been pocketed by Ms. Catolico.
unreasonable search and seizure is directed against. The
restraint stayed with the State and did not shift to anyone
else. Co asked Catolico to explain, within twenty-four hours,
her side of the reported irregularity. Catolico asked for
additional time to give her explanation,12 and she was
Corolarilly, alleged violations against unreasonable granted a 48-hour extension from 1 to 3 February 1990.
search and seizure may only be invoked against the State However, on 2 February 1990, she was informed that
by an individual unjustly traduced by the exercise of effective 6 February 1990 to 7 March 1990, she would be
sovereign authority. To agree with appellant that an act of placed on preventive suspension to protect the interests
a private individual in violation of the Bill of Rights should of the company.
also be construed as an act of the State would result in
serious legal complications and an absurd interpretation
of the constitution. In a letter dated 2 February 1990, Catolico requested
access to the file containing Sales Invoice No. 266 for her
to be able to make a satisfactory explanation. In said letter
Similarly, the admissibility of the evidence procured by an she protested Saldaña's invasion of her privacy when
individual effected through private seizure equally Saldaña opened an envelope addressed to Catolico. In a
applies, in pari passu, to the alleged violation, non- letter15 to Co dated 10 February 1990, Catolico, through
governmental as it is, of appellant's constitutional rights to her counsel, explained that the check she received from
privacy and communication. YSP was a Christmas gift and not a "refund of overprice."
She also averred that the preventive suspension was ill-
motivated, as it sprang from an earlier incident between
The judgment of conviction finding appellant guilty beyond her and Co's secretary, Irene Soliven. On 5 March 1990,
reasonable doubt of the crime charged is hereby WATEROUS Supervisor Luzviminda Bautro, issued a
AFFIRMED. memorandum16 notifying Catolico of her termination.

WATEROUS DRUG CORP v. NLRC Catolico filed before the Office of the Labor Arbiter a
complaint for unfair labor practice, illegal dismissal, and
illegal suspension. Labor Arbiter Alex Arcadio Lopez
FACTS: WATEROUS Control Clerk Eugenio Valdez found no proof of unfair labor practice against petitioners.
informed WATEROUS Vice President-General Co that he Nevertheless, he decided in favor of Catolico because
noticed an irregularity involving Catolico (pharmacist of petitioners failed to "prove what [they] alleged as
WATEROUS) and Yung Shin Pharmaceuticals, Inc. complainant's dishonesty," and to show that any
(hereafter YSP). Previews P.O.s issued to YSP, Inc. investigation was conducted. Hence, the dismissal was
showed that the price per bottle is P320.00 while P.O. No. without just cause and due process. He thus declared the
19045 is priced at P384.00 or an over price of P64.00 per dismissal and suspension illegal but disallowed
bottle (or total of P640.00). WDRC paid the amount of reinstatement, as it would not be to the best interest of the
P3,840.00 thru MBTC Check No. 222832 dated parties. Accordingly, he awarded separation pay to
December 15, 1988. Verification was made to YSP, Inc. Catolico computed at one-half month's pay for every year
to determine the discrepancy and it was found that the of service; back wages for one year; and the additional
cost per bottle was indeed overpriced. YSP, Inc. sum of P2,000.00 for illegal suspension "representing 30
Accounting Department (Ms. Estelita Reyes) confirmed days work." Arbiter Lopez computed the award in favor of
that the difference represents refund of jack-up price of Catolico as follows:
ten bottles of Voren tablets per sales invoice no. 266 as
per their check voucher no. 629552 (shown to the
undersigned), which was paid to Ms. Catolico through
30 days Preventive Suspension P2,000.00
China Bank check no. 892068 dated November 9, 1989.
The undersigned talked to Ms. Catolico regarding the Backwages 26,858.50
check but she denied having received it and that she is
unaware of the overprice. However, upon conversation 1/12 of P26,858.50 2,238.21
with Ms. Saldana, EDRC Espana Pharmacy Clerk, she
Separation pay (3 years) 4,305.15
confirmed that the check amounting to P640.00 was
actually received by Ms. Catolico. As a matter of fact, Ms. —————
Catolico even asked Ms. Saldana if she opened the
envelope containing the check but Ms. Saldana answered TOTAL AWARD P35,401.86
————— protection against unreasonable searches and seizures
refers to the immunity of one's person from interference
by government and cannot be extended to acts committed
Petitioners seasonably appealed from the decision and by private individuals so as to bring it within the ambit of
urged the NLRC to set it aside because the Labor Arbiter alleged unlawful intrusion by the government.
erred in finding that Catolico was denied due process and
that there was no just cause to terminate her services.
ISSUE: Whether or not the dismissal of the private
respondent is in violation of the Constitution, under the Bill
NLRC affirmed the findings of the Labor Arbiter on the of Rights
ground that petitioners were not able to prove a just cause
for Catolico's dismissal from her employment. It found that
petitioner's evidence consisted only of the check of RULING:
P640.00 drawn by YSP in favor of complainant, which her
co-employee saw when the latter opened the envelope.
But, it declared that the check was inadmissible in A thorough review of the record leads us to no other
evidence pursuant to Sections 2 and 3(1 and 2) of Article conclusion than that, except as to the third ground, the
III of the Constitution.20 instant petition must fail.

NLRC affirmed the findings of the Labor Arbiter on the Concededly, Catolico was denied due process.
ground that petitioners were not able to prove a just cause Procedural due process requires that an employee be
for Catolico's dismissal from her employment. It found that apprised of the charge against him, given reasonable time
petitioner's evidence consisted only of the check of to answer the charge, allowed ample opportunity to be
P640.00 drawn by YSP in favor of complainant, which her heard and defend himself, and assisted by a
co-employee saw when the latter opened the envelope. representative if the employee so
But, it declared that the check was inadmissible in
evidence pursuant to Sections 2 and 3(1 and 2) of Article desires.23 Ample opportunity connotes every kind of
III of the Constitution.20 It concluded: assistance that management must accord the employee
to enable him to prepare adequately for his defense,
including legal representation.24
With the smoking gun evidence of respondents being
rendered inadmissible, by virtue of the constitutional right
invoked by complainants, respondents' case falls apart as In the case at bar, although Catolico was given an
it is bereft of evidence which cannot be used as a legal opportunity to explain her side, she was dismissed from
basis for complainant's dismissal. the service in the memorandum of 5 March 1990 issued
by her Supervisor after receipt of her letter and that of her
counsel. No hearing was ever conducted after the issues
The NLRC then dismissed the appeal for lack of merit, but were joined through said letters. The Supervisor's
modified the dispositive portion of the appealed decision memorandum spoke of "evidences [sic] in [WATEROUS]
by deleting the award for illegal suspension as the same possession," which were not, however, submitted. What
was already included in the computation of the aggregate the "evidences" [sic] other than the sales invoice and the
of the awards in the amount of P35,401.86. check were, only the Supervisor knew.

Hence, this petition. Catolico was also unjustly dismissed. It is settled that the
burden is on the employer to prove just and valid cause
(1st and 2nd grounds) Petitioners insist that Catolico had for dismissing an employee, and its failure to discharge
been receiving "commissions" from YSP, or probably from that burden would result in a finding that the dismissal is
other suppliers, and that the check issued to her on 9 unjustified.25 Here, WATEROUS proved unequal to the
November 1989 was not the first or the last. They also task.
maintained that Catolico occupied a confidential position
and that Catolico's receipt of YSP's check, aggravated by
her "propensity to violate company rules," constituted It is evident from the Supervisor's memorandum that
breach of confidence. And contrary to the findings of Catolico was dismissed because of an alleged anomalous
NLRC, Catolico was given ample opportunity to explain transaction with YSP. Unfortunately for petitioners, their
her side of the controversy. Petitioners submit that, in light evidence does not establish that there was an
of the decision in the People v. Marti,21 the constitutional overcharge.
Control Clerk Eugenio C. Valdez, who claims to have People vs. Mendoza
discovered Catolico's inappropriate transaction, stated in 301 SCRA 66
his affidavit:
FACTS:

5. I verified the matter to YSP Phils. to determine the On November 11, 1988, Octavio Mendoza, his wife
discrepancy and I found out that the cost per bottle was Cecilia and their 10-year old daughter Charmaine went to
indeed overpriced. The Accounting Department of YSP the birthday party of a relative of Octavio in Mcdo,
Phils. through Ms. Estelita Reyes confirmed that there Harrison Plaza. During the party, Octavio left without
was really an overprice and she said that the difference telling his wife and kid, and went to KFC.
was refunded through their check voucher no. 629552
which was shown to me and the payee is Melodia
Catolico, through a China Bank Check No. 892068 dated Since Cecilia and Charamaine couldn’t find him in the
November 9, 1989. party, they went home in Las Pinas. They arrived home at
7 pm and left again to go to Cecilia’s parents in Bacoor to
bring perfume. Octavio still was not home.
It clearly appears then that Catolico's dismissal was
based on hearsay information. Estelita Reyes never
testified nor executed an affidavit relative to this case; They got home around 9pm and saw Octavio’s car parked
thus, we have to reject the statements attributed to her by in the garage of their neighbor. All the lights were opened
Valdez. Hearsay evidence carries no probative value.27 but the front door was locked. After a while, Octavio
opened the back door and let them in. He was drunk and
told Charmaine to get cold water and douse him. She
Catolico's dismissal then was obviously grounded on followed and was instructed to go to her room. She went
mere suspicion, which in no case can justify an and got ready for bed. She heard her parents arguing
employee's dismissal. Suspicion is not among the valid about them leaving the party without Octavio. Afterwards,
causes provided by the Labor Code for the termination of she heard three gunshots, ran to their room and saw her
mom on the floor bleeding. She also saw her dad hide a
employment;31 and even the dismissal of an employee gun under the bed.
for loss of trust and confidence must rest on substantial
grounds and not on the employer's arbitrariness, whims,
caprices, or suspicion. Octavio then called his brother-in-law Sgt. Antonio Gabac.
When Gabac arrived, they all brought him to Perpetual
Help Hospital where Cecilia was declared dead on arrival.
As regards the constitutional violation upon which The policemen investigated Gabac and found a .38
the NLRC anchored its decision, we find no reason to caliber revolver in his waist. Gabac told them that Octavio
revise the doctrine laid down in People vs. Marti34 handed it over to him as soon as he arrived at the crime
that the Bill of Rights does not protect citizens from scene. Cecilia’s father, Alipio Eusebio learned of his
unreasonable searches and seizures perpetrated by daughter’s death and that valuables were being taken
private individuals. It is not true, as counsel for away from her house. He and his sons decided to go there
Catolico claims, that the citizens have no recourse and decided to remove, together with his sons, the
against such assaults. On the contrary, and as said remaining pieces of property therein, including accused-
counsel admits, such an invasion gives rise to both appellant's personal effects such as memorandum receipt
criminal and civil liabilities. and mission order authorizing Mendoza to carry weapon.

The instant petition is hereby DISMISSED and the At court, Charmaine testified that she saw her father hide
challenged decision and resolution of the National Labor the gun under the bed. In her second testimony, she said
Relations Commission dated 30 September 1993 and 2 she saw no such act. Octavio also denied that he killed
December 1993, respectively, in NLRC-NCR CA No. his wife and that he owned that gun. He said that the
005160-93 are AFFIRMED, except as to its reason for memorandum receipt and mission order were illegally
upholding the Labor Arbiter's decision, viz., that the procured by Eusebio in violation of his right against
evidence against private respondent was inadmissible for unreasonable search and seizure.
having been obtained in violation of her constitutional
rights of privacy of communication and against
unreasonable searches and seizures which is hereby set
aside.
ISSUE: RULING: Yes

WON Octavio’s constitutional right against unreasonable The contention of the accused appellant that the suitcase
search and seizure violated when Eusebio took the was opened and searched without his consent hence, a
memorandum receipt and mission order and brought it to violation of his constitutional right against unreasonable
court. search and seizure is devoid of merit.

RULING In the case before us, the baggage of the accused-


appellant was searched by the vessel security personnel.
NO. The Court held that Eusebio did not violate Octavio’s It was only after they found shabu inside the suitcase that
constitutional right against unreasonable search and they called the Philippine Coast Guard for assistance. The
seizure. The constitutional protection against search and seizure of the suitcase and the contraband
unreasonable searches and seizures refers to immunity items was therefore carried out without government
of one’s person from interference from THE intervention, and hence, the constitutional protection
GOVERNMENT and it cannot be extended to acts against unreasonable search and seizure does not apply.
committed by PRIVATE INDIVIDUALS. In the case at bar,
the articles were discovered by Cecilia’s father, Alipio
Eusebio, a private individual. Besides, other evidence
proved his guilt. Test showed nitrate in his hands and the The vessel security officer in the case at bar is a private
testimony of his daughter was accorded great weight. He employee and does not discharge any governmental
was charged of the crime parricide with the special function. In contrast, police officers are agents of the state
aggravating circumstance of the use of an unlicensed tasked with the sovereign function of enforcement of the
firearm. law. Historically and until now, it is against them and other
agents of the state that the protection against
unreasonable searches and seizures may be invoked.
People vs Bongcarawan
G.R. No. 143944. (July 11, 2002)
things in possession of a person are presumed by law to
be owned by him.[31 To overcome this presumption, it is
FACTS: necessary to present clear and convincing evidence to the
contrary. In this case, the accused points to a certain
Alican Alex Macapudi as the owner of the contraband, but
Accused Basher Bongcarawan y Macarambon was presented no evidence to support his claim.
charged for violation of Section 16, Article III of RA 6425,
otherwise known as the Dangerous Drugs Act of 1972, as
amended by RA 7659 for the possession, custody and WHEREFORE, the decision of the Regional Trial Court is
control eight (8) packs of shabu. affirmed. penalty of Reclusion Perpetua and to pay a fine
of Five Hundred Thousand Pesos (P500,000.00) without
subsidiary imprisonment in case of insolvency.
Evidence for the prosecution shows that on March 11,
1999 an interisland passenger ship, M/V Super Ferry 5,
sailed from Manila to Iligan City. At about 3:00 a.m. on
March 13, 1999, the vessel was about to dock at the port
of Iligan City when its security officer, Mark Diesmo,
received a complaint from passenger Lorena Canoy
about her missing jewelry. Canoy suspected one of her
co-passengers at cabin no. 106 as the culprit. After the
body search, The security requested to open the
samsonite suitcase thus revealing a brown bag and small
plastic packs containing white crystalline substance. The
security then reported the suspected drugs.

ISSUE:

W/n the the search and seizure was valid and therefore
admissible as evidence?
I. JAIL SAFETY CONSTITUTIONALITY OF CHECKPOINTS AND “AREA TARGET
ZONINGS”

PEOPLE VS CONDE
FACTS:
VALMONTE vs.DE VILLA
The witness to the robbery was only the one who saw the G.R. No. 83988 (September 29, 1989)
victim got stabbed. The victim was an alien (Indian). FACTS
There were 2 aliens. The witness was able to identify the
On 20 January 1987, the National Capital Region District
accused.
Command (NCRDC) was activated for the purpose of
These accused after they were identified, and eventually establishing an effective territorial defense, maintaining
apprehended by the policemen and they were jailed. peace and order, and providing an atmosphere conducive
There was no evidence that the officers saw the stabbing to the social, economic and political development of the
of these victims. However, when the wife of one of the National Capital Region. As part of its duty to maintain
accused visited the police station, she brought a knife in peace and order, the NCRDC installed checkpoints in
her bag. When she passed through the inspection in the various parts of Valenzuela, Metro Manila. Petitioners
jail, the knife was seen she may not pass through the aver that, because of the installation of said checkpoints,
checkpoint for it may be used to kill or injure someone. the residents of Valenzuela are worried of being harassed
That piece of evidence was used as evidence against the and of their safety being placed at the arbitrary, capricious
accused. and whimsical disposition of the military manning the
checkpoints, considering that their cars and vehicles are
The weapon used and the identity of the assailant was the being subjected to regular searches and check-ups,
link to the stabbing. It was argued by the accused that this especially at night or at dawn, without the benefit of a
knife could not be admitted as an evidence because it was search warrant and/or court order. Their alleged fear for
procured by the policeman without any warrant, pursuant their safety increased when, at dawn of 9 July 1988,
to illegal search. Benjamin Parpon, a supply officer of the Municipality of
Valenzuela, Bulacan, was gunned down allegedly in cold
ISSUE:
blood by the members of the NCRDC manning the
W/N jail searches are reasonable forms of searches. checkpoint. Petitioner Valmonte also claims that, on
several occasions, he had gone thru these checkpoints
RULING: Yes. The Court said that the knife is admissible where he was stopped and his car subjected to
as evidence even if there was no warrant when the search search/check-up without a court order or search warrant.
was done. The Court said it was valid when the knife was
found inside the bag of the wife when she was visiting her ISSUE Whether or not checkpoint is unconstitutional.
husband who is a detainee. The police officer testified that
the search that was done to her is part of the Police
Standard Operating Procedure and recognized as part of RULING No.
the precautionary measures by the police to safeguard
the safety of the detainees as well as the overall security Petitioners' concern for their safety and apprehension at
of the jail premises. being harassed by the military manning the checkpoints
are not sufficient grounds to declare the checkpoints as
It’s a reasonable form of search. What is protected under per se illegal. The constitutional right against
Article III Section 2, are unreasonable searches. Even if unreasonable searches and seizures is a personal right
there was no search warrant, it is reasonable and invocable only by those whose rights have been
therefore the item was admissible. infringed, or threatened to be infringed. What constitutes
a reasonable or unreasonable search and seizure in
any particular case is purely a judicial question,
determinable from a consideration of the circumstances
involved.

Petitioner Valmonte's general allegation to the effect that


he had been stopped and searched without a search
warrant by the military manning the checkpoints, without
more, i.e., without stating the details of the incidents which
amount to a violation of his right against unlawful search
and seizure, is not sufficient to enable the Court to
determine whether there was a violation of Valmonte's
right against unlawful search and seizure. Not all
searches and seizures are prohibited. Those which are
reasonable are not forbidden. A reasonable search is not
to be determined by any fixed formula but is to be resolved Between the inherent right of the state to protect its
according to the facts of each case. existence and promote public welfare and an individual's
right against a warrantless search which is however
Where, for example, the officer merely draws aside the reasonably conducted, the former should prevail.
curtain of a vacant vehicle which is parked on the public
fair grounds, 7 or simply looks into a vehicle, 8 or flashes
a light therein, 9 these do not constitute unreasonable
PEOPLE VS. EXALA
search. 221 SCRA 494 (1993)
True, the manning of checkpoints by the military is
susceptible of abuse by the men in uniform, in the same FACTS
manner that all governmental power is susceptible of
abuse. But, at the cost of occasional inconvenience, On November 2, 1982, a private jeepney driven by the
discomfort and even irritation to the citizen, the accused - appellant, Restituto Bocalan,was stopped at a
checkpoints during these abnormal times, when police checkpoint for routing inspection regarding
conducted within reasonable limits, are part of the price unlicensed firearms and other prohibited items. Along with
we pay for an orderly society and a peaceful community. Bocalan were his co-accused Jaime Fernandez and
Rodelio Exala. Pfc. Ricardo Galang, a member of the
WHEREFORE, the petition is DISMISSED. inspection team, went near the jeep and asked the
occupants if there were firearms inside, and in which the
occupants answered in the negative. Pfc. Galang then
VALMONTE vs.DE VILLA noticed a black bag inside the jeepney, as he became
G.R. No. 83988, May 24, 1990
suspicious, he asked the occupants of what the bag
contained, but nobody answered. Pfc. Galang then
FACTS opened the bag and found a number of marijuana.
Thereafter, the accused were brought to the police station
In the Court's decision dated 29 September 1989, for further investigation. The accused were all charged for
petitioners' petition for prohibition seeking the declaration violation of Section 4, Article II of R.A. 6425, as amended.
of the checkpoints as unconstitutional and their Bocalan now assails his conviction. He contends that the
dismantling and/or banning, was dismissed. Petitioners trial court erred in admitting the bag as evidence against
have filed the instant motion and supplemental motion for him since it was obtained through a warrantless search.
reconsideration of said decision.

ISSUE
ISSUE
Whether or not the warrantless search was valid and
Whether or not checkpoint is unconstitutional. legal, thus proper in using the marijuana as evidence for
convicting the accused (Bocalan) of the crime charged
against him.
RULING

NO. The setting up of the questioned checkpoints in


HELD
Valenzuela (and probably in other areas) may be
considered as a security measure to enable the NCRDC Yes. Since the search was conducted prior to the arrest,
to pursue its mission of establishing effective territorial Bocalan argues that it was not incident to a lawful arrest.
defense and maintaining peace and order for the benefit This issue was never raised in the proceedings. Bocalan
of the public. Checkpoints may also be regarded as never objected to the admissibility of the evidence on the
measures to thwart plots to destabilize the government, in ground that the same was obtained in a warrantless
the interest of public security. search. Consequently, he is deemed to have waived his
objection on the legality of the search and the admissibility
of the evidence obtained in the course thereof. In view of
In this connection, the Court may take judicial notice of such waiver, the court is bound to admit the evidence. But
the shift to urban centers and their suburbs of the even assuming arguendo that there was no waiver, still
insurgency movement, so clearly reflected in the appellant's contention deserves scant consideration.
increased killings in cities of police and military men by There are indeed instances where search and seizure can
NPA "sparrow units," not to mention the abundance of be effected without necessarily being preceded by an
unlicensed firearms and the alarming rise in lawlessness arrest. The instant case is an incident to or an offshoot of
and violence in such urban centers, not all of which are a lawful "stop-and- search" at a military or police
reported to media. checkpoint. The checkpoint in the instant case was
established in line with "Operational Bakal" the main
object of which was to search for unlicensed firearms and prohibit the respondents from further conducting these
other prohibited items in the possession of unauthorized drives.
persons passing through it. When the jeep carrying the
contraband passed through the checkpoint, it was flagged
down and the occupants were asked routine questions. In ISSUE: Whether or Not the saturation drive committed
the course thereof, Pfc. Galang noticed a black leather consisted of violation of human rights.
bag the sides of which were bulging. He asked what the
contents of the bag were. None of the accused answered.
At that moment, the demeanor of the accused changed;
RULING:
they became suspiciously quiet and nervous as if they
were concealing something from Pfc. Galang. The
accused clearly appeared to be in abject fear of being
discovered. Such peculiar apprehensiveness if not No, The areal target zonings in this petition were intended
restrained reaction of the accused, which did not appear to flush out subversives and criminal elements particularly
normal, provided the probable cause justifying a more because of the blatant assassinations of public officers
extensive search that led to the opening of the bag and and police officials by elements supposedly coddled by
the discovery of the prohibited stuff. Significantly, there the communities where the "drives" were conducted.
was no sign of any protest or objection to the search. The
It is clear from the pleadings of both petitioners and
accused remained silent even after their arrest. Their
respondents, however, that there was no rebellion or
submissive stance after the discovery of the bag of
criminal activity similar to that of the attempted coup d'
marijuana, as well as the absence of any protest on their
etats. There appears to have been no impediment to
part when arrested, not only casts serious doubts on their
securing search warrants or warrants of arrest before any
professed innocence but also confirms their
houses were searched or individuals roused from sleep
acquiescence to the search. Clearly then, there was
were arrested. There is no strong showing that the
waiver of the right against unreasonable search and
objectives sought to be attained by the "areal zoning"
seizure.
could not be achieved even as the rights of squatter and
low income families are fully protected.

Guanzon vs De villa Herein lies the problem of the Court. We can only guess
G.R. No. 80508 (January 30, 1990) the truth. Everything before us consists of allegations.
According to the petitioners, more than 3,407 persons
were arrested in the saturation drives covered by the
FACTS:
petition. No estimates are given for the drives in Block 34,
Dagat-dagatan, Navotas; Apelo Cruz Compound, Pasig;
and Sun Valley Drive near the Manila International Airport
This is a petition for prohibition with preliminary injunction area. Not one of the several thousand persons treated in
to prohibit the military and police officers represented by the illegal and inhuman manner described by the
public respondents from conducting "Areal Target petitioners appears as a petitioner or has come before a
Zonings" or "Saturation Drives" in Metro Manila. trial court to present the kind of evidence admissible in
courts of justice. Moreover, there must have been tens of
thousands of nearby residents who were inconvenienced
According to the petitioners, the "areal target zonings" or in addition to the several thousand allegedly arrested.
saturation drives" are in critical areas pinpointed by the None of those arrested has apparently been charged and
military and police as places where the subversives are none of those affected has apparently complained.
hiding. The arrests range from seven (7) persons during
the July 20 saturation drive in Bangkusay, Tondo to one
thousand five hundred (1,500) allegedly apprehended on In the meantime and in the face of a prima facie showing
November 3 during the drive at Lower Maricaban, Pasay that some abuses were probably committed and could be
City. The petitioners claim that the saturation drives follow committed during future police actions, we have to
a common pattern of human rights abuses. In all these temporarily restrain the alleged banging on walls, the
drives, it is alleged that the following were committed. kicking in of doors, the herding of half-naked men to
assembly areas for examination of tattoo marks, the
violation of residences even if these are humble shanties
The forty one (41) petitioners state that they are all of legal of squatters, and the other alleged acts which are
age, bona fide residents of Metro Manila and taxpayers shocking to the conscience. WHEREFORE, the petition is
and leaders in their respective communities De Villa et al. hereby REMANDED to the Regional Trial Courts.
allege that the accusations about the deliberate disregard
of human rights are total lies. They cite Art VII, sec 17 and
18174 as legal bases for the drives. Petitioners seek to
Abenes VS CA the latter being able to present any license or permit to
G.R. No. 156320 (February 14, 2007) possess the same, such fact alone is not conclusive proof
that he was not lawfully authorized to carry such firearm.
FACTS: In other words, such fact does not relieve the prosecution
from its duty to establish the lack of a license or permit to
carry the firearm by clear and convincing evidence, like a
certification from the government agency concerned. 24
RODOLFO ABENES Y GACUTAN was accused of the
offense of ILLEGAL POSSESSION OF HIGH POWERED Thus, for failure of the prosecution to prove beyond
FIREARM & ITS AMMUNITIONS (Violation of P.D. No. reasonable doubt that petitioner was carrying a firearm
1866, as amended by R.A. No. 8294), committed as without prior authority, license or permit, the latter must
follows: be exculpated from criminal liability under P.D. No. 1866,
as amended.

With respect to the charge of violating Section 261(q)


That on May 8, 1998, at about 10:30 a.m. within the of B.P. Blg. 881, as amended, otherwise known as the
Election period which is from January 11, 1998 to June Omnibus Election Code, the Court is constrained to affirm
30, 1998, in Danlugan, Pagadian City, Philippines, a the conviction of the petitioner, since the prosecution
checkpoint in Barangay Danlugan at said city, for the
successfully discharged its burden of proof.
purpose of enforcing the Gun Ban which was then being
implemented by the COMELEC. SPO3 Cipriano Q. 2. Yes, This Court has ruled that not all checkpoints are
Pascua was the designated team leader. SPO1 Requejo illegal. Those which are warranted by the exigencies of
and SPO3 Pascua noticed that a holstered firearm was public order and are conducted in a way least intrusive to
tucked at the right waist of Abenes. The firearm was motorists are allowed. For, admittedly, routine
readily visible to the policemen; it was not covered by the checkpoints do intrude, to a certain extent, on motorists’
shirt worn by Abenes. Abenes was then asked by SPO3 right to "free passage without interruption," but it cannot
Pascua whether he had a license and authority to carry be denied that, as a rule, it involves only a brief detention
the firearm, and whether his possession was exempted of travelers during which the vehicle’s occupants are
from the Gun Ban being enforced by the COMELEC. required to answer a brief question or two. For as long as
Accused answered in the affirmative. The policemen then the vehicle is neither searched nor its occupants
demanded for the pertinent documents to be shown to subjected to a body search, and the inspection of the
support Abenes’ claim. He could not show any. Hence, vehicle is limited to a visual search, said routine checks
SPO1 Requejo confiscated Abenes’ firearm. cannot be regarded as violative of an individual’s right
against unreasonable search. In fact, these routine
checks, when conducted in a fixed area, are even less
ISSUE: intrusive.

1. W/n the petitioner’s constitutional right against unlawful


search and seizure violated, Given the circumstances,
The checkpoint herein conducted was in pursuance of the
and the evidence adduced. gun ban enforced by the COMELEC. The COMELEC
2. W/n the checkpoint was legally set up? would be hard put to implement the ban if its deputized
agents were limited to a visual search of pedestrians. It
would also defeat the purpose for which such ban was
instituted. Those who intend to bring a gun during said
RULING:
period would know that they only need a car to be able to
easily perpetrate their malicious designs.

1. No, In the instant case, the firearm was seized from the
petitioner when in plain view, the policemen saw it tucked
The facts adduced do not constitute a ground for a
into his waist uncovered by his shirt. It must be
violation of the constitutional rights of the accused against
emphasized that the policemen discovered the firearm
illegal search and seizure. PO3 Suba admitted that they
[on] the person of the [petitioner] shortly after he alighted
were merely stopping cars they deemed suspicious, such
from the vehicle and before he was frisked. SPO3
as those whose windows are heavily tinted just to see if
Pascua’s testimony[,] corroborated by that of SPO1
the passengers thereof were carrying guns. At best they
Requejo[,] convincingly established that the holstered .45
would merely direct their flashlights inside the cars they
caliber pistol tucked at the right waist of the [petitioner]
would stop, without opening the car’s doors or subjecting
was readily visible to the policemen (TSN, August 24,
its passengers to a body search. There is nothing
1998, pp. 18, 37). While the prosecution was able to
discriminatory in this as this is what the situation
establish the fact that the subject firearm was seized by
demands.17 (Emphasis supplied)
the police from the possession of the petitioner, without
device or arrangement." Is an extension of a telephone
unit such a device or arrangement as would subject the
WHEREFORE, the petition is partly GRANTED. The user to imprisonment ranging from six months to six years
Decision dated November 29, 2002 of the Court of with the accessory penalty of perpetual absolute
Appeals is REVERSED and SET ASIDE. Petitioner disqualification for a public officer or deportation for an
Rodolfo Abenes Y Gacutan is ACQUITTED from the alien?
charge of illegal possession of firearm

WIRE TAPPING
The law refers to a "tap" of a wire or cable or the use of a
"device or arrangement" for the purpose of secretly
overhearing, intercepting, or recording the
GAANAN vs.IAC communication. There must be either a physical
G.R. No. L-69809 October 16, 1986
interruption through a wiretap or the deliberate installation
of a device or arrangement in order to overhear, intercept,
FACTS or record the spoken words.

Complainant Atty. Pintor and Montebon offered to


withdraw the complaint for direct assault they filed against
An extension telephone cannot be placed in the same
Laconico after demanding P8,000 from him. This demand
category as a dictaphone, dictagraph or the other devices
was heard by Atty. Gaanan through a telephone
enumerated in Section 1 of RA No. 4200 as the use
extension as requested by Laconico so as to personally
thereof cannot be considered as "tapping" the wire or
hear the proposed conditions for the settlement. Atty.
cable of a telephone line. The telephone extension in this
Pintor was subsequently arrested in an entrapment
case was not installed for that purpose. It just happened
operation upon receipt of the money. Since Atty. Gaanan
to be there for ordinary office use. It is a rule in statutory
listened to the telephone conversation without
construction that in order to determine the true intent of
complainant''s consent, complainant charged Gaanan
the legislature, the particular clauses and phrases of the
and Laconico with violation of the Anti- Wiretapping Act
statute should not be taken as detached and isolated
(RA 4200).
expressions, but the whole and every part thereof must
be considered in fixing the meaning of any of its parts.

ISSUE

Whether or not an extension telephone is among the Further, our lawmakers intended to discourage, through
prohibited devices in Section 1 of the Act, such that its use punishment, persons such as government authorities or
to overhear a private conversation would constitute representatives of organized groups from installing
unlawful interception of communications between the two devices in order to gather evidence for use in court or to
parties using a telephone line. intimidate, blackmail or gain some unwarranted
advantage over the telephone users. Consequently, the
mere act of listening, in order to be punishable must
strictly be with the use of the enumerated devices in RA
RULING
4200 or others of similar nature. An extension telephone
NO. The Court ruled in favor of the petitioner. The is not among such devices or arrangements.
petitioner was acquitted of the crime of violation of Rep.
Act No. 4200, otherwise known as the Anti-Wiretapping
Act.

Because of technical problems caused by the sensitive


nature of electronic equipment and the extra heavy loads
which telephone cables are made to carry in certain
areas, telephone users often encounter what are called
"crossed lines". An unwary citizen who happens to pick
up his telephone and who overhears the details of a crime
might hesitate to inform police authorities if he knows that
he could be accused under Rep. Act 4200 of using his
own telephone to secretly overhear the private
communications of the would be crime.

The main issue in the resolution of this petition, however,


revolves around the meaning of the phrase "any other
REMEDIES IN CASES OF VIOLATION was impossible for the judges who issued the warrants to
have found the existence of probable cause, for the same
A. EXLUSIONARY RULE
presupposes the introduction of competent proof that the
STONEHILL vs DIOKNO party against whom it is sought has performed particular
G.R. No. L-19550 June 19, 1967 acts, or committed specific omissions, violating a given
provision of our criminal laws. As a matter of fact, the
FACTS: applications involved in this case do not allege any
specific acts performed by herein petitioners. It would be
Respondents issued, on different dates, 42 search a legal heresy, of the highest order, to convict anybody of
warrants against petitioners personally, and/or a “violation of Central Bank Laws, Tariff and Customs
corporations for which they are officers directing peace Laws, Internal Revenue (Code) and Revised Penal
officers to search the persons of petitioners and premises Code,” — as alleged in the aforementioned applications
of their offices, warehouses and/or residences to search — without reference to any determinate provision of said
for personal properties “books of accounts, financial laws or codes.
records, vouchers, correspondence, receipts, ledgers,
journals, portfolios, credit journals, typewriters, and other The grave violation of the Constitution made in the
documents showing all business transactions including application for the contested search warrants was
disbursement receipts, balance sheets and profit and loss compounded by the description therein made of the
statements and Bobbins(cigarettes)” as the subject of the effects to be searched for and seized, to wit:
offense for violations of Central Bank Act, Tariff and
Customs Laws, Internal Revenue Code, and Revised
Penal Code. “Books of accounts, financial correspondence, receipts,
ledgers, typewriters, and other documents and/or papers
showing all business transactions including disbursement
Upon effecting the search in the offices of the receipts, balance sheets and related profit and loss
aforementioned corporations and on the respective statements.”
residences of the petitioners, there seized documents,
papers, money and other records. Petitioners then were
subjected to deportation proceedings and were Thus, the warrants authorized the search for and seizure
constrained to question the legality of the searches and of records pertaining to all business transactions of
seizures as well as the admissibility of those seized as Stonehill et al, regardless of whether the transactions
evidence against them. were legal or illegal. The warrants sanctioned the seizure
of all records of Stonehill et al and the aforementioned
corporations, whatever their nature, thus openly
On March 20, 1962, the SC issued a writ of preliminary contravening the explicit command of the Bill of Rights —
injunction and partially lifted the same on June 29, 1962 that the things to be seized be particularly described —
with respect to some documents and papers. as well as tending to defeat its major objective: the
elimination of general warrants. The Moncado doctrine is
likewise abandoned and the right of the accused against
a defective search warrant is emphasized.
ISSUE:

Whether or not the search warrant issue is valid.


Indeed, the non-exclusionary rule is contrary, not only to
the letter, but also, to the spirit of the constitutional
RULING: injunction against unreasonable searches and seizures.
To be sure, if the applicant for a search warrant has
NO. The constitution protects the people’s right against competent evidence to establish probable cause of the
unreasonable search and seizure. It provides; (1) that no commission of a given crime by the party against whom
warrant shall issue but upon probable cause, to be the warrant is intended, then there is no reason why the
determined by the judge in the manner set forth in said applicant should not comply with the requirements of the
provision; and (2) that the warrant shall particularly fundamental law. Upon the other hand, if he has no such
describe the things to be seized. In the case at bar, none competent evidence, then it is not possible for the Judge
of these are met. The warrant was issued from mere to find that there is probable cause, and, hence, no
allegation that Stonehill et al committed a “violation of justification for the issuance of the warrant. The only
Central Bank Laws, Tariff and Customs Laws, Internal possible explanation (not justification) for its issuance is
Revenue (Code) and Revised Penal Code.” In other the necessity of fishing evidence of the commission of a
words, no specific offense had been alleged in said crime. But, then, this fishing expedition is indicative of the
applications. The averments thereof with respect to the absence of evidence to establish a probable cause.
offense committed were abstract. As a consequence, it
Possession of Firearms and Ammunition as penalized
under P.D. No. 1866, §1.
Moreover, the theory that the criminal prosecution of
those who secure an illegal search warrant and/or make
unreasonable searches or seizures would suffice to
protect the constitutional guarantee under consideration, ISSUE:
overlooks the fact that violations thereof are, in general, Whether or not the Search Warrant issued by Judge
committed By agents of the party in power, for, certainly, Teodorico Durias is invalid for failure to comply with the
those belonging to the minority could not possibly abuse basic requirements of the Constitution.
a power they do not have. Regardless of the handicap
under which the minority usually — but, understandably
— finds itself in prosecuting agents of the majority, one
RULING:
must not lose sight of the fact that the psychological and
moral effect of the possibility of securing their conviction, NO. The Court held that petitioner waived any objection
is watered down by the pardoning power of the party for based on the illegality of the search.
whose benefit the illegality had been committed.

Petitioner assails the absence of a written deposition


PASTRANO vs. CA showing that the judge had examined the complainant
G.R. No. 104504 October 31, 1997
and his witnesses by means of searching questions in
writing and under oath as required by Rule 126, §4 of the
FACTS: Rules on Criminal Procedure.

A group of students went to see Capt. Rodolfo Mañoza,


then intelligence operations officer of the Philippine
Rule 126, §4 indeed requires the examination of the
Constabulary, at Camp Naranjo, at Oroquieta City. They
complainant and his witnesses to be put in writing and
reported having seen Clyde Pastrano beaten up by his
under oath. But although this is a ground for quashing a
father, petitioner Pedrito Pastrano. The students were
search warrant in this case, petitioner did nothing to this
willing to testify but expressed fear of the petitioner who,
end. He did not move to quash the information before the
according to them, had firearms. Clyde Pastrano had died
trial court. Nor did he object to the presentation of the
and it was suspected he had been the victim of foul play.
evidence obtained as being the product of an illegal
Two sons of Pedrito Pastrano by his estranged wife also
search.
saw Capt. Mañoza, seeking his assistance in connection
with the death of their brother Clyde. The brothers
reported that their father and his common-law wife were
keeping unlicensed firearms in their house. They Petitioner thus waived any objection based on the
executed a joint affidavit which they stated that they had illegality of the search. As held in People v. Omaweng, the
personal knowledge of the fact that their father Pedrito right to be secure against unreasonable searches and
Pastrano was keeping three (3) firearms of different seizures, like any other right, can be waived and the
calibers in the bedroom of his house. waiver may be made either expressly or impliedly.

On the basis of the affidavit of the Pastrano brothers, The Court find that the prosecution clearly established the
Capt. Mañoza applied for a search warrant on the same elements of the crime charged and that the Court of
day. Appeals and the trial court correctly found petitioner guilty
beyond reasonable doubt of the crime of Illegal
Possession of Firearms and Ammunition.
After examining complainant and the two brothers, Judge
Teodorico M. Durias of the Municipal Trial Court of
Oroquieta City (Branch I) issued a search warrant which
Capt. Mañoza and his men later served at the residence
of Pedrito Pastrano at Capitol Drive, Oroquieta City.
Seized from petitioner's dwelling was a sack containing
the following:XXX

On the basis of the evidence thus seized, petitioner


Pedrito Pastrano y Capapas and his common-law wife,
Erlinda Ventir y Sanchez, were charged with Illegal
B. CIVIL ACTION FOR DAMAGES injuries suffered because of respondents’ confiscation of
their private belongings, the violation of their right to
remain silent and to counsel and their right to protection
ABERCA vs. VER against unreasonable searches and seizures and against
G.R. No. L-69866 April 15, 1988 torture and other cruel and inhuman treatment.

FACTS:
The question became moot and academic since the
Petitioners brought suit alleging that General Fabian Ver suspension of the PWHC had been lifted with the
had ordered the Task Force Makabansa of the AFP to issuance of then Pres. Corazon Aquino of Proclamation
conduct "preemptive strikes against known communist No. 2 on March 25, 1986.
terrorists'' underground houses" in Metro Manila. The
TFM raided some places using defective warrants; they
seized personal belongings of petitioners; they had been
Forbes vs Chuoco Tiaco
interrogated in violation of their right to silence and to G.R. No. L-6157 (July 30, 1910)
counsel; they had been tortured and intimidated.
Petitioners asked for payment of damages for violations
of their constitutional rights. FACTS:

ISSUE: On the 1st of April, 1910, the defendant Chuoco Tiaco


(alias Choa Tea) filed a suit in the Court of First Instance
of the city of Manila against the plaintiffs in which
substantially the following allegations and petition were
Whether or not the suspension of the privilege of the writ
made, alleging that on the 19th of August, 1909, under
of habeas corpus bars a civil action for damages for illegal
the orders of the said W. Cameron Forbes, Governor-
searches conducted by military personnel and other
General of the Philippine Islands, he was deported
violations of rights and liberties guaranteed under the
therefrom and sent to Amoy, China, by the aforesaid J. E.
Constitution.
Harding and C. R. Trowbridge, chiefs, as above stated, of
the police and of the secret service, respectively, of the
city of Manila, and that having been able to return to these
RULING: Islands he feared, as it was threatened, that he should be
NO. The suspension of the privilege of the writ of habeas again deported by the said defendants, concluding with a
corpus does not destroy petitioners’ right and cause of petition that a preliminary injunction should be issued
action for damages for illegal arrest and detention and against the plaintiffs in this case prohibiting them from
other violations of their constitutional rights. The deporting the defendant, Chuoco Tiaco (alias Choa Tea),
suspension does not render valid an otherwise illegal and that they be sentenced to pay him P20,000 as an
arrest or detention. What is suspended is merely the right indemnity.
of the individual to seek release from detention through
the writ of habeas corpus as a speedy means of obtaining
his liberty. The plaintiff countered a demurrer against the same and
presented a motion asking that the injunction be
dissolved, the grounds of the demurrer being that the
Moreover, as pointed out by petitioners, their right and facts set out in the complaint did not constitute a motive
cause of action for damages are explicitly recognized in of action, and that the latter was one in which the court
PD 1755 which amended Art. 1146 of the Civil Code by lacked jurisdiction to issue such an injunction against the
adding the following text: However, when the action (for plaintiffs for the reasons set out in the complaint;
injury to the rights of the plaintiff or for quasi-delict) arises notwithstanding which, the defendant A. S. Crossfield
from or out of any act, activity or conduct of any public overruled the demurrer and disallowed the motion,
officer involving the exercise of powers or authority arising leaving the complaint and the injunction standing.
from martial law including the arrest, detention and/or trial
of the plaintiff, the same must be brought within one year.
The purpose of the first suits, was to make the Governor-
General personally liable for the damages of his acts
Even assuming that the suspension of the privilege of the which was done in the exercise of his duties, as well as to
writ of habeas corpus suspends petitioners’ right of action prevent him from exercising the same power in the future.
for damages for illegal arrest and detention, it does not This sufficiently appears by the declarations, which
and cannot suspend their rights and causes of action for suggest and do not exclude official action, and is alleged
in the complaints for prohibition. On April 19, 1910, the
Philippine legislature then passed an act which grants the WHEN ARREST MAY BE MADE WITHOUT A WARRANT
governor- general the authority to deport."in the exercise
of authority vested in him by law," enacted that his action
was "approved and ratified and confirmed by the A. STRICT ENFORCEMENT OF RULE
subsequent law.
LUMANOG V. PEOPLE
630 SCRA 42 SEPTEMBER 7, 2010
ISSUE: WON the Governor General, as Chief Executive,
can be sued in a civil action. FACTS: Appellants were the accused perpetrators of the
ambush-slay of former Chief of the Metropolitan
Command Intelligence and Security Group of the
RULING: No, That the Government of the United States Philippine Constabulary (now the Philippine National
in the Philippine Islands is a government possessed with Police), Colonel Rolando N. Abadilla.
"all the military, civil, and judicial powers necessary to
govern the Philippine Islands" and as such has the power Joel, who was believed to be the murderer, was
and duty, through its political department, to deport aliens arrested six (6) days after the murder on June 19, 1996.
whose presence in the territory is found to be injurious to The murder was committed on June 13, 1996. He was
the public good and domestic tranquility of the people. arrested without warrant.

That the Governor-General, acting in his political and


executive capacity, is invested with plenary power to ISSUE: W/N the arrest without warrant is valid
deport obnoxious aliens, whose continued presence in
the territory is found by him to be injurious presence to the
public interest, and in the method of deporting or expelling
them, he may use such method as his official judgment RULING: No, the arrest without warrant six days after the
and good conscience may dictate. commission of the crime is not valid. The period of time
when the appellant was arrested is no longer the purview
of hot arrest pursuit which is one of the instances of valid
warrantless arrest.
That this power to deport or expel obnoxious aliens being
invested in the political department of the Government,
the judicial department will not, in the absence of express
PEOPLE V. ARANETA
legislative authority, intervene for the purpose of 634 SCRA 475 OCTOBER 20, 2010
controlling such power, nor the purpose of inquiring
whether or not he is liable in damages for the exercise
thereof. FACTS: A confidential information arrived at the drug
enforcement unit of Pasig Police Station about the alleged
peddling of illegal drugs of the live-in couple Botong and
Therefore the lower court was without jurisdiction to Malou. The team arrived at the target place at 4:10 in the
consider the particular questions presented in the cause, morning.
and it is hereby ordered and decreed that the writ of Upon arrival, PO2 Damasco and the informant went near
prohibition shall be issued, directed to the defendant, the the appellants who were standing just outside their house.
Hon. A. S. Crossfield, perpetually prohibiting him from The informant and appellants exchanged greetings and
proceeding in the cause in which Chuoco Tiaco the parties exchanged conversations. The latter then
(alias Choa Tea) is plaintiff and W. Cameron Forbes, immediately gave her the marked P100 bill. Malou
Charles R. Trowbridge, and J.E. Harding are defendants, called Botong and when the latter came out, Malou
and to dismiss said action, as well as to enter an order handed to him the marked money. Botong then gave
dissolving the injunction granted by him in said cause Malou a plastic sachet which she handed to PO2
against the said defendants. Damasco.

After examining the plastic sachet, PO2 Damasco


immediately gave the pre-arranged signal to the other
members of the team who thereafter rushed to the scene.
PO2 Damasco arrested Malou while SPO2 Zigapan
arrested Botong. SPO2 Zigapan recovered from Botong
the marked P100 bill and after frisking him, the police
officer found in Botong’s pocket one plastic sachet of what
looked like marijuana and eight plastic sachets containing
white crystalline substance.
ISSUE: W/N the arrest without warrant is valid

RULING: Yes, the arrest is valid even without warrant. ISSUE: W/N the arrest is valid

A search warrant or warrant of arrest was not needed


because it was a buy-bust operation and the accused
were caught in flagrante delicto in possession of, and RULING: Yes, the warrantless arrest of the appellant is
selling, dangerous drugs to the poseur-buyer. It was valid.
definitely legal for the buy-bust team to arrest, and search, SEC. 5. Arrest without warrant; when lawful. — A
them on the spot because a buy-bust operation is a peace officer or a private person may, without a warrant,
justifiable mode of apprehending drug pushers. A buy- arrest a person: (a) When, in his presence, the person to
bust operation is a form of entrapment whereby ways and be arrested has committed, is actually committing, or is
means are resorted to for the purpose of trapping and attempting to commit an offense; (b) When an offense has
capturing the lawbreakers in the execution of their in fact been committed and he has personal knowledge of
criminal plan. In this jurisdiction, the operation is legal and facts indicating that the person to be arrested has
has been proven to be an effective method of committed it; and, (c) When the person to be arrested is
apprehending drug peddlers, provided due regard to a prisoner who has escaped from a penal establishment
constitutional and legal safeguards is undertaken. or place where he is serving final judgment or temporarily
confined while his case is pending, or has escaped while
being transferred from one confinement to another.
PEOPLE V. UYBOCO
640 SCRA 146 JANUARY 19, 2011 The second instance of lawful warrantless arrest
covered by paragraph (b) cited above necessitates two
stringent requirements before a warrantless arrest can be
FACTS: Nimfa and her wards (Jeson Kevin and Jeson effected: (1) an offense has just been committed; and (2)
Kirby) were kidnapped by appellants and were brought to the person making the arrest has personal knowledge of
a house in Paranaque. While still in garage of the house, facts indicating that the person to be arrested has
Nimfa was able to sneak out of the car and place a call to committed it.
the secretary of her employer to inform the latter that they
were in Merville Subdivision. She came back to the car Records show that both requirements are present in the
undetected and after a while, she and her wards were instant case. The police officers present in Magallanes
asked to alight from the car and they were locked inside Commercial Center were able to witness the pay-off
the comfort room. which effectively consummates the crime of kidnapping.
They all saw appellant take the money from the car trunk
Through further negotiations, the parties agreed of Jepson. Such knowledge was then relayed to the other
to a ransom of P1.5 Million Pesos and the said ransom police officers stationed in Fort Bonifacio where appellant
was then brought by Jepson and taken by the kidnappers was expected to pass by.
in their agreed location.
Personal knowledge of facts must be based on probable
P/Insp. Escandor together with two other police cause, which means an actual belief or reasonable
officers brought a camera to cover the supposed pay-off. grounds of suspicion. The grounds of suspicion are
He took a total of 24 shots.20 He identified Macias reasonable when, in the absence of actual belief of the
together with appellant in Magallanes Commercial Center arresting officers, the suspicion that the person to be
and the latter as the one who took the ransom. arrested is probably guilty of committing the offense is
P/Supt. Chan was one of the team leaders dispatched to based on actual facts, i.e., supported by circumstances
take a video coverage on the supposed pay-off. He sufficiently strong in themselves to create the probable
witnessed the pay-off and identified appellant as the one cause of guilt of the person to be arrested. A reasonable
who took the bag containing the ransom money from the suspicion, therefore, must be founded on probable cause,
car trunk of Jepson. coupled with good faith on the part of the peace officers
making the arrest. Section 5, Rule 113 of the 1985 Rules
P/Supt. Cruz is assigned with the primary task of on Criminal Procedure does not require the arresting
apprehending the kidnappers of Dichaves’ children and officers to personally witness the commission of the
helper. At around 7:45 p.m., they heard on their radio that offense with their own eyes.
the suspect’s vehicle, a red Nissan Sentra was heading in
their direction. A few minutes later, they saw the red car It is sufficient for the arresting team that they were
and tailed it until it reached Dasmariñas Village in Makati. monitoring the pay-off for a number of hours long enough
They continuously followed the car inside the village. for them to be informed that it was indeed appellant, who
When said car slowed down, they blocked it and was the kidnapper. This is equivalent to personal
immediately approached the vehicle. They introduced knowledge based on probable cause.
themselves as police officers and accosted the suspect,
who turned out to be appellant.
B. EXCEPTION TO STRICT ENFORCEMENT: ILLEGAL PEOPLE V. SEMBRANO
POSSESSIONS OF GUNS OR DRUGS 628 SCRA 328 AUGUST 16, 2010

PEOPLE V. PENAFLORIDA FACTS: A buy bust operation was conducted. The police
551 SCRA 111 (APRIL 10, 2008)
officers waited until appellant arrived at around 5:00
o’clock in the afternoon. Upon appellant’s arrival, the
FACTS: SPO3 Vicente Competente (Competente) confidential informant introduced PO1 Manaol to him as
narrated that he received a tip from an asset that a bundle an interested buyer of shabu. PO1 Manaol handed the
of marijuana was being transported by appellant to two marked One Hundred Peso bills to appellant, who, in
Huyon-huyon from another barangay in Tigaon, turn, handed one (1) plastic sachet containing white
Camarines Sur. A team was then organized. crystalline substance to him. The transaction having been
consummated, PO1 Manaol executed their pre-arranged
The team boarded the police mobile car and signal and scratched his head. When the other members
proceeded to Sitio Nasulan in Barangay Huyon- of the team saw PO1 Manaol execute the pre-arranged
huyon.5 They overtook appellant who was on a bicycle. signal, they immediately proceeded to their location and
The police officers flagged appellant down and found arrested appellant.
marijuana wrapped in a cellophane and newspaper
together with other grocery items. The amount PO1 Manaol recovered the suspected shabu subject of
of P1550.00 was also found in appellant's possession. the sale from appellant and placed his initials JAM
The police officers confiscated these items and took thereon. PO1 Bagay was also able to retrieve the buy-
photographs thereof. Appellant was then brought to the bust money from appellant’s right hand. A follow-up frisk
headquarters where he was booked. on appellant resulted in the confiscation of two other
plastic sachets of white crystalline substance suspected
In assailing his conviction, appellant appellant resorts to to be shabu, from the right hand pocket of his shorts.
a challenge on the validity of his arrest predicated on lack Immediately after retrieving the evidence, PO1 Bagay
of a warrant of arrest. marked the confiscated sachets with his initials KJB.

ISSUE: W/N the arrest without warrant was valid ISSUE: W/N the appellant was illegally arrested

RULING: Yes, the arrest without warrant was valid. RULING: No, the appellant was not illegally arrested.
The OSG correctly justifies the failure to apply for On the legality of the warrantless arrest, We
an arrest warrant because at that point, time was of the reiterate that appellant was arrested during an
essence in appellant's apprehension, noting in the same entrapment operation where he was caught in flagrante
breath that there is no law requiring investigation and delicto selling shabu. When an arrest is made during an
surveillance upon receipt of tips from assets before entrapment operation, it is not required that a warrant be
conducting police operations. secured in line with the provisions of Rule 113, Section
The police was tipped off at around 1:00 p.m. that 5(a) of the Revised Rules of Court allowing warrantless
appellant was transporting marijuana to Huyon-huyon. arrests, to wit:
Certainly, they had no time to secure an arrest warrant as
appellant was already in transit and already committing a
crime. The arrest was effected after appellant was caught Section 5. Arrest without warrant; when lawful. – A peace
in flagrante delicto. He was seen riding his bicycle and officer or a private person may, without a warrant, arrest
carrying with him the contraband, hence, demonstrating a person:
that a crime was then already being committed. Under the
circumstances, the police had probable cause to believe (a) When, in his presence, the person to be arrested has
that appellant was committing a crime. Thus, the committed, is actually committing, or is attempting to
warrantless arrest is justified. commit an offense.

xxx

A buy-bust operation is a form of entrapment which in


recent years has been accepted as a valid and effective
mode of apprehending drug pushers. If carried out with
due regard for constitutional and legal safeguards, a buy-
bust operation, such as the one involving appellant,
deserves judicial sanction. Consequently, the warrantless Objections to the legality of arrests must, however, be
arrest and warrantless search and seizure conducted on made prior to the entry of plea at arraignment; otherwise,
the person of appellant were allowed under the they are considered waived.
circumstances. The search, incident to his lawful arrest,
needed no warrant to sustain its validity.Thus, there is no We have also ruled that an accused may be estopped
doubt that the sachets of shabu recovered during the from assailing the illegality of his arrest if he fails to move
legitimate buy-bust operation, are admissible and were for the quashing of the information against him before his
properly admitted in evidence against him. arraignment. And since the legality of an arrest affects
only the jurisdiction of the court over the person of the
Appellant’s defenses of denial and frame-up are both self- accused, any defect in his arrest may be deemed cured
serving and uncorroborated, and must fail in light of when he voluntarily submitted to the jurisdiction of the trial
straightforward and positive testimony of poseur-buyer court as what was done by the appellants in the instant
identifying him as the seller of shabu. The twin defenses case. Not only did they enter their pleas during
of denial and frame-up hold little weight vis-à-vis the arraignment, but they also actively participated during the
strong evidence gathered by the prosecution in proving trial which constitutes a waiver of any irregularity in their
his complicity to the offenses. arrest.

C. EFFECTS OF DECLARATION OG ILLEGAL ARREST

VALDEZ V. PEOPLE
538 SCRA 611 NOVEMBER 23, 2007
PEOPLE V. BIYOC
532 SCRA 611 DECEMBER 7, 2007
FACTS: Bautista testified that at around 8:00 to 8:30 p.m.
of 17 March 2003, he was conducting the routine patrol
FACTS: Herein appellant was charged with qualified rape
along the National Highway in Barangay San Benito
of AAA. After the incident, CCC thus immediately
Norte, Aringay, La Union together with Aratas and Ordoño
confronted AAA who did confirm that appellant had
when they noticed petitioner, lugging a bag, alight from a
inserted his penis inside her vagina that afternoon, and
mini-bus. The tanods observed that petitioner, who
that appellant had been doing the same act to her since
appeared suspicious to them, seemed to be looking for
she was nine years old. Incensed, CCC accompanied
something. They thus approached him but the latter
AAA the following day, December 6, 2000, to the
purportedly attempted to run away. They chased him, put
Department of Social Welfare and Development (DSWD)
him under arrest and thereafter brought him to the house
to report the incident.
of Barangay Captain Orencio Mercado (Mercado) where
From the DSWD, AAA and her mother, accompanied by he, as averred by Bautista, was ordered by Mercado to
a social worker, proceeded to the police station of San open his bag. Petitioner’s bag allegedly contained a pair
Mateo, Rizal where they lodged a complaint against of denim pants, eighteen pieces of eggplant and dried
appellant. At the police station, AAA and CCC were marijuana leaves wrapped in newspaper and cellophane.
interviewed by PO1 Florescita S. Javier. It was then that petitioner was taken to the police station
for further investigation.
PO1 Javier, together with AAA and CCC thereafter
proceeded to the family home, and on their way, they met
appellant. PO1 Javier at once informed him of his rights,
ISSUE: W/N the warrantless arrest effected against the
arrested him, and brought him to the police station. AAA's
appellant by the barangay tanods was unlawful
and CCC's statements were thereupon taken.

Appellant claims that his arrest was illegal


because a "warrantless arrest was effected even before RULING: Evidence shows that the arrest of appellant was
the statement of the private complainant was taken. unlawful. However, at the outset, we observe that
nowhere in the records can we find any objection by
petitioner to the irregularity of his arrest before his
ISSUE: W/N the objection of appellant was valid arraignment. Considering this and his active participation
in the trial of the case, jurisprudence dictates that
petitioner is deemed to have submitted to the jurisdiction
RULING: In the present case, appellant failed to question of the trial court, thereby curing any defect in his arrest.
the illegality of his arrest before entering his plea, hence, The legality of an arrest affects only the jurisdiction of the
he is deemed to have waived the same. court over his person.18 Petitioner’s warrantless arrest
therefore cannot, in itself, be the basis of his acquittal.
PEOPLE V. SANTOS the subsequent warrantless arrests were likewise legally
555 SCRA 578 JUNE 26, 2008 effected. Furthermore, any search resulting from the
lawful warrantless arrests was also valid, because the
FACTS: A buy bust operation was conducted. Upon appellants committed a crime in flagrante delicto; that is,
reaching the designated place, PO3 Luna and the the persons arrested committed a crime in the presence
informant alighted from their vehicle, while the rest of the of the arresting officers.
team were left inside. The informant then pointed to two
persons standing along the target area, one of whom was
Monching Labo, later identified as appellant Ramon Catoc Ople v Torres
y Picayo. After approaching, the informant introduced 293 SCRA 141 (1998)
PO3 Luna as a shabu customer to one of the persons,
later identified as appellant Jerry Santos y Macol. Facts:
Appellant Santos then asked PO3 Luna how much worth
of shabu he was buying and asked for the money. PO3 Petitioner Ople prays that this Court invalidate
Luna gave appellant Santos the buy-bust money Administrative Order No. 308 entitled “Adoption of a
consisting of a pre-marked P100.00 bill. Appellant Santos National Computerized Indentification Reference System”
handed this money to appellant Catoc, who took out from on two important constitutional grounds, viz: one, it is
his pocket a sealed transparent plastic sachet containing usurpation of the power of Congress to legislate, and two,
a white crystalline substance, which he handed back to it impermissibly intrudes on our citizenry’s protected zone
appellant Santos. When appellant Santos gave the plastic of privacy.
sachet to PO3 Luna, the latter nabbed the former and
introduced himself as a policeman. At that point, the other
members of the team arrived and likewise held and Issue:
arrested appellant Catoc.
Whether or not AO No. 308 violates the constitutional right
After their arraignment, the accused objected to the to privacy.
alleged irregularity of their arrest.

Ruling:
ISSUE: W/N the appellants did a timely objection to the
alleged irregularity of their arrest Yes. Zones of privacy are recognized and protected in our
laws. The Civil Code provides that “every person shall
respect the dignity, personality, privacy and peace of mind
of his neighbours and other persons” and punishes as
RULING: The claim of appellants that their warrantless
actionable torts several acts by a person of meddling and
arrests were illegal also lacks merit. The Court notes that
prying into the privacy of another. It also holds a public
nowhere in the records did we find any objection by
officer or employee or any private individual liable for
appellants to the irregularity of their arrests prior to their
damages for any violation of the rights and liberties of
arraignment. We have held in a number of cases that the
another person, and recognizes the privacy of letter and
illegal arrest of an accused is not a sufficient cause for
other private communications. The Revised Penal Code
setting aside a valid judgment rendered upon a sufficient
makes a crime the violation of secrets by an officer, the
complaint after a trial free from error; such arrest does not
revelation of trade and industrial secrets, and trespass to
negate the validity of the conviction of the accused. It is
dwelling. Invasion of privacy is an offense in special laws
much too late in the day to complain about the warrantless
like the Anti- Wiretapping Law, the Secrecy of Bank
arrest after a valid information has been filed, the accused
Deposits Act and the Intellectual Property Code. The
arraigned, trial commenced and completed, and a
Rules of Court on privileged communication likewise
judgment of conviction rendered against him.
recognize the privacy of certain information.
Nevertheless, our ruling in People v. Cabugatan provides
that: The rule is settled that an arrest made after an
entrapment does not require a warrant inasmuch as it is Unlike the dissenters, we rescind from the premise that
considered a valid warrantless arrest pursuant to Rule the right to privacy is a fundamental right guaranteed by
113, Section 5(a) of the Rules of Court, which states: the Constitution; hence, it is the burden of government to
SEC. 5. Arrest without warrant; when lawful. - A peace show that AO No. 308 is justified by some compelling
officer or a private person may, without a warrant, arrest state interest and that it is narrowly drawn. AO No. 308 is
a person: (a) When, in his presence, the person to be predicated on two considerations: (1) the need to provide
arrested has committed, is actually committing, or is our citizens and foreigners with the facility to conveniently
attempting to commit an offense. transact business with basic service and social security
providers and other government instrumentalities and (2)
As we have already declared the legality of the buy-bust
the need to reduce, if not totally eradicate, fraudulent
operation that was conducted by the police, it follows that
transaction and misrepresentations by person seeking PRIVACY
basic services. It is debatable whether these interests are
compelling enough to warrant the issuance of AO No.
308. But what is not arguable is the broadness, the IN RE: CAMILO SABIO
vagueness, the overbreadth of AO No. 308 which if 504 SCRA 704 OCTOBER 17, 2006
implemented will put our people’s right to privacy in clear
and present danger. FACTS: Two decades ago, on February 28, 1986, former
President Corazon C. Aquino installed her regime by
issuing Executive Order (E.O.) No. 1,1 creating the
AO No. 308 falls short of assuring that personal Presidential Commission on Good Government (PCGG).
information which will be gathered about our people will She entrusted upon this Commission the herculean task
only be processed for unequivocally specified purposes. of recovering the ill-gotten wealth accumulated by the
That lack of proper safeguards in this regard od AO No. deposed President Ferdinand E. Marcos, his family,
308 may interfere with individual’s liberty of abode and relatives, subordinates and close associates.2 Section 4
travel by enabling authorities to track down his movement; (b) of E.O. No. 1 provides that: "No member or staff of the
it may also enable unscrupulous persons to access Commission shall be required to testify or produce
confidential information and circumvent the right against evidence in any judicial, legislative or administrative
self-incrimination; it may pave the way for “fishing proceeding concerning matters within its official
expeditions” by government authorities and evade the cognizance." Apparently, the purpose is to ensure
right against unreasonable searches and seizures. The PCGG's unhampered performance of its task.
possibilities of abuse and misuse of PRN, biometrics and
computer technology are accentuated when we consider On February 20, 2006, Senator Miriam Defensor
that the individual lacks control over what can be read or Santiago introduced Philippine Senate Resolution No.
placed on his ID, much less verify the correctness of the 455 (Senate Res. No. 455),4 "directing an inquiry in aid of
date encoded. They threaten the very abuses that the Bill legislation on the anomalous losses incurred by the
of Rights seeks to prevent. Philippines Overseas Telecommunications Corporation
(POTC), Philippine Communications Satellite Corporation
(PHILCOMSAT), and PHILCOMSAT Holdings
Corporation (PHC) due to the alleged improprieties in
The right to privacy is one of the most threatened rights of
their operations by their respective Board of Directors.
man living in a mass society. The threats emanate from
various sources – governments, journalists, employers, On May 8, 2006, Chief of Staff Rio C. Inocencio,
social scientists, etc. In the case at bar, the threat comes under the authority of Senator Richard J. Gordon, wrote
from the executive branch of government which by issuing Chairman Camilo L. Sabio of the PCGG, one of the herein
AO No. 308 pressures the people to surrender their petitioners, inviting him to be one of the resource persons
privacy by giving information about themselves on the in the public meeting jointly conducted by the Committee
pretext that it will facilitate delivery of basic services. on Government Corporations and Public Enterprises and
Given the record -keeping power of the computer, only the Committee on Public Services. The purpose of the public
indifferent fail to perceive the danger that AO No. 308 meeting was to deliberate on Senate Res. No. 455. On
gives the government the power to compile a devastating May 9, 2006, Chairman Sabio declined the invitation
dossier against unsuspecting citizens. It is timely to take because of prior commitment. At the same time, he
not of the well-worded warning of Kalvin, Jr., “the invoked Section 4(b) of E.O. No. 1 earlier quoted.
disturbing result could be that everyone will live burdened Moreover, even with further notices, Sabio still did not
by an unerasable record of his past and his limitations. In comply.
a way, the threat is that because of its record-keeping, the
society will have lost its benign capacity to forget.” Moreover, n G.R. No. 174177, petitioners Philcomsat
Oblivious to this counsel, the dissents still say we shout Holdings Corporation and its directors and officers
not be too quick in labelling the right to privacy as a alleged, among others, that the subpoenae violated
fundamental right. We close with the statement that the petitioners' rights to privacy and against self-incrimination.
right to privacy was not engraved in our Constitution for
flattery.
ISSUE: W/N there is a violation of their right to privacy

RULING: No, their right to privacy has not been violated


by the Senate Committees.

Zones of privacy are recognized and protected in our


laws. Within these zones, any form of intrusion is
impermissible unless excused by law and in accordance
with customary legal process. The meticulous regard we administration. In Valmonte v. Belmonte, the Court
accord to these zones arises not only from our conviction remarked that as public figures, the Members of the
that the right to privacy is a "constitutional right" and former Batasang Pambansa enjoy a more limited right to
"the right most valued by civilized men," but also from our privacy as compared to ordinary individuals, and their
adherence to the Universal Declaration of Human Rights actions are subject to closer scrutiny. Taking this into
which mandates that, "no one shall be subjected to consideration, the Court ruled that the right of the people
arbitrary interference with his privacy" and "everyone has to access information on matters of public concern
the right to the protection of the law against such prevails over the right to privacy of financial transactions.
interference or attacks."
Under the present circumstances, the alleged anomalies
Our Bill of Rights, enshrined in Article III of the in the PHILCOMSAT, PHC and POTC, ranging in millions
Constitution, provides at least two guarantees that of pesos, and the conspiratorial participation of the PCGG
explicitly create zones of privacy. It highlights a person's and its officials are compelling reasons for the Senate to
"right to be let alone" or the "right to determine what, how exact vital information from the directors and officers of
much, to whom and when information about himself shall Philcomsat Holdings Corporations, as well as from
be disclosed." Section 2 guarantees "the right of the Chairman Sabio and his Commissioners to aid it in
people to be secure in their persons, houses, papers and crafting the necessary legislation to prevent corruption
effects against unreasonable searches and seizures of and formulate remedial measures and policy
whatever nature and for any purpose." Section 3 renders determination regarding PCGG's efficacy. There being no
inviolable the "privacy of communication and reasonable expectation of privacy on the part of those
correspondence" and further cautions that "any evidence directors and officers over the subject covered by Senate
obtained in violation of this or the preceding section shall Res. No. 455, it follows that their right to privacy has not
be inadmissible for any purpose in any proceeding." been violated by respondent Senate Committees.

In evaluating a claim for violation of the right to privacy, a


court must determine whether a person has exhibited a
SJS V. DDB
reasonable expectation of privacy and, if so, whether that 570 SCRA 410 NOVEMBER 3, 2008
expectation has been violated by unreasonable
government intrusion.50 Applying this determination to
these cases, the important inquiries are: first, did the FACTS: In these kindred petitions, the constitutionality of
directors and officers of Philcomsat Holdings Corporation Section 36 of Republic Act No. (RA) 9165, otherwise
exhibit a reasonable expectation of privacy?; known as the Comprehensive Dangerous Drugs Act of
and second, did the government violate such 2002, insofar as it requires mandatory drug testing of
expectation? candidates for public office, students of secondary and
tertiary schools, officers and employees of public and
The answers are in the negative. Petitioners were invited private offices, and persons charged before the
in the Senate's public hearing to deliberate on Senate prosecutor's office with certain offenses, among other
Res. No. 455, particularly "on the anomalous losses personalities, is put in issue.
incurred by the Philippine Overseas Telecommunications
Corporation (POTC), Philippine Communications Satellite
Corporation (PHILCOMSAT), and Philcomsat Holdings
As far as pertinent, the challenged section reads as
Corporations (PHC) due to the alleged improprieties in the
follows:
operations by their respective board of directors."
Obviously, the inquiry focus on petitioners' acts
committed in the discharge of their duties as officers and
directors of the said corporations, particularly Philcomsat SEC. 36. Authorized Drug Testing. - Authorized drug
Holdings Corporation. Consequently, they have no testing shall be done by any government forensic
reasonable expectation of privacy over matters involving laboratories or by any of the drug testing laboratories
their offices in a corporation where the government has accredited and monitored by the DOH to safeguard the
interest. Certainly, such matters are of public concern and quality of the test results. x x x The drug testing shall
over which the people have the right to information. employ, among others, two (2) testing methods, the
screening test which will determine the positive result as
This goes to show that the right to privacy is not absolute well as the type of drug used and the confirmatory test
where there is an overriding compelling state interest. which will confirm a positive screening test. x x x The
In Morfe v. Mutuc, the Court, in line with Whalen v. following shall be subjected to undergo drug testing:
Roe,employed the rational basis relationship test when it
held that there was no infringement of the individual's right
to privacy as the requirement to disclosure information is
xxxx
for a valid purpose, i.e., to curtail and minimize the
opportunities for official corruption, maintain a standard of
honesty in public service, and promote morality in public
(c) Students of secondary and tertiary schools. - Students For one, the provisions constitute undue delegation of
of secondary and tertiary schools shall, pursuant to the legislative power when they give unbridled discretion to
related rules and regulations as contained in the school's schools and employers to determine the manner of drug
student handbook and with notice to the parents, undergo testing. For another, the provisions trench in the equal
a random drug testing x x x; protection clause inasmuch as they can be used to harass
a student or an employee deemed undesirable. And for
(d) Officers and employees of public and private offices. - a third, a person’s constitutional right against
Officers and employees of public and private offices, unreasonable searches is also breached by said
whether domestic or overseas, shall be subjected to provisions.
undergo a random drug test as contained in the
company's work rules and regulations, x x x for purposes (Atty. Laserna v. DDB & PDEA | G.R. 158633)
of reducing the risk in the workplace. Any officer or
employee found positive for use of dangerous drugs shall Petitioner Atty. Manuel J. Laserna, Jr., as citizen and
be dealt with administratively which shall be a ground for taxpayer, also seeks in his Petition for Certiorari and
suspension or termination, subject to the provisions of Prohibition under Rule 65 that Sec. 36(c), (d), (f), and (g)
Article 282 of the Labor Code and pertinent provisions of of RA 9165 be struck down as unconstitutional for
the Civil Service Law; infringing on the constitutional right to privacy, the right
against unreasonable search and seizure, and the right
xxxx against self-incrimination, and for being contrary to the
due process and equal protection guarantees.
(f) All persons charged before the prosecutor's office with
a criminal offense having an imposable penalty of
imprisonment of not less than six (6) years and one (1)
day shall undergo a mandatory drug test; ISSUE: W/N paragraphs (c), (d), (f), and (g) of Sec. 36,
RA 9165 violate the right to privacy
(g) All candidates for public office whether appointed or
elected both in the national or local government shall
undergo a mandatory drug test. RULING: The Court is of the view and so holds that the
provisions of RA 9165(c) requiring mandatory, random,
and suspicionless drug testing of students are
(Pimentel v. COMELEC | G.R. No. 16158) constitutional. Indeed, it is within the prerogative of
educational institutions to require, as a condition for
On Dec. 23, 2003, the COMELEC issued Resolution No. admission, compliance with reasonable school rules and
6486, prescribing the rules and regulations for the regulations and policies. To be sure, the right to enroll is
mandatory drug testing of candidates for public office in not absolute; it is subject to fair, reasonable, and equitable
connection with the May 2004 elections. Pimentel claims requirements. A random drug testing of students in
that Sec. 36 (g) of RA 9165 and COMELEC Resolution secondary and tertiary schools is not only acceptable, but
No. 6486 illegally impose an additional qualification on may even be necessary if the safety and interest of the
candidates for senator. He points out that, subject to the student population, doubtless a legitimate concern of the
provisions on nuisance candidates, a candidate for government, are to be promoted and protected.
senator needs only to meet the qualifications laid down in
Sec. 3, Art. VI of the Constitution, to wit: (1) citizenship,
(2) voter registration, (3) literacy, (4) age, and (5)
residency. Beyond these stated qualification Just as in the case of secondary and tertiary level
requirements, candidates for senator need not possess students, the mandatory but random drug test prescribed
any other qualification to run for senator and be voted by Sec. 36 of RA 9165(d) for officers and employees of
upon and elected as member of the Senate. The public and private offices is justifiable, albeit not exactly
Congress cannot validly amend or otherwise modify these for the same reason. The Court notes in this regard that
qualification standards, as it cannot disregard, evade, or petitioner SJS, other than saying that “subjecting almost
weaken the force of a constitutional mandate, or alter or everybody to drug testing, without probable cause, is
enlarge the Constitution. unreasonable, an unwarranted intrusion of the individual
right to privacy,” has failed to show how the mandatory,
random, and suspicionless drug testing under Sec. 36(c)
and (d) of RA 9165 violates the right to privacy and
(SJS v. DDM & PDEA | G.R. 157870) constitutes unlawful and/or unconsented search under
In its Petition for Prohibition under Rule 65, petitioner Art. III, Secs. 1 and 2 of the Constitution. Petitioner
Social Justice Society (SJS), a registered political party, Laserna’s lament is just as simplistic, sweeping, and
seeks to prohibit the Dangerous Drugs Board (DDB) and gratuitous and does not merit serious consideration.
the Philippine Drug Enforcement Agency (PDEA) from
enforcing paragraphs (c), (d), (f), and (g) of Sec. 36 of RA
9165 on the ground that they are constitutionally infirm.
The essence of privacy is the right to be left alone. In testing can never be random or suspicionless. The ideas
context, the right to privacy means the right to be free from of randomness and being suspicionless are antithetical to
unwarranted exploitation of one’s person or from intrusion their being made defendants in a criminal complaint.
into one’s private activities in such a way as to cause They are not randomly picked; neither are they beyond
humiliation to a person’s ordinary sensibilities; and while suspicion. When persons suspected of committing a
there has been general agreement as to the basic crime are charged, they are singled out and are
function of the guarantee against unwarranted search, impleaded against their will. The persons thus charged,
“translation of the abstract prohibition against by the bare fact of being haled before the prosecutor’s
‘unreasonable searches and seizures’ into workable office and peaceably submitting themselves to drug
broad guidelines for the decision of particular cases is a testing, if that be the case, do not necessarily consent to
difficult task,” to borrow from C. Camara v. Municipal the procedure, let alone waive their right to privacy. To
Court. Authorities are agreed though that the right to impose mandatory drug testing on the accused is a
privacy yields to certain paramount rights of the public blatant attempt to harness a medical test as a tool for
and defers to the state’s exercise of police power. criminal prosecution, contrary to the stated objectives of
RA 9165. Drug testing in this case would violate a
persons’ right to privacy guaranteed under Sec. 2, Art. III
As the warrantless clause of Sec. 2, Art III of the of the Constitution. Worse still, the accused persons are
Constitution is couched and as has been held, veritably forced to incriminate themselves.
“reasonableness” is the touchstone of the validity of a
government search or intrusion. While every officer and
employee in a private establishment is under the law PRIVACY OF COMMUNICATION
deemed forewarned that he or she may be a possible
subject of a drug test, nobody is really singled out in
advance for drug testing. The goal is to discourage drug Zulueta v CA
use by not telling in advance anyone when and who is to 253 SCRA 699 (1996)
be tested. And as may be observed, Sec. 36(d) of RA
9165 itself prescribes what, in Ople, is a narrowing FACTS:
ingredient by providing that the employees concerned
shall be subjected to “random drug test as contained in Petitioner Cecilia Zulueta is the wife of private respondent
the company’s work rules and regulations x x x for Alfredo Martin. On March 26, 1982, petitioner entered the
purposes of reducing the risk in the work place.” It is to be clinic of her husband, a doctor of medicine, and in the
noted the very reason RA 9165 was enacted is to presence of her mother, a driver and private respondent’s
safeguard the well-being of the citizens from the secretary, forcibly opened the drawers and cabinet in her
deleterious effects of dangerous drugs. husband’s clinic and took 157 documents consisting of
private correspondence between Dr. Martin and his
alleged paramours, greeting cards, cancelled checks,
Paragraph (f) of RA 9165 was declared unconstitutional diaries, Dr. Martin’s passport, and photographs. The
by the Court. Unlike the situation covered by Sec. 36(c) documents and papers were seized for use in evidence in
and (d) of RA 9165, the Court finds no valid justification a case for legal separation and for disqualification from
for mandatory drug testing for persons accused of crimes. the practice of medicine which petitioner had filed against
In the case of students, the constitutional viability of the her husband.
mandatory, random, and suspicionless drug testing for
students emanates primarily from the waiver by the
students of their right to privacy when they seek entry to ISSUE: Whether or not the documents and papers in
the school, and from their voluntarily submitting their question are inadmissible in evidence
persons to the parental authority of school authorities. In
the case of private and public employees, the
constitutional soundness of the mandatory, random, and RULING:
suspicionless drug testing proceeds from the
reasonableness of the drug test policy and requirement. YES. The constitutional injunction declaring “the privacy
of communication and correspondence to be inviolable” is
no less applicable simply because it is the wife (who
The Court finds the situation entirely different in the case thinks herself aggrieved by her husband’s infidelity) who
of persons charged before the public prosecutor’s office is the party against whom the constitutional provision is to
with criminal offenses punishable with six (6) years and be enforced. The only exception to the prohibition in the
one (1) day imprisonment. The operative concepts in the Constitution is if there is a “lawful order from a court or
mandatory drug testing are “randomness” and when public safety or order requires otherwise, as
“suspicionless.” In the case of persons charged with a prescribed by law.” Any violation of this provision renders
crime before the prosecutor’s office, a mandatory drug
the evidence obtained inadmissible “for any purpose in No. The opening and reading of the detainees’ letters in
any proceeding.” the present case did not violate the detainees’ right to
privacy of communication. The letters were not in sealed
envelope. The inspection of the folded letters is a valid
The intimacies between husband and wife do not justify measure as it serves the same purpose as the opening of
any one of them in breaking the drawers and cabinets of sealed letters for the inspection of contraband.
the other and in ransacking them for any tell-tale evidence
of marital infidelity. A person, by contracting marriage,
does not shed his/her integrity or his right to privacy as an The letters alleged to have been read by the ISAFP
individual and the constitutional protection is ever authorities were not confidential letters between the
available to him or to her. detainees and their lawyers. The petitioner who received
the letters from detainees Trillanes and Maestrecampo
was merely acting as the detainees’ personal courier and
The law insures absolute freedom of communication not as their counsel when he received the letters for
between the spouse by making it privileged. Neither mailing. In the present case, since the letters were not
husband nor wife may testify for or against the other confidential communication between the detainees and
without the consent of the affected spouse while the their lawyers, the officials of the ISAFP Detention Center
marriage subsists. Neither may be examined without the could read the letters. If the letters are marked confidential
consent if the other as to any communication received in communication between the detainees and their lawyers,
confidence by one from the other during the marriage, the detention officials should not read the letters but only
save for specified exceptions. But one thing is freedom of open the envelopes for inspection in the presence of the
communication; quite another is a compulsion for each detainees.
one to share what one knows with the other. And this has
nothing to do with the duty of fidelity that each owes to the
other. That a law is required before an executive officer could
intrude on a citizen’s privacy rights is a guarantee that is
available only to the public at large but not to persons who
In Re Alejano are detained or imprisoned. The right to privacy of those
468 SCRA 188 (2005) detainees is subject to Section 4 of RA 7438, as well as
to the limitations inherent in lawful detention or
imprisonment. By the very fact of their detention, pre-trial
FACTS:
detainees and convicted prisoners have a diminished
This case results from the Oakwood incident in July 2003. expectation of privacy rights.

One of the petitioners’ argument is that the officials of the


ISAFP Detention Center violated the detainees’ right to
privacy when ISAFP officials opened and read the letters
handed by detainees Trillanes and Maestrecampo to one
of the petitioners for mailing. Petitioners point out that the
letters were not in a sealed envelope but simply folded
because there were no envelopes in the ISAFP Detention
Center. Petitioners contend that the Constitution prohibits
the infringement of a citizen’s privacy rights unless
authorized by law. The Solicitor General does not deny
that the ISAFP officials opened the letters.

ISSUE:

Whether or not the officials of ISAFP Detention Center


violated detainees’ right to privacy of communication.

RULING:
III. FREEDOM OF EXPRESSION RULING:

No. Anti-wiretapping law does not endanger the national


PRIOR RESTRAINTS/SUBSEQUENT PUNISHMENT
security of the State. We rule that not every violation of a
law will justify straitjacketing the exercise of freedom of
speech and of the press. Our laws are of different kinds
Chavez v. Gonzales and doubtless, some of them provide norms of conduct
which even if violated have only an adverse effect on a
FACTS: As a consequence of the public release of copies persons private comfort but does not endanger national
of the “Hello Garci” compact disc audiotapes involving a security. There are laws of great significance but their
wiretapped mobile phone conversation between then- violation, by itself and without more, cannot support
President Gloria Arroyo and Comelec Commissioner suppression of free speech and free press. In fine,
Virgilio Garcillano, respondent DOJ Secretary Gonzales violation of law is just a factor, a vital one to be sure, which
warned reporters that those who had copies of the CD should be weighed in adjudging whether to restrain
and those broadcasting or publishing its contents could freedom of speech and of the press. The totality of the
be held liable under the Anti-Wiretapping Act. He also injurious effects of the violation to private and public
stated that persons possessing or airing said tapes were interest must be calibrated in light of the preferred status
committing a continuing offense, subject to arrest by accorded by the Constitution and by related international
anybody. Finally, he stated that he had ordered the covenants protecting freedom of speech and of the press.
National Bureau of Investigation to go after media In calling for a careful and calibrated measurement of the
organizations “found to have caused the spread, the circumference of all these factors to determine
playing and the printing of the contents of a tape.” compliance with the clear and present danger test, the
Court should not be misinterpreted as devaluing violations
of law. By all means, violations of law should be
vigorously prosecuted by the State for they breed their
Meanwhile, respondent NTC warned in a press release own evil consequence. But to repeat, the need to prevent
all radio stations and TV network owners/operators that their violation cannot per se trump the exercise of free
the conditions of the authorization and permits issued to speech and free press, a preferred right whose breach
them by government like the Provisional Authority and/or can lead to greater evils. For this failure of the
Certificate of Authority explicitly provides that they shall respondents alone to offer proof to satisfy the clear and
not use their stations for the broadcasting or telecasting present danger test, the Court has no option but to uphold
of false information or willful misrepresentation. The NTC the exercise of free speech and free press. There is no
stated that the continuous airing or broadcast of the “Hello showing that the feared violation of the anti-wiretapping
Garci” taped conversations by radio and TV stations is a law clearly endangers the national security of the State.
continuing violation of the Anti-Wiretapping Law and the
conditions of the Provisional Authority and/or Certificate
of Authority. It warned that their broadcast/airing of such
false information and/or willful misrepresentation shall be
a just cause for the suspension, revocation and/or
cancellation of the licenses or authorizations issued to the
said media establishments.

Subsequently, a dialogue was held between the NTC and


the Kapisanan ng mga Brodkaster sa Pilipinas (KBP)
which resulted in the issuance of a Joint Press Statement
which stated, among others, that the supposed
wiretapped tapes should be treated with sensitivity and
handled responsibly.

ISSUE:

Whether or not there’s a purported violation of law such


as the Anti-Wiretapping Law justify straitjacketing the
exercise of freedom of speech and of the press.
Newsounds v. Dy the 2001 elections, Bombo Radyo was aggressive in
exposing the widespread election irregularities in Isabela
that appear to have favored Respondent Dy and other
FACTS:
members of the Dy political dynasty. Respondents efforts
Petitioners are authorized by law to operate radio stations to close petitioners radio station clearly intensified
in Cauayan City, and had been doing so for some years immediately before the May 2004 elections, where a
undisturbed by local authorities. Beginning in 2002, former employee of DZNC Bombo Radyo, Grace Padaca,
respondents in their official capacities impeded the ability was mounting a credible and ultimately successful
of petitioners to freely broadcast, if not broadcast at all. challenge against the incumbent Isabela governor, who
These actions have ranged from withholding permits to happened to be the brother of respondent Dy. It also
operate to the physical closure of those stations. bears notice that the requirements required of petitioners
Petitioner were required to submit requirements for the by the Cauayan City government are frankly beyond the
reclassification of the land wherein the said stations are pale and not conventionally adopted by local
operating. Such requirements then as required were governments throughout the Philippines.
never listed in the list of requirements in the
renewal/application of any permit issued by Cauayan City.
And notably, petitioners had never been required to Soriano v. Laguardia
submit such papers before.

FACTS:

ISSUE: W/N the right of petitioner’s freedom of Petitioner Eliseo S. Soriano, the former Ang Dating Daan
expression was violated by the respondents by the host, uttered the following statements in his TV program
closure of the station. against Michael Sandoval (Iglesia ni Cristo’s minister and
regular host of the TV program Ang Tamang Daan):

Lehitimong anak ng demonyo! Sinungaling ! Gago ka


RULING: talaga, Michael! Masahol ka pa sa
YES. The circumstances of this case dictate that
respondents’ closure of petitioners’ radio stations is
clearly tainted with ill motives. It must be pointed out that Espinosa, Forto, Gascon, Ibrahim, Sidaya, Valdevieso,
in the 2001 elections, Bombo Radyo was aggressive in Villamor, Zapanta
exposing the widespread election irregularities in Isabela
that appear to have favored respondent Dy and other
members of the Dy political dynasty. Bombo Radyo is a putang babae, o di ba? ‘Yung putang babae, ang
rival station of DWDY who is also owned by the family DY. gumagana lang doon, ‘yung ibaba, dito kay Michael, ang
Also, in an article found in the Philippine Daily inquirer gumagana ang itaas, o di ba? O, masahol pa sa putang
dated February 2004, respondent Dy was quoted as babae ‘yan. Sobra ang kasinungalingan ng mga
saying that he will "disenfranchise the radio station." Such demonyong ito.
statement manifests and confirms that respondents’
denial of petitioners’ renewal applications on the ground
that the Property is commercial is merely a pretext and
As a result, The MTRCB initially slapped Soriano’s Ang
that their real agenda is to remove petitioners from
Dating Daan, with a 20-day preventive suspension after a
Cauayan City and suppress the latter’s voice. This is a
preliminary conference. Later, in a decision, it found him
blatant violation of the petitioners constitutional right to
liable for his utterances, and was imposed a three-month
press freedom.
suspension from his TV program Ang Dating Daan.
Soriano challenged the order of the MTRCB.

The requirements required of petitioners are frankly


beyond the pale and not conventionally adopted by local
ISSUE:
governments throughout the Philippines. Prior to 2002,
petitioners had not been frustrated in securing the various W/N the 20-day preventive suspension order of MTRCB
local government requirements for the operation of their was an unconstitutional abridgement of the freedom of
stations. It was only in the beginning of 2002, after the speech and expression and an impermissible prior
election of respondent Ceasar Dy as mayor of Cauayan, restraint.
that the local government started to impose these new
requirements substantiating the conversion of CDCs
property for commercial use. Petitioners admit that during
RULING: Borjal v CA, Wenceslao
301 SCRA 1 (1999)
NO. It has been established in this jurisdiction that
unprotected speech or low-value expression refers to
libelous statements, obscenity or pornography, false or FACTS:
misleading advertisement, insulting or fighting words, i.e.,
those which by their very utterance inflict injury or tend to
incite an immediate breach of peace and expression Francisco Wenceslao was one of the heroes of EDSA,
endangering national security. and the chairman and director of the First National
Conference on Land Transportation. Apparently this
The Court finds that petitioners statement can be treated conference attracted media mileage and drew public
as obscene, at least with respect to the average child. attention.
Hence, it is, in that context, unprotected speech. Where a
language is categorized as indecent, as in petitioners Sometime in 1989, a series of articles written by petitioner
utterances on a general-patronage rated TV program, it Borjal was published on different dates in his column
may be readily proscribed as unprotected speech. Jaywalker. The article dealt with anomalous activities
such as solicitations and unauthorized use of names of
Pres. Cory Aquino of an “organizer of a conference”
CONTENT-BASED RESTRICTIONS without naming or identifying Wenceslao. Neither did it
refer to the FNCLT as the conference therein mentioned.
A. FREEDOM OF EXPRESSION AND NATIONAL SECURITY
Wenceslao filed a case for libel against petitioner Borjal.
Soliven Vs Makasiar; Beltran vs Makasiar
G.R. No. 82585 (November 14, 1988)
ISSUE: Whether or not Borjal is protected by freedom of
FACTS: Soliven broadcasted the statement that expression
President Cory hid under the bed during a coup d’etat.
Luis Beltran is among the petitioners in this case. He,
together with others, was charged with libel by the then RULING:
president Corazon Aquino. Cory herself filed a complaint-
affidavit against him and others. Beltran questioned the
issuance of the warrant. He said the judge did not Yes. There is no denying that the questioned articles dealt
“personally examine” the witnesses, which is with matters of public interest.
unconstitutional. In an arrest warrant, the judge may rely
on the evidences. He need not call the witnesses. The court held that Wenceslao is deemed a public figure.
A public figure is a person who, by his accomplishments,
fame, mode of living, or by adopting a profession or calling
ISSUE: W/N petitioners here can be sued for libel. which gives the public a legitimate interest in his doings,
his affairs and his character, has become a "public
personage."
RULING: No. Libel is an actual and malicious imputation The FNCLT was air undertaking infused with public
of a prime vice or defect against any person (RPC). Even interest. It was promoted as a joint project of the
dead persons are subject to libel. The SC held to look at government and the private sector, and organized by top
the context of his statement. He was referring to her government officials and prominent businessmen. For
official functions, hence it is protected by freedom of this reason, it attracted media mileage and drew public
speech. If you’re a public official, people are allowed to attention not only to the conference itself but to the
comment on your official functions. What is not covered personalities behind as well. As its Executive Director and
by freedom of speech are comments on private matters. spokesman, private respondent consequently assumed
the status of a public figure.
Vasquez v. CA Tulfo v. People
314 SCRA 460 (2000) 565 SCRA 283 (2008)

FACTS: FACTS:

Rodolfo Vasquez is a resident of the Tondo Foreshore Atty. Carlos Ding So of the Bureau of Customs filed and
Area. Sometime in April 1986, he and some 37 families charged petitioner Erwin Tulfo, as author or writer of the
from the area went to see then National Housing Authority daily tabloid Remate, with the crime libel. That private
General Manager Lito Atienza regarding their complaint respondent was indicated as an extortionist, a corrupt
against Brgy. chairman Jaime Olmedo about his alleged public official, smuggler and having his wealth illegally.
engagement in land grabbing. After their meeting with Tulfo had no reliable source.
Atienza and other NHA Officials, petitioner and his
companions were met and interviewed by a newspaper
reporters at the NHA compound concerning their RTC found petitioners guilty if the crime of libel. CA
complaint. The next day the news articles were published affirmed the judgement of the trial court. Hence, Tulfo
in the paper Ang Tinig ng Masa. In the article, published appealed and raised that the said article is qualified
the complaint of Vasquez against Brgy. Chairman privilege communication and is written without malice.
Olmedo about the said land grabbing.

ISSUE: Whether or not the article is defamatory and


Based on the newspaper article, Olmedo filed a complaint petitioner is liable for libel
for libel against petitioner alleging that the latter’s
statements cast aspersion on him and damaged his
reputation.
RULING:

ISSUE: Whether or not petitioner is liable for libel


Yes. The Court has long respected the freedom of the
press, and upheld the same when it came to
commentaries made on public figures and matters of
RULING: public interest. Even in cases wherein the freedom of the
press was given greater weight over the rights of
individuals, the Court, however, has stressed that such
No. The standard actual malice in New York Times vs freedom is not absolute and unbounded. The exercise of
Sullivan is to be applied in criminal prosecutions for libel. this right or any right enshrined in the Bill of Rights,
indeed, comes with an equal burden of responsible
If the defamatory statements is made against a public exercise of that right. The recognition of a right is not free
official with respect to the discharge of his official duties license for the one claiming it to run roughshod over the
and functions and the truth of the allegation is shown, the rights of others.
accused will be entitled to an acquittal even though he
does not prove that the imputation was published with
good motives and for justifiable ends.
The exercise of press freedom must be done "consistent
For that matter, even if the defamatory statement is false, with good faith and reasonable care. The doctrine of fair
no liability can attach if it relates to official conduct, unless comment means that while in general every discreditable
the public official concerned proves that the statements imputation publicly made is deemed false, because every
was made with actual malice — that is, with knowledge man is presumed innocent until his guilt is judicially
that it was false or with reckless disregard of whether it proved, and every false imputation is deemed malicious,
was false or not. nevertheless, when the discreditable imputation is
directed against a public person in his public capacity, it
is not necessarily actionable. In order that such
discreditable imputation to a public official may be
actionable, it must either be a false allegation of fact or a
comment based on a false supposition. If the comment is
an expression of opinion, based on established facts, then
it is immaterial that the opinion happens to be mistaken,
as long as it might reasonably be inferred from the facts.
B. FREEDOM OF EXPRESSION AND THE RIGHT OF RULING:
PRIVACY
YES. The production and filming by petitioners of the
projected motion picture "The Four Day Revolution" does
AYER PRODUCTIONS vs. not, in the circumstances of this case, constitute an
HON.IGNACIO M. CAPULONG and JUAN PONCE ENRILE unlawful intrusion upon private respondent's "right of
privacy."
FACTS:

The subject matter of "The Four Day Revolution" relates


to the non- bloody change of government that took place
Petitioner Ayer Productions is a movie production
at Epifanio de los Santos Avenue in February 1986, and
company which 1 envisioned, sometime in 1987, the for
the train of events which led up to that denouement.
commercial viewing and for Philippine and international
Clearly, such subject matter is one of public interest and
release, the histolic peaceful struggle of the Filipinos at
concern. Indeed, it is, petitioners' argue, of international
EDSA. Petitioners discussed this Project with a local
interest. The subject thus relates to a highly critical stage
movie producer who suggested that they consult with the
in the history of this countryand as such, must be
appropriate government agencies and also with General
regarded as having passed into the public domain and as
FVR and Senator Enrile, who had played major roles in
an appropriate subject for speech and expression and
the events proposed to be filmed. The proposed motion
coverage by any form of mass media. The subject mater,
picture entitled "The Four Day Revolution" was endorsed
as set out in the synopsis provided by the petitioners and
by the MTRCB as well as the other government agencies
quoted above, does not relate to the individual life and
consulted. General Fidel Ramos also signified his
certainly not to the private life of private respondent Ponce
approval of the intended film production. In a letter,
Enrile. Unlike in Lagunzad, which concerned the life story
petitioner informed private respondent Juan Ponce Enrile
of Moises Padilla necessarily including at least his
about the projected motion picture enclosing a synopsis
immediate family, what we have here is not a film
of it. On 21 December 1987, private respondent Enrile
biography, more or less fictionalized, of private
replied that "[he] would not and will not approve of the
respondent Ponce Enrile. "The Four Day Revolution" is
use, appropriation, reproduction and/or exhibition of his
not principally about, nor is it focused upon, the man Juan
name, or picture, or that of any member of his family in
Ponce Enrile' but it is compelled, if it is to be historical, to
any cinema or television production, film or other medium
refer to the role played by Juan Ponce Enrile in the
for advertising or commercial exploitation" and further
precipitating and the constituent events of the change of
advised petitioners that 'in the production, airing, showing,
government in February 1986.
distribution or exhibition of said or similar film, no
reference whatsoever (whether written, verbal or visual)
should not be made to [him] or any member of his family,
much less to any matter purely personal to them. It A public figure has been defined as a person who, by his
appears that petitioners acceded to this demand and the accomplishments, fame, or mode of living, or by adopting
name of private respondent Enrile was deleted from the a profession or calling which gives the public a legitimate
movie script, and petitioners proceeded to film the interest in his doings, his affairs, and his character, has
projected motion picture. become a 'public personage.' He is, in other words, a
celebrity. Obviously to be included in this category are
those who have achieved some degree of reputation by
appearing before the public, as in the case of an actor, a
Enrile filed a Complaint with application for TRO and Wilt
professional baseball player, a pugilist, or any other
of Pretion with the RTC seeking to enjoin petitioners from
entertainment.
producing the movie alleging that without private
respondent's consent and over his objection, producing
the movie constitutes an obvious violation of his right of
privacy which was granted. Private respondent is a "public figure" precisely because,
inter alia, of his participation as a principal actor in the
culminating events of the change of government in
February 1986. Because his participation therein was
ISSUE:
major in character, a film reenactment of the peaceful
W/N granting of the Writ of Preliminary Injunction against revolution that fails to make reference to the role played
the petitioners is a restraint on their right of the free by private respondent would be grossly unhistorical. The
expression? right of privacy of a "public figure" is necessarily narrower
than that of an ordinary citizen. Private respondent has
not retired into the seclusion of simple private citizenship.
he continues to be a "public figure." After a successful
political campaign during which his participation in the
EDSA Revolution was directly or indirectly referred to in ISSUE:
the press, radio and television, he sits in a very public
place, the Senate of the Philippines. W/N constitutional privilege granted under the freedom of
speech and the press against liability for damages extend
to Phil. Journal in this case?

The line of equilibrium in the specific context of the instant


case between the constitutional freedom of speech and of
expression and the right of privacy, may be marked out in RULING:
terms of a requirement that the proposed motion picture
must be fairly truthful and historical in its presentation of
events. There must, in other words, be no knowing or NO. The freedom of speech and of the press is not
reckless disregard of truth in depicting the participation of absolute. Enshrined in Section 4, Article of the Bill of
private respondent in the EDSA Revolution. 16 There Rights of the 1987 Constitution, it states, No law shall be
must, further, be no presentation of the private life of the passed abridging the freedom of speech, of expression,
unwilling private respondent and certainly no revelation of or of the press, or the right of the people peaceably to
intimate or embarrassing personal facts. The proposed assemble and petition the government for redress of
motion picture should not enter into what Mme. Justice grievances.
Melencio-Herrera in Lagunzad referred to as "matters of
essentially private concern." To the extent that "The Four
Day Revolution" limits itself in portraying the participation But not all speech is protected. The right of free speech is
of private respondent in the EDSA Revolution to those not absolute at all times and under all circumstances.
events which are directly and reasonably related to the There are certain well-defined and narrowly limited
public facts of the EDSA Revolution, the intrusion into classes of speech, the prevention and punishment of
private respondent's privacy cannot be regarded as which has never been thought to raise any Constitutional
unreasonable and actionable. Such portrayal may be problem. These include the
carried out even without a license from private
respondent. - lewd and obscene, the profane, the libelous, and - the
insulting or fighting words – - those which by their very
utterance inflict injury or - tend to incite an immediate
PHIL. JOURNAL V. THEONEN breach of the peace.

FACTS: It has been well observed that such utterances are no


For almost a century, this Court has sought that elusive essential part of any exposition of ideas, and are of such
equilibrium between the law on defamation on one hand, slight social value as a step to truth that any benefit that
and the constitutionally guaranteed freedoms of speech may be derived from them is clearly outweighed by the
and press on the other. This case revisits that search. social interest in order and morality.

Francis Thoenen, is a retired engineer permanently Libel is not protected speech. Article 353 of the Revised
residing in this country with his Filipina wife and their Penal Code defines libel as a public and malicious
children. He filed a civil case for damages against imputation of a crime, or of a vice or defect, real or
peittioners after having been identified in a brief news item imaginary, or any act, omission, condition, status, or
w/ a headline Swiss Shoots Neighbors Pets. He claims circumstance tending to cause the dishonor, discredit, or
that the report was false and defamatory, and that the contempt of a natural or juridical person, or to blacken the
petitioners acted irresponsibly in failing to verify the truth memory of one who is dead.
of the same prior to publication.

In the instant case, even if we assume that the letter


The petitioners claim that Lee, as the reporter assigned to written by the spurious Atty. Angara is privileged
cover news events in the CID, acquired a copy of a letter communication, it lost its character as such when the
from a trusted source in the CIDs Intelligence Division. matter was published in the newspaper and circulated
They claimed to have reasonable grounds to believe in among the general population. A written letter containing
the truth and veracity of the information derived (from libelous matter cannot be classified as privileged when it
their) sources which served as the basis for the article. is published and circulated in public, which was what the
petitioners did in this case.
C. FREEDOM OF EXPRESSION AND THE ADMINISTRATION IN THE MATTER-MACASAET FREEDOM of the press and
OF JUSTICE (CONTEMPT OF COURT) judicial independence (kalayaan ng pamamahayag at
kalayaang panghukuman)

IN RE: PUBLISHED ALLEGED THREATS AGAINST FACTS:


MEMBERS OF THE COURT IN THE PLUNDER LAW CASE
HURLED BY ATTY. LEONARD DE VERA The case stemmed from certain articles that appeared in
the Business Circuit column of Amado P. Macasaet in the
FACTS: Malaya, a newspaper of general circulation of which he is
the publisher. The articles, containing statements and
Respondent admitted the report in the November 6, 2002 innuendoes about an alleged bribery incident in the
issue of the Inquirer that he suggested that the Court must Supreme Court involving J. Consuelo Ynares- Santiago,
take steps to dispel once and for all these ugly rumors and came out in four (4) issues of the newspaper.
reports that the Court would vote in favor of or against the
validity of the Plunder Law to protect the credibility of the
Court. He explained therein: ISSUE:

Whether or not respondent can invoke the defense of


In short, the integrity of the Court, including the names of freedom of speech.
the Honorable Members who were being unfairly dragged
and maliciously rumored to be in favor or against one side
of the issue, was being viciously attacked. To remain RULING:
silent at this time when the Honorable Court was under
siege by what appeared to be an organized effort to No. The Court have no problems with legitimate criticisms
influence the court in their decision would and could lend pointing out flaws in our decisions, judicial reasoning, or
credence to these reports coming from anonymous even how we run our public offices or public affairs. They
sources. should even be constructive and should pave the way for
a more responsive, effective and efficient judiciary.

RULING: Whether or not the repot made by respondent


is a protected speec. Unfortunately, the published articles of respondent
Macasaet are not of this genre. On the contrary, he has
crossed the line, as his are baseless scurrilous attacks
which demonstrate nothing but an abuse of press
No. Freedom of speech includes the right to know and freedom. They leave no redeeming value in furtherance
discuss judicial proceedings, but such right does not of freedom of the press. They do nothing but damage the
cover statements aimed at undermining the Courts integrity of the High Court, undermine the faith and
integrity and authority, and interfering with the
confidence of the people in the judiciary, and threaten the
administration of justice. Freedom of speech is not
doctrine of judicial independence.
absolute, and must occasionally be balanced with the
requirements of equally important public interests, such
as the maintenance of the integrity of the courts and
orderly functioning of the administration of justice.

Thus, the making of contemptuous statements directed


against the Court is not an exercise of free speech; rather,
it is an abuse of such right. Unwarranted attacks on the
dignity of the courts cannot be disguised as free speech,
for the exercise of said right cannot be used to impair the
independence and efficiency of courts or public respect
therefor and confidence therein.

Clearly, respondents utterances pressuring the Court to


rule in favor of the constitutionality of the Plunder Law or
risk another series of mass actions by the public cannot
be construed as falling within the ambit of constitutionally-
protected speech.
D. SYMBOLIC EXPRESSION E. ASSEMBLY AND PETITION

Texas v. Johnson Dela Cruz v. CA


419 US 109 (1986) 305 SCRA 303 (1999)

FACTS: FACTS:

During the 1984 Republican National Convention, Petitioners are public school teachers from various
respondent Johnson participated in a political schools in Metro Manila who were simultaneously
demonstration to protest against Reagan Administration charged, preventively suspended, and eventually
and some Dallas-based corporations. During the dismissed in October 1990 by the Secretary of the
demonstration Johnson burned an American flag outside Department of Education, Culture and Sports (DECS) in
of the convention center in Dallas, Texas. He was connection with the administrative complaints filed before
arrested and charged with violating a Texas statute that its office by their respective principals for participating in
prevented the desecration of a venerated object, including a mass action/strike and subsequently defying the return-
the American flag, if such action were likely to incite anger to-work order by DECS constituting grave misconduct.,
in others. A Texas court tried and convicted Johnson. He gross neglect of duty, gross violation of Civil Service Law,
appealed, arguing that his actions were "symbolic Rules and Regulations and reasonable office regulations,
speech" protected by the First Amendment. refusal to perform official duty, gross insubordination
conduct prejudicial to the best interest of the service and
absence without official leave (AWOL), in violation of
ISSUE: Whether or not flag burning constitutes "symbolic Presidential Decree 807.
speech" protected by the First Amendment

The petitioners averred that they were denied of their right


RULING: to freedom to assembly and that they were dismissed
illegally.

ISSUE: Whether or not the petitioners’ exercise of their


Yes. The SC said that it respects freedom of expression, right to freedom to assembly and petition were valid
freedom to burn the flag, and then you can because it is
part of our democracy.

Under the circumstances, Johnson's burning of the flag RULING:


constituted expressive conduct, permitting him to invoke
the First Amendment. The State conceded that the
conduct was expressive. Occurring as it did at the end of No. The SC said that the court denies the claim that the
a demonstration coinciding with the Republican National teachers were thereby denied their rights to peaceably
Convention; the expressive, overtly political nature of the assemble and petition the government for redress of
conduct was both intentional and overwhelmingly grievances reasoning that this constitutional liberty to be
apparent. upheld, like any other liberty, must be exercised within
reasonable limits so as not to prejudice the public welfare.
But the public school teachers in the case of the 1990
mass actions did not exercise their constitutional rights
within reasonable limits. On the contrary, they committed
acts prejudicial to the best interest of the service by
staging the mass protests on regular school days,
abandoning their classes and refusing to go back even
after they had been ordered to do so. Had the teachers
availed of their free time — recess, after classes,
weekends or holidays — to dramatize their grievances
and to dialogue with the proper authorities within the
bounds of law, no one — not the DECS, the CSC or even
the Supreme Court — could have held them liable for their
participation in the mass actions.
CONTENT-NEUTRAL RESTRICTIONS independent from the exit polls. The holding and the
reporting of the results of exit polls cannot undermine
A. REGULATION OF POLITICAL CAMPAIGN/ELECTION
ACTIVITY those of the elections, since the former is only part of the
latter. If at all, the outcome of one can only be indicative
of the other.
ABS-CBN v Comelec
323 SCRA 811 (2000)
Moreover, the prohibition incidentally prevents the
collection of exit poll data and their use for any purpose.
FACTS:
The valuable information and ideas that could be derived
from them, based on the voters' answer to the survey
questions will forever remain unknown and unexplored.
In this case the comelec issued a resolution approving the Unless the ban is restrained, candidates, researchers,
issuance of a restraining order to stop ABS CBN or any social scientists and the electorate in general would be
other groups, its agents or representatives from deprived of studies on the impact of current events and of
conducting exit surveys. The electoral body believed that election-day and other factors on voters' choices
such project might conflict with the official Comelec count,
as well as the unofficial quick count of the National
Movement for Free Elections (Namfrel). It also noted that SWS vs. COMELEC
it had not authorized or deputized Petitioner ABS-CBN to G.R. No. 147571 (May 5, 2001)
undertake the exit survey.

On May 9, 1998, this Court issued the Temporary


Restraining Order prayed for by petitioner. We directed
the Comelec to cease and desist, until further orders, from FACTS:
implementing the assailed Resolution or the restraining
order issued pursuant thereto, if any. In fact, the exit polls
were actually conducted and reported by media without Petitioners assail the prohibition of COMELEC on the
any difficulty or problem. publication of election surveys 7 days prior to an election
under the Fair Election Act as unconstitutional. SC
agreed, ruling that the prohibition was invalid because (1)
ISSUE: Whether or not the Comelec acted with grave it imposed a prior restraint on the freedom of expression,
abuse of discretion in prohibiting ABS CBN in conducting (2) it had a direct and total suppression of a category of
exit polls during the elections expression even though such suppression is only for a
limited period, and (3) the governmental interest sought
to be promoted could have been achieved by means other
than the suppression of freedom of expression.
RULING:

Petitioners brought an action for prohibition to enjoin the


Yes. The petition was granted and the TRO issued by the
COMELEC from enforcing Sec. 5.4 of the Fair Election
Court was declared null and void.
Act in the 2001 elections, which provides: Surveys
affecting national candidates shall not be published 15
days before an election and surveys affecting local
Such arguments are purely speculative and clearly candidates shall not be published 7 days before an
untenable. election.

First, by the very nature of a survey, the interviewees or Restriction on publication of election survey results
participants are selected at random, so that the results will constitutes a prior restraint on the exercise of freedom of
as much as possible be representative or reflective of the speech without any clear and present danger to justify
general sentiment or view of the community or group such restraint. Publication of survey results in prior
polled. Second, the survey result is not meant to replace elections does not cause confusion among voters. There
or be at par with the official Comelec count. It consists is no evidence that election surveys cause an immediate
merely of the opinion of the polling group as to who the and inevitable danger to the voting process. There was no
electorate in general has probably voted for, based on the reason for voters to be denied access to the results of
limited data gathered from polled individuals. Finally, not election surveys which are relatively objective.
at stake here are the credibility and the integrity of the
elections, which are exercises that are separate and
ISSUE: W/N Sec. 5.4 of the Fair Election Act is B. FREEDOM OF ASSEMBLY
constitutional (NO, it is unconstitutional)

BAYAN vs ERMITA
488 SCRA 226 (2006)

RULING:
FACTS:

Petitioners come in three groups. Bayan, et al, Jess del


Petition for prohibition GRANTED. Sec. 5.4 of R.A. No. Prado, et al, , Kilusang Mayo Uno (KMU), et al, KMU, et
9006 declared UNCONSTITUTIONAL. al.

Since the prohibition lays down a prior restraint on


freedom of speech, expression, and the press and
The rally was scheduled to proceed along España
considering the preferred status of said constitutional
Avenue in front of the University of Santo Tomas and
rights, there is a presumption of invalidity due to the
going towards Mendiola. Police blocked them and
nature of the prohibition. The government has the burden
prevented them from proceeding further. They were then
of justifying the restraint. Applicability of Art. IX-C, Sec. 4
forcibly dispersed, causing injuries on one of them. Three
The provision on the COMELEC’s supervisory power to
other rallyists were arrested.
regulate the enjoyment or utilization of franchise for the
operation of media of communication, and the
corresponding presumption of validity attached to the
exercise of such power, does not apply. It is limited to In the case of Bayan et al they are citizens and taxpayers
ensuring equal opportunity, time, space, and the right to of the Philippines and they allege that their rights as
reply as well as uniform and reasonable rates of charges organizations and individuals were violated when the rally
for the use of such media facilities for public information they participated in on October 6, 2005 was violently
campaigns and forums among candidates. dispersed by policemen implementing Batas Pambansa
(B.P.) No. 880.

Test to determine validity As held in United States v.


O’Brien, government regulation is justified (1) if it is within All petitioners assail Batas Pambansa No. 880, some of
the constitutional power of the Government; (2) if it them in toto and others only Sections 4, 5, 6, 12, 13(a),
furthers a substantial governmental interest; (3) if the and 14(a), as well as the policy of CPR, "Calibrated
interest is unrelated to the suppression of expression; and Preemptive Response". They seek to stop violent
(4) if the incidental restriction is no greater than is dispersals of rallies under the "no permit, no rally" policy
essential to the furtherance of that interest. and the CPR policy recently announced.

Conclusion The SC held that the prohibition was invalid Bayan et al argued that B.P. No. 880 requires a permit
because (1) it imposed a prior restraint on the freedom of before one can stage a public assembly regardless of the
expression, (2) it had a direct and total suppression of a presence or absence of a clear and present danger. It
category of expression even though such suppression is also curtails the choice of venue and is thus repugnant to
only for a limited period, and (3) the governmental interest the freedom of expression clause as the time and place
sought to be promoted could have been achieved by of a public assembly form part of the message for which
means other than the suppression of freedom of the expression is sought. Furthermore, it is not content-
expression. neutral as it does not apply to mass actions in support of
the government. As a content-based legislation, it cannot
pass the strict scrutiny test.

ISSUE: Whether or not the implementation of B.P. No.


880 violated their rights as organizations and individuals
when the rally they participated in on October 6, 2005
RULING:

On the other hand, B.P. No. 880 cannot be condemned


as unconstitutional; it does not curtail or unduly restrict
B.P. No. 880 requires a permit for all who would publicly freedoms; it merely regulates the use of public places as
assemble in the nation’s streets and parks. to the time, place and manner of assemblies. Far from
being insidious, "maximum tolerance" is for the benefit of
rallyists, not the government. The delegation to the
The first point to mark is that the right to peaceably mayors of the power to issue rally "permits" is valid
assemble and petition for redress of grievances is, because it is subject to the constitutionally-sound "clear
together with freedom of speech, of expression, and of the and present danger" standard.
press, a right that enjoys primacy in the realm of
constitutional protection. For these rights constitute the
very basis of a functional democratic polity, without which In this Decision, the Court goes even one step further in
all the other rights would be meaningless and safeguarding liberty by giving local governments a
unprotected. deadline of 30 days within which to designate specific
freedom parks as provided under B.P. No. 880. If, after
that period, no such parks are so identified in accordance
Rights to peaceful assembly to petition the government with Section 15 of the law, all public parks and plazas of
for a redress of grievances and, for that matter, to the municipality or city concerned shall in effect be
organize or form associations for purposes not contrary to deemed freedom parks; no prior permit of whatever kind
law, as well as to engage in peaceful concerted activities. shall be required to hold an assembly therein. The only
These rights are guaranteed by no less than the requirement will be written notices to the police and the
Constitution, particularly Sections 4 and 8 of the Bill of mayor’s office to allow proper coordination and orderly
Rights, Section 2(5) of Article IX, and Section 3 of Article activities.
XIII.

C. FREEDOM OF ASSOCIATION AND THE RIGHT TO STRIKE


It is very clear, therefore, that B.P. No. 880 is not an
absolute ban of public assemblies but a restriction that
simply regulates the time, place and manner of the GSIS vs KAPISANAN NG MGA MANGGAGAWA
assemblies, it as a "content-neutral" regulation of the G.R. No. 170132 (December 6, 2006)
time, place, and manner of holding public assemblies. A
fair and impartial reading of B.P. No. 880 thus readily
FACTS:
shows that it refers to all kinds of public assemblies that
would use public places. The reference to "lawful cause"
does not make it content-based because assemblies
really have to be for lawful causes, otherwise they would There was a 4 day rally in front of GSIS, attended byGSIS
not be "peaceable" and entitled to protection. Neither are personnel, among them members of the herein
the words "opinion," "protesting" and "influencing" in the respondent Kapisanan Ng Mga Manggagawa sa GSIS
definition of public assembly content based, since they (“KMG” or the “Union”), a public sector union of GSIS
can refer to any subject. The words "petitioning the rank-and-file employees.
government for redress of grievances" come from the
wording of the Constitution, so its use cannot be avoided.
Finally, maximum tolerance is for the protection and On or about October 10, 2004, the manager of the
benefit of all rallyists and is independent of the content of GSIS Investigating Unit issued a memorandum
the expressions in the rally. directing 131 union and non-union members to show
cause why they should not be charged administratively
for their participation in said rally. In reaction, KMG
Furthermore, the permit can only be denied on the ground sought reconsideration of said directive on the ground,
of clear and present danger to public order, public safety, among others, that the subject employees resumed
public convenience, public morals or public health. work on October 8, 2004 in obedience to the return-to-
work order thus issued. The plea for reconsideration
was, however, effectively denied by the filing, on
October 25, 2004, of administrative charges against
The so-called calibrated preemptive response policy has
some 110 KMG members for grave misconduct and
no place in our legal firmament and must be struck down
conduct prejudicial to the best interest of the service.
as a darkness that shrouds freedom. It merely confuses
our people and is used by some police agents to justify
abuses.
In the concrete, the appellate court concluded that 2004, the stubborn fact remains that the erring
inasmuch as GSIS employees are not barred from employees, instead of exploring non-crippling activities
forming, joining or assisting employees’ organization, during their free time, had taken a disruptive approach
petitioner Garcia could not validly initiate charges to attain whatever it was they were specifically after.
against GSIS employees waging or joining rallies and
demonstrations notwithstanding the service-disruptive
effect of such mass action. To petitioner Garcia, as President and General
Manager of GSIS, rests the authority and responsibility,
under Section 45 of Republic Act No. 8291, the GSIS
ISSUE: WON the strike conducted by the GSIS Act of 1997, to remove, suspend or otherwise discipline
employees were valid GSIS personnel for cause. At bottom then, petitioner
Garcia, by filing or causing the filing of administrative
charges against the absenting participants of the
RULING: October 4-7, 2004 mass action, merely performed a
duty expected of him and enjoined by law. Regardless
of the mood petitioner Garcia was in when he signed
the charge sheet, his act can easily be sustained as
NO. The 1987 Constitution expressly guaranteeing, for
legally correct and doubtless within his jurisdiction.
the first time, the right of government personnel to self-
organization to complement the provision according
workers the right to engage in “peaceful concerted
activities, including the right to strike in accordance
with law.”. GSIS vs VILLAVIZA
G. R. No. 190291 (July 27, 2010)

It was against the backdrop of the aforesaid provisions FACTS:


of the 1987 Constitution that the Court resolved
Bangalisan v. Court of Appeals. In it, we held, citing
MPSTA v. Laguio, Jr., that employees in the public
service may not engage in strikes or in concerted and Garcia, as President and General Manager of the
unauthorized stoppage of work; that the right of GSIS, filed separate formal charges against
government employees to organize is limited to the respondents and eventually found them guilty for
formation of unions or associations, without including Grave Misconduct and/or Conduct Prejudicial to the
the right to strike. Specifically, the right of civil servants Best Interest of the Service and meting out the
to organize themselves was positively recognized in penalty of one (1) year suspension plus penalties. The
Association of Court of Appeals Employees vs. Ferrer- charges contained that respondent, wearing red shirt
Caleja. But, as in the exercise of the rights of free together with some employees, marched to or
expression and of assembly, there are standards for appeared simultaneously at or just outside the
allowable limitations such as the legitimacy of the office of the Investigation Unit in a mass
purpose of the association, [and] the overriding demonstration/rally of protest and support for
considerations of national security. penalized employees, the latter having surreptitiously
entered the GSIS premises. On appeal, CSC found
that the acts of respondents in going to the GSIS-IU
office wearing red shirts to witness a public hearing
As regards the right to strike, the Constitution itself do not amount to a concerted activity or mass action
qualifies its exercise with the provision “in accordance proscribed above. CSC added that their actuations can
with law.” This is a clear manifestation that the state be deemed an exercise of their constitutional right to
may, by law, regulate the use of this right, or even deny freedom of expression. The CA found no cogent reason
certain sectors such right. Public employees going on to deviate therefrom.
disruptive unauthorized absences to join concerted
mass actions may be held liable for conduct prejudicial
to the best interest of the service.
ISSUE: Whether or not the unruly mass gathering
of twenty employees during office hours, inside office
premises to protest falls within the purview of the
Indeed, for four straight days, participating KMG constitutional guarantee to freedom of expression and
members and other GSIS employees staged a walk out peaceful assembly. NO.
and waged or participated in a mass protest or
demonstration right at the very doorstep of the GSIS
main office building. In whatever name respondent
desires to call the four-day mass action in October
RULING: theater operators and viewers that some contents of Kapit
are not fit for the young. A good portion of the film shots
concentrated on some women erotically dancing naked,
Yes. As defined in Section 5 of CSC Resolution No. or at least nearly naked, on the theater stage. A motion
02-1316 which serves to regulate the political rights for reconsideration was filed by Gonzales, in behalf of
of those in the government service, the concerted Malaya Films, stating that the classification of the film "For
activity or mass action proscribed must be coupled Adults Only" was without basis. For petitioners, such
with the intent of effecting work stoppage or service classification "is without legal and factual basis and is
disruption in order to realize their demands of force exercised as impermissible restraint of artistic expression.
concession. Wearing similarly colored shirts, attending The film is an integral whole and all its portions, including
a public hearing at the GSIS-IU office, bringing with those to which the Board now offers belated objection, are
them recording gadgets, clenching their fists, some essential for the integrity of the film. Viewed as a whole,
even badmouthing the guards and PGM Garcia, are there is no basis even for the vague speculations
acts not constitutive of an (i) intent to effect work advanced by the Board as basis for its classification." The
stoppage or service disruption and (ii) for the purpose Board further resolves to direct the Chairman of the Board
of realizing their demands of force concession. to withhold the issuance of the permit to exhibit until these
deficiencies are supplied." Gonzales, et. al. filed the
petition for certiorari with the Supreme Court.
Not all collective activity or mass undertaking of
government employees is prohibited. Otherwise, we
would be totally depriving our brothers and sisters in ISSUE: Whether the Board of Review for Motion Pictures
the government service of their constitutional right to and Television have the power to classify the movie “Kapit
freedom of expression. Government workers, whatever sa Patalim” under the classification “For AdultsOnly” and
their ranks, have as much right as any person in the impose conditions to edit the material to allow it a
land to voice out their protests against what they “General patronage” rating.
believe to be a violation of their rights and interests.
Civil Service does not deprive them of their freedom of
expression. It would be unfair to hold that by joining RULING:
the government service, the members thereof have
renounced or waived this basic liberty. This freedom (1) Motion pictures are important both as a medium for the
can be reasonably regulated only but can never be communication of ideas and the expression of the artistic
taken away. Respondents freedom of speech and of impulse. The "importance of motion pictures as an organ
expression remains intact, and CSCs Resolution No. of public opinion lessened by the fact that they are
021316 defining what a prohibited concerted activity designed to entertain as well as to inform." There is no
or mass action has only tempered or regulated these clear dividing line between what involves knowledge and
rights. Measured against that definition, respondents what affords pleasure. If such a distinction were
actuations did not amount to a prohibited concerted sustained, there is a diminution of the basic right to free
activity or mass action. expression.

(2) Censorship or previous restraint certainly is not all


there is to free speech or free press. To avoid an
D. MOVIE CENSORSHIP unconstitutional taint on its creation, the power of the
Board is limited to the classification of films. It can, to
safeguard other constitutional objections, determine what
GONZALES vs. KALAW KATIGBAK motion pictures are for general patronage and what may
137 SCRA 647 require either parental guidance or be limited to adults
only. That is to abide by the principle that freedom of
FACTS: expression is the rule and restrictions the exemption. The
power to exercise prior restraint is not to be presumed,
rather the presumption is against its validity.

Gonzalez is the President of the Malaya Films, a movie


production. Maria Kalaw Katigbak is the Chairman and
(3) The test, to repeat, to determine whether freedom of
Brig. Gen. Wilfredo C. Estrada is the Vice-Chairman of the
Board of Review for Motion Pictures and Television. In a expression may be limited is the clear and present danger
resolution of a sub-committee of the Board of 23 October of an evil of a substantive character that the State has a
right to prevent. Such danger must not only be clear but
1984, a permit to exhibit the film "Kapit sa Patalim" under
also present. There should be no doubt that what is feared
the classification "For Adults Only," with certain changes
may be traced to the expression complained of. The
and deletions enumerated was granted. The film in issue
causal connection must be evident. Also, there must be
was given an adult classification to serve as a warning to
reasonable apprehension about its imminence. The time President the classification of its TV Series No. 128
element cannot be ignored. Nor does it suffice if such which allowed it through a letter of former Executive
danger be only probable. Secretary Edelmiro A. Amante, Sr., addressed for
Henrietta S. Mendez reversing the decision of the
respondent Board. According to the letter the episode
(4) The law, however, frowns on obscenity. All ideas in is protected by the constitutional guarantee of free
having even the slightest redeeming social importance speech and expression and no indication that the
unorthodox ideas, controversial ideas, even ideas episode poses any clear and present danger.
hateful to the prevailing climate of opinion have the Petitioner also filed Civil Case. Petitioner alleged that
full protection of the guaranties, unless excludable the respondent Board acted without jurisdiction or with
because they encroach upon the limited area of more grave abuse of discretion in requiring petitioner to
important interests. But implici t in the history of the submit the VTR tapes of itsTV program and in x-
First Amendment is the rejection of obscenity as utterly rating them. In their Answer, respondent Board
without redeeming social importance. invoked its power under PD No. 19861 in relation to
Article 201 of the Revised Penal Code. The Iglesia
ni Cristo insists on the literal translation of the bible
and says that our (Catholic) veneration of the Virgin
(5) This being a certiorari petition, the question before
Mary is not to be condoned because nowhere it is
the Court is whether or not there was a grav e abuse
found in the bible. The board contended that it
of discretion. That there was an abuse of discretion
outrages Catholic and Protestant's beliefs. RTC ruled
by respondent Board is evident in the light of the
in favor of petitioners. CA however reversed it hence
difficulty and travail undergone by petitioners before
this petition.
Kapit sa Patalim was classified as "For Adults Only,"
without any deletion or cut. Moreov er its perception ISSUE: WON the "ang Iglesia Ni Cristo" program is
of what constitutes obscenity appears to be unduly not constitutionally protected as a form of religious
restrictive. This Court concludes then that there was exercise and expression.
an abuse of discretion.

RULING:
(7) All that remains to be said is that the ruling is to
be limited to the concept of obscenity applicable to
motion pictures. It is the consensus of this Court that Yes. Any act that restrains speech is accompanied
where television is concerned, a less liberal approach with presumption of invalidity. It is the burden of the
calls for observance. This is so because unlike motion respondent Board to overthrow this presumption. If it
pictures where the patrons have to pay their way, fails to discharge this burden, its act of censorship
television reaches every home where there is a set. will be struck down. This is true in this case. So-
Children then will likely will be among the avid viewers called "attacks" are mere criticisms of some of the
of the programs therein shown. deeply held dogmas and tenets of other religions .
RTC’s ruling clearly suppresses petitioner's freedom of
speech and interferes with its right to free exercise
of religion. “attack” is different from “offend” any race
or religion. The respondent Board may disagree with
IGLESIA NI CRISTO VS CA
259 SCRA 529 the criticisms of other religions by petitioner but that
gives it no excuse to interdict such criticisms, however,
unclean they may be. Under our constitutional
FACTS: scheme, it is not the task of the State to favor any
religion by protecting it against an attack by another
religion. Religious dogmas and beliefs are often at
Petitioner has a television program entitled “Ang Iglesia war and to preserve peace among their followers,
ni Cristo". The program presents and propagates especially the fanatics, the establishment clause of
petitioner's religious beliefs, doctrines and practices freedom of religion prohibits the State from leaning
often times in comparative studies with other religions. towards any religion. Respondent board cannot censor
Petitioner submitted to the respondent Board of the speech of petitioner Iglesia ni Cristo simply
Review for Moving Pictures and Television the VTR because it attacks other religions, even if said religion
tapes of its TV program to be aired. The Board happens to be the most numerous church in our
classified the series as "X" or not for public viewing country. The basis of freedom of religion is freedom
on the ground that they "offend and constitute an of thought and it is best served by encouraging the
attack against other religions which is expressly marketplace of dueling ideas. There is no showing
prohibited by law." It appealed to the Office of the whatsoever of the type of harm the tapes will bring
about especially the gravity and imminence of the The broadcast media have also established a uniquely
threatened harm. Prior restraint on speech, including pervasive presence in the lives of all Filipinos.
religious speech, cannot be justified by hypothetical Newspapers and current books are found only in
fears but only by the showing of a substantive and metropolitan areas and in the poblaciones of
imminent evil. It is inappropriate to apply the clear municipalities accessible to fast and regular
and present danger test to the case at bar because transportation. Even here, there are low income masses
the issue involves the content of speech and not the who find the cost of books, newspapers, and magazines
time, place or manner of speech. beyond their humble means. Basic needs like food and
shelter inevitably enjoy high priorities.

E. RADIO BROADCAST
All forms of media, whether print or broadcast, are entitled
to the broad protection of the freedom of speech and
Eastern Broadcasting Corp. vs. Jose P. Dans, Jr. expression clause. The test for limitations on freedom of
expression continues to be the clear and present danger
G.R. No. L-59329. July 19, 1985 rule—that words are used in such circumstances and are
of such a nature as to create a clear and present danger
that they will bring about the substantive evils that the
FACTS: lawmaker has a right to prevent. The clear and present
danger test, however, does not lend itself to a simplistic
and all embracing interpretation applicable to all
A petition was filed to reopen The radio station DYRE. utterances in all forums.
DYRE was “summarily closed” on grounds of national
security. The radio station was allegedly used to incite
people to sedition. Petitioner, DYRE petitioners contends F. FREEDOM OF INFORMATION
that they were denied due process. There was no hearing
to establish factual evidence for the closure. Furthermore,
the closure of the radio station violates freedom of Neri vs. Senate Committee on Accountability of
speech and expression. Before the court could even Public Officers and Investigations
promulgate a decision, petitioner through its president,
Mr. Espina, filed a motion to withdraw the petition. The G.R. No. 180643. March 25, 2008
rights of the station were sold to a new owner, Mr.
Pastrana, who is no longer interested in pursuing the said
case. Despite the case becoming moot and academic, the FACTS:
Court still finds that there is a need to pass a resolution
for the guidance of inferior courts and administrative
tribunals in matters as this case. DOTC entered into a contract with ZTE for the supply of
equipment and services for the National Broadband
Network (NBN) Project in the amount of approximately
ISSUE: USD 329.5M (or P16 Billion).

Whether or not the closure of DYRE is a violation of the The NBN Project was initially approved as a Build-
freedom of speech and expression clause. Operate-Transfer (BOT) project but NEDA acquiesced to
convert it into a government-to-government project, to be
financed through a loan from the Chinese Government.
RULING:

The NBN Project became the subject of investigation of


Yes. The closure of the petitioner’s radio station on several senate committees. NEDA Chair Romulo Neri
grounds of national security without elaboration of the was summoned to appear and testify. He disclosed that
grounds and without hearing deserves to be condemned then COMELEC Chairman Benjamin Abalos offered him
in no uncertain terms for it is manifest that due process P200 Million in exchange for his approval of the NBN
was not observed. If there is an idea which should be Project. He informed President Arroyo about the bribery
impressed in the minds of those who wield power it is that attempt and that she instructed him not to accept the
power must be used in a reasonable manner. bribe. However, when probed further on what they
discussed about the NBN Project, Neri refused to answer,
invoking "executive privilege". In particular, he refused to
answer the questions on (a) whether or not President and the source code of the in-house COMELEC programs
Arroyo followed up the NBN Project, (b) whether or not called the Data Capturing System (DCS) utilities.
she directed him to prioritize it, and (c) whether or not she
directed him to approve after being informed of the bribe
attempt. For his refusal to answer, the Senate cited Neri On August 26, 2009 COMELEC replied that the source
in contempt. code CenPEG wanted did not yet exist for the reasons: 1)
that it had not yet received the baseline source code of
the provider, Smartmatic, since payment to it had been
ISSUE: withheld as a result of a pending suit; 2) its customization
of the baseline source code was targeted for completion
in November 2009 yet; 3) under Section 11 of R.A. 9369,
Whether or not executive privilege violates Art. III, Sec. 7 the customized source code still had to be reviewed by
of the 1987 Constitution. “an established international certification entity,” which
review was expected to be completed by the end of
February 2010; and 4) only then would the AES be made
available for review under a controlled environment.
RULING:
Rejecting COMELEC’s excuse, on October 5, 2009
CenPEG filed the present petition for mandamus, seeking
No, invoking executive privilege does not violate Art. III, to compel COMELEC to immediately make its source
Sec. 7 of the 1987 Constitution. The Court stated that codes available to CenPEG and other interested parties.
“there is a recognized public interest in the confidentiality
ISSUE:
of such information is a recognized principle in other
democratic states. The right to information is not an
absolute right. Restrictions to such right include: (1)
matters of national security, (2) trade secrets and banking Whether or not Commission on Elections (COMELEC)
transactions, (3) criminal matters, and (4) other should disclose the source code for the Automated
confidential information. Matters of national security Election System (AES) technologies it used in the 2010
include state secrets with regard to military and diplomatic national and local elections
matters, as well as information on inter-governmental
exchanges prior to the finality of treaties and executive
agreements. For these state secrets to be protected, it RULING:
must be examined in strict confidence and given
scrupulous protection.
YES. The pertinent portion of Section 12 of R.A. 9369 is
clear in that “once an AES technology is selected for
In the case at bar, it was considered privilege because it implementation, the Commission shall promptly make the
was within the scope of presidential communication and source code of that technology available and open to any
correspondence. So, the Court allowed such non- interested political party or groups which may conduct
appearance for it was based on the principle of separation their own review thereof.” The COMELEC has offered no
of powers. reason not to comply with this requirement of the law.
Indeed, its only excuse for not disclosing the source code
was that it was not yet available when CenPEG asked for
Center for People Empowerment in Governance vs. it and, subsequently, that the review had to be done,
Commission on Elections apparently for security reason, “under a controlled
G.R. No. 189546. September 21, 2010 environment.” The elections had passed and that reason
is already stale.
FACTS:

On May 26, 2009 petitioner Center for People


Empowerment in Governance (CenPEG), a
nongovernment organization, wrote respondent
COMELEC, requesting a copy of the source code of the
Precinct Count Optical Scan (PCOS) programs, the Board
of Canvassers Consolidation/Canvassing System (BOC
CCS) programs for the municipal, provincial, national, and
congressional canvass, the COMELEC server programs,
G. ACADEMIC FREEDOM

De La Salle University, Inc. vs. Court of Appeals


Garcia vs. The Faculty Admission Committee, Loyola G.R. No. 127980. December 19, 2007
School of Theology
G. R. No. L-40779. November 28, 1975
FACTS:
FACTS:

Members of the Tau Gamma Phi were expelled by the De


la Salle University and the College of Saint Benilde Joint
Epicharis T. Garcia studied M.A. Theology because she
Disciplinary Board because of their involvement in an
wants to become a priest. She was a lay woman, she
offensive action, causing injuries to petitioner James Yap
enrolled in the first semester of M.A. Theology. Her
and three other members of the Domino Lux Fraternity.
enrollment was denied. Fr. Lambino, the head of the
When they tried to enroll, they were barred to enroll. They
School of Loyola said “you are very smart, you are very
went to Court to ask for the TRO. The Court granted the
brilliant, but we cannot allow you to enroll in M.A.
TRO directing De la Salle University to accept the
Theology because M.A. Theology is a prerequisite for the
students as enrollees. Undaunted, De la Salle University
seminary, and later on for priesthood.” She went to Court.
and College of Saint Benilde denied their enrollment
She filed a mandamus case to compel the Ateneo School
claiming Academic Freedom
of Theology to accept her enrollment because she wanted
to become a priest.

ISSUE:
ISSUE:

Whether or not De la Salle University can validly deny the


enrollment of private respondents considering their right
Whether or not petitioner can compel the Faculty
of education and the University’s right to Academic
Admission Committee of the Loyola School of Theology
Freedom.
to let her continue studying therein.

RULING:
RULING:

YES. Section 5(2), Article XIV of the Constitution


NO. The internal conditions for academic freedom in a
guaranties all institutions of higher learning academic
university are that the academic staff should have de facto
freedom. This institutional academic freedom includes the
control of the following functions: (i) the admission and
right of the school or college to decide for itself, its aims
examination of students; (ii) the curricula for courses of
and objectives, and how best to attain them free from
study; (iii) the appointment and tenure of office of
outside coercion or interference save possibly when the
academic staff; and (iv) the allocation of income among
overriding public interest calls for some restraint.
the different categories of expenditure. It would be a poor
According to present jurisprudence, academic freedom
prospect for academic freedom if universities had to rely
encompasses the independence of an academic
on the literal interpretation of their constitutions in order to
institution to determine for itself (1) who may teach, (2)
acquire for their academic members control of these four
what may be taught, (3) how it shall teach, and (4) who
functions, for in one constitution or another most of these
may be admitted to study.
functions are laid on the shoulders of the lay governing
body. Indeed, while it is categorically stated under the Education
Act of 1982 that students have a right “to freely choose
SC held that part of academic freedom is barring the
their field of study, subject to existing curricula and to
enrollment of this student. The College of Theology had
continue their course therein up to graduation,”77 such
all the right to bar the enrollment of this student.
right is subject to the established academic and
disciplinary standards laid down by the academic
institution. Petitioner DLSU, therefore, can very well
exercise its academic freedom, which includes its free
choice of students for admission to its school.
IV. FREEDOM OF RELIGION ISSUE:

NON-ESTABLISHMENT CLAUSE
Whether or not Muslim employees be granted of their two
requests above mentioned.

Re: Request of Muslim Employees in the Different Courts


in Iligan City (Re: Office Hours)
A.M. No. 02-2-10-SC. December 14, 2005 RULING:

No. Only the first request, which is to hold office hours


FACTS: from 7:30am to 3:30pm without lunch break or coffee
breaks during the month of Ramadan, can be granted and
not the second one. Said requests are grounded on
In their Letter to Executive Judge Salazar, RTC Iligan Section 5, Article 3 of the Constitution, particularly the free
City, several Muslim employees requested that they be exercise clause to one's religion. The court said that this
allowed to enjoy the following privileges: (1) to hold office clause is of two-fold, freedom to believe which is absolute
hours from 7:30 a.m. to 3:30 p.m. without lunch break or and freedom to act on one's beliefs as subject to
coffee breaks during the month of Ramadan; (2) to be regulation since it involves matters affecting public
excused from work from 10:00 a.m. to 2:00 p.m. every welfare. The Court recognizes that the observance of
Friday (Muslim Prayer Day) during the entire calendar Ramadan and the Friday Muslim Prayer Day is integral to
year. The Muslim employees invoked P.D. No. 291 as the Islamic faith. However, only the first request finds
amended by P.D. No. 322, which was issued to reinforce support in Section 3 (a) of P.D. No. 291, as amended by
national unity by recognizing Muslim holidays and making P.D. No. 322, there is no basis for the second request. In
them part of our national holidays. fact, allowing the second request would mean diminution
of 12 hours from the prescribed government working
hours.
Section 3 of the same law, as amended, provides that The performance of religious practices, whether by the
during the fasting season on the month of Ramadan, all Muslim employees or those belonging to other religious
Muslim employees in the national government, denominations, should not prejudice the courts and the
government-owned or controlled corporations, provinces, public. Indeed, the exercise of religious freedom does not
cities, municipalities and other instrumentalities shall exempt anyone from compliance with reasonable
observe office hours from 7:30 a.m. to 3:30 p.m. without requirements of the law, including civil service laws.
lunch break or coffee breaks, and that there shall be no
diminution of salary or wages, provided, that the
employeewho is not fasting is not entitled to the benefit of TARUC VS. DELA CRUZ
this provision. It also provides that "regulations for the
implementation of this section shall be issued together
with the implementing directives on Muslim holidays.” FACTS :

The petitioners were members of PIC in Surigao while the


respondents de la Cruz and Florano were the bishop and
Pursuant thereto, the Civil Service Commission (CSC) parish priest. Petitioner, Taruc, tried to organize a mass
promulgated Resolution No. 811277 stating that during to be celebrated by Fr. Ambong during the fiesta of
“Ramadan," the fasting month of the Muslims, the Civil Socorro. When Taruc informed the Bishop of his plan, the
Service official time of 8:00 to 12:00 and1:00 to 5:00 is Bishop was against it for the reason that Fr. Ambong was
hereby modified to 7:30 AM to 3:30 P.M. without noon not a member of the clergy of Surigao. Bishop de la Cruz
break and the difference of 2 hours is not counted as declared petitioners expelled/excommunicated from the
undertime. The resolution also declared that during Friday PIC because the priest still celebrated the mass.
Muslims are excused from work from 10o'clock in the Subsequently, Fr. Ambong filed a case before the court
morning to 2 o'clock in the afternoon. In another for him to be reinstated.
resolution, CSC clarified that the term "Friday “in the
above resolution is not limited to the Fridays during the
month of Ramadan but refers to "all Fridays of the ISSUE : Whether or not the court has jurisdiction
calendar year."
RULING: The Supreme Court held that the court has no
jurisdiction. The Church and the State are to be separate
and distinct from each other. The expulsion or
excommunication of members of religious institutions is a
matter best left to the discretion of the officials.
ESTRADA VS. ESCRITOR TOLENTINO VS SECRETARY
FACTS:

Soledad Escritor is a court interpreter in the RTC. She FACTS:


lives with a man who is not her husband (Quilapio) and
they have a son. Her husband died before she entered These are motions seeking reconsideration for dismissing
into the judiciary while Quilapio is still married to another. the petitions filed in the cases for the declaration of
unconstitutionality the Expanded Value-Added Tax Law.
According to complainant (Estrada), Escritor should not The Philippine Press Institute (PPI) contends that by
be allowed to be employed in the judiciary for it will appear removing the exemption of the press from the VAT while
as if the court allows such act. maintaining those granted to others, the law discriminates
the press. It is averred, “even nondiscriminatory taxation
Escritor claims that their conjugal arrangement is of constitutionally guaranteed freedom is
permitted by her religion (Jehovah’s Witness). They unconstitutional.”
allegedly have a ‘Declaration of Pledging Faithfulness’
under the approval of their congregation which is effective ISSUE:
when legal impediments render it impossible for a couple
to legalize their union. Whether or not sales tax on bible sales is violative of
religious and press freedom?
ISSUE: Whether or not the state can use compelling state
interest to determine that Escritor’s conduct is immoral RULING:
and therefore must be administratively charged? The Supreme Court held no. The VAT is not a license tax
RULING: Supreme Court held that no, because it was and not included in religious exercises. It is not a tax on
found out that it was part of their religious practice. the exercise of a privilege, much less a constitutional
Freedom of religion was preferred over by the court than right. It is imposed on the sale, barter, lease or exchange
gross immorality. of goods or properties or the sale or exchange of services
and the lease of properties purely for revenue purposes.
To subject the press to its payment is not to burden the
exercise of its right any more than to make the press pay
A. TAX EXEMPTION
income tax or subject it to general regulation is not to
violate its freedom under the Constitution.

BISHOP OF NUEVA SEGOVIA VS. PROVINCIAL BOARD

B. PUBLIC AID TO RELIGION


FACTS:

The Church owns a parcel of land in Ilocos del Norte. On


AGLIPAY VS RUIZ
the south side was the church yard, the convent and a
vegetable garden. On the center was the church. And on
the north side was an old cemetery. FACTS:

ISSUE: Whether or not the whole lot is exempted from Mons. Gregorio Aglipay seeks the issuance from the court
tax? of a writ of prohibition to prevent the Director of Posts
from issuing and selling postage stamps commemorative
RULING: The Supreme Court that it is exempted from tax. of the 33rd International Eucharistic Congress. Petitioner
Section 344 (c) of the Administrative Code provides the contends that such act is a violation of the Constitutional
exemption in favor of the convent in the payment of the provision stating that no public funds shall be
tax refers to the home of the priest who presides over the appropriated or used in the benefit of any church, system
church and who has to take care of himself in order to of religion, etc.
discharge his duties. It not only includes the land actually
occupied by the church, but also the adjacent ground ISSUE: Whether or not there was a violation of the non-
destined to the ordinary incidental uses of man. establishment clause?

RULING: Supreme Court held no. The issuance of the


postage stamps by the Director of Posts and the
Secretary of Public Works and Communications was not
inspired by any sectarian feeling to favor a particular
church or religious denomination. The stamps were not
issued and sold for the benefit of the Roman Catholic
Church. From the letter of the Director of Posts, the only
purpose in issuing and selling the stamps was "to B. FREEDOM TO PROPAGATE RELIGIOUS DOCTRINES
advertise the Philippines and attract more tourists to this
country."
AMERICAN BIBLE SOCIETY vs. CITY OF MANILA

FREE EXERCISE CLAUSE


FACTS:

Plaintiff-appellant is a foreign, non-stock, non-


A. FLAG SALUTE profit, religious, missionary corporation duly registered
and doing business in the Philippines through its
Philippine agency established in Manila in November,
EBRALINAG VS. DIVISION SUP’T OF SCHOOLS 1898, with its principal office at 636 Isaac Peral in said
City. The defendant-appellee is a municipal corporation
with powers that are to be exercised in conformity with the
FACTS: provisions of Republic Act No. 409, known as the Revised
The petitioners were expelled from their classes by the Charter of the City of Manila.
public school authorities for refusing to salute the flag, American Bible Society has been distributing and
sing the national anthem and recite the patriotic pledge as selling bibles and/or gospel portions throughout the
required by R.A. 1265 and Department Order No. 8 Philippines and translating the same into several
making the flag ceremony compulsory in all educational Philippine dialect City Treasurer of Manila informed
institutions. Jehovah's Witnesses admitted that they American Bible Society that it was violating several
taught their children not to salute the flag, sing the Ordinances for operating without the necessary permit
national anthem, and recite the patriotic pledge for they and license, thereby requiring the corporation to secure
believe that those are"acts of worship" or "religious the permit and license fees covering the period from 4Q
devotion" which they "cannot conscientiously give to 1945-2Q 1953.
anyone or anything except God". They think that
compelling the flag salute and pledge transcends To avoid closing of its business, American Bible
constitutional limitations on the State's power and invades Society paid the City of Manila its permit and license fees
the sphere of the intellect and spirit which the Constitution under protest.
protects.
American Bible filed a complaint, questioning the
ISSUE: Whether or not the students be dismissed for constitutionality and legality of the Ordinances 2529 and
refusing to take part, on account of their religious beliefs, 3000, and prayed for a refund of the payment made to the
in the flag ceremony? City of Manila. They contended:

RULING: Supreme Court held no. A similar exemption a. They had been in the Philippines since
may be accorded to the Jehovah's Witnesses with regard 1899 and were not required to pay any license fee or sales
to the observance of the flag ceremony out of respect for tax
their religious beliefs, however "bizarre" those beliefs may
b. it never made any profit from the sale of its
seem to others. Nevertheless, their right not to participate
bibles
in the flag ceremony does not give them a right to disrupt
such patriotic exercises. If they quietly stand at attention ISSUE:
during the flag ceremony while their classmates and
teachers salute the flag, sing the national anthem and Whether or not American Bible Society liable to
recite the patriotic pledge, we do not see how such pay sales tax for the distribution and sale of bibles
conduct may possibly disturb the peace, or pose "a grave
RULING:
and present danger of a serious evil to public safety,
public morals, public health or any other legitimate public NO. While it is true the price asked for the
interest that the State has a right (and duty) to prevent.” religious articles was in some instances a little bit higher
than the actual cost of the same, but this cannot mean
that plaintiff was engaged in the business or occupation
of selling said "merchandise" for profit. For this reasons,
the provisions of City Ordinance No. 2529, as amended,
which requires the payment of license fee for conducting
the business of general merchandise, cannot be applied
to plaintiff society, for in doing so, it would impair its free
exercise and enjoyment of its religious profession and
worship, as well as its rights of dissemination of religious
beliefs.
The constitutional guaranty of the free exercise used, the various members of the enumeration are to be
and enjoyment of religious profession and worship carries taken separately.
with it the right to disseminate religious information. Any
restraint of such right can only be justified like other Accordingly, “charitable” and “religious,” which
restraints of freedom of expression on the grounds that are integral parts of an enumeration using the disjunctive
there is a clear and present danger of any substantive evil “or” should be given different, distinct, and disparate
which the State has the right to prevent." (Tañada and meanings. There is no compelling consideration why the
Fernando on the Constitution of the Philippines, Vol. I, 4th same treatment or usage of these words cannot be made
ed., p. 297). applicable to the questioned provisions of Presidential
Decree No. 1564.

CENTENO V. VILLALON
C. EXEMPTION FROM UNION SHOP

FACTS:
VICTORIANO V. ELIZALDE
Sometime in the last quarter of 1985, the officers
of a civic organization known as the Samahang
Katandaan ng Nayon ng Tikay launched a fund drive for FACTS:
the purpose of renovating the chapel of Barrio Tikay,
Malolos, Bulacan. Petitioner Martin Centeno, the Victoriano was an employee of the Elizalde Rope
chairman of the group, together with Vicente Yco, Factory, Inc. As such employee, he was a member of the
approached Judge Adoracion G. Angeles, a resident of Elizalde Rope Workers’ Union which had a closed shop
Tikay, and solicited from her a contribution of P1,500.00. agreement with the Company that membership in the
It is admitted that the solicitation was made without a Union shall be required as a condition of employment for
permit from the Department of Social Welfare and all its permanent employees.
Development. Being a member of a religious sect that prohibits
As a consequence, an information was filed the affiliation of its members with any labor organization,
against petitioner Martin Centeno, together with Religio Appellee presented his resignation to appellant Union in
Evaristo and Vicente Yco, for violation of Presidential 1962, and when no action was taken thereon, he
Decree No. 1564, or the Solicitation Permit Law, before reiterated his resignation on September 3, 1974.
the Municipal Trial Court of Malolos, Bulacan, Branch 2, Thereupon, the Union wrote a formal letter to the
and docketed as Criminal Case No. 2602. Petitioner filed Company asking the latter to separate Appellee from the
a motion to quash the information2 on the ground that the service in view of the fact that he was resigning from the
facts alleged therein do not constitute an offense, claiming Union as a member.
that Presidential Decree No. 1564 only covers The management of the Company in turn notified
solicitations made for charitable or public welfare Appellee and his counsel that unless the Appellee could
purposes, but not those made for a religious purpose such achieve a satisfactory arrangement with the Union, the
as the construction of a chapel. This was denied by the Company would be constrained to dismiss him from the
trial court, and petitioner’s motion for reconsideration service. This prompted Appellee to file an action for
having met the same fate, trial on the merits ensued. injunction, docketed as Civil Case No. 58894 in the Court
ISSUE: of First Instance of Manila to enjoin the Company and the
Union from dismissing Appellee. In its answer, the Union
Whether or not the phrase “charitable purposes” invoked the “union security clause” of the collective
should be construed in the broadest sense so as to bargaining agreement; assailed the constitutionality of
include a religious purpose. Republic Act No. 3350; and contended that the Court had
no jurisdiction over the case, pursuant to Republic Act No.
RULING:
875, Sections 24 and 9 (d) and (e).
NO. The acts of the petitioners cannot be
ISSUE:
punished under the said law because the law does not
contemplate solicitation for religious purposes. Whether or not RA No. 3550 is unconstitutional
for infringing on the fundamental freedom to form
In the provisions of the Constitution and the
associations.
statutes mentioned above, the enumerations therein
given which include the words “charitable” and “religious” RULING:
make use of the disjunctive “or.” In its elementary sense,
“or” as used in a statute is a disjunctive article indicating NO. RA No. 3350 merely excludes ipso jure from
an alternative. It often connects a series of words or the application and coverage of the closed shop
propositions indicating a choice of either. When “or” is agreement the employees belonging to any religious
sects which prohibit affiliation of their members with any According to the Marcoses, such act deprives
labor organization. What the exception provides, them of their right to life, liberty, property without due
therefore, is that members of said religious sects cannot process and equal protection of the laws. They also said
be compelled or coerced to join labor unions even when that it deprives them of their right to travel which according
said unions have closed shop agreements with the to Section 6, Article 3 of the constitution, may only be
employers; that in spite of any closed shop agreement, impaired by a court order.
members of said religious sects cannot be refused
employment or dismissed from their jobs on the sole ISSUE:
ground that they are not members of the collective WON there is violation of the Marcoses right to
bargaining union. abode and right to travel?
It is clear, therefore, that the assailed Act, far from RULING:
infringing the constitutional provision on freedom of
association, upholds and reinforces it. It does not prohibit NO. It must be emphasized that the individual
the members of said religious sects from affiliating with right involved is not the right to travel from the Philippines
labor unions. It still leaves to said members the liberty and to other countries or within the Philippines. These are
the power to affiliate, or not to affiliate, with labor unions. what the right to travel would normally connote.
If, notwithstanding their religious beliefs, the members of
The right to return to one’s country is not among
said religious sects prefer to sign up with the labor union,
the rights specifically guaranteed in the Bill of Rights,
they can do so. If in deference and fealty to their religious
which treats only of the liberty of abode and the right to
faith, they refuse to sign up, they can do so; the law does
travel, but it is our well considered view that the right to
not coerce them to join; neither does the law prohibit them
return may be considered, as a generally accepted
from joining; and neither may the employer or labor union
principle of international law and, under our Constitution,
compel them to join. Republic Act No. 3350, therefore,
is part of the law of the land [Art. II, Sec. 2 of the
does not violate the constitutional provision on freedom of
Constitution]. However, it is distinct and separate from the
association.”
right to travel and enjoys a different protection under the
International Covenant of Civil and Political Rights,
i.e.,against being “arbitrarily deprived” thereof [Art. 12 (4)].
V. LIBERTY OF ABODE AND OF TRAVEL
The issue before the Court is novel and without
precedent in Philippine, and even in American
MARCOS V. MANGLAPUS jurisprudence. Consequently, resolution by the Court of
the well- debated issue of whether or not there can be
limitations on the right to travel in the absence of
FACTS: legislation to that effect is rendered unnecessary.
Former President Ferdinand E. Marcos was
deposed from the presidency via the non-violent “people
power” revolution and was forced into exile. Marcos, in his SILVERIO VS. CA
deathbed, has signified his wish to return to the
Philippines to die. But President Corazon Aquino,
FACTS:
considering the dire consequences to the nation of his
return at a time when the stability of government is On 26 January 1988, or more than two (2) years
threatened from various directions and the economy is after the filing of the Information, respondent People of the
just beginning to rise and move forward, has stood firmly Philippines filed an Urgent ex parte Motion to cancel the
on the decision to bar the return of Marcos and his family. passport of and to issue a hold-departure Order against
accused-petitioner on the ground that he had gone
Aquino barred Marcos from returning due to
abroad several times without the necessary Court
possible threats & following supervening events. Marcos
approval resulting in postponements of the arraignment
filed for a petition of mandamus and prohibition to order
and scheduled hearings.
the respondents to issue them their travel documents and
prevent the implementation of President Aquino’s Overruling opposition, the Regional Trial Court,
decision to bar Marcos from returning in the Philippines. on 4 April 1988, issued an Order directing the Department
of Foreign Affairs to cancel Petitioner’s passport or to
Petitioner questions Aquino’s power to bar his
deny his application therefor, and the Commission on
return in the country. He also questioned the claim of the
Immigration to prevent Petitioner from leaving the country.
President that the decision was made in the interest of
This order was based primarily on the Trial Court’s finding
national security, public safety and health. Petitioner also
that since the filing of the Information on 14 October 1985,
claimed that the President acted outside her jurisdiction.
“the accused has not yet been arraigned because he has
never appeared in Court on the dates scheduled for his
arraignment and there is evidence to show that accused
Ricardo C. Silverio, Sr. has left the country and has gone COJUANGCO VS. SANDIGANBAYAN
abroad without the knowledge and permission of this
Court” (Rollo, p. 45). Petitioner’s Motion for
FACTS:
Reconsideration was denied on 28 July 1988.
Petitioner invokes his constitutional right to due
Petitioner contends that respondent Court of
process, a speedy trial, and a speedy determination of his
Appeals erred in not finding that the Trial Court committed
cases before all judicial, quasi-judicial and administrative
grave abuse of discretion amounting to lack of jurisdiction
bodies. Further, he prays for the issuance of a Temporary
in issuing its Orders, dated 4 April and 28 July 1988, (1)
Restraining Order and/or Writ of Preliminary Injunction
on the basis of facts allegedly patently erroneous,
enjoining respondent Sandiganbayan (First Division) from
claiming that the scheduled arraignments could not be
further enforcing and/or implementing its order dated
held because there was a pending Motion to Quash the
February 20, 1995 which bans petitioner from leaving the
Information; and (2) finding that the right to travel can be
country except upon prior approval by said court.
impaired upon lawful order of the Court, even on grounds
other than the “interest of national security, public safety Criminal Case No. 22018 is an offshoot of a
or public health.” complaint filed on January 12, 1990, by the Office of the
Solicitor General before the Presidential Commission on
ISSUE:
Good Government (PCGG), docketed as I.S. No. 74,
Whether or not there is a violation to Right To against the former Administrator of the Philippine Coconut
Travel Restricted By Conditions Of Bail Authority (PCA) and the former members of the PCA
Governing Board, petitioner among them, for violation of
Republic Act No. 3019, the Anti-Graft and Corrupt
Practices Act, as amended.
RULING:
In said complaint, the respondents were charged
NO. The condition imposed upon an accused on
“for having conspired and confederated together and
bail to make himself available at all times whenever the
taking undue advantage of their public positions and/or
Court requires his presence operates as a valid restriction
using their powers, authority, influence, connections or
of his right to travel.
relationship with the former President Ferdinand E.
Article III, Section 6 of the 1987 Constitution Marcos and former First Lady, Imelda Romualdez-Marcos
should be interpreted to mean that while the liberty of without authority granted a donation in the amount of Two
travel may be impaired even without Court Order, the Million Pesos (P2,000,000.00) to the Philippine Coconut
appropriate executive officers or administrative Producers Federation (COCOFED), a private entity, using
authorities are not armed with arbitrary discretion to PCA special fund, thereby giving COCOFED unwarranted
impose limitations. They can impose limits only on the benefits, advantage and preference through manifest
basis of “national security, public safety, or public health” partiality, evident bad faith and gross inexcusable
and “as may be provided by law,” a limitive phrase which negligence to the grave (sic) and prejudice of the Filipino
did not appear in the 1973 text (The Constitution, Bernas, people and to the Republic of the Philippines.”
Joaquin G., S.J., Vol. I, First Edition, 1987, p. 263).

Apparently, the phraseology in the 1987


ISSUE:
Constitution was a reaction to the ban on international
travel imposed under the previous regime when there was Whether or not there is a violation to the Right to
a Travel Processing Center, which issued certificates of travel.
eligibility to travel upon application of an interested party
(See Salonga v. Hermoso & Travel Processing Center,
No. 53622, 25 April 1980, 97 SCRA 121).
RULING:

YES. Prescinding from our initial declaration that


the issuance of warrant of arrest against petitioner by
respondent court is invalid, it now becomes necessary
that there be strong and compelling reasons to justify the
continued restriction on petitioner’s right to travel abroad.

Admittedly, all of petitioner’s previous requests to


travel abroad has been granted and that, as confirmed by
the Office of the Solicitor General, that petitioner has
always returned to the Philippines and complied with the
restrictions imposed on him. The necessity of further
denying petitioner’s right to travel abroad, with attendant
restrictions, appears less than clear.
The risk of flight is further diminished in view of RULING:
petitioner’s recent reinstatement as Chairman and Chief
Executive Officer of San Miguel Corporation, though he
has now more justification to travel so as to oversee the Yes. There is excessive bail. The Court finds the setting
entire operations of that company. In this regard, it has to of amount to P5.5 million as unreasonable, excessive,
be conceded that his assumption of such vital post has and constitutes an effective denial of petitioner’s right to
come at a time when the current economic crisis has bail and believes the amount of P200,000 is more
adversely affected the international operations of many reasonable.
companies, including San Miguel. The need to travel
abroad frequently on the part of petitioner, to formulate Pertaining to standards for fixing bail, bail is not intended
and implement the necessary corporate strategies and as a punishment nor as a satisfaction of civil liability which
decisions, could not be forestalled. These considerations should necessarily await the judgement of the appellate
affecting the petitioner’s duties to a publicly held court. Imposing bail in an excessive amount could render
company, militate against imposing further restrictions on meaningless the right to bail. The Court will not hesitate
petitioner’s right to travel abroad. to exercise its supervisory powers over lower courts
should the latter, after holding the accused entitled to bail,
effectively deny the same by imposing a prohibitory sum
YAP JR. V. CA or exacting unreasonable conditions. The amount should
358 SCRA 564 (2001) be high enough to assure the presence of the accused
when required but no higher than is reasonably calculated
to fulfil this purpose.
FACTS:

No. The conditions imposed were not violative of


The right against excessive bail, and the liberty of petitioner’s rights. Petitioner was found to have left the
abode and travel, are being invoked to set aside two (2) country several times while the case was pending, the
resolutions of the Court of Appeals which fixed bail at Court of Appeals required the confiscation of his passport
P5,500,000.00 and imposed conditions on change of and the issuance of a hold departure order against him. It
residence and travel abroad. Petitioner was convicted of was also shown that petitioner used different names in his
estafa by the RTC of Pasig City for misappropriating business transactions and had several abodes in different
amounts equivalent to P5.5 million. Petitioner filed a parts of the country. Under the circumstances of this case,
notice of appeal, and moved to be allowed provisional we find that appropriate conditions have been imposed in
liberty under the cash bond he had filed earlier in the the bail bond to ensure against the risk of flight,
proceedings. The motion was denied by the trial court. particularly, the combination of the hold departure order
and the requirement that petitioner inform the court of any
change of residence and of his whereabouts.
The Solicitor General commented on the motion and
opined that the petitioner may be allowed to post bail in
the amount of P5.5 million and be required to secure a
certification/guaranty from the Mayor of the place of his
residence that he is a resident of the area and that he will MIRASOL V. DPWH
remain to be so until final judgment is rendered or in case 490 SCRA 318 (2006)
he transfers residence, it must be with prior notice to the
court and private complainant. Petitioner contended in his FACTS:
reply that the proposed bail was violative of his right
against excessive bail and prays that bail be reduced.
On January 10, 2001, petitioners filed before the trial court
a Petition for Declaratory Judgment with Application for
ISSUE: Whether or not there is excessive bail. Temporary Restraining Order and Injunction, asserting
Whether or not the conditions imposed in the bail bond that Department of Public Works and Highways’ (DPWH)
impair petitioner’s liberty of abode and travel. Department Order No. 74, No. 215, and the Revised
Rules and Regulations on Limited Access Facilities of the
Toll Regulatory Board (TRB) violate Republic Act No.
2000 (RA 2000) or the Limited Access Highway Act.
Petitioners also seek to declare Department Order No.
123 (DO 123) and Administrative Order No. 1 (AO 1)
unconstitutional. Petitioner states that the petition is in
order to prevent the enforcement of the total ban on
motorcycles along the entire breadth of North and South
Luzon Expressways and the Manila-Cavite (Coastal used by a certain vehicle, to maximum vehicle height.
Road) Toll Expressway under DO 215. Furthermore, None of the rules outlined in AO 1 strikes us as arbitrary
Department Order No. 123 was issued which allowed and capricious. AO 1 does not infringe upon petitioners’
motorcycles with engine displacement of 400 cubic right to travel but merely bars motorcycles, bicycles,
centimeters inside limited access facilities only (toll ways). tricycles, pedicabs, and any non-motorized vehicles as
Trial Court issued decision dismissing petition but the mode of traveling along limited access highways.
declaring invalid DO 123. Petitioners moved for
reconsideration.
The petition is partly granted. DO No. 74, 215, and 123
and the Revised Rules and Regulations on Limited
ISSUE: Access Facilities of the Toll Regulatory Board is declared
void. However, Administrative Order No. 1 of the
Department of Public Works and Communications is
Whether or not DO 74, DO 215 and the TRB regulations declared valid.
contravene RA 2000.

OFFICE OF THE ADMINISTRATIVE SERVICES vs.


MACARINE
Whether or not AO 1 and DO 123 are unconstitutional. 677 SCRA 1 (2012)

FACTS:

RULING:
The Office of the Court Administrator (OCA) filed the
present administrative case against Judge Ignacio B.
Yes. The Court held that the RTC’s ruling is based on a Macarine (respondent) for violation of OCA Circular No.
wrong premise. 49-2003. The Circular required that all foreign travels of
judges and court personnel, regardless of the number of
days, must be with prior permission from the Court. A
DO 74 and DO 215 are void. The RTC assumed that the travel authority must be secured from the OCA. Judges
DPWH derived its authority from its predecessor, the must submit requirements that should be submitted to and
Department of Public Works and Communications, under received by the OCA at least two weeks before the
Section 4 of RA 2000. However, such assumption fails to intended time of travel. No action shall be taken on
consider the evolution of the Department of Public Works requests for travel authority with incomplete
and Communications. requirements. Judges and personnel who shall leave the
country without travel authority issued by [the OCA] shall
be subject to disciplinary action.

Under EO 125 and EO 125-A, which further reorganized


the DOTC, the authority to administer and enforce all
laws, rules and regulations relative to transportation is On August 13, 2009, the respondent wrote then Court
clearly with the DOTC. Thus, DO 74 and DO 215 are void Administrator, requesting for authority to travel to
as the DPWH has no authority to declare certain Hongkong with his family for the period of September 10
expressways as limited access facilities but rather it is the - 14, 2009 where he would celebrate his 65th birthday.
DOTC which is authorized. Respondent however did not submit the corresponding
requirements and so his request remained enacted upon
but the respondent still proceeded with his travel abroad
without the required travel authority from the OCA.
Yes. DO 123 is void for want of authority of the DPWH to
promulgate it. Petitioners argue that AO 1 violates their
right to travel.
ISSUE: Whether or not the OCA Circular violated the
liberty of abode and travel by the defendant.

The Court finds that AO 1 does not impose unreasonable


restrictions. It merely outlines several precautionary
measures, to which toll way users must adhere. These
rules were designed to ensure public safety and the
uninhibited flow of traffic within limited access facilities.
They cover several subjects, from what lanes should be
RULING: him. The questioning secured a signed confession that
was admitted to trial. His court-appointed lawyer objected
the presentation of the confession as it was not obtained
No. The OCA circular was not violative of the voluntarily and should therefore be excluded but such was
respondent’s constitutional rights to travel. overruled and defendant was sentenced to 20-30 years of
imprisonment.

While it is true that the right to travel is guaranteed by


Constitution. However, the exercise of such right is not ISSUE:
absolute. Section 6, Article III of the 1987 Constitution
allows restrictions on one’s right to travel provided that
such restriction is in the interest of national security, public Whether or not the statement obtained from the defendant
safety or public health as may be provided by law. This, who was subjected to custodial police interrogation is
however, should by no means be construed as limiting the admissible against him in a criminal trial.
Court’s inherent power of administrative supervision over
lower courts. OCA Circular No. 49-2003 does not restrict
but merely regulates, by providing guidelines to be Whether or not the procedures which assure that
complied by judges and court personnel, before they can individual is accorded his privilege under the Fifth
go on leave to travel abroad. To “restrict” is to restrain or Amendment to the Constitution not be compelled to
prohibit a person from doing something; to “regulate” is to incriminate himself are necessary.
govern or direct according to rule.
RULING:

For traveling abroad without having been officially allowed


by the Court, the respondent is guilty of violation of Office No. The U.S Supreme Court held that the prosecution
of the Court Administrator (OCA) Circular No. 49- 2003. may not use statements, whether exculpatory or
Punishable by suspension from office without salary and inculpatory, stemming from custodial interrogation of the
other benefits for not less than one (1) month nor more defendant unless it demonstrates the use of procedural
than three (3) months; or a fine of more than P10,000.00 safeguards effective to secure the privilege against self-
but not exceeding P20,000.00. incrimination, and that the defendant not only understood
his constitutional rights, but voluntarily waived them.
Failure to file an application for leave and a report on his
caseload prior to his travel abroad, the Court held Judge
Ignacio B. Macarine admonished, with warning against
Yes. The procedures which assure that individual is
repetition of similar violation.
accorded his privilege under the Fifth Amendment
necessary. The U.S Court held that there can be no doubt
that the Fifth Amendment privilege is available outside of
VI. RIGHTS OF PERSONS UNDER CUSTODIAL
INTERROGATION
criminal court proceedings and serves to protect persons
in all settings in which their freedom of action is curtailed
in any significant way from being compelled to incriminate
MIRANDA VS. ARIZONA themselves. Furthermore, the Court held that without
384 U.S. 436 (1996) proper safeguards the process of in-custody interrogation
of persons suspected or accused of crime contains
inherently compelling pressures which work to undermine
FACTS: the individual’s will to resist and to compel him to speak
where he would otherwise do so freely.

On March 13, 1963, the police arrested the indigent


Mexican Ernesto Miranda based on circumstantial Therefore, the defendant must be warned prior to any
evidence which links him to a kidnapping and rape of an questioning that he has the right to remain silent, that
eighteen-year old woman. The police took him into a anything he says can be used against him in a court of
special interrogation room for two (2) hours where they law, that he has the right to the presence of an attorney,
were cut him off from the outside world. The defendant and that if he cannot afford an attorney, one will be
was not given a full and effective warning of his rights at appointed for him prior to any questioning if he so desires.
the outset of the interrogation process. At no time was
Miranda been told his right to counsel, was not advised of
his right to remain silent, nor was he informed that his
statements during the interrogation would be used against
HO WAI PANG V. PEOPLE evidence, provided they are relevant to the issue and are
659 SCRA 624 (October 19, 2011) not otherwise excluded by law or rules, are not affected
even if obtained or taken in the course of custodial
FACTS: investigation.

On September 6, 1991, at around 11:30 P.M, Flight No. In the case at bench, petitioner did not make any
068 of the United Arab Emirates Airlines from Hongkong confession or admission during his custodial
arrived at the Ninoy Aquino International Airport (NAIA). investigation. The prosecution did not present any
Among the passengers were thirteen (13) Hongkong extrajudicial confession extracted from him as evidence
nationals who came to the Philippines as tourists. At the of his guilt. Moreover, no statement was taken from
arrival area, Sonny Wong presented a Baggage petitioner during his detention and subsequently used in
Declaration Form to Customs Examiner Gilda L. Cinco evidence against him. Verily, in determining the guilt of
(Cinco), which examined the baggage of each of the the petitioner and his co-accused, the trial court based its
thirteen (13) passengers. From the first traveling bag, she Decision on the testimonies of the prosecution witnesses
saw few personal belongings such as used clothing, and on the existence of the confiscated shabu. As the
shoes and chocolate boxes. The following bags also Court held in People v. Buluran, any allegation of violation
contained chocolate boxes of the same size as those in of rights during custodial investigation is relevant and
the first bag. Becoming suspicious, she took out four (4) material only to cases in which an extrajudicial admission
of the chocolate boxes and opened one of them. Instead or confession extracted from the accused becomes the
of chocolates, what she saw inside was white crystalline basis of their conviction.
substance contained in a white transparent plastic. Cinco
immediately called her superiors which advised her to call
the Narcotics Command (NARCOM) and the police. All in Petitioner’s conviction in the present case was on the
all, 18 chocolate boxes were recovered from the bags of strength of his having been caught in flagrante delicto
the six accused. transporting shabu into the country and not on the basis
of any confession or admission.

Miranda rule not applicable to confessions executed


The thirteen (13) tourists were brought to the National before Jan. 17, 1973.
Bureau of Investigation (NBI) for further questioning. Out
of the 13 tourists, the NBI found evidence for violation of
R.A. No. 6425 only as against petitioner and his five co-
LUZ v. PEOPLE
accused. Petitioner claimed that when they confessed,
they were not assisted by counsel, not informed of rights
to remain silent and competent counsel of his choice. FACTS: PO2 Emmanuel L. Alteza, who was then
Therefore, they contend that it was a violation of their right assigned at the Sub-Station 1 of the Naga City Police
because they were already in custodial investigation. Station as a traffic enforcer, substantially testified that on
March 10, 2003 at around 3:00 o’clock in the morning, he
saw the accused, who was coming from the direction of
ISSUE: Whether or not the Court of Appeals erred in not Panganiban Drive and going to Diversion Road, Naga
excluding evidence taken during the custodial City, driving a motorcycle without a helmet; that this
investigation. prompted him to flag down the accused for violating a
municipal ordinance which requires all motorcycle drivers
RULING: No. Petition is denied and the assailed decision to wear helmet (sic) while driving said motor vehicle; that
and resolution of the Court of Appeals are affirmed. he invited the accused to come inside their sub-station
since the place where he flagged down the accused is
almost in front of the said sub-station; that while he and
While there is no dispute that petitioner was subjected to SPO1 Rayford Brillante were issuing a citation ticket for
all the rituals of a custodial questioning by the customs violation of municipal ordinance, he noticed that the
authorities and the NBI in violation of his constitutional accused was uneasy and kept on getting something from
right under Section 12 [31] of Article III of the Constitution, his jacket; that he was alerted and so, he told the accused
we must not, however, lose sight of the fact that what said to take out the contents of the pocket of his jacket as the
constitutional provision prohibits as evidence are only latter may have a weapon inside it; that the accused
confessions and admissions of the accused as against obliged and slowly put out the contents of the pocket of
himself. Thus, the Court held that The infractions of the his jacket which was a nickel-like tin or metal container
so-called “Miranda rights” render inadmissible only the about two (2) to three (3) inches in size, including two (2)
extrajudicial confession or admission made during cellphones, one (1) pair of scissors and one (1) Swiss
custodial investigation; The admissibility of other knife; that upon seeing the said container, he asked the
accused to open it; that after the accused opened the RULING: No.
container, he noticed a cartoon cover and something
beneath it; and that upon his instruction, the accused
spilled out the contents of the container on the table which We find the Petition to be impressed with merit, but not for
turned out to be four (4) plastic sachets, the two (2) of the particular reasons alleged.
which were empty while the other two (2) contained
suspected shabu.
First - There was no valid arrest of petitioner. When he
was flagged down for committing a traffic violation, he was
During trial, petitioner testified for himself and raised the not, ipso facto and solely for this reason, arrested.
defense of planting of evidence and extortion. RTC
convicted petitioner of illegal possession of dangerous
drugs5 committed on 10 March 2003. It found the
Arrest is the taking of a person into custody in order that
prosecution evidence sufficient to show that he had been
he or she may be bound to answer for the commission of
lawfully arrested for a traffic violation and then subjected
an offense.10 It is effected by an actual restraint of the
to a valid search, which led to the discovery on his person
person to be arrested or by that person’s voluntary
of two plastic sachets later found to contain shabu. The
submission to the custody of the one making the arrest.
RTC also found his defense of frame-up and extortion to
Neither the application of actual force, manual touching of
be weak, self-serving and unsubstantiated. RODEL LUZ
the body, or physical restraint, nor a formal declaration of
y ONG GUILTY beyond reasonable doubt for the crime of
arrest, is required. It is enough that there be an intention
violation of Section 11, Article II of Republic Act No. 9165.
on the part of one of the parties to arrest the other, and
that there be an intent on the part of the other to submit,
under the belief and impression that submission is
Upon review, the CA affirmed the RTC’s Decision. Hence, necessary.
this petition.

Under R.A. 4136, or the Land Transportation and Traffic


Petitioner claims that there was no lawful search and Code, the general procedure for dealing with a traffic
seizure, because there was no lawful arrest. He claims violation is not the arrest of the offender, but the
that the finding that there was a lawful arrest was confiscation of the driver’s license of the latter:
erroneous, since he was not even issued a citation ticket
or charged with violation of the city ordinance. Even
assuming there was a valid arrest, he claims that he had
SECTION 29. Confiscation of Driver's License. — Law
never consented to the search conducted upon him.
enforcement and peace officers of other agencies duly
deputized by the Director shall, in apprehending a driver
for any violation of this Act or any regulations issued
On the other hand, finding that petitioner had been pursuant thereto, or of local traffic rules and regulations
lawfully arrested, the RTC held thus: The accused himself not contrary to any provisions of this Act, confiscate the
admitted that he was not wearing a helmet at the time license of the driver concerned and issue a receipt
when he was flagged down by the said police officers, prescribed and issued by the Bureau therefor which shall
albeit he had a helmet in his possession. Obviously, there authorize the driver to operate a motor vehicle for a period
is legal basis on the part of the apprehending officers to not exceeding seventy-two hours from the time and date
flag down and arrest the accused because the latter was of issue of said receipt. The period so fixed in the receipt
actually committing a crime in their presence, that is, a shall not be extended, and shall become invalid
violation of City Ordinance No. 98-012. In other words, the thereafter. Failure of the driver to settle his case within
accused, being caught in flagrante delicto violating the fifteen days from the date of apprehension will be a
said Ordinance, he could therefore be lawfully stopped or ground for the suspension and/or revocation of his
arrested by the apprehending officers. license.

ISSUE: W/N there was a valid arrest that may allow the Similarly, the Philippine National Police (PNP) Operations
public officer to conduct a warrantless search and seizure. Manual12 provides the following procedure for flagging
down vehicles during the conduct of checkpoints:

SECTION 7. Procedure in Flagging Down or Accosting


Vehicles While in Mobile Car. This rule is a general
concept and will not apply in hot pursuit operations. The This ruling does not imply that there can be no arrest for
mobile car crew shall undertake the following, when a traffic violation. Certainly, when there is an intent on the
applicable: x x x part of the police officer to deprive the motorist of liberty,
or to take the latter into custody, the former may be
deemed to have arrested the motorist. In this case,
m. If it concerns traffic violations, immediately issue a however, the officer’s issuance (or intent to issue) a traffic
Traffic Citation Ticket (TCT) or Traffic Violation Report citation ticket negates the possibility of an arrest for the
(TVR). Never indulge in prolonged, unnecessary same violation.
conversation or argument with the driver or any of the
vehicle’s occupants;
Even if one were to work under the assumption that
petitioner was deemed "arrested" upon being flagged
At the time that he was waiting for PO3 Alteza to write his down for a traffic violation and while awaiting the issuance
citation ticket, petitioner could not be said to have been of his ticket, then the requirements for a valid arrest were
"under arrest." There was no intention on the part of PO3 not complied with.
Alteza to arrest him, deprive him of his liberty, or take him
into custody. Prior to the issuance of the ticket, the period
during which petitioner was at the police station may be This Court has held that at the time a person is arrested,
characterized merely as waiting time. In fact, as found by it shall be the duty of the arresting officer to inform the
the trial court, PO3 Alteza himself testified that the only latter of the reason for the arrest and must show that
reason they went to the police sub-station was that person the warrant of arrest, if any. Persons shall be
petitioner had been flagged down "almost in front" of that informed of their constitutional rights to remain silent and
place. Hence, it was only for the sake of convenience that to counsel, and that any statement they might make could
they were waiting there. There was no intention to take be used against them.14 It may also be noted that in this
petitioner into custody. case, these constitutional requirements were complied
with by the police officers only after petitioner had been
arrested for illegal possession of dangerous drugs.
However, we decline to accord talismanic power to the
phrase in the Miranda opinion emphasized by
respondent. Two features of an ordinary traffic stop Second - There being no valid arrest, the warrantless
mitigate the danger that a person questioned will be search that resulted from it was likewise illegal.
induced "to speak where he would not otherwise do so
freely," Miranda v. Arizona, 384 U. S., at 467. First,
detention of a motorist pursuant to a traffic stop is The following are the instances when a warrantless
presumptively temporary and brief. Second, search is allowed:
circumstances associated with the typical traffic stop are
not such that the motorist feels completely at the mercy of (i) a warrantless search incidental to a lawful arrest;
the police. The usual traffic stop is more analogous to a
(ii) search of evidence in "plain view;"
so-called "Terry stop," see Terry v. Ohio, 392 U. S. 1
(1968), than to a formal arrest. x x x The comparatively (iii) search of a moving vehicle;
nonthreatening character of detentions of this sort
explains the absence of any suggestion in our opinions (iv) consented warrantless search;
that Terry stops are subject to the dictates of Miranda.
(v) customs search;
The similarly noncoercive aspect of ordinary traffic stops
prompts us to hold that persons temporarily detained (vi) a "stop and frisk" search; and
pursuant to such stops are not "in custody" for the
purposes of Miranda. (vii) exigent and emergency circumstances.15

It also appears that, according to City Ordinance No. 98- None of the above-mentioned instances, especially a
012, which was violated by petitioner, the failure to wear search incident to a lawful arrest, are applicable to this
a crash helmet while riding a motorcycle is penalized by case.
a fine only. Under the Rules of Court, a warrant of arrest
need not be issued if the information or charge was filed
for an offense penalized by a fine only. It may be stated It must be noted that the evidence seized, although
as a corollary that neither can a warrantless arrest be alleged to be inadvertently discovered, was not in "plain
made for such an offense. view." It was actually concealed inside a metal container
inside petitioner’s pocket. Clearly, the evidence was not and frisk is merely a limited protective search of outer
immediately apparent.16 clothing for weapons.20

Neither was there a consented warrantless search. In Robinson, supra, we noted the two historical rationales
Consent to a search is not to be lightly inferred, but shown for the "search incident to arrest" exception:
by clear and convincing evidence.17 It must be voluntary
in order to validate an otherwise illegal search; that is, the (1) the need to disarm the suspect in order to take him
consent must be unequivocal, specific, intelligently given into custody, and
and uncontaminated by any duress or coercion. While the (2) the need to preserve evidence for later use at trial. x x
prosecution claims that petitioner acceded to the x But neither of these underlying rationales for the search
instruction of PO3 Alteza, this alleged accession does not incident to arrest exception is sufficient to justify the
suffice to prove valid and intelligent consent. In fact, the search in the present case.
RTC found that petitioner was merely "told" to take out the
contents of his pocket.18
We have recognized that the first rationale—officer
safety—is "‘both legitimate and weighty,’" x x x The threat
Whether consent to the search was in fact voluntary is a to officer safety from issuing a traffic citation, however, is
question of fact to be determined from the totality of all the a good deal less than in the case of a custodial arrest. In
circumstances. Relevant to this determination are the Robinson, we stated that a custodial arrest involves
following characteristics of the person giving consent and "danger to an officer" because of "the extended exposure
the environment in which consent is given: which follows the taking of a suspect into custody and
(1) the age of the defendant; transporting him to the police station." 414 U. S., at 234-
235. We recognized that "[t]he danger to the police officer
(2) whether the defendant was in a public or a secluded flows from the fact of the arrest, and its attendant
location; proximity, stress, and uncertainty, and not from the
grounds for arrest." Id., at 234, n. 5. A routine traffic stop,
(3) whether the defendant objected to the search or on the other hand, is a relatively brief encounter and "is
passively looked on; more analogous to a so-called ‘Terry stop’ . . . than to a
(4) the education and intelligence of the defendant; formal arrest." Berkemer v. McCarty, 468 U. S. 420, 439
(1984). See also Cupp v. Murphy, 412 U. S. 291, 296
(5) the presence of coercive police procedures; (1973) ("Where there is no formal arrest . . . a person
might well be less hostile to the police and less likely to
(6) the defendant’s belief that no incriminating evidence
take conspicuous, immediate steps to destroy
would be found;
incriminating evidence").
(7) the nature of the police questioning;
The foregoing considered, petitioner must be acquitted.
(8) the environment in which the questioning took place; While he may have failed to object to the illegality of his
and arrest at the earliest opportunity, a waiver of an illegal
warrantless arrest does not, however, mean a waiver of
(9) the possibly vulnerable subjective state of the person the inadmissibility of evidence seized during the illegal
consenting. warrantless arrest. The subject items seized during the
illegal arrest are inadmissible.25 The drugs are the very
corpus delicti of the crime of illegal possession of
It is the State that has the burden of proving, by clear and dangerous drugs. Thus, their inadmissibility precludes
positive testimony, that the necessary consent was conviction and calls for the acquittal of the accused.26
obtained, and was freely and voluntarily given.19 In this
case, all that was alleged was that petitioner was alone at
the police station at three in the morning, accompanied by Petitioner Rodel Luz y Ong is hereby ACQUITTED and
several police officers. These circumstances weigh ordered immediately released from detention, unless his
heavily against a finding of valid consent to a warrantless continued confinement is warranted by some other cause
search. or ground.

Neither does the search qualify under the "stop and frisk"
rule. While the rule normally applies when a police officer
observes suspicious or unusual conduct, which may lead
him to believe that a criminal act may be afoot, the stop
TEST OF VALIDITY OF WAIVER OF MIRANDA RIGHTS confessions obtained during custodial investigation does
PEOPLE VS. RIBADAJO
not have any retroactive effect.
142 SCRA 637 (1986)

All confessions, whether or not executed before the 1973


FACTS:
Constitution took effect are subject thereto and must
comply therewith.

There were originally six accused which were all


members of the Sigue-Sigue Commando Gang, however
JOSE D. FILOTEO, JR v. SANDIGANBAYAN
one of them died during the pendency of the case. On G.R. NO. 79543 (October 16, 1996)
November 18,1971, at about 7:55 P.M, the evidence on
record shows that the prisoners from Brigade C,
Muntinlupa, Rizal, succeeded in opening the door of their FACTS:
dormitory by means of a false key (tin can) and attacked
the inmates from dormitory 3-a, while the latter were then
getting their food rations from the delivery truck wherein Petitioner was a police investigator of the Western Police
the victim was among them. A prisoner (Catamura) was District in Metro Manila. Together with his co-accused, he
stabbed simultaneously which resulted to his death. In an was being charged of being a mastermind of the armed
investigation conducted by the Investigation Section of hijacking of a postal delivery van. This happened in
the New Bilibid Prisons, all the accused executed Bulacan, wherein the said accused, two of whom were
statements admitting their participation in the slaying of armed with guns, conspiring, confederating together and
Bernardo Cutamora. Their motive was to avenge the helping one another, did then and there willfully,
throwing of human waste on them. unlawfully and feloniously with intent of gain and by
means of violence, threat and intimidation, stop the Postal
Delivery Truck while it was travelling along the said
Appellants claim that they have not been informed of their municipality, at the point of their guns, and then take, rob
right to silence and counsel during custodial investigation. and carry away with them the following:(1) Postal Delivery
They withdrew their pleas of guilty, and repudiated their Truck, (2) Social Security System Medicare Checks and
confessions, claiming that they had signed the same Vouchers (3) Social Security System Pension Checks and
under duress. Vouchers (4) Treasury Warrants, and (5) Several Mail
Matters from abroad.

ISSUE: Whether or not the trial court erred in admitting as


evidence, and in giving weight to the supposed The Sandiganbayan found them GUILTY as co-principals
extrajudicial confession of the accused which is violative beyond reasonable doubt of the violation of Section 2 (e),
of the Miranda Right. in relation to Section 3 (b) of Presidential Decree No. 532,
otherwise known as the Anti-Piracy and Anti-Highway
Robbery Law of 1974. On appeal to the SC, the
petitioners claim that the rights of an accused under
RULING:
Article III, Section 12 of the 1987 Constitution which
includes the right against an uncounseled waiver of the
right to counsel is applicable to him retroactively, even
No. The trial court did not err in admitting as evidence, the though his custodial investigation took place in 1983 --
confession of the accused. Also, claim of respondent is long before the effectivity of the new Constitution. This is
bereft of merit. because it is favorable to him as an accused pursuant to
Under the 1935 Constitution, one cannot invoke the rights Art. 22 of the RPC.
under custodial investigation. Appellant made the
confession on November 18, 1971. As to the appellants’
claim that they have not been informed of their right to ISSUE: Whether or not Article III, Section 12 of the 1987
silence and to counsel during custodial investigation, Constitution shall be given retroactive effect and
suffice it to state that the proscription against the petitioner’s extrajudicial confession be held as
admissibility of confessions obtained from an accused inadmissible evidence.
during the period of custodial interrogation, in violation of
procedural safeguards, applies to confessions obtained
after the effectivity of the 1973 Constitution. No law gives
the accused the right to be informed before the enactment
of the 1973 Constitution, even if presented after January
17, 1973. That Constitutional guaranty relative to
RULING: not police investigators, for as public officials it was
incumbent upon them to observe the express mandate of
the Constitution. While these rights may be waiver
NO. Petitioner's contention that Article III, Section 12 of executed in the presence of counsel. He concludes that
the 1987 Constitution should be given retroactive effect his extrajudicial confession is inadmissible in evidence.
for being favorable to him as an accused, cannot be ISSUE: Whether or not Juanito’s extrajudicial confession
sustained. before the barangay captain was admissible.
While Article 22 of the Revised Penal Code provides that
"penal laws shall have a retroactive effect insofar as they
favor the person guilty of a felony who is not a habitual RULING:
criminal," what is being construed here is a constitutional
provision specifically contained in the Bill of Rights which
is obviously not a penal statute. A bill of rights is a Yes, as to his confession with the barangay captain
declaration and enumeration of the individual rights and Ceniza, it has been held that the constitutional provision
privileges which the Constitution is designed to protect on custodial investigation does not apply to a
against violations by the government, or by individuals or spontaneous statement, not elicited through questioning
groups of individuals. It is a charter of liberties for the by the authorities but given in an ordinary manner
individual and a limitation upon the power of the state. whereby the suspect orally admits having committed the
Penal laws, on the other hand, strictly and properly are crime. Neither can it apply to admissions or confessions
those imposing punishment for an offense committed made by a suspect in the commission of a crime before
against the state which the executive of the state has the he is placed under investigation. What the Constitution
power to pardon. In other words, a penal law denotes bars is the compulsory disclosure of incriminating facts or
punishment imposed and enforced by the state for a crime confessions. In the instant case, Juanito voluntarily
or offense against its law. narrated to Ceniza that he rapes Genelyn and thereafter
threw her body into the ravine. This narration was
spontaneous answer, freely and voluntarily given in an
NOT APPLICABLE TO RES GESTAE ordinary manner. It was given before he was arrested or
STATEMENTS/SPONTANEOUS STATEMENTS
place under custody for investigation in connection with
the commission of the offense. Moreover, Juanito did not
offer any evidence of improper or ulterior motive on the
PEOPLE OF THE PHILIPPINES v. JUANITO BALOLOY party of Ceniza, which could have compelled her testify
G.R. NO. 140740 (April 12, 2002)
falsely against him.

FACTS:
BENJAMIN JESALVA v. PEOPLE OF THE PHILIPPINES
G.R. NO. 187725 (January 19, 2011)

At Barangay Inagasan, Aurora, Zamboanga del Sur, on


the evening of August 3 1996, the body of 11 years old FACTS:
Genelyn Camacho was found at the waterfalls at the said
barangay. Autopsy report found the Genelyn was raped
before she was drowned. The one who caused its On September 11, 1992, the Chief of Police of Sorsogon,
discovery was accused-appellant Juanito Baloloy himself, Sorsogon, filed a criminal complaint for Frustrated Murder
who claimed that he had caught sight of it while he was against petitioner. Four days thereafter, or on September
catching frogs in the nearby creek. While in the wake of 15, 1992, the complaint was amended, charging petitioner
Genelyn, Juanito confessed to the barangay captain that with the crime of Murder. After conducting a hearing on
he only wanted to frighten the girl but ended up raping and the bail application of petitioner, the Municipal Trial Court
throwing her body in the ravine. While in the custody of (MTC) of Sorsogon, Sorsogon, on December 18, 1992,
authorities, he was asked incriminating questions by granted him bail. On January 11, 1993, the MTC
Judge Dicon who justified his actions saying that Juanito recommended the filing of Murder against petitioner, and
was not yet in custodial investigation.
then ordered the transmittal of the records of the case to
Based on his alleged extrajudicial confession, coupled the Provincial Prosecutor of Sorsogon. Thus, petitioner
with circumstantial evidence, the trail court violated was charged with the crime of Murder in an Information
Section 12 (1) of Article III of the barangay captain Ceniza dated January 26, 1993.
and Judge Dicon. According to him, the two failed to The prosecution highlighted that, per testimony of Gloria
inform him of his Constitutional rights before they took it Haboc, Leticia disclosed to her that petitioner was
upon themselves to elicit from him the incriminatory courting Leticia. However, Leticia told petitioner that they
information. It is of no moment that Ceniza and Dicon are
should just remain as friends because she was already and penalized under Articles 217 and 171 (8), in relation
married, and that she loved her handsome husband. to Article 48 of the Revised Penal Code.

Petitioner denied that he killed Leticia. He testified that he In July of 1990, the National Power Corporation ("NPC")
did not have any reason to kill her, and that he had many became embroiled in a controversy involving the
reasons why he should not kill her. disappearance of P183,805,291.25 of its funds which
were originally on deposit with the Philippine National
Bank, NPC Branch ("PNB") but were subsequently used
RTC ruled against petitioner. CA affirmed RTC’s decision. to purchase two (2) managers’/cashier’s checks in order
to comply with its loan obligations to the Asian
Development Bank ("ADB").
Petitioner avers that the statements he made at the police
station are not admissible in evidence, considering that he
was, technically, under custodial investigation, and that The prosecution theorizes that the accused diverted the
there was no waiver of his right to remain silent. funds covered by the two PNB Manager’s checks by
falsifying a commercial document called an "Application
for Cashier’s Check" (ACC) by inserting an account
number (A/C #111-1212-04) of a private individual after
ISSUE: Whether or not the RTC as well as the CA erred
the name of the payee, UCPB, T.M. Kalaw Branch.
in ruling that the statement made by petitioner were
admissible

In his defense, appellant asserts that there was no


evidence that he committed any of the acts alleged in the
RULING:
information, particularly the intercalation on the ACC; that
he deposited the checks subsequently issued or that he
received the proceeds thereof; or that he conspired with
The assailed statements herein were spontaneously any of his co-accused. He claims that his conviction was
made by petitioner and were not at all elicited through based on the alleged sworn statement and the transcript
questioning. It was established that petitioner, together of stenographic notes of a supposed interview with
with his cousin Fiscal Jayona, personally went to the appellant by the NPC personnel and the report of the
police station and voluntarily made the statement that National Bureau of Investigation (NBI).
Leticia jumped out of his vehicle at around 12:30 a.m. of
September 9, 1992. The RTC and the CA did not,
therefore, err in holding that the constitutional procedure ISSUE: Whether or not his sworn statement is
for custodial investigation is not applicable in the instant
incompetent evidence and is in violation of Article III,
case.
Section 12 of the 1987 Constitution.

NOT APPLICABLE TO STATEMENTS GIVEN IN


ADMNISTRATIVE INVESTIGATIONS RULING:

The fact that an NBI investigation was being


contemporaneously conducted at the time the sworn
PEOPLE OF THE PHILIPPINES v. JOSE TING LAN UY statement was taken will not extricate appellant from his
G.R. NO. 157399 (November 17, 2005)
predicament. The essence of the constitutional safeguard
is protection from coercion. The interview where the
FACTS: sworn statement is based was conducted by NPC
personnel for the NPC’s administrative investigation. Any
investigation conducted by the NBI is a proceeding
separate, distinct and independent from the NPC inquiry
For allegedly diverting and collecting funds of the National
and should not be confused or lumped together with the
Power Corporation (NPC) intended for the purchase of
latter.
US Dollars from the United Coconut Planters Bank
(UCPB), respondent along with Ernesto Gamus, Jaime Appellant finally contends that both the NBI Investigation
Ochoa and Raul Gutierrez were indicted before the Report and the transcript of stenographic notes are
Sandiganbayan for the complex crime of Malversation hearsay for having been made extra-judicially. The
through Falsification of Commercial Documents defined record, however, shows that the prosecution presented
the team leader of the NBI investigators who conducted and intelligently waive these rights and agree to answer
the investigation, although his testimony was dispensed or make a statement. But unless and until such warnings
with as the parties stipulated on the existence and due and waivers are demonstrated by the prosecution at the
execution of the NBI Investigation report albeit without trial, no evidence obtained as a result of interrogation can
admitting the truth of its contents. If at all, the admission be used against him.
of the report’s existence is an acknowledgment that it is
neither spurious nor counterfeit.
It bears noting, however, that when the prosecution
formally offered its evidence, petitioners failed to file any
objection thereto including their extra-judicial admissions.
At any rate, the Court answers the issue in the affirmative.
ROSARIO v. PEOPLE OF THE PHILIPPINES
G.R. NO. 159734 (November 29, 2006)

The employee may, of course, refuse to submit any


FACTS: statement at the investigation; that is his privilege. But if
he should opt to do so, in his defense to the accusation
against him, it would be absurd to reject his statements,
Rosario Astudillo and Filipina Orellana were hired by whether at the administrative investigation, or at a
Western Marketing Corporation (Western), as subsequent criminal action brought against him, because
salespersons, while Roberto Benitez and Flormarie Robel he had not been accorded, prior to his making and
were hired as floor manager and service-in- presenting them, his “Miranda rights” which, to repeat, are
charge/cashier-reliever, respectively. In the course of relevant in custodial investigations. The Court of Appeals
preparing the monthly sales report of the branch, the did not thus err in pronouncing that Astudillo and Orellana
Branch Accountant noticed discrepancies in the monthly were not under custodial investigation to call for the
sales report. The case of the missing invoices and the presence of counsel of their own choice, hence, their
shortage of cash sales collection were then reported to written incriminatory statements are admissible in
Western‘s branch manager Lily Chan Ong (Lily). evidence.

Petitioners were found guilty of qualified theft by the RTC CUSTODIAL PHASE OF INVESTIGATIONS – POLICE LINEUPS
for feloniously taking, stealing and carrying away two (2)
booklets of Sales Invoices Nos. from128351 to 128400 of
the said corporation and thereafter use the said invoices ALEJANDRO B. DE LA TORRE v. COURT OF APPEALS
in the preparation of fictitious sales and withdrawals of G.R. NO. 102786 (August 14, 1998)
merchandise from Western Marketing Corp.
FACTS: An electrical engineer of MERALCO assigned to
inspect six electric meters installed in the premises of the
ISSUE: Whether or not extra-judicial admissions taken Cathay Pacific Steel and Smelting Corporation
before an employer in the course of an administrative (CAPASSCO) on De la Cruz Street in San Bartolome,
inquiry are admissible in a criminal case. Novaliches, Quezon City, discovered that the said electric
meters were missing. He reported the loss to the
MERALCO office in Ortigas Avenue, Pasig City. They
RULING: suspected that CAPASSCO employees must have
damaged the electric meters while tampering with them
and that to conceal the attempt, the employees must have
removed the electric meters. They expressed suspicion
Chief Justice Warren summarized the procedural
that MERALCO personnel were involved.
safeguards laid down for a person in police custody, "in-
custody interrogation" being regarded as the Patrolman Enopia, who was assigned to the case,
commencement of an adversary proceeding against the proceeded to the scene of the crime and inquired from
suspect. He must be warned prior to any questioning that people he saw there if they had seen the electric meters
he has the right to remain silent, that anything he says can being taken down from the post near the gate of
be used against him in a court of law, that he has the right CAPASSCO. According to one of those he asked, on April
to the presence of an attorney, and that if he cannot afford 11, 1989 four crewmembers in a MERALCO service truck,
an attorney one will be appointed for him prior to any with the number 522 painted on its side, removed the
questioning if he so desires. Opportunity to exercise those electric meters. Acting on this lead, Enopia asked
rights must be afforded to him throughout the MERALCO for the identities of the men, one of whom
interrogation. After such warnings have been given, such turned out to be petitioner de la Torre.
opportunity afforded him, the individual may knowingly
On July 4, 1989, the crewmembers were taken to the NPD The accused-appellant Pavillare prays for an acquittal
headquarters for investigation. They were included in a based on reasonable doubt.While under police custody
line-up of eight (8) persons. Garcia pointed to petitioner the appellant was required to stand in a police line-up
de la Torre as the leader of the group which took down where he was supposedly identified by the private
the electric meters from the CAPASSCO premises. complainant as one of his abductors. He claims that he
was identified by the private complainant as one of his
Petitioner de la Torre claims he was not informed of his abductors because the Indians needed a "scapegoat" for
right to remain silent and to have the assistance of the other four cases of kidnapping of Indian nationals then
counsel during the investigation conducted on July 4, pending. The appellant argues that the private
1989 at the NPD headquarters, where the crewmembers complainant could not identify his captors by himself
of MERALCO service truck number 522 were presented which is shown by the inconsistencies in his testimony
in a police line-up. He further invokes the exclusionary and by the improper suggestion made by the investigating
rule in par. 3 of the same 12 that "any confession or police officer pointing to the accused-appellant as one of
admission obtained in violation of [this rule] shall be the malefactors.
inadmissible in evidence against him."
The Solicitor-General filed brief praying for the
ISSUE: Whether or not Petitioner’s Constitutional rights affirmance in toto of the appealed decision. The appellee
were violated during the police line-up. contends that in court the private complainant
RULING: NO. A police line-up is not considered part of unhesitatingly and consistently identified the accused-
any custodial inquest because it is conducted before that appellant Pavillare as one of the kidnappers. Throughout
stage is reached. his narration of the incident in court the complainant
referred to Pavillare as one of the kidnappers because he
In the instant case, petitioner de la Torre, together with was the one who made the phone call and the one who
the other crewmembers of MERALCO truck number 522, received the ransom money. The complainant's failure to
was merely included in a line-up of eight (8) persons from state an accurate description of the kidnappers in his
which he was picked out by Garcia as the leader of the sworn statement does not belie his identification of
group which had removed the electric meters from the Pavillare in court as it is the general rule that affidavits are
CAPASSCO premises. Until then, the police investigation often inaccurate and incomplete. The argument of the
did not focus on petitioner. Indeed, no questions were put accused-appellant that his identification in the police line-
to him. Rather, the questions were directed to witnesses up was made with improper motive either from the other
of the complainant. There is, therefore, no basis for Indian nationals who were at the police station or from
petitioner's allegations that his rights as a suspect in a SPO1 Frias is without evidentiary basis.
custodial interrogation were violated.

ISSUE: WON petitioner is entitled to the rights under the


PEOPLE V PAVILLARE Miranda Doctrine?
G.R. No. 129970 April 5, 2000

FACTS: Accused-appellants were charged and convicted RULING:


of kidnapping one Sukhjinder Singh at the corner of Scout
Reyes and Roces Avenue, this City, and thereafter Section 12 (1), Art. III of the Constitution states that “Any
brought him at the corner of Aurora Boulevard and Boston person under investigation for the commission of an
street, this City, for the purpose of extorting ransom offense shall have the right to remain silent and to have
money in the amount of P20,000.00 Philippine currency, competent and independent counsel preferably of his own
thereby detaining and depriving him of his liberty for more choice. If the person cannot afford the services of
than three hours, to the damage and prejudice of the said counsel, he must be provided with one. These rights
offended party. cannot be waived except in writing and in the presence of
counsel.”

On April 29, 1996 both accused were arraigned and both


pleaded "not guilty" Thus the prohibition for custodial investigation conducted
without the assistance of counsel. Any evidence obtained
in violation of the constitutional mandate is inadmissible
in evidence.
The trial court rendered judgment finding Eduardo
Pavillare guilty beyond reasonable doubt of having
committed the crime of kidnapping for the purpose of
ransom. The prohibition however, does not extend to a person in
a police line-up because that stage of an investigation is
not yet a part of custodial investigation. It has been
repeatedly held that custodial investigation commences Accused-appellant contends that the evidence against
when a person is taken into custody and is singled out as him is insufficient to prove his guilt beyond reasonable
a suspect in the commission of the crime under doubt.
investigation and the police officers begin to ask
questions on the suspect’s participation therein and which
tend to elicit an admission. The stage of an investigation ISSUE: WON the constitutional rights of Casimiro was
wherein a person is asked to stand in a police line-up has violated?
been held to be outside the mantle of protection of the
right to counsel because it involves a general inquiry into
an unsolved crime and is purely investigatory in nature. It
RULING: Accused-appellant Albert Casimiro is acquitted
has also been held that an uncounseled identification at
on the ground of reasonable doubt.
the police line-up does not preclude the admissibility of an
in-court identification. The identification made by the
private complainant in the police line-up pointing to
Pavillare as one of his abductors is admissible in evidence The receipt states that a brick of dried marijuana leaves
although the accused-appellant was not assisted by was delivered by the suspect to a poseur buyer and
counsel. signed by accused-appellant Albert Casimiro as
“suspect/owner.” In effect, accused-appellant admitted
that he delivered a prohibited drug to another, which is an
TEST OF VALIDITY OF WAIVER OF MIRANDA RIGHTS offense under the law. Having been made without the
assistance of counsel, it cannot be accepted as proof that
marijuana was seized from him. It is inadmissible in
evidence.
PEOPLE V CASIMIRO
R. No. 146277 June 20, 2002

The warning was incomplete. It did not include a


FACTS: On the 17th day of August 1999, in the City of
statement that, if accused-appellant could not afford
Baguio, Philippines, and within the jurisdiction of this
counsel, one would be assigned to him. The warning was
Honorable Court, the above-named accused, did then
perfunctory, made without any effort to find out if he
and there willfully, unlawfully and feloniously sell and/or
understood it. It was merely ceremonial and inadequate
deliver to
in transmitting meaningful information to the suspect. We
SPO2 Dorotheo Supa of the 14th Regional Field Office, cannot say that, in signing the receipt without a lawyer,
Narcotics Unit, posing as buyer, about nine hundred fifty accused-appellant acted willingly, intelligently, and freely.
(950) grams of marijuana dried leaves in brick form. What is more, the police investigators did not pause long
enough and wait for accused-appellant to say whether he
was willing to answer their questions even without the
assistance of counsel or whether he was waiving his right
Upon arraignment, accused-appellant pleaded not guilty
to remain silent at all.
to the crime charged.

Indeed, there is failure in this case to observe standard


After arresting accused-appellant, the policemen took him
operating procedure for a buy-bust operation. The
to the 14th Narcom Office, where PO2 Supa, SPO2
government’s drive against illegal drugs deserves
Madlon, and PO3 Piggangay wrote their initials on the
everybody’s support. But it is precisely when the
brick of marijuana before giving it to the evidence
government’s purposes are beneficent that we should be
custodian. The policemen prepared a booking sheet and
most on our guard to protect these rights. As Justice
arrest report, affidavits, and a request for the laboratory
Brandeis warned long ago, "the greatest dangers to
examination of the confiscated marijuana. They also
liberty lurk in the insidious encroachment by men of zeal,
prepared a "receipt of property seized.
well meaning but without understanding." Our desire to
stamp out criminality cannot be achieved at the expense
of constitutional rights. For these reasons, we cannot
Accused-appellant signed the receipt without the uphold the conviction of accused-appellant.
assistance of counsel.

The trial court rendered a decision finding accused-


appellant guilty of the crime charged.
PEOPLE V SAYABOC The right to be informed requires "the transmission of
G.R. No. 147201 January 15, 2004 meaningful information rather than just the ceremonial
and perfunctory recitation of an abstract constitutional
principle."27 It should allow the suspect to consider the
effects and consequences of any waiver he might make
FACTS: That on or about December 2, 1994, in the of these rights. More so when the suspect is one like
Municipality of Solano, Province of Nueva Vizcaya, Sayaboc, who has an educational attainment of Grade IV,
Philippines Benjamin Sayaboc was accused of killing was a stranger in Nueva Vizcaya, and had already been
certain Joseph Galam y Antonio. At their arraignment under the control of the police officers for two days
accused pleaded not guilty to the charge of murder. previous to the investigation, albeit for another offense.

We likewise rule that Sayaboc was not afforded his


constitutional right to a competent counsel. While we are
On 8 March 1995, the caretaker and the waitress unable to rule on the unsubstantiated claim that Atty.
identified Benjamin Sayaboc at the PNP Provincial Cornejo was partial to the police, still, the facts show
Headquarters in Bayombong as the gunman who shot through the testimonies of Sayaboc and prosecution
Joseph Galam to death. Sayaboc was taken to the witness SPO4 Cagungao that Atty. Cornejo remained
Provincial Command Headquarters in Bayombong, silent throughout the duration of the custodial
Nueva Vizcaya to take his statement but before taking the
investigation.
statement of Sayaboc, he advised the latter of his
constitutional rights. Then Sayaboc told him that he
wanted to have a counsel of his own choice. But since
Sayaboc could not name one, the police officers brought Jurisprudence provides that extrajudicial confessions are
Atty. Rodolfo Cornejo of the PAO, who then conferred presumed to be voluntary. The condition for this
with Sayaboc for a while. After Cagungao heard Sayaboc presumption, however, is that the prosecution is able to
say, "okay," he continued the investigation, during which show that the constitutional requirements safeguarding
Atty. Cornejo remained silent the entire time. However, an accused’s rights during custodial investigation have
Cagungao would stop questioning Sayaboc whenever been strictly complied with, especially when the
Atty. Cornejo would leave to go to the comfort extrajudicial confession has been denounced. The
room.11 That night Sayaboc executed an extrajudicial rationale for this requirement is to allay any fear that the
confession12 in Ilocano dialect. He therein confessed to person being investigated would succumb to coercion
killing Joseph Galam at the behest of Marlon Buenviaje while in the unfamiliar or intimidating environment that is
for the sum of P100,000. He likewise implicated Miguel inherent in custodial investigations. Therefore, even if the
Buenviaje and Patricio Escorpiso. The confession was confession may appear to have been given voluntarily
also signed by Atty. Cornejo and attested to by one Fiscal since the confessant did not file charges against his
Melvin Tiongson. alleged intimidators for maltreatment, the failure to
properly inform a suspect of his rights during a custodial
investigation renders the confession valueless and
inadmissible.
ISSUE: WON Sayaboc’s extrajudicial confession was
admissible?

The right to be informed requires “the transmission of


meaningful information rather than just the ceremonial
RULING: Apart from the absence of an express waiver of
and perfunctory recitation of an abstract constitutional
his rights, the confession contains the passing of principle.” It should allow the suspect to consider the
information of the kind held to be in violation of the right effects and consequences of any waiver he might make
to be informed under Section 12, Article III of the of these rights.
Constitution. In People v. Jara,26 the Court explained:

The stereotyped "advice" appearing in practically all


extrajudicial confessions which are later repudiated has The desired role of counsel in the process of custodial
assumed the nature of a "legal form" or model. Police investigation is rendered meaningless if the lawyer merely
investigators either automatically type it together with the gives perfunctory advice as opposed to a meaningful
curt "Opo" as the answer or ask the accused to sign it or advocacy of the rights of the person undergoing
even copy it in their handwriting. Its tired, punctilious, questioning. If the advice given is so cursory as to be
fixed, and artificially stately style does not create an useless, voluntariness is impaired.
impression of voluntariness or even understanding on the
part of the accused. The showing of a spontaneous, free,
and unconstrained giving up of a right is missing.
Wherefore the court held that Sayaboc should only be that the accused is thrust into an unfamiliar atmosphere
guilty of homicide. running through menacing police interrogation
procedures where the potentiality for compulsion,
physical or psychological is forcefully apparent. It is not
PEOPLE V BAGNATE intended as a deterrent to the accused from confessing
G.R. Nos. 133685-86 May 20, 2004 guilt if he voluntarily and intelligently so desires but to
protect the accused from being coerced to admit any that
is untrue. To be an effective counsel, a lawyer need not
FACTS: There were two criminal charges for Bagnate.
challenge all the questions being propounded to his client.
First for killing Auria Broña by attacking and hacking with
The presence of a lawyer is not intended to stop an
a bolo inflicting upon her mortal wounds which caused her accused from saying anything which might incriminate
death and the other by unlawfully and feloniously having him but, rather, it was adopted in our Constitution to
a sexual intercourse with Rosalie Rayal, against her will, preclude the slightest coercion as would lead the accused
whom the accused attacked and hacked afterwards to admit something false. The counsel, however, should
inflicting upon her mortal wounds on the different parts of never prevent an accused from freely and voluntarily
her body, which caused her death. telling the truth.

On arraignment bagnate pleaded not guilty to both Thus, what the Constitution regards as inadmissible in
charges against him. evidence is confession given by an accused without
having been informed of his right to remain silent, or,
without having been given competent and independent
Appellant was brought before Judge Arsenio Base, Jr. of counsel, preferably his own choice, or if he cannot afford
the Municipal Trial Court of Tabaco, Albay. Judge Base the services of counsel, he was not provided with one; or
requested the presence of Atty. Brotamonte and the waiver of his rights was not in writing and not in the
subsequently examined the voluntariness and veracity of presence of counsel; or, that he was tortured, forced,
the confession as well as the authenticity of the signatures threatened, intimidated, by violence or any other means
of appellant and Atty. Brotamonte. that vitiated his free will. There is nothing in the
Constitution that mandates a counsel to inform an
accused of the possible penalty for the crime he
There were no eyewitnesses to the incident; only the committed. Neither would a presumption arise that the
extra-judicial confession of appellant showed how the counsel is incompetent or not independent just because
crimes were committed by him. he failed to apprise the accused that the imposable
penalty for the crime he was about to admit is death. After
Appellant repudiated his extra-judicial confession before all, the imposable penalty is totally immaterial to the
the trial court and assailed its admissibility alleging that it resolve of an accused to admit his guilt in the commission
was executed in violation of his constitutional rights, of a crime. To be considered competent and independent
particularly his right to a competent and independent for the purpose of assisting an accused during a custodial
counsel of his own choice; and that he was not fully investigation, it is only required for a lawyer to be: “. . . .
apprised of the consequences of his confession. willing to fully safeguard the constitutional rights of the
accused, as distinguished from one who would merely be
giving a routine, peremptory and meaningless recital of
ISSUE: WON the extrajudicial confession was the individual’s constitutional rights. In People v. Basay
admissible? (219 SCRA 404, 418) this Court stressed that an
accused’s right to be informed of the right to remain silent
and to counsel contemplates the transmission of
RULING: Yes, the extrajudicial confession of the accused meaningful information rather than just the ceremonial
and perfunctory recitation of an abstract constitutional
was admissible.
principle.”
To be admissible in evidence, an extra-judicial confession
must be express and voluntarily executed in writing with
the assistance of an independent and competent counsel,
and a person under custodial investigation must be
continuously assisted by counsel from the very start
thereof. The presence of counsel is intended to secure the
voluntariness of the extra-judicial confession, and the
assistance given must be independent and competent,
that is, providing full protection to the constitutional rights
of the accused. The rule is premised on the presumption
A. GALIT RULE B. NEW RULE ON WAIVER

PEOPLE V GALIT SANTOS V SANDIGANBAYAN


G.R No. L-51770 March 20, 1985 G.R. No. 71523-25 December 8, 2000

FACTS: Finding accused Alfredo Fajardo, Jr. alias Boy


Fajardo, Marcelo Desiderio y Silvestre, Jesus Estacio y
FACTS: The prisoner was arrested for killing the victim oil Estrella and Rolando Santos y Ramirez alias Mickey
the occasion of a robbery. He had been detained and Mouse, guilty as co-principals in the three (3) separate
interrogated almost continuously for five days, to no avail. complex crimes of Estafa Thru Falsification of Public
He consistently maintained his innocence. There was no Documents. Accused Estacio, Fajardo, Jr., Santos and
evidence to link him to the crime. Obviously, something Desiderio appear to have been detained at the NBI as of
drastic had to be done. A confession was absolutely February 16, 1982 by virtue of a Presidential Commitment
necessary. So the investigating officers began to maul Order, although all of them were later bonded and
him and to torture him physically. Still the prisoner insisted released on different dates, except Santos who has
on his innocence. His will had to be broken. A confession remained in custody up to the present.
must be obtained. So they continued to maltreat and beat
him. 'They covered his face with a rag and pushed his
face into a toilet bowl full of human waste. The prisoner
could not take any more. His body could no longer endure Upon arraignment, accused Fajardo, Jr. @ Boy Fajardo,
the pain inflicted on him and the indignities he had to Desiderio, Estacio, Valentino and Santos, assisted by
suffer. His will had been broken. He admitted what the their respective counsel, pleaded not guilty to the crimes
investigating officers wanted him to admit and he signed charged.
the confession they prepared. Later, against his will, he
posed for pictures as directed by his investigators,
purporting it to be a reenactment. On cross-examination, Estacio admitted that during his
stay at the NBI for about two months, his wife and children
would visit him every week and he could talk to them
freely.24 He was transferred to Muntinlupa and detained
at the Death Row for two years. On March 22, 1982,
ISSUE: WON the confession was valid? Agent Ranin took his second statement that was a
continuation of his first statement. He was unable to read
his supplementary statement because of fear of Agent
RULING: No.The confession is not valid. Ranin, who was scaring him. He stressed that the
statements he made before the NBI were not true and that
he only signed those documents because he was afraid
of Agent Ranin.
At the time a person is arrested, it shall be the duty of the
arresting officer to inform him of the reason for the arrest
and he must be shown the warrant of arrest, if any. He
shall be informed of his constitutional rights to remain Petitioners assert that there was no proof beyond
silent and to counsel, and that any statement he might reasonable doubt that they committed the crimes charged
make could be used against him. The person arrested principally because the extrajudicial confessions of
shall have the right to communicate with his lawyer, a petitioner Estacio and Valentino are inadmissible in
relative, or anyone he chooses by the most expedient evidence as their right to counsel was violated when said
means—by telephone if possible—or by letter or confessions were executed;
messenger. It shall be the responsibility of the arresting
officer to see to it that this is accomplished. No custodial
investigation shall be conducted unless it be in the ISSUE: WON the extrajudicial confessions cannot be
presence of counsel engaged by the person arrested, by used as evidence
any person on his behalf, or appointed by the court upon
petition either of the detainee himself or by anyone on his
behalf The right to counsel may be waived but the waiver RULING: Relevant to petitioners’ contention on the
shall not be valid unless made with the assistance of admissibility of the extrajudicial confessions of petitioner
counsel. Any statement obtained in violation of the Estacio and Valentino is Article IV, Section 20 of the 1973
procedure herein laid down, whether exculpatory or Constitution providing for the rights of an accused during
inculpatory, in whole or in part, shall be inadmissible in custodial investigation. It reads:
evidence.
"No person shall be compelled to be a witness against
himself. Any person under investigation for the
commission of an offense shall have the right to remain
silent and to counsel, and to be informed of such rights. People vs. Mendoza
No force, violence, threat, intimidation, or any other G.R. No. 143702, September 13, 2001
means which vitiates the free will shall be used against
him. Any confession obtained in violation of this section FACTS: On July 7, 1994, at around 1:30 a.m., the victim
shall be inadmissible in evidence." Hernandez Abatay and his companion Jose Neri
On the other hand, the first paragraph of Article III, Tajanlangit were at the corner of Quezon and Ledesma
Section 12 of the 1987 Constitution states: Streets in Iloilo City waiting for a jeepney to take them

"(1) Any person under investigation for the commission of home. They had just come from work at a supermarket.
an offense shall have the right to be informed of his right Tajanlangit left Abatay to urinate nearby. He was about
to remain silent and to have competent and independent 15 feet away from Abatay when he saw accused-
counsel preferably of his own choice. If the person cannot appellant Zaldy Mendoza and a companion approach
afford the services of counsel, he must be provided with Abatay. The two men robbed Abatay. Accused-
one. These rights cannot be waived except in writing and appellant’s companion held Abatay’s hands behind his
in the presence of counsel." back while accused-appellant took Abatay’s wrist watch
and money. Accused-appellant then stabbed Abatay in
the abdomen. Abatay ran away but accused-appellant
pursued him. PO3 Tan then took accused-appellant to the
A comparison of these provisions would readily show that St. Paul’s
the 1973 Constitution does not specify the right against
uncounselled waiver of the right to counsel, which is found Hospital where Abatay was confined. In the presence of
in paragraph 1, Section 12, Article III of the 1987 Tan and some nurses and the attending physician in the
Constitution. However, the latter constitutional provision emergency room, Abatay pointed to accused-appellant as
cannot be applied to extrajudicial confessions made prior one of those who had held him up and then stabbed him.
to its date of effectivity. In Filoteo, Jr. vs. Sandiganbayan, At that time, Abatay was in good condition and even
this Court held that: spoke with Tan. Tan then placed accused-appellant under
arrest

and took him to the police station for investigation.


“x x x the specific provision of the 1987 Constitution Accused-appellant allegedly pointed to Marco Aguirre as
requiring that a waiver by an accused of his right to his companion when they held up Abatay. Accused was
counsel during custodial investigation must be made with convicted, and upon appeal, one of his arguments is that
the assistance of counsel may not be applied retroactively the confession he made to PO3 Daniel Tan at the St.
or in cases where the extrajudicial confession was made Paul’s Hospital that he and Marco Aguirre had robbed
prior to the effectivity of said Constitution. Accordingly, Abatay is inadmissible in evidence because it was given
waivers of the right to counsel during custodial without the assistance of counsel while he (accused-
investigation without the benefit of counsel during the appellant) was in custody.
effectivity of the 1973 Constitution should, by such
argumentation, be admissible. Although a number of
cases held that extrajudicial confessions made while the
1973 Constitution was in force and effect, should have ISSUE: W/N the confession made by the accused is
been made with the assistance of counsel, the definitive admissible as evidence.
ruling was enunciated only on April 26, 1983 when this
Court, through Morales, Jr. vs. Enrile, issued the
guidelines to be observed by law enforcers during RULING: Accused-appellant argues that the confession
custodial investigation. The Court specifically ruled that he made to PO3 Daniel Tan at the St. Paul’s Hospital that
‘(t)he right to counsel may be waived but the waiver shall he and Marco Aguirre had robbed Abatay is inadmissible
not be valid unless made with the assistance of counsel.’ in evidence because it was given without the assistance
of counsel while he (accused-appellant) was in custody.
Indeed, the confession is inadmissible in evidence under
Clearly then, the Morales-Galit rulings are inapplicable in Article III, Section 12(1) and (3) of the Constitution,
these cases as the extrajudicial confessions in question because it was given under custodial investigation and
here, were taken on February 13, February 17 and March was made without the assistance of counsel. However,
22, 1982, long before the date of promulgation of the the defense failed to object to its presentation during the
Morales Decision on April 26, 1983. Prior to this date, the trial with the result that the defense is deemed to have
guidelines requiring that waiver of the right to counsel by waived objection to its admissibility. No error was,
an accused can be properly made only with the presence therefore, incurred by the trial court in admitting evidence
and assistance of counsel, had yet to be formulated and of the confession
pronounced by this Court.
THE BURDEN OF PROVING VOLUNTARINESS OF WAIVERS IS time, he was asked whether he was willing to give a
ON THE PROSECUTION
statement and he said he was. This is sufficient. Contrary
to accusedappellant’s

People vs. Bacor contention, there is no need for a separate and express
G.R. No. 122895, April 30, 1999 written waiver of his constitutional rights. Accused-
appellant was not arrested. He presented himself to the
authorities to confess to the crime because, he said, he
FACTS: On March 17, 1991, at about 9:00 o’clock in the
was being bothered by his conscience. By voluntarily
evening, more or less, in barangay Señor, municipality of
executing his extrajudicial confession, which he did in the
Sinacaban, province of Misamis Occidental, Philippines,
presence of and with the assistance of counsel and after
and within the jurisdiction of this Honorable Court, the
having been informed of his constitutional rights,
aforementioned accused with intent to kill, conspiring,
accused-appellant effectively waived his right to remain
confederating and helping one another, did then and
silent.
there, willfully, unlawfully, feloniously and treacherously
attack, assault and shoot one DIONISIO ALBORES with
the use of a shotgun while the latter was inside his
EXCLUSIONARY RULE
dwelling, unaware, unarmed and defenseless, thereby
inflicting multiple gun shot wounds on different vital parts
of his body causing his [instantaneous] death. On June 6,
1991, appellant approached Jesus Bernido, Chief of the People vs. Andan
G.R. No. 116437, March 3, 1997
Intelligence Section of the Sinacaban Police Station and
told the latter that he was the one responsible for the
killing of Dionisio Albores . In view thereof, Bernido asked FACTS: That on or about the 19th day of February 1994,
appellant what prompted him to surrender and appellant in the municipality of Baliuag, province of Bulacan,
told him that it was due to his Philippines, the above-named accused, with lewd design,
by means of violence and intimidation, did then and there
guilty conscience. Consequently, Bernido, accompanied
wilfully, unlawfully and feloniously have carnal knowledge
by SPO3 Maharlika Ydulzura, Chief Investigator of the
of one Marianne Guevarra y Reyes against her will and
Sinacaban Police Station, and two (2) other police escorts
without her consent; and the above-named accused in
brought appellant to the Public Attorney’s Office (PAO) in
order to suppress evidence against him and delay (sic)
Oroquieta City.
the identity of the victim, did then and there wilfully,
unlawfully and feloniously, with intent to kill the said
Marianne Guevarra y Reyes, attack, assault and hit said
Upon arraignment on September 4, 1991, accused- victim with concrete hollow blocks in her face and in
appellant pleaded not guilty. Trial then ensued. After the different parts of her body, thereby inflicting upon her
prosecution rested its case, the defense demurred to the mortal wounds which directly caused her death. The
evidence on the ground that accused’s extrajudicial police were able to identify the accused, and was brought
confession which is the only piece of evidence connecting into their custody. In the presence of the mayor, the
him to the commission of the murder, is inadmissible for police, representatives of the media and appellant’s own
any purpose. wife and son, appellant confessed his guilt. He disclosed
how he killed Marianne and volunteered to show them the
place where he hid her bags. He asked for forgiveness
ISSUE: W/N the confession made by the accused is from Larin and Dizon whom he falsely implicated saying
admissible as evidence. he did it because of ill-feelings against them.

RULING: Accused-appellant claims that he gave the However, upon appeal, accused-appellant assailed the
confession without being warned of his constitutional admission of the testimonies of the policemen, the mayor
rights. This is not true. The record shows that he was and the news reporters because they were made during
advised of his rights, particularly the right to remain silent, custodial investigation without the assistance of counsel.
not only once but thrice: first, by his counsel, Atty. Meriam
Anggot of Public Attorney’s Office (PAO); second, by
SPO3 Maharlika Ydulzura, the investigator who took ISSUE: W/N the confession made by the accused is
accused-appellant’s confession; and lastly, by the branch admissible as evidence.
clerk of court of the Regional Trial Court of Oroquieta City,
Atty. Nora Montejo-Lumasag, before whom accused-
appellant swore to the veracity of his confession. Each
RULING: Plainly, any person under investigation for the matters some of them containing U.S. Dollar Bills in the
commission of an offense shall have the right: (1) to aggregate amount of $500, or its peso equivalent in the
remain silent; (2) to have competent and independent amount of P11,000.00, Philippine Currency, to the
counsel preferably of his own choice; and (3) to be damage and prejudice of the different addressee or the
informed of such rights. These rights cannot be waived government in the aforesaid mentioned amount.
except in writing and in the presence of counsel. Any
confession or admission obtained in violation of this
provision is inadmissible in evidence against him. The Romero, Marcelo, and Pasicolan were asked to affix their
exclusionary rule is premised on the presumption that the signatures on the envelopes of the letters. They did so in
defendant is thrust into an unfamiliar atmosphere and the presence of the members of the NBI Administrative
runs through menacing police interrogation procedures and Investigative Staff and the people transacting
where the potentiality for compulsion, physical and business with the NBI at that time. According to Director
psychological, is forcefully apparent. The incommunicado Ranin, they required the accused to do this in order to
character of custodial interrogation or investigation also identify the letters as the very same letters confiscated
obscures a later judicial determination of what really from them.
transpired.

It should be stressed that the rights under Section 12 are


accorded to “[a]ny person under investigation for the ISSUE: W/N the letters signed by the accused during
commission of an offense.” An investigation begins when custodial investigation without the assistance of counsel
it is no longer a general inquiry into an unsolved crime but is admissible as evidence
starts to focus on a particular person as a suspect, i.e.,
when the police investigator starts interrogating or
exacting a confession from the suspect in connection with RULING: To be sure, the use of specimen handwriting in
an alleged offense. As intended by the 1971 Beltran is different from the use of petitioner’s signature in
Constitutional Convention, this covers “investigation this case. In that case, the purpose was to show that the
conducted by police authorities which will include specimen handwriting matched the handwriting in the
investigations conducted by the municipal police, the PC document alleged to have been falsified and thereby
and the NBI and such other police agencies in our show that the accused was the author of the crime
government.” (falsification) while in this case the purpose for securing
the signature of petitioner on the envelopes was merely
When the police arrested appellant, they were no longer
to authenticate the envelopes as the ones seized from
engaged in a general inquiry about the death of Marianne.
him and Ronnie Romero. However, this purpose and
Indeed, appellant was already a prime suspect even
petitioner’s signatures on the envelope, when coupled
before the police found him at his parents’ house. This is
with the testimony of prosecution witnesses that the
clear from the testimony of SPO4 Danilo S. Bugay, the
envelopes seized from petitioner were those given to him
police chief investigator of the crime.
and Romero, undoubtedly help establish the guilt of
petitioner. Since these signatures are actually evidence of
admission obtained from petitioner and his co-accused
Marcelo vs. Sandiganbayan
under circumstances contemplated in Art. III, §§12(1) and
G.R. No. 109242, January 26, 1999
17 of the Constitution, they should be excluded. For
indeed, petitioner and his co-accused signed following
FACTS: That on or about February 17, 1989, in the their arrest. Hence, they were at the time under custodial
Municipality of Makati, Metro Manila, Philippines, and investigation, defined as questioning initiated by law
within the jurisdiction of enforcement officers after a person has been taken into
custody or otherwise deprived of his freedom of action in
this Honorable Court, the accused, ARNOLD a significant way. Under the Constitution, among the
PASICOLAN, a public officer, being then an Emergency rights of a person under custodial investigation is the right
Laborer assigned as bag opener at the printed matters to have competent and independent counsel preferably of
section of Makati Central Post Office, and taking his own choice and if the person cannot afford the
advantage of his official position by having access to the services of counsel, that he must be provided with one.
mail matters in conspiracy with accused RONNIE S.
ROMERO and LITO MARCELO, both private individuals,
did

then and there wilfully, unlawfully and feloniously with


grave abuse of confidence, and with intent of gain and
without the consent of the owners thereof, take, steal and
carry away from the Central Post Office of Makati one bag
containing assorted mail
People vs. Janson Constitution abhors an uncounselled confession or
G.R. No. 125938, April 4, 2003 admission and whatever information is derived therefrom
shall be regarded as inadmissible in evidence against the
FACTS: That on or about the 24th day of March 1986, at confessant.
about 10:00 o’clock in the evening at Barangay Mateo,
Municipality of
Under the Constitution and existing law as well as
Kidapawan, Province of Cotabato, Philippines, the above- jurisprudence, a confession to be admissible must satisfy
named accused JOEL JANSON, RICKY PINANTAO alias the following requirements: (1) it must be voluntary; (2) it
OGCO in company with alias ABDUL, alias PUTO, JOHN must be made with the assistance of competent and
DOE and PETER DOE, constituting a band and armed independent counsel; (3) it must be express; and (4) it
with long and short firearms, must be in writing. The purpose of providing counsel to a
conspiring, confederating and mutually helping one person under custodial investigation is to curb the
another, with intent to gain, with force and intimidation, did uncivilized practice of extracting confession by coercion
then and there willfully, unlawfully and feloniously take no matter how slight, as would lead the accused to admit
and carry away, at gun point, cash money in something false. What is sought to be avoided is the evil
of extorting from the very mouth of the person undergoing
the amount of P1,400.00, three (3) pieces of wrist interrogation for the commission
watches, one (1) can coffee beans and one (1) chicken
and if converted into cash it of an offense, the very evidence with which to prosecute
and thereafter convict him. These constitutional
amounted to P1,845.00 or a total amount of Three guarantees have been made available to protect him from
Thousand Two Hundred Fourty Five Pesos (P3,245.00), the inherently coercive psychological, if not physical,
Philippine Currency, atmosphere of such investigation.

owned by Mr. & Mrs. CESARIO ALCANTARA; and on the


same occasion, the above-named accused, with the use
VII. RIGHT TO BAIL
of force, violence and intimidation and armed with
firearms, did then and there willfully, unlawfully and
feloniously take turns in having carnal knowledge with one
WHEN RIGHT MAY BE INVOKED
MARITESS ALCANTARA, a girl about 13

years old, daughter of Mr. & Mrs. CESARIO People vs. Sandiganbayan
ALCANTARA, against her will and consent, to the G.R. No. 158754, August 10, 2007
damage and prejudice of the aforesaid persons in the
aforesaid amount.
FACTS: On April 4, 2001, the Ombudsman issued a Joint
Resolution finding probable cause warranting the filing
with the Sandiganbayan of several criminal Informations
Appellant JOEL JANSON, for his own defense, declared against the former President and the other respondents
that he was assisted by a lawyer when he was therein. One of the Informations was for the crime of
investigated and made to sign a sworn statement before plunder under Republic Act [RA] No. 7080 and among the
the police on June 26, 1986. But he denied the accusation respondents was herein petitioner Jose “Jinggoy”
against him and claimed that he was not assisted by Estrada, then mayor of San Juan, Metro Manila.
counsel during the custodial investigation. He claimed
that he did not know how to read or write, and that he was
made to execute a sworn statement before a certain
policeman named Ulep. Only after the investigation did On April 25, 2001, the respondent court issued a warrant
Atty. Zerrudo sign the document. On cross-examination, of arrest for [Jinggoy] and his co-accused. On its basis,
he said that he was put in jail for another crime, robbery. [Jinggoy] and his co-accused were placed in custody of
the law.

ISSUE: W/N the sworn statement is admissible as


evidence. On April 30, 2001, [Jinggoy] filed a “Very Urgent Omnibus
Motion” alleging that: (1) no probable cause exists to put
him on trial and hold him liable for plunder, it appearing
that he was only allegedly involved in illegal gambling and
RULING: Clearly, the alleged extrajudicial confession of not in a “series or combination of overt or criminal acts”
appellant Joel Janson cannot be admitted in evidence. as required in R.A. No. 7080; and (2) he is entitled to bail
The manner by which it was obtained violated accused’s as a matter of right. [He] prayed that he be excluded from
constitutional right to counsel. It is well-settled that the
the Amended Information …. In the alternative, [he] also VALERIO VS CA
prayed that he be allowed to post bail. G.R. No. 164311-12 / 2007-10-10

FACTS: On march 18, 2000 at around 12:30 a.m Jun


From the denial action of the Sandiganbayan immediately Valerio, chief OF the Office OF the government corporate
adverted to, Jinggoy interposed a petition for certiorari counsel was shot and killed in front of his house at No. 82
before this Court claiming that the respondent Mapang-akit ST. Diliman Quezon City. An information for
Sandiganbayan committed grave abuse of discretion in murder was filled against Antonio E. Cabador, Martin M.
denying his right to post bail pending the resolution of the Jimenez, Samuel C. Baran and Geronimo S. Quinta, Who
Plunder case. shot the victim with an unlisenced .38 Caliber armscor
revolver, hitting him on different parts inflicting fatal
wounds that cause the immediate death.While
On April 17, 2002, Jinggoy filed before the information of parricide was filed against the victim’s wife,
Sandiganbayan an Omnibus Application for Bail10 Milagroso F. Valerio who allegedly ordered them to shoot
against which the prosecution filed its comment and Atty Jun Valerio for a consideration, promise or reward
opposition. Bail hearings were then conducted, followed and was also found to have relationship with Antonio
by the submission by the parties of their respective Cabador. Milagro Valerio filed and application for bail
memoranda. claiming that the evidence of guilt against her was not
strong. Which the RTC granted but denied the motion to
In the herein assailed Resolution11 of March 6, 2003, convert Samuel as stats witness. On March 5, 2002
respondent Sandiganbayan (Special Division) granted Milagros posted bail bond furnished by Central Security
the omnibus application for bail for ₱500,000.00. and Insurance Company, and was ordered release.
Meanwhile, Antonio Caborador was arrested. In his
Sworn statement he stated that it was Milagroso who
Petitioner assailed the decision, hence this petition. planned the murder and pleaded guilty to charge of
murder. Petitioners contended that Milagros is not entitled
to bail as the evidence of guilt against her is strong. Then
bank on the testimony OF Modesto Cabador, victim’s
ISSUE: W/N bail should be granted.
cousin that he heard Milagros impatiently Ask Antonio
about Their plot to kill Jun Valerio. Petitioners also assert
that Antonio’s plea of guilty to the charge of conspiring
RULING: Even if the capital offense charged is bailable with Milagros in the murder of Jun Valerio indicated strong
owing to the weakness of the evidence of guilt, the right evidence of guilt against Milagros. Herein petitioners,
to bail may justifiably still be denied if the probability of Laarni N. Valerio, sister of the victim and the People of the
escape is great. Here, ever since the promulgation of the Philippines, elevated the case to the Court of Appeals
assailed Resolutions a little more than four (4) years ago, ascribing grave abuse of description but Appellate court
Jinggoy does not, as determined by Sandiganbayan, found no abuse of discretion.
seem to be a flight risk.

ISSUE: Whether or not Milagros is entitled for bail


With the view we take of this case, the respondent court considering she was pointed Antonio Cabador as the
did not commit grave abuse of discretion in issuing its mastermind of the incident.
assailed resolutions, because the grant of bail therein is
predicated only on its preliminary appreciation of the
evidence adduced in the bail hearing to determine
RULING: No Milagros is Not entitled to bail. Bail is not a
whether or not deprivation of the right to bail is warranted.
matter of right in cases where person is charged with
Needless to stress, a grant of bail
capital offense or an offense punishable by reclusion
does not prevent the trier of facts, the same Anti-Graft Perpetua or life imprisonment. Article 114, section 7 of the
Court, from making a final assessment of the evidence Revised Rules of Criminal Procedure, states, “No person
after full trial on charged with a capital offense, or an offense punishable
by reclusion Perpetua or life imprisonment, shall be
the merits. As jurisprudence teaches: x x x Such admitted to bail when evidence of guilt is strong,
appreciation [of evidence] is at best preliminary and regardless of the stage of the crimimal action”. In this
should not prevent the trial judge from making a final case, the trial court had disregarded the glaring Face that
assessment of the evidence before him after full trial. It is the killer himself had confesses to the crime and has
not an uncommon occurrence that an accused person implicated that Milagros as the mastermind. When Taken
granted bail is convicted in due course. in conjuction with the Other evidence on Record, these
facts show very strongly that Milagros may have
participated as principal by inducement in the murder of
Jun Valerio. It was this a grave Errol or a grave abide of meetings for five (5) days or more in a week Will virtually
discretion Committee by the trial court to grant her make him a free man with all the privileges appurtenant
application for bail. The Appellate court clearly Committee to his position. Such an aberrant situation not only
a reversible Errol affirming the trial court’s decision elevates accused-appellant’s status to that of a special
granting bail to Milagros Valerio. WHEREFORE, the Class, it also would be a mockery of the purpose of the
instant consolidated petitions or GRANTED. The correction system.xxx WHEREFORE, the petition is
assaulted Decisions dated April 28, 2004 and resolution DENIED SO ORDERED.
dated July 1, 2004 of the court of appeals in CA-G.R. SP
Nos. 71900 and 72132, affirming the subject resolution
dated February 26, 2002 and orders of the Regional trial QUI VS PEOPLE
court of Quezon City,Branch 81, in Criminal Case Nos. Q- G.R. NO 196161/ 2012-09-26
00-93291 and Q-00-93292, are REVERSED AND SET
ASIDE. The Regional Trial Court of Quezon City, Branch
FACTS: On December 1999, Quezon City, Philippines.
81 is directed to CANCEL the bail posted by accused
Petitioner feloniously committed acts of cruelty, and Child
Milagros E. Valerio and to order her Immediate arrest and
abuse upon Christian John Ignacio, 8 years old. Thus
Detention.
petitioner Cyril Calpito Qui was charged with two (2)
counts of violation of RA 7610 of Special Protection of
Children Agiants Child Abuse, Exploitaion and
TRILLANES VS PIMENTEL
Discrimination Act before the Regional Trial Court (RTC),
G.R NO. 179817/ 2008-06-27
Branch 94 Quezon City. And on June 18, 2010 petitioner
was convicted as charged and sentenced to two equal
FACTS: On July 27, 2003, a group of more than 300 periods of imprisonment. On July 2010 petitioner filed
heavily armed soldiers led by junior officers of the Armed before the Appellate court an Urgent Petition/ Application
Forces of the Philippines (AFP) stormed into the for Bail which the OSG urged to deny on the ground of
Oakwood Premier Apartments in Makati City and publicly petitioners propensity to evade the law and that she is a
demanded resignation of the President and key official flight risk, as she failed to attend several hearings before
dubbed as the “Oakwood Incident”. Later that day, the RTC resulting the issuance of three (3) warrants for
President Gloria Macapagal Arroyo issued Proclamation her arrest. The Court of Appeals denied petitioner’s
No. 427 and General Order No. 4 declaring stats of application for bail, hence this petition.
rebellion and calling out the Armed Forces so suppression
the rebellion. After series of negotiations, military soldiers
surrendered that evening. In the after math petitioner ISSUE: Whether or not petitioner is entitled to bail
Antonio F. Trillanes IV was charged, along with his pending trial
comrades, with coup d’etat before the Regional Trial
Court (RTC) of Makati. Close to foul years later, petitioner,
who has remained in detention won a seat in the senate
RULING: Petition is Bereft of Merit. The CA properly
with Six-year term commencing at noon on June 30,2007.
exercised its discretion in denying petitioner’s application
Before the comencement of his term petitioner filed with
for bail pending trial on the ground that she is a flight risk,
the RTC, Makati City l, Branch 148 an “Omnibus Motion
a bail-negating Sec. 5(d) of Rule 114. The CA’s
for Leave of Court to be Allowed to Attend Senate
determination is not without factual mooring. The
Sessions and Related Request” The Trial Court denial the
undisputed fact that petitioner did not attend hearing
request in the Omnibus Motion.Hence the present petition
before the RTC is undeniable indicative of petitioner’s
for certiorari to Set aside orders of the trial court and for
propensity to trifle with court processes and shoulder
prohibition and mandamus.
weight heavily against grant of bail pending trial.
Moreover, petitioners penchant to disobey court
processes is also deduced from the fact that she lied in
ISSUE: whether or not Trillanes’ election as a senator order to justify her Non-appearance on March 8, 2010
provides legal justification to allow him the application of hearing before the RTC. Said her father was hospitalized
Bail. and died days later when in fact her father died a year
ago. And her transfer of residence without informing her
bondsman and the trial court can be viewed inclination to
RULING: Petition is bereft of merit The Court of Appeals evade court appearance. Also her argument that she has
exercised its discretion in denying petitioners application the Constitutional right to bail and that the evidence of
for bail pending Appeal. He was considered a high Risk guilt against her is not strong is spurious. Certainly, after
for flight by not attending several hearings before the one is convicted by trial court, the presumption of
RTC. Also He did not inform his transferred of residence innocence, and with it, the constitutional right to bail, End
hence bail is not allowed. xxx Alowing accused-Appellant WHEREFORE, petitioner’s application for bail pending
to attend congressional sessions and Committee appeal is DENIED. SO ORDERED.
WHEN BAIL IS A MATTER OF RIGHT, WHEN IT IS A MATTER
OF DISCRETION
A. BAIL IN MILITARY COURTS

GACAL VS JUDGE INFANTE


A.M. NO. RTJ-04-1845/ 2011-10-05 COMENDADOR VS DE VILLA
G.R. NO. 93177/ 1991-08-02

FACTS: Atty. Franklin Gacal, the private Prosecutor for


murder arising from the kiling of Felomino O. Occasion, FACTS: Private respondent in this case is involve in a
charges Judge Jaime I. Infante, presiding Judge of consolidated case of officers of the Armed Forces of the
Branch 38 of the RTC to whose Criminal case was raffled Philippines facing prosecution for alleged failed
for arraignment and trial, with gross ignorance of the law, participation in the failed coup d’etat on December 1-9,
gross incompetence, and evident partiality, for the latter’s 1989. Charges against them include Mutiny, conduct of
failure to Set a hearing before granting bail to the accused unbecomming an officer and a gentlemen, and various
and for releasing him immediately after allowing bail. crimes in relation to murder. In the pre trial investigation
Judge infante would excuse himself from blame and (PTI) panel issued several letters of notice to the
responsibility by insisting that the hearing was no ko her petitioners for counter-affidavit and of the affidavit of their
necessary considering that the accused had not filed a witnesses. All were move to delay and the petitioners
petition for bail; in that much as no application for bail had contended that there was no pre-trial investigation done.
been filed my the accused, his taon orders of April 23, Jacinto Ligot applied for bail but was denied by GCM no.
2003 where not orders granting an application for bail, but 14. The RTC granted him provisional Liberty but He was
were instead his approval of the bail bond posted;and that not released immediately, then RTC ruled that the right to
Atty. Gacal’s very urgent motion and Other motions and bail covers military men facing court-martial proceedings.
written submission lacked the requisites written
conformity of the public prosecutor, rendering them null
and void. ISSUE: Whether or not the AFP personnel are entitled to
bail?

ISSUE: whether or not grant of bail by the respondent


Judge is valid. RULING: No Supreme held that the right to bail invoked
by the petitioner has traditionally not been recognized and
is not available in the military, as an exception to the
RULING: The court rules in the negative. In the light of General rule embodied in the Bill of Rights.
the foregoing facts, the failure of Judge Infante to conduct
a hearing prior to the grant of bail in capital offenses was
inexcusable and reflected gross ignorance of the law and ASWAT VS GALIDO
the rules as well as a cavalier disregard of its requirement. G.R. NO. 88555/ 1991-11-12
He well knew that the determination of whether or not the
evidence of guilt is strong was a matter of judicial
FACTS: Petitioner Eduardo N. Aswat and victim Felix B.
discretion. And that the discretion lay not in the
Nebres were both enlisted men of the Armed Forces of
determination whether or not a hearing shoulder be held,
the Philippines (AFP) respectively holding the ranks
but in the appreciation and evaluation of the weight of the
Private First Class and Corporation. Both assigned to the
Prosecutor’s evidence of guilt against the accused. His
SOLCOM . Aswat was detailed as caretaker of Brigadier
fault was made worse by his granting bail despite the
General Galido’s rest house while Nebres was assigned
absence of petition for bail from the accused.
as personal driver of Brigadier General Galido’s wife. On
Consequently, any order he issued in the absence of the
december 1988 both was involved in a Shooting incident
requisite evidence was not a product of sound judicial
at Dominican Hills, Baguio City, which resulted to the
discretion but of whom and caprice and outright
death of Nebres. Thereafter, petitioner voluntarily
arbitrariness.
surrendered to Baguio City police authorities and was
briefly incarcerated at the Baguio City Jail until He was
transferred to a SOLCOM detention cell on December
1988. On April 20 1989 He was charged before a
SOLCOM General Court Martial with violaytion of Article
94 of the Articles of War, the specification being homicide.
Petitioner filed an instant petition contending that He is
entitled to be released on bail as a matter of right pursuing
to Section 13, Article III of the Constitution.
complying with the constitutional mandate of speedy trial.
If notice to sureties is not served, no trial can be had.
ISSUE: Whether or not peritioner is entitled to right to bail.
DE LA CAMARA VS ENAGE
G.R. Nos. L-32951-2 September 17, 1971

RULING: No petitioner is not entitled to bail The


justification for this exception was well explained by the FACTS: Petitioner, Ricardo, de la Camara, Municipal
Solicitor General as follows: ‘The unique structure of the Mayor of Magsaysay, Misamis Oriental, was arrested on
military shoulder be enough reason to exempt military November 7, 1968 and detained at the Provincial Jail of
men from the constitutional coverage on the right to bail. Agusan, for his alleged participation in the killing of
Aside from structural peculiarity, it is vital to note that fourteen and the wounding of twelve other laborers of the
mutinous soldiers operate within the framework of the Tirador Logging Co., at Nato, Esperanza, Agusan del Sur,
democratic system, are allowed the fiduciary use of on August 21, 1968. Then on January 14, 1969, came an
firearms by the government for the discharge of their application for bail filed by petitioner with the lower court,
duties and responsibilities and are pair out of the premised on the assertion that there was no evidence to
revenues collected from the people. All Other insurgent link him with such fatal incident of August 21, 1968.
elements carry out their activities outside of and against Respondent Judge, on August 10, 1970, issued an order
the existing political system’. xxx xxx xxx The argument granting petitioner's application for bail, admitting that
that denial from the military of the right to bail would there was a failure on the part of the prosecution to prove
violate the equal protection clause is not acceptable. This that petitioner would flee even if he had the
guarantee requires equal treatment only to persona or opportunity,but fixed the amount of the bail bond at the
things similarly situated and does not apply where the excessive amount of P1,195,200.00,the sum of
subject of the treatment is substantially different from P840,000.00 for the information charging multiple murder
others. They cannot say that they have been and P355,200.00 for the offense of multiple frustrated
discriminated because they are not allowed the same murder. Then came the allegation that on August 12,
right that is extended to civilians. ACCORDINGLY, the 1970, the Secretary of Justice, Vicente Abad Santos,
Court revolves to DISMISS the petition for lack of Merit. upon being informed of such order, sent a telegram to
SO ORDERED. respondent Judgestating that the bond required "is
excessive" and suggesting that a P40,000.00bond, either
in cash or property, would be reasonable.
STANDARDS FOR FIXING BAIL

ISSUE: Whether or not the bail was excessive


VILLASENOR VS ABANO
(G.R. No. L-23599) 1967 RULING: Yes. Before conviction, every person is bailable
except if charged with capital offenses when the evidence
of guilt is strong. It is not beyond the realm of probability,
FACTS: Reynaldo Villasenor was charge with the murder however, that a person charged with a crime, especially
of Boac police sergeant Alfonso Madla. He was admitted so where his defense is weak, would just simply make
a P60K bail which was later on reduced to P40k. himself scarce and thus frustrate the hearing of his case.
Petitioner posted a property bond, was set at provisional A bail is intended as a guarantee that such an intent would
liberty. However the court further required that the be thwarted. Where, however, the right to bail exists, it
persons who can only post the property bond should be should not be rendered nugatory by requiring a sum that
residents of Marinduque actually staying therein which is excessive. So the Constitution commands. It is
must be in the possession and ownership of the said understandable why. If there were no such prohibition, the
resident for 5 years. right to bail becomes meaningless. Nothing can be
ISSUE: Whether or not the bond requirement is clearer, therefore, than that the challenged order of
excessive? August 10, 1970 fixing the amount of P1,195,200.00 as
the bail that should be posted by petitioner, the sum of
RULING: NO. It does not transgress the excessive bail P840,000.00 for the information charging multiple murder,
prohibition. The respondent Judge reasons out that it has there being fourteen victim, and the sum of P355,200 for
been his experience that “it is hard to send notices to the other offense of multiple frustrated murder, there
people outside the province.” The posture taken by being twelve victims, is clearly violative of constitutional
respondent judge does not offend the good sense of provision. Under the circumstances, there being only two
justice. Bail is given to secure appearance of the accused. offenses charged, the amount required as bail could not
If bondsmen reside in faraway places, even if within possibly exceed P50,000.00 for the information for
Philippines, the purpose of bail may be frustrated. There murder and P25,000.00 for the other information for
is insufficiency of the mails as an effective means of frustrated murder. Nor should it be ignored in this case
communication. And then, there is the problem of that the Department of Justice did recommend the total
sum of P40,000.00 for the two offenses. There is an
attempt on the part of respondent Judge to justify what, (2) No. The right to change abode and travel within the
on its face, appears to be indefensible by the alleged Philippines, being invoked by petitioner are not absolute
reliance on Villaseñor v. Abano.9 The guidelines in the rights. The court has the power to prohibit a person
fixing of bail was there summarized, in the opinion of admitted to bail from leaving the Philippines. This is s
Justice Sanchez, as follows: "(1) ability of the accused to necessary consequence of the nature and function of bail
give bail; (2) nature of the offense; (3) penalty for the bond. If accused were allowed to leave the Philippines
offense charged; (4) character and reputation of the without sufficient reason, he may be placed beyond the
accused; (5) health of the accused; (6) character and reach of the courts.
strength of the evidence; (7) probability of the accused
appearing in trial; (8) forfeiture of other bonds; (9) whether
the accused wasa fugitive from justice when arrested; and RIGHT TO BAIL AND RIGHT TO TRAVEL ABROAD
(10) if the accused is under bond for appearance at trial
in other cases." 10 Respondent Judge, however, did
ignore this decisive consideration appearing at the end of MANOTOC JR. VS COURT OF APPEALS
the above opinion: "Discretion, indeed, is with the court (G.R. No. L-62100, May 30 1986)
called upon to rule on the question of bail. We must stress,
however, that where conditions imposed upon a
defendant seeking bail would amount to a refusal thereof FACTS: Petitioner was charged with estafa. Then he
and render nugatory the constitutional right to bail, we will posted bail. Petitioner wanted to go abroad, so he filed a
not hesitate to exercise our supervisory powers to provide motion before the trial court stating as ground therefor his
the required remedy. In the same breath that he was told desire to go to the United States relative to his business
he could be bailed out, the excessive amount required transactions and opportunities. The prosecution opposed
could only mean that provisional liberty would be beyond the said motion and after due hearing, the same was
his reach. It would have been more forthright if he were denied. Petitioner contends that having been admitted to
informed categorically that such a right could not be bail as a matter of right, neither the courts which granted
availed of. him bail nor the Securities and Exchange Commission
could prevent him from exercising his constitutional right
to travel.
YAP JR. VS CA ISSUE: Whether or not there petitioner’s constitutional
(G.R No. 141529, June 6, 2001) right to travel has been violated?

RULING: No. A court has the power to prohibit a person


FACTS: Francisco yap was convicted of estafa for admitted to bail from leaving the Philippines. This is a
misappropriating money in the amount of P5.5 million. necessary consequence of the nature and function of bail
Then after, he filed for bail and was granted fixing the bond. The condition imposed upon petitioner to make
same in the amount similar to that of his civil liability which himself available at all times whenever the court requires
is P5.5 miliion. He also contested the condition imposed his presence operates as a valid restriction on his right to
by the court that he secure a certification/guaranty to travel. Its object is to relieve the accused of imprisonment
travel abroad, violates his right to travel. and the state of the burden of keeping him, pending trial,
Issues: (1) Whether or not the proposed bail of P5.5 and at the same time, to put the accused as much under
Million was violative of petitioner’s right against excessive the power of the court as if he were in custody of the
bail? (2) Whether or not the condition imposed by the proper officer, and to secure the appearance of the
court is violative of his liberty of abode and right to travel? accused so as to answer the call of the court and do what
the law may require of him.
RULING: (1) Yes. The court find the amount of P5.5
million to be unreasonable, excessive and constitutes an Manotoc failed to satisfy the trial courts and the appellate
effective denial of petitioner’s right to bail. The purpose of court of the urgency of his travel, the duration thereof, as
the bail is to guarantee the appearance of the accused in well as the consent of his surety to the proposed travel.
the trial. The amount should be high enough to assure the
presence of the accused when required but no higher
than is reasonably calculated to fulfil this purpose. To fix
bail at an amount equivalent to the civil liability is to permit
the impression that the amount paid as bail is an exaction
of the civil liability that accuse is charged of; this we
cannot allow because bail is not intended as a
punishment, nor as satisfaction of civil liability which
should necessarily await the judgement of the appellate
court.
SANTIAGO VS VASQUEZ his right to travel so that he may be dealt with in
(G.R. Nos. 99289-90, January 27, 1993) accordance with law. The offended party in any criminal
proceeding is the People of the Philippines. It is to their
FACTS: An information was filed against petitioner best interest that criminal prosecutions should run their
Santiago for the violation of Anti-Graft and corrupt course and proceed to finality without undue delay, with
practices act. Petitioner posted bail but later on asked for an accused holding himself amenable at all times to court
the cancellation of her bail bond and that she be allowed orders and processes.
provisional release on recognizance. The Sandiganbayan
deffered it. The latter issued a hold departure order
against Santiago by reason of the announcement she WAIVER OF THE RIGHT TO BAIL
made that she would be leaving for the U.S to accept a
fellowship in Harvard. Hence, a motion was submitted
before the Supreme Court she argues that her right to PEOPLE VS DONATO
travel is impaired. (G.R No. 79269, June 5, 1991)

ISSUE: Whether or not there is a violation of Petitioner’s


right to travel? FACTS: Private respondent Rodolfo Salas and his co-
accused were charged of rebellion. The former filed a
RULING: No. The hold departure order is but an exercise petition for bail, which was opposed because respondent
of the court’s inherent power to preserve and to maintain is not entitled to bail anymore since rebellion became a
the effectiveness of its jurisdiction over the case and the capital offense under PD 1996, 942 and 1834 amending
person of the accused. Under the obligations assumed by Art. 135 of the RPC. However the PD’s were repealed and
the petitioner in her bail bond she holds herself amenable thus restoring Art. 135 of the RPC. Judge Donato now
at all times to the orders and processes of the court, she granted the bail which was fixed at P30K but was
may legally be prohibited from leaving the country during increased to P50K and imposed a condition that he shall
the pendency of the case. report to court once every two months within the first ten
days of every period. Petitioner filed a motion for
reconsideration indirectly asking the court to deny bail
SILVERIO VS COURT OF APPEALS considering the inevitable probability that the accused will
(G.R. No. 94284, APRIL 8, 1991) not comply with the condition.

A petition for habeas corpus for private respondent and


FACTS: Ricardo Silverio was charged with violation of his co-accused was filed but was dismissed on the basis
section 20 (4) of the revised Securities Act. He then of the agreement of the parties under which herein private
posted bail for his provisional liberty. After more than two respondent Salas “will remain in legal custody and will
years after the filing of the information, respondent people face trial before the court having custody over his person”
of the Philippines filed a motion to cancel the passport of and the warrants for the arrest of his co-accused are
and to issue a hold departure order against accused on deemed recalled and they shall be immediately released
the ground that he had gone abroad several times without but shall submit themselves to the court having
the necessary court approval resulting in postponement jurisdiction over their person.
of the arraignment and scheduled hearings.
Issues: (1) Whether or not private respondent has the
Thereafter, RTC issued an order directing the Department right to bail? (2) Whether or not Accused validly waived
of Foreign Affairs to cancel Petitioner’s passport or to such right?
deny his application therefor, and the Commission on
Immigration to prevent petitioner from leaving the country. RULING: (1) Yes. The Supreme Court ruled that Bail is a
matter of right in this case since the crime is not a capital
ISSUE: Whether or not there is an impairment of offense, therefor prosecution has no right to present
petitioner’s right to travel? evidence. It is only when capital offense that the right
becomes discretionary. However it was wrong for Judge
RULING: No. Bail is the security given for the release of
Donato to change the amount of bail from 30K TO 50K
a person in custody of the law furnished by him or a
without hearing the prosecution.
bondsman, conditioned upon his appearance before any
court when so required by the Court or the Rules. The (2) Yes. In this case, accused validly waived his right to
foregoing condition imposed upon an accused to make bail. The agreement that herein private respondent will
himself available at all times whenever the court requires remain in actual physical custody of the court as
his presence operates as a valid restriction of his right to distinguished from the stipulation concerning his co-
travel. accused, who were to be released in view of the recall of
the warrants of arrest against them. The stipulation that
Holding an accused in criminal case within the reach of
only the warrants of arrest for Josefina Cruz and Jose Milo
the courts by preventing his departure from the
Conception shall be recalled and that only they shall be
Philippines must be considered as a valid restriction on
released, further confirmed the agreement that herein instances, without hearing the side of the prosecution,
private respondent shall remain in custody of the law or speaks poorly of his competence in applying the law and
detention or confinement. Bail is the security given for the jurisprudence on the matter.
release of a person in custody of law. Hence, there was a
waiver of his right. Right to bail is one of the constitutional
rights that can be waived. It is a right which is personal to PEOPLE v. PANES
the accused and whose waiver would not be contrary to
law, public order, public policy, morals or good customs,
or prejudicial to a third person with a right recognized by FACTS:
law. The above-named accused, conspiring, confederating
and mutually helping one another to better realize their
purpose armed with a knife and a .38 caliber revolver
LARDIZABAL v. REYES respectively, with treachery and/or evident premeditation,
did then and there willfully, unlawfully, and feloniously
FACTS: assault, attack, stab and shut Nicanor Tamorite with the
knife and .38 caliber revolver with which they were then
On October 25, 1993, a minor named Cynthia L. provided, inflicting upon the said Nicanor Tamorite stab
Lardizabal, through Atty Luis L. Lardizabal filed a wounds and gun \shot wounds on the different parts of his
complaint with ignorance of the law resulting in gross body which caused his death immediately thereafter.
miscarriage of justice against Judge Oscar A. Reyes.
The prosecution recommended no bail for the provisional
The complaint alleges that on September 1, 1993 that liberty of the accused. The trial court issued a warrant of
complainant filed a criminal complaint for rape. arrest against the accused. the accused filed a petition for
bail, which was opposed by the prosecution.
Respondent judge issued an order directing the arrest of
the accused, but at the same time, motu proprio, fixed the The trial court, however, did not hear the petition for bail.
bail of the accused in the amount of P80,000.00 without Neither did the accused invoke the right to bail at any
any application on the part of the accused to be admitted stage of the trial and the trial court rendered judgment and
to bail and without conducting any hearing thereon. The convicted both the accuse guilty of the crime.
accused then filed a motion to reduce bail from
P80,000.00 to P20,000.00, respondent judge, again, In the appeal, accused questioned the trial court's failure
without prior notice and hearing, reduced the bail to (a) to hear the petition for bail; (b) to consider defense of
P40,000.00 relative in favor of Ramil Manes; and (c) to take note that
Sergon Manes was a mere victim of Tamorite's unlawful
ISSUE: W/N Judge Reyes’ fixing and granting bail motu aggression.The appeal has no merit. The trial court did
proprio proper not err in finding appellants guilty of murder.

RULING: ISSUE: W/N both the accused should be granted the right
to bail?
The rule is explicit that when an accused is charged with
a serious offense punishable with reclusion perpetua, bail RULING:
may be granted only after a motion for that purpose has
been filed and a hearing thereon conducted by a judge to Under the law, in offenses punishable by reclusion
determine whether the prosecution’s evidence of guilt is perpetua, life imprisonment or death, the accused has no
strong.—The Court does not accept as satisfactory right to bail when evidence of guilt is strong. The court
respondent’s explanation that good faith urged him to fix must hear a petition for bail to determine whether the
and grant bail motu proprio for the provisional release of evidence of guilt is strong before deciding to grant or deny
the accused charged with rape. By so doing, he acted bail to the accused. While the accused can apply for bail
irregularly, thereby depriving the prosecution of an and have the court hear his application summarily and
opportunity to interpose objections to the grant of bail. The promptly, such right may be waived expressly or
rule is explicit that when an accused is charged with a impliedly.
serious offense punishable with reclusion perpetua, such
as rape, bail may be granted only after a motion for that
purpose has been filed by the accused and a hearing In this case, the trial court proceeded to try the case
thereon conducted by a judge to determine whether or not without resolving the petition for bail that appellants filed.
the prosecution’s evidence of guilt is strong. Respondent However, the latter did not call the attention of the trial
could not have arrived at a fair conclusion that the court to their unresolved application for bail. It was only in
evidence was not strong enough to deny bail to the the appeal that they raised this issue. Thus, for failure to
accused when the prosecution had not been heard on the bring to the attention of the trial court at the earliest
matter. Respondent’s unjustified haste in granting bail opportune time, appellants are deemed to have waived
and thereafter reducing the amount thereof, in both their right to bail.
What is more, the issue has been rendered academic by
the conviction of the accused. When an accused is
charged with a capital offense, or an offense punishable ISSUE: The merit of the contention of Chairman Sabio
by reclusion perpetua, or life imprisonment or death, and and his Commissioners that their refusal to appear before
evidence of guilt is strong, bail must be denied, as it is respondent Senate Committees is justified
neither a matter of right nor of discretion. Thus, the RULING:
Supreme Court affirmed the decision of the RTC.

No. Article VI, Section 21 of the 1987 Constitution grants


VIII. PRIVILEGE AGAINST SELF-INCRIMINATION
the power of inquiry not only to the Senate and the House
of Representatives, but also to any of their respective
SCOPE committees.

Furthermore, the respondents have not violated any civil


In Re: Sabio right of the individual petitioners, such as their (a) right to
privacy; and (b) right against self-incrimination; and sixth,
the inquiry does not constitute undue encroachment into
FACTS: justiciable controversies. One important limitation on the
Congress' power of inquiry is that "the rights of persons
appearing in or affected by such inquiries shall be
Years back, Pres. Cor. Aquino issued a EO no. 1 creating respected." This is just another way of saying that the
the Presidential Commission on Good Government power of inquiry must be "subject to the limitations placed
(PCGG). It is entrusted to them to recover the ill-gotten by the Constitution on government action."
wealth accumulated by the deposed President Ferdinand
As to Right to incrimination;
E. Marcos, his family, relatives, subordinates and close
associates.

It must be emphasized that this right maybe invoked by


the said directors and officers of Philcomsat Holdings
As provided by Section 4 (b) of E.O. No. 1: "No member
Corporation only when the incriminating question is being
or staff of the Commission shall be required to testify or
asked, since they have no way of knowing in advance the
produce evidence in any judicial, legislative or
nature or effect of the questions to be asked of them."
administrative proceeding concerning matters within its
That this right may possibly be violated or abused is no
official cognizance." Apparently, the purpose is to ensure
ground for denying respondent Senate Committees their
PCGG's unhampered performance of its task.
power of inquiry. The consolation is that when this power
is abused, such issue may be presented before the
courts. At this juncture, what is important is that
Senator Santiago introduced a resolution “directing an respondent Senate Committees have sufficient Rules to
inquiry in aid of legislation on the anomalous losses guide them when the right against self-incrimination is
incurred by the POTC, PHILCOMSAT and Philcomsat invoked.
Holdings due to the alleged improprieties in their
operations by their respective Board of Directors”.
Let it be stressed at this point that so long as the
constitutional rights of witnesses, like Chairman Sabio
With this, the PCGG Chairman Camilo Sabio was invited and his Commissioners, will be respected by respondent
by the Senate to be one of its resource persons. Senate Committees, it their duty to cooperate with them
in their efforts to obtain the facts needed for intelligent
legislative action. The unremitting obligation of every
However, Chairman Sabio declined, invoking Section 4 citizen is to respond to subpoenae, to respect the dignity
(b) of EO No. 1. Another notice was sent to Sabio of the Congress and its Committees, and to testify fully
requiring him to appear and testify on the same subject with respect to matters within the realm of proper
matter but the same did not comply. Sabio again sent a investigation.
letter reiterating his position.

Hence, this prompted Senator Gordon to issue an Order


requiring Chairman Sabio and Commissioners Abcede,
Conti, Javier and Nario to show cause why they should
not be cited in contempt of the Senate.
SJS vs DDB RULING:

FACTS:

YES, Sec. 36(g) of RA 9165 and COMELEC Resolution


Three consolidated petitions assailing the constitutionality No. 6486 impose an additional qualification for candidates
of Section 36 of RA 9165 or the Comprehensive for senator; NO, Congress CANNOT enact a law
Dangerous Drugs Act of 2002 insofar as it requires prescribing qualifications for candidates for senator in
mandatory drug testing of candidates for public office, addition to those laid down by the Constitution.
students of secondary and tertiary schools, officers and
employees of public and private offices, and persons In essence, Pimentel claims that Sec. 36(g) of RA 9165
charged before the prosecutor’s office with certain and COMELEC Resolution No. 6486 illegally impose an
offenses. additional qualification on candidates for senator. He
points out that, subject to the provisions on nuisance
candidates, a candidate for senator needs only to meet
the qualifications laid down in Sec. 3, Art. VI of the
The mandatory drug testing imposes an additional
Constitution, to wit: (1) citizenship, (2) voter registration,
qualification for Senators beyond that which are provided
(3) literacy, (4) age, and (5) residency. Beyond these
by the Constitution. No provision in the Constitution
stated qualification requirements, candidates for senator
authorizes the Congress or the COMELEC to expand the
need not possess any other qualification to run for senator
qualification requirements of candidates for senator.
and be voted upon and elected as member of the Senate.
The Congress cannot validly amend or otherwise modify
these qualification standards, as it cannot disregard,
Meanwhile, SJS contends that Section 36(c)(d)(f) and (g) evade, or weaken the force of a constitutional mandate,
are constitutionally infirm as it constitutes undue or alter or enlarge the Constitution.
delegation of legislative power when they give unbridled
discretion to schools and employers to determine the
manner of drug testing. It also violates the equal
Pimentel’s contention is well-taken. Accordingly, Sec.
protection clause as it can be used to harass a student or
36(g) of RA 9165 should be, as it is hereby declared as,
employee deemed undesirable. The constitutional right
unconstitutional.
against unreasonable searches is also breached.

Sec. 36(g) of RA 9165, as sought to be implemented by


Atty. Manuel J. Laserna, Jr., as a citizen and taxpayers
the assailed COMELEC resolution, effectively enlarges
maintains that said provision should be struck down as
the qualification requirements enumerated in the Sec. 3,
unconstitutional for infringing on the constitutional right to
Art. VI of the Constitution. As couched, said Sec. 36(g)
privacy, the right against unreasonable search and
unmistakably requires a candidate for senator to be
seizure, and the right against self-incrimination, and for
certified illegal-drug clean, obviously as a pre-condition to
being contrary to the due process and equal
the validity of a certificate of candidacy for senator or, with
protection guarantees.
like effect, a condition sine qua non to be voted upon and,
if proper, be proclaimed as senator-elect. The COMELEC
resolution completes the chain with the proviso that “[n]o
ISSUE: Do Sec. 36(g) of RA 9165 and COMELEC person elected to any public office shall enter upon the
Resolution No. 6486 impose an additional qualification for duties of his office until he has undergone mandatory drug
candidates for senator? Corollarily, can Congress enact a test.” Viewed, therefore, in its proper context, Sec. 36(g)
law prescribing qualifications for candidates for senator in of RA 9165 and the implementing COMELEC Resolution
addition to those laid down by the Constitution add another qualification layer to what the 1987
Constitution, at the minimum, requires for membership in
the Senate. Whether or not the drug-free bar set up under
the challenged provision is to be hurdled before or after
election is really of no moment, as getting elected would
be of little value if one cannot assume office for non-
compliance with the drug-testing requirement.
IN WHAT PROCEEDINGS AVAILABLE must be revealed, such desirable objective should not be
accomplished according to means and methods offensive
Pascual vs Board of Medical Examiners to the high sense of respect accorded to the human
personality.

FACTS:
Galman vs Pamaran

An administrative case was filed against Pascual Jr.


Salvador Gatbonton and Enriqueta Gatbonton. FACTS:

It was alleged therein that at the initial hearing of an Years back Sen. Aquino was killed inside the premises of
administrative case7 for alleged immorality, counsel for the Manila International Airport (now NAIA).
complainants announced that he would present as his first With this, to investigate the death of the Senator and his
witness herein petitioner-appellee, who was the suspected gunman, PD 1886 was promulgated creating
respondent in such malpractice charge. Thereupon, an ad hoc Fact Finding Board or the Agrava Board.
petitioner-appellee, through counsel, made of record his
objection, relying on the constitutional right to be exempt Among the witnesses who appeared, testified and
from being a witness against himself. produced evidence before the Board were the private
respondents General Fabian C. Ver, Major General
The Board of Examiners took note of such a plea but Prospero Olivas,Sgt. Pablo Martinez, Sgt. Tomas
scheduled Pascual to testify in the next hearing unless in Fernandez, Sgt. Leonardo Mojica, Sgt. Pepito Torio,
the meantime he could secure a restraining order from a Sgt.Prospero Bona and AIC Aniceto Acupido.
competent authority.
These Private respondents were charged as accessories
Pascual filed with the Court of First Instance of Manila an for killing the said senator and gunman (GALMAN).
action for prohibition with prayer for preliminary injunction
against the Board of Medical Examiners. The lower court During their court trial, the prosecution offered in evidence
ordered that a writ of preliminary injunction issue against their individuals testimonies before the Agrava Court.
the Board commanding it to refrain from hearing or further
proceeding with such an administrative case and to await In return, the private respondents, in a Motion to Exclude
the judicial disposition of the matter. Evidence, objected to the admission of said exhibits
contending that its admission will be in derogation of their
With this, a decision was rendered by the lower court constitutional right against self-incrimination and violative
finding the claim of Pascual to be well-founded and of the immunity granted by P.D. No. 1886.
prohibiting the Board "from compelling the petitioner to act
and testify as a witness for the complainant in said
investigation without his consent and against himself." ISSUE: Whether or not the testimonies made by the
ISSUE: Whether a medical practitioner charged with private respondent before the Board is immune from
malpractice in administrative case can avail of the prosecution by the virtue of their right against self-
constitutional guarantee not to be a witness against incrimination that is granted by PD 1886.
himself. RULING:

RULING: No. PD1886 merely grants immunity from use of any


Yes. The case for malpractice and cancellation of the statement that was given before the board, but not from
license to practice medicine while administrative in prosecution. Merely testifying and producing evidence do
character possesses a criminal or penal aspect. An not render the witness immune from prosecution
unfavorable decision would result in the revocation of the notwithstanding his invocation of the right against self-
license of the respondent to practice medicine. incrimination. He is merely saved from the use against
Consequently, he can refuse to take the witness stand. him of such statement and nothing more.

The right against self-incrimination extends not only to


right to refuse to answer questions put to the accused
while on witness stand, but also to forgo testimony, to
remain silent and refuse to take the witness stand when
called by as a witness by the prosecution. The reason is
that the right against self incrimination, along with the
other rights granted to the accused, stands for a belief that
while a crime should not go unpunished and that the truth
In Re Sabio

“USE IMMUNITY” VS. “TRANSACTIONAL IMMUNITY”


FACTS:
A. USE AND FRUIT IMMUNITY

Pursuant to Senate Resolution No. 455, Senator Gordon Galman vs Pamaran


requested PCGG Chairman Sabio and his
Commissioners to appear as resource persons in the
public meeting jointly conducted by the Committee on FACTS:
Government Corporations and Public
Years back Sen. Aquino was killed inside the premises of
Enterprises and Committee on Public Services.
the Manila International Airport (now NAIA).

With this, to investigate the death of the Senator and his


Chairman Sabio declined the invitation because of prior suspected gunman, PD 1886 was promulgated creating
commitment, and at the same time invoked Section 4(b) an ad hoc Fact Finding Board or the Agrava Board.
of EO No. 1: “No member or staff of the Commission shall
Among the witnesses who appeared, testified and
be required to testify or produce evidence in any judicial,
produced evidence before the Board were the private
legislative or administrative proceeding concerning
respondents General Fabian C. Ver, Major General
matters within its official cognizance.”
Prospero Olivas,Sgt. Pablo Martinez, Sgt. Tomas
Fernandez, Sgt. Leonardo Mojica, Sgt. Pepito Torio,
Sgt.Prospero Bona and AIC Aniceto Acupido.
ISSUE: Whether or not Section 4(b) of E.O. No.1 limits
power of legislative inquiry by exempting all PCGG These Private respondents were charged as accessories
members or staff from testifying in any judicial, legislative for killing the said senator and gunman (GALMAN).
or administrative proceeding.
During their court trial, the prosecution offered in evidence
their individuals testimonies before the Agrava Court.

RULING: No. Article VI, Section 21 of the 1987 In return, the private respondents, in a Motion to Exclude
Constitution grants the power of inquiry not only to the Evidence, objected to the admission of said exhibits
Senate and the House of Representatives, but also to any contending that its admission will be in derogation of their
of their respective committees. Clearly, there is a direct constitutional right against self-incrimination and violative
conferral of investigatory power to the committees and it of the immunity granted by P.D. No. 1886.
means that the mechanism which the Houses can take in
order to effectively perform its investigative functions are
also available to the committees. ISSUE: Whether or not the testimonies made by the
private respondent before the Board is immune from
It can be said that the Congress’ power of inquiry has
prosecution by the virtue of their right against self-
gained more solid existence and expansive construal.
incrimination that is granted by PD 1886.
Considering these jurisprudential instructions, Section
4(b) is directly repugnant with Article VI, Section 21.
Section 4(b) exempts the PCGG members and staff from RULING:
the Congress’ power of inquiry. This cannot be
countenanced. Nowhere in the Constitution is any
provision granting such exemption. The Congress’
No. PD1886 merely grants immunity from use of any
power of inquiry, being broad, encompasses everything
statement that was given before the board, but not from
that concerns the administration of existing laws as well
prosecution. Merely testifying and producing evidence do
as proposed or possibly needed statutes. It even
not render the witness immune from prosecution
extends “to government agencies created by Congress
notwithstanding his invocation of the right against self-
and officers whose positions are within the power of
incrimination. He is merely saved from the use against
Congress to regulate or even abolish.” PCGG belongs to
him of such statement and nothing more.
this class.

A statute may be declared unconstitutional because it is


not within the legislative power to enact; or it creates or
establishes methods or forms that infringe constitutional
principles; or its purpose or effect violates the Constitution
or its basic principles.
IX. RIGHT TO SPEEDY DISPOSITION OF CASES appellant to show cause why his appeal should not be
dismissed.

Dimarucot vs People
In the case at bar, there is no showing that petitioner was
FACTS: served with a notice requiring him to show cause why his
appeal should not be dismissed for failure to file
appellant's brief. The purpose of such a notice is to give
an appellant the opportunity to state the reasons, if any,
Petitioner is the accused in a criminal case for Frustrated
why the appeal should not be dismissed because of such
Murder in the Regional TrialCourt of Malolos, Bulacan.
failure, in order that the appellate court may determine
Upon receiving the notice to file appellants brief, petitioner whether or not the reasons, if given, are satisfactory.
thru his counsel de parte requested and was granted
additional period of twenty (20) days within which to fileaid
brief. This was followed by three (3) successive motions Petitioner cannot simply harp on the mistakes and
for extension which were all granted by the Court of negligence of his lawyer allegedly beset with personal
Appeals. problems and emotional depression. The negligence and
mistakes of counsel are binding on the client. There are
exceptions to this rule, such as when the reckless or gross
The CA issued a Resolution dismissing the appeal for the negligence of counsel deprives the client of due process
accused-appellant failed to file his appellants brief within of law, or when the application of the general rule results
the reglementary period which expired. in the outright deprivation of one's property or liberty
through a technicality. However, in this case, we find no
reason to exempt petitioner from the general rule.
Petitioner filed a motion for reconsideration, his counsel
admitting that he was at fault in failing the appellants brief
due to personal problems emanating from his counsel’s The admitted inability of his counsel to attend fully and
wife recent surgical operations. ably to the prosecution of his appeal and other sorts of
excuses should have prompted petitioner to be more
vigilant in protecting his rights and replace said counsel
with a more competent lawyer. Instead, petitioner
ISSUE: Whether or not the petitioner’s contention is valid
continued to allow his counsel to represent him on appeal
and even up to this Court, apparently in the hope of
moving this Court with a fervent plea for relaxation of the
RULING: rules for reason of petitioner's age and medical condition.
Verily, diligence is required not only from lawyers but also
from their clients
No. It is not valid.

As stated in Section 8, paragraph 1, Rule 124 of


the Revised Rules of Criminal Procedure, as amended,

SEC. 8. Dismissal of appeal for abandonment or failure to


prosecute. - The Court of Appeals may, upon motion of
the appellee or motu proprio and with notice to the
appellant in either case, dismiss the appeal if the
appellant fails to file his brief within the time prescribed by
this Rule, except where the appellant is represented by a
counsel de oficio.

It is clear under the foregoing provision that a criminal


case may be dismissed by the CA motu proprio and with
notice to the appellant if the latter fails to file his brief
within the prescribed time. The phrase "with notice to the
appellant" means that a notice must first be furnished the
Angeles vs Sempio-diy was convicted was serious and his liberty was at stake.
She adds that the death threats she and the members of
her judicial staff received from May to July 2009, caused
FACTS:
them disorientation and contributed further to the delay in
the resolution of the subject motion.

Judge Sempio-Dy is the judge in criminal cases file by


Judge Angeles.
A judge ought to know the cases submitted to her for
decision or resolution and is expected to keep her own
record of cases so that she may act on them promptly. It
Complainant Judge Angeles filed an administrative is incumbent upon her to devise an efficient recording and
complaint for disbarment and dismissal from judiciary filing system in her court so that no disorderliness can
service against respondent Judge Sempio Diy which affect the flow of cases and their speedy disposition.
stemmed from consolidated criminal case that filed. Proper and efficient court management is as much her
responsibility. She is the one directly responsible for the
proper discharge of her official functions.
Judge Angeles further alleges that she was the private
complainant in the above-mentioned cases which by
order of respondent judge were submitted for decision The Court reminds the respondent of her duty to closely
and set for promulgation. However, said rendering of supervise and monitor the monthly docket inventories to
decision and promulgation of judgment incur delay after a forestall future occurrences of this nature. The petition
lapse of 90 days and six (6) months, respectively, from was admonished.
the time it was submitted for resolution to the time it was
promulgated.
Raymundo vs Andoy

In return, Respondent judge belies to the accusations


hurled at her by complainant. The former counters that FACTS:
she decided subject cases in due time and within the
extended period granted by the Supreme Court.

The complainant, Cirila Raymundo, sometime in 2000,


ISSUE: Should Judge Dy be punished for the delay filed six counts of violation of BP Blg 22 against
Hermelinda Chang before the MTC of Cainta, Rizal to the
respondent judge presided over the court.
RULING:

In time, after the trial ended the complainant filed with the
No. Upon investigation, OCA found that Judge Sempio MTC an urgent ex parte motion to render decision. And
Diy cannot be held guilty of unreasonable delay in almost two years again, on March 12, 2008, the
rendering the Joint Decision in Criminal Case Nos. Q-95- complainant filed another ex parte motion to render
61294 and Q-95-62690 given her seasonably-filed decision. The respondent judge did not act on these
requests for extension of time. The requests were all motions on the reason of the heavy case on the part of
granted by this Court in the November 24, 2008 the Judge.
Resolution, giving respondent a total extension period of
ninety (90) days from September 18, 2008. The OCA,
however, opined that respondent should be In the findings of the OCA: The OCA explained that while
administratively sanctioned for incurring delay in the the Court is not unaware of the heavy caseload of judges,
resolution of accused Carino’s Urgent Motion for nothing in the records shows that the respondent judge
Reconsideration. asked for an extension of time to decide the subject
criminal cases. In addition, the respondent judge failed to
consider that the subject cases required a quicker
Respondent Judge Sempio Diy claims that the delay in resolution as they were covered by the Rule on Summary
submitting accused’s motion for reconsideration was due Procedure.
to inadvertence and without bad faith on her part. She
explains that she opted to wait for the defence to file its
reply to the prosecution’s comment on the motion for
reconsideration because the offense of which accused
ISSUE: Whether or not the respondent judge is guilty of respondent trial courts, except in one case, wherein the
the charges against him trial court declared the law unconstitutional and dismissed
the case.

RULING:
BP 22 punishes a person "who makes or draws and
issues any check on account or for value, knowing at the
Yes. As the Court discussed, it was clear that the time of issue of insufficient funds in or credit with the
respondent judge failed to observe the mandated period drawee bank for payment of said check in full upon
of time to decide cases under the Rule on Summary presentment, which check is subsequently dishonored by
Procedure. Following Section 17 of this Rule, he should the drawee bank for insufficiency of funds or credit or
have rendered a decision within 30 days from the would have been dishonored for the same reason had not
termination of trial on August 4, 2004. Even assuming that the drawer, without any valid reason, ordered the bank to
the subsequent resettings of the cases for trial were valid, stop payment."
he should have rendered a decision within 30 days from
October 12, 2005, or the date the cases were finally
considered submitted for decision. His failure to meet this The statute likewise imposes the same penalty on "any
deadline is a patent indication that he did not take into person who, having sufficient funds in or credit with the
account and had disregarded the Rule on Summary drawee bank when he makes or draws and issues a
Procedure. check, shall fail to keep sufficient funds or to maintain a
credit to cover the full amount of the check if presented
within a period of 90 days from the date appearing
The Constitution mandates that all cases or matters filed thereon, for which reason it is dishonored by the drawee
before all lower courts shall be decided or resolved within bank. An essential element of the offense is "knowledge"
90 days from the time the case is submitted for decision. on the part of the maker or drawer of the check of
Failure to comply with the mandated period constitutes a insufficiency of his funds in or credit with the bank to cover
serious violation of the constitutional right of the parties to the check upon its presentment.
a speedy disposition of their cases a lapse that
undermines the people’s faith and confidence in the
judiciary, lowers its standards and brings it to disrepute. Among the constitutional objections raised against BP 22,
the most serious is the alleged conflict between the
statute and the constitutional provision forbidding
As correctly pointed out by the OCA, while the imprisonment for debt. It is contended that the statute
respondent judge attributed his failure to render a runs counter to the inhibition in the Bill of Rights which
decision to the heavy caseload in his sala, he did not ask states, "No person shall be imprisoned for debt or non-
for an extension of time to decide the cases. This failure payment of a poll tax."
to decide within the required period, given that he could
have asked for an extension, is inexcusable; it constitutes
neglect of duty as well as gross inefficiency that ISSUE: Has BP 22 transgressed the constitutional
collectively warrant administrative sanction. inhibition against imprisonment for debt?

X. SUBSTANTIVE RIGHTS UNDER THE DUE PROCESS RULING: No.


CLAUSE

WHAT ACTS CANNOT BE CRIMINALIZED

A. DEBTS AND CIVIL OBLIGATIONS The Court held that BP 22 does not conflict with the
constitutional inhibition against imprisonment for debt.
LOZANO vs. MARTINEZ G.R. NO. L-63419 (1986) The enactment of BP 22 is a declaration by the legislature
that as a matter of public policy, the making and issuance
of a worthless check is deemed a public nuisance to be
FACTS:
abated by the imposition of penal sanctions. Organic
The subject petitions arose from cases involving provisions relieving from imprisonment for debt were
prosecution of offenses under BP 22 or the Bouncing intended to prevent commitment of debtors to prison for
Checks Law. The defendants in these cases moved liabilities arising from actions ex contractu. The inhibition
seasonably to quash the informations on the ground that was never meant to include damages arising in actions ex
the acts charged did not constitute an offense, the statute delicto, for the reason that damages recoverable therein
being unconstitutional. The motions were denied by the do not arise from any contract entered into between the
parties but are imposed upon the defendant for the wrong From the above definition, it is clear that a memorandum
he has done and are considered as punishment, nor to check, which is in the form of an ordinary check, is still
fines and penalties imposed by the courts in criminal drawn on a bank and should therefore be distinguished
proceedings as punishments for crime from a promissory note, which is but a mere promise to
pay. In the business community, a promissory note,
certainly, has less impact and persuadability than a check
The gravamen of the offense punished by BP 22 is the act A memorandum check must therefore fall within the ambit
of making and issuing a worthless check or a check that of B.P. 22 which does not distinguish but merely provides
is dishonored upon its presentation for payment. It is not that "any person who makes or draws and issues any
the non-payment of an obligation which the law punishes. check knowing at the time of issue that he does not have
The law is not intended or designed to coerce a debtor to sufficient funds in or credit with the drawee bankwhich
pay his debt. The thrust of the law is to prohibit, under pain check is subsequently dishonored shall be punished by
of penal sanctions, the making of worthless checks and imprisonment. Ubi lex non distinguit nec nos distinguere
putting them in circulation. debemus.

PEOPLE vs. NITAFAN 202 SCRA 726 (1992) A memorandum check, upon presentment, is generally
accepted by the bank. The mere act of issuing a worthless
check, whether as a deposit, as a guarantee, or even as
FACTS: an evidence of a pre-existing debt, is malum prohibitum.
Thus, a memorandum check is still covered under BP 22
and can still be prosecuted for violation of the bouncing
Private respondent K.T. Lim was charged before checks law.
respondent court with violation of B.P. 22 in an
Information alleging that he made or draw and issued to
Fatima Cortez Sasaki a Philippine Trust Company Check VERGARA vs. GEDORIO 402 SCRA 520 (2003)
dated February 9, 1985 in the amount of P143,000 well-
knowing that at that time, he did not have sufficient funds
in or credit with the drawee bank. The check was FACTS:
subsequently dishonored by the drawee bank for
insufficiency of funds, and despite receipt of notice of
such dishonor, said accused failed to pay said Fatima Petitioners are tenants of Berlito P. Taripe on a property
Cortez Sasaki the amount of said check or to make in Paranaque City. In 2001, they were arrested by
arrangement for full payment of the same within five (5) authority of a warrant of arrest stemmed from a motion
banking days after receiving said notice. filed by Eleuteria P. Bolaño, as Special Administratrix of
the estate of late Anselma P. Allers, praying that
petitioners be held guilty of indirect contempt for not
Private respondent moved to quash the Information on complying with probate court's order directing them to pay
the ground that the facts charged did not constitute a their monthly rentals to respondent Bolaño
felony as B.P. 22 was unconstitutional and that the check
he issued was a memorandum check which was in the
nature of a promissory note, perforce, civil in nature. Pending the settlement of the estate of the deceased
Anselma P. Allers, respondent Bolaño included the said
property leased by Taripe to petitioners in the inventory of
ISSUE: WON the issuance of a worthless memorandum the estate. The probate court ordered that Taripe’s
check is punishable? lessees and listed in the

RULING: Yes Inventory to pay their respective monthly rental regularly


starting the month of August, 1999, including arrears if
A memorandum check is in the form of an ordinary check, any, to the duly appointed Special Administratrix Mrs.
with the word "memorandum", "memo" or "mem" written Eleuteria P. Bolaño.
across its face, signifying that the maker or drawer
engages to pay the bona fide holder absolutely, without
any condition concerning its presentment. Such a check Some of the petitioners, together with the other tenants of
is an evidence of debt against the drawer, and although the property, informed the probate court that they are
may not be intended to be presented, has the same effect "freezing" their monthly rentals as they are in a quandary
as an ordinary check, and if passed to a third person, will as to whom to pay the rentals.
be valid in his hands like any other check.
B. ACTS WHICH WHEN DONE WERE INNOCENT

(EX POST FACTO LAWS)


Bolaño then filed a motion to cite Vergara, et. al. in
contempt. The probate court found Vergara, et. al. guilty
of indirect contempt and ordered them to pay a fine of
P30,000 each and to undergo imprisonment until they KAY VILLEGAS KAMI 35 SCRA 429 (1970)
comply with the probate court's order for them to pay
rentals. Vergara et. al. wrote the probate court to withdraw FACTS:
the “indirect contempt” and stated that their failure to
attend the hearing was due to financial constraints, most
of them working on construction sites, receiving minimum
wages, and repeated that the reason why they are A petition for declaratory relief was filed by Kay Villegas
freezing the monthly rentals is that they are uncertain as Kami, Inc. claiming to be duly recognized as an existing
to whom to remit it. non-stock and non-profit corporation. It also prayed for the
determination of the validity of Section 8 of R.A. 6132
which prohibits any candidate for delegate to the
Constitutional Convention to allow himself to be
ISSUE: Whether imprisonment due to non-payment of represented as being a candidate of any political party or
rentals, made in contravention of the trial court’s order, is any other organization or organized group of whatever
within the purview of the right against imprisonment from nature.
debt?

Kay Villegas Kami, Inc avers that it has printed materials


RULING: No. designed to propagate its ideology and program of
government and that it intends to pursue its purposes by
supporting delegates to the Constitutional Convention
Imprisonment for contempt as a means of coercion for who will propagate the same ideology.
civil purpose cannot be resorted to until all other means
fail, but the court's power to order the contemnor's
detension continues so long as the contumacy persists. According to the group, RA 6132 violates due process,
right of association, and freedom of expression. It also
claims that the act is in the nature of an ex post facto law.
In Philippine jurisdiction, Section 20, Article 3 of the 1987
Philippine Constitution expressly provides that no person
shall be imprisoned for debt. Debt, as used in the ISSUE: WON the challenged provision constitutes an ex
Constitution, refers to civil debt or one not arising from a post facto law?
criminal offense. It means any liability to pay arising out of
a contract, express or implied.

RULING: No. An ex post facto law is one which:

In the present case, petitioners, as recognized lessees of


the estate of the deceased, were ordered by the probate
court to pay the rentals to the administratrix. Petitioners (1) makes criminal an act done before the passage
did not comply with the order for the principal reason that of the law and which was innocent when done, and
they were not certain as to the rightful person to whom to punishes such an act;
pay the rentals because it was a certain Berlito P. Taripe
who had originally leased the subject property to them.
Clearly, the payment of rentals is covered by the (2) aggravates a crime, or makes it greater than it
constitutional guarantee against imprisonment. was, when committed;

(3) changes the punishment and inflicts a greater


punishment than the law annexed to the crime when
committed;

(4) alters the legal rules of evidence, and authorizes


conviction upon less or different testimony than the law
required at the time of the commission of the offense;
A behest loan is under-collateralized, the borrower
corporation is undercapitalized, direct or indirect
(5) assuming to regulate civil rights and remedies endorsement by high government officials like presence
only, in effect imposes penalty or deprivation of a right for of marginal notes, use of corporate layering, extraordinary
something which when done was lawful; and speed in which the loan release was made, stockholders,
officers or agents of the borrower corporation are
identified as cronies, among others.
(6) deprives a person accused of a crime of some
lawful protection to which he has become entitled, such
as the protection of a former conviction or acquittal, or a After examination of the loan transactions, the Committee
proclamation of amnesty. determined that it is a behest loan because the
stockholders and officers of PEMI were known cronies of
then President Ferdinand Marcos; the loan was under-
From the aforesaid definition as well as classification of collateralized; and that the PEMI was undercapitalized at
ex post facto laws, the constitutional inhibition refers only the time the loan was granted.
to criminal laws which are given retroactive effect.

Atty. Orlando L. Salvador, Consultant of the Fact-Finding


While it is true that Sec. 18 penalizes a violation of any Committee, and representing the PCGG,
provision of R.A. No. 6132 including Sec. 8 (a) thereof,
the penalty is imposed only for acts committed after the filed with the Office of the Ombudsman a sworn complaint
approval of the law and not those perpetrated prior for violation of Sections 3(e) and (g) of R.A.3019, or the
thereto. There is nothing in the law that remotely Anti-Graft and Corrupt Practices Act, against the
insinuates that Secs. 8(a) and 18, or any other provision respondents Placido I. Mapa, Jr. et al.
thereof, shall apply to acts carried out prior to its approval.
On the contrary, Sec. 23 directs that the entire law shall
be effective upon its approval. It was approved on August After considering the Committee's allegation, the
24, 1970. Ombudsman dismissed the complaint complaint holding
that the offenses charged had already prescribed and that
Administrative Order No. 13 and Memorandum Order No.
Side ISSUE: Sec. 8 of R.A. 6132 not violative of due 61 are ex post facto laws. According to it, the said orders
process, right of association and freedom of expression. violate the prohibition against ex post facto laws for
ostensibly inflicting punishment upon a person for an act
done prior to their issuance and which was innocent when
done.
R.A. 6132 is designed to prevent the clear and present
danger of the twin substantive evils, namely, the
prostitution of electoral process and denial of the equal
protection of the laws. ISSUE: WON AO 13 is an ex post facto law?

SALVADOR vs. MAPA 539 SCRA 37 (2007) RULING: No. An ex post facto law is one which:

FACTS:
(1) makes criminal an act done before the passage
of the law and which was innocent when done, and
punishes such an act;
Fidel Ramos issued A.O. 13 which created the Ad Hoc
Fact Finding committee on behest loans. A loan
transaction between Metals Exploration Asia, Inc. (MEA),
(2) aggravates a crime, or makes it greater than it
now Philippine Eagle Mines, Inc. (PEMI) and the
was, when committed;
Development Bank of the Philippines (DBP) were referred
to the Presidential Ad Hoc Fact Finding-Committee for
investigation as being a behest loan. Subsequently,
Memorandum Order No. 61 was issued to define criteria (3) changes the punishment and inflicts a greater
in determining behest loans. punishment than the law annexed to the crime when
committed;
(4) alters the legal rules of evidence, and authorizes PEOPLE vs. CASTA 565 SCRA 341 (2008)
conviction upon less or different testimony than the law
required at the time of the commission of the offense;
FACTS:

The prosecution charged the appellant before the RTC


(5) assuming to regulate civil rights and remedies with the crime of murder by inflicting upon Danilo Camba
only, in effect imposes penalty or deprivation of a right for the injuries using a knife which caused the latter’s
something which when done was lawful; and instantaneous death. The appellant pleaded not guilty to
the charge upon arraignment. However, at the police
station, the appellant confessed to the killing of Danilo
after being informed of his constitutional rights and in the
(6) deprives a person accused of a crime of some
presence of counsel.
lawful protection to which he has become entitled, such
as the protection of a former conviction or acquittal, or a The RTC convicted the appellant guilty beyond
proclamation of amnesty. reasonable doubt of the crime of Murder and hereby
sentences him to suffer the penalty of reclusion perpetua
and to indemnify the heirs of the deceased.
Orders issued are not ex post facto laws (only for penal
The Information indicates that the crime was committed
laws). The constitutional doctrine that outlaws an ex post
on August 20, 1989 which was before the effectivity of
facto law generally prohibits the retrospectivity of penal
Republic Act No. 7659 on December 31, 1993 amending
laws.
Article 248 of the Revised Penal Code on murder, raising
the penalty to reclusion perpetua to death. Prior to its
amendment the penalty for the crime of murder under
Penal laws are those acts of the legislature which prohibit Article 248 of the Revised Penal Code was reclusion
certain acts and establish penalties for their violations; or temporal in its maximum period to death.
those that define crimes, treat of their nature, and provide
for their punishment. In light of the greater penalty that attaches under the
amendment, the previous penalty of reclusion temporal in
its maximum period to death will have to be imposed in
The subject administrative and memorandum orders order not to run afoul of the constitutional prohibition
clearly do not come within the shadow of this definition. against ex post facto laws.
Administrative Order No. 13 creates the Presidential Ad
Hoc Fact-Finding Committee on Behest Loans, and
provides for its composition and functions. It does not ISSUE: WON the penalty imposed will be based on RA
mete out penalty for the act of granting behest loans. 7659
Memorandum Order No. 61 merely provides a frame of
reference for determining behest loans. Not being penal
laws, Administrative Order No. 13 and Memorandum RULING: The punishment for his acts must be based on
Order No. 61 cannot be characterized as ex post facto the law applicable during the time he killed the victim in
laws. There is, therefore, no basis for the Ombudsman to 1993. He cannot be prosecuted for the new law under
rule that the subject administrative and memorandum 1998 because it provides greater penalty.
orders are ex post facto.
Under Section 22 of Article III of the 1987 Constitution, no
ex post facto law or bill of attainder shall be enacted. An
ex post facto law, among others, is one that changes the
Side ISSUE: The offense charged under R.A. 3019 has
penalty and inflicts a greater punishment than what the
not prescribed. Since the prescriptive period commenced
law annexed to the crime when committed - the situation
to run on the date of the discovery of the offenses, and
that would obtain if the amendment under RA 7659 would
since discovery could not have been made earlier than
be applied.
October 8, 1992, the date when the Committee was
created, the criminal offenses allegedly committed by the Considering that the appellant has in his favor the
respondents had not yet prescribed when the complaint mitigating circumstance of voluntary surrender with no
was filed on October 4, 1996. aggravating circumstance to offset it, he was sentenced
to suffer the indeterminate penalty of imprisonment for 10
years and 1 day of prision mayor maximum, as minimum,
to 17 years 4 months and 1 day of reclusion temporal
maximum, as maximum.
NASI-VILLAR vs. PEOPLE 571 SCRA 202 (2008) RULING: No.

FACTS:
The basic rule is that a criminal act is punishable under
the law in force at the time of its commission. Thus, she
was properly convicted under the Labor Code, and not
This case originated from an Information for Illegal
under R.A. 8042.
Recruitment as defined under Sections 6 and 7 of R.A.
8042 filed by the Office of the Provincial Prosecutor of
Davao del Sur on 5 October 1998 for acts committed by
petitioner and one Dolores Placa in January 1993. The There is neither violation of the prohibition against ex post
aforenamed accused recruited Nila Panilag for facto law nor a retroactive application of R.A. 8042, as
employment abroad and demanded and received the alleged by petitioner. An ex post facto law is one which,
amount of P6,500 as placement fee, the said accused among others, aggravates a crime or makes it greater
being a non-licensee or non-holder of authority to engage than it was when committed or changes the punishment
in the recruitment of workers abroad. and inflicts a greater punishment than the law annexed to
the crime when committed. Penal laws and laws which,
while not penal in nature, nonetheless have provisions
defining offenses and prescribing penalties for their
The RTC hereby finds accused Rosario Nasi-Villar guilty
violation operate prospectively. Penal laws cannot be
beyond reasonable doubt of Illegal Recruitment and
given retroactive effect, except when they are favorable
based on the penalty set forth under the Labor Code, as
to the accused.
amended, said accused is hereby sentenced to an
indeterminate penalty ranging from 4 YEARS as minimum
to 5 YEARS as maximum. The CA declared that petitioner
should be charged under Art. 13(b) of Labor Code and not R.A. No. 8042 amended pertinent provisions of the Labor
under R.A. 8042 since said law was approved on 7 June Code and gave a new definition of the crime of illegal
1995 and took effect on 15 July 1995. recruitment and provided for its higher penalty. There is
no indication in R.A. No. 8042 that said law, including the
penalties provided therein, would take effect retroactively.
A law can never be considered ex post facto as long as it
Petitioner alleged that CA erred in failing to consider that
operates prospectively since its strictures would cover
R.A. 8042 cannot be given retroactive effect and that the
only offenses committed after and not before its
decision of RTC constitutes a violation of the
enactment.
constitutional prohibition against ex post facto law.

(BILL OF ATTAINDER)
Since R.A. No. 8042 did not yet exist in January 1993
when the crime was allegedly committed, petitioner
argues that law cannot be used as the basis of filing a
PEOPLE vs. FERRER 48 SCRA 382 (1972)
criminal action for illegal recruitment. What was applicable
in 1993 is the Labor Code, where under Art. 38, in relation
to Art. 39, the violation of the Code is penalized with FACTS:
imprisonment of not less than 4 years nor more than 8
years or a fine of not less than P20,000 and not more than A criminal complaint for violation of section 4 of the Anti-
P100,000 or both. On the other hand, Sec. 7(c) of R.A. Subversion Act was filed against the respondent Feliciano
No. 8042 penalizes illegal recruitment with a penalty of Co on the alleged that he became an officer and/or
imprisonment of not less than 6 years and 1 day but not ranking leader of the Communist Party of the Philippines,
more than 12 years and a fine not less than P200,000 nor an outlawed and illegal organization aimed to overthrow
more than P500,000. Thus, the penalty of imprisonment the Government of the Philippines by means of force,
provided in the Labor Code was increased by R.A. 8042. violence, deceit, subversion, or any other illegal means
Petitioner concludes that the charge and conviction of an for the purpose of establishing in the Philippines a
offense carrying a penalty higher than that provided by the totalitarian regime and placing the government under the
law at the time of its commission constitutes a violation of control and domination of an alien power, by being an
the prohibition against ex post facto law and the instructor in the Mao Tse Tung University, the training
retroactive application of R.A. 8042. school of recruits of the New People's Army, the military
arm of the said Communist Party of the Philippines. Co
moved to quash on the ground that the Anti-Subversion
Act is a bill of attainder.
ISSUE: WON RA 8042 is an ex post facto law?
Meanwhile, another criminal complaint was filed with the now Mines and Geosciences Bureau, for the issuance of
same court, charging the respondent Nilo Tayag and five the corresponding license to exploit said marble deposits.
others with subversion on the alleged that the above- The Mines and Geosciences Bureau granted a license to
named accused knowingly, willfully and by overt acts exploit marble deposits. Subsequently, Maceda through
organized, joined and/or remained as officers and/or Proclamation No. 84, unilaterally cancelled such license
ranking leaders, of the KABATAANG MAKABAYAN, a because it violated Section 69 of PD 463. The latter
subversive organization. Tayag moved to quash, reason was confirmed by the language of Proclamation
impugning the validity of the statute on the grounds that No. 84. According to this law, public interest would be
(1) it is a bill of attainder; (2) it is vague; (3) it embraces served by reverting the parcel of land that was excluded
more than one subject not expressed in the title; and (4) by Proclamation No. 2204 to the former status of that land
it denies him equal protection of laws. as part of the Biak-na-Bato national park. The RTC held
that said proclamation was an ex post facto law as it
automatically cancelled the license without trial.
ISSUE: Is the Act a Bill of Attainder? ISSUE: WON the Constitutional prohibition against ex
post facto law applies to Proclamation No. 84?

RULING: Yes.

Article III, Section 1 (11) of the Constitution states that "No RULING:
bill of attainder or ex post facto law shall be enacted." A
bill of attainder is a legislative act which inflicts
punishment without trial. Its essence is the substitution of The Court did not sustain the argument that Proclamation
a legislative for a judicial determination of guilt. The No. 84 is a bill of attainder; that is, a "legislative act which
constitutional ban against bills of attainder serves to inflicts punishment without judicial trial. Neither is the
implement the principle of separation of powers by cancellation of the license a punishment within the
confining legislatures to rule-making. purview of the constitutional proscription against bills of
attainder. There is no merit in the argument that the
In this case, the Anti-Subversion Act Was condemned as proclamation is an ex post facto law. There are six
a bill of attainder because it "tars and feathers" the recognized instances when a law is considered as such:
Communist Party of the Philippines as a "continuing
menace to the freedom and security of the country; its
existence, a 'clear, present and grave danger to the
security of the Philippines.' However, the Court held that 1) it criminalizes and punishes an action that was
the Anti-Subversion Act is a bill of attainder. done before the passing of the law and that was innocent
when it was done;
When the Act is viewed in its actual operation, it will be
seen that it does not specify the Communist Party of the
Philippines or the members thereof for the purpose of 2) it aggravates a crime or makes it greater than it
punishment. What it does is simply to declare the Party to was when it was committed;
be an organized conspiracy for the overthrow of the
Government for the purposes of the prohibition, stated in
section 4, against membership in the outlawed
3) it changes the punishment and inflicts one that is
organization. The "Communist Party of the Philippines" is
greater than that imposed by the law annexed to the crime
used solely for definitional purposes. In fact the Act
when it was committed;
applies not only to Communist Party of the Philippines but
also to "any other organization having the same purpose
and their successors." Its focus is not on individuals but
on conduct. 4) it alters the legal rules of evidence and authorizes
conviction upon a less or different testimony than that
required by law at the time of commission of offense;
REPUBLIC vs. RMDC 426 SCRA 517 (2004)

5) it assumes the regulation of civil rights and


FACTS: remedies only, but in effect imposes a penalty or a
Petitioners, after having been granted permission to deprivation of a right as a consequence of something that
prospect for marble deposits in the mountains of Biak-na- was considered lawful when it was done; and
Bato in Bulacan, succeeded in discovering marble
deposits of high quality and in commercial quantities in
Mount Mabio which forms part of the Biak-na-Bato 6) it deprives a person accused of a crime of some
mountain range. They applied with the Bureau of Mines, lawful protection to which he or she become entitled, such
as the protection of a former conviction or an acquittal or RULING: Yes.
the proclamation of an amnesty.

The return-to-work order is issued pending the


Proclamation No. 84 does not fall under any of the determination of the legality or illegality of the strike. It is
enumerated categories; hence, it is not an ex post facto not correct to say that it may be enforced only if the strike
law. It is settled that an ex post facto law is limited in its is legal and may be disregarded if the strike is illegal, for
scope only to matters criminal in nature. Proclamation 84, the purpose precisely is to maintain the status quo while
which merely restored the area excluded from the Biak- the determination is being made.
na-Bato national park by cancelling respondents' license,
is not penal in nature.
While one purpose of the return-to-work order is to protect
the workers who might otherwise be locked out by the
WHAT PUNISHMENTS CANNOT BE IMPOSED employer for threatening or waging the strike, the more
important reason is to prevent impairment of national
interest in case the operations of the company are
A. INVOLUNTARY SERVITUDE disrupted by a refusal of the strikers to return to work as
directed. In the instant case, stoppage of work in the firm
SARMIENTO vs. TUICO 162 SCRA 676 (1988) will be hurtful not only to both the employer and the
employees but also to the national economy because of
the resultant reduction in our export earnings and our
FACTS: dollar reserves.

Petitioner Asian Transmission Corporation terminated the It is also important to emphasize that the return-to-work
services of Catalino Sarmiento, vice-president of the Bisig order not so much confers a right as it imposes a duty;
ng Asian Transmission Labor Union (BATU), for allegedly and while as a right it may be waived, it must be
carrying a deadly weapon in the company premises. As a discharged as a duty even against the worker's will.
result, the BATU filed a notice of strike on May 26, 1986, Returning to work in this situation is not a matter of option
claiming that the ATC had committed an unfair labor or voluntariness but of obligation. The worker must return
practice. The conciliatory conference failed to settle the to his job together with his co-workers so the operations
dispute. of the company can be resumed and it can continue
serving the public and promoting its interest. That is the
real reason such return can be compelled.
Noting that the impending strike would prejudice the
national interest as well as the welfare of some 350
workers and their families, the Ministry of Labor and The Court held that the return-to-work order should
Employment issued an order certifying the labor dispute benefit only those workers who complied and must be
to the NLRC and it enjoined the management from lacking entitled to be paid for work they have actually performed.
out its employees and the union from declaring a strike or Conversely, those workers who refused to obey said
similar concerted order and instead waged the restrained strike are not
action. The MOLE issued a return-to-work order and that entitled to be paid for work not done or to reinstatement
the union and the striking workers should return to work to the positions they have abandoned by their refusal to
immediately to resume operations. Meantime, the return thereto as ordered.
defendants were charged with staging an illegal strike,
barricading the gates of the ATC plant and preventing the
workers through intimidation, harassment and force from
reporting for work.

ISSUE: WON a return-to-work order may be validly


issued by the National Labor Relations Commission
pending determination of the legality of the strike?
B. EXCESSIVE FINE Two theories:

PEOPLE vs. DELA CRUZ 92 PHIL 906 (1953) 1. Prohibition applies to legislation only, and not to
the courts' decision imposing penalties within the limits of
FACTS: the statute

Having retailed a can of milk at ten centavos more than 2. Authorities are not lacking to the effect that the
the ceiling price, Pablo de la Cruz was sentenced, after fundamental prohibition likewise restricts the judge's
trial, in the court of first instance of Manila, to power and authority
imprisonment for 5 years, and to pay a fine of P5000 plus
costs. He was also barred from engaging in wholesale
and retail business for 5 years. Referring to the penalty provided in Republic Act No. 509,
under the first theory the section would violate the
Constitution, if the penalty is excessive under any and all
In this appeal he argues that the trial judge erred: (a) in circumstances, the minimum being entirely out of
not holding that the charge was fabricated; proportion to the kind of offenses prescribed. If it is not,
the imposition by the judge of a stiff penalty - but within
the limits of the section - will not be deemed
unconstitutional. The second theory would contrast the
(b) in imposing a punishment wholly disproportionate
penalty imposed by the court with the gravity of the
to the offense and therefore unconstitutional and (c) in not
particular crime or misdemeanor, and if notable disparity
invalidating Republic Act No. 509 in so far as it prescribed
results, it would apply the constitutional brake, even if the
excessive penalties.
statute would, under other circumstances, be not extreme
or oppressive.

509) The evidence shows that in the morning of


October 14, 1950, Eduardo Bernardo, Jr. went to the
The Court held that the penalty was not excessive
defendant's store in Sampaloc, Manila, and purchased
because milk, as a precious commodity, should be
from him a six-ounce tin of "Carnation" milk for thirty
protected. Taking into consideration the duration of the
centavos. As the purchase had been made for Ruperto
offense, the Court held that he may even have profited
Austria, who was not in good terms with Pablo de la Cruz
more. Hence, the penalty is not excessive because it was
the matter reached the City Fiscal's office and resulted in
not disproportionate to the offense committed.
this criminal prosecution, because Executive Order No.
331 (issued by authority of Republic Act No. 509 fixed 20
centavos as the maximum price for that kind of
PEOPLE vs. DACUYCUY 173 SCRA 901 (1989)
commodity.

FACTS:
ISSUE: WON the penalty was excessive?

Private respondents Celestino Matondo, Segundino


RULING: No. Caval and Cirilo Zanoria, public school officials of Leyte,
were charged before the municipal court for violation of
RA 4670. At the arraignment, they pleaded not guilty to
The Constitution directs that "Excessive fines shall not be the charge. Thereafter, they orally moved to quash the
imposed, nor cruel and unusual punishment inflicted." complaint for lack of jurisdiction over the offense allegedly
The prohibition of cruel and unusual punishments is due to the correctional nature of the penalty of
generally aimed at the form or character of the imprisonment prescribed for the offense; but it was
punishment rather than its severity in respect of duration denied.
or amount. To justify a court's declaration of conflict with
the Constitution, the prison term must be so
disproportionate to the offense committed as to shock the So they filed a motion for reconsideration of the denial
moral sense of all reasonable men as to what is right and order and alleged that Section 32 of RA 4670 is
proper under the circumstances. unconstitutional for: (1) It imposes a cruel and unusual
punishment, the term of imprisonment being unfixed and
may run to reclusion perpetua; and (2) It also constitutes
an undue delegation of legislative power, the duration of AGBANLOG vs. PEOPLE 220 SCRA 530 (1993)
the penalty of imprisonment being solely left to the
discretion of the court.
FACTS:

Feliciano Agbanlog was the Officer-In-Charge of the


ISSUE: WON the indefinite penalty of imprisonment Office of Municipal Treasurer of Aglipay, Quirino for the
constitutes a cruel and unusual punishment? period from March 24, 1986 to May 31, 1986. When
audited by COA Auditing Examiner of the Provincial
Auditor's Office of Cobarroguis, Quirino, on August 4,
1986 for the aforesaid period of his incumbency as Acting
RULING: No.
Municipal Treasurer, Feliciano Agbanlog was found short
in his cash and accounts in the sum of P21,940.70

It is contended that RA 4670 is unconstitutional on the


ground that the imposable but indefinite penalty of
A written demand to explain the shortage and to pay the
imprisonment provided therein constitutes a cruel and
amount thereof was neither answered nor acted upon by
unusual punishment, in defiance of the express mandate
the said accountable officer. Consequently, a Report was
of the Constitution. This contention is inaccurate and
made by Examining Auditors manifesting their findings
should be rejected.
and recommending the institution of administrative and/or
criminal charges against Acting Municipal Treasurer
Feliciano Agbanlog. At the outset, the Auditors found the
A punishment authorized by statute is not cruel or unusual accused Agbanlog short in the amount of P32,950.34
or disproportionate to the nature of the offense unless it is
a barbarous one unknown to the law or so wholly
disproportionate to the nature of the offense as to shock
Upon the finding that P11,009.64 of this amount was
the moral sense of the community.
chargeable to the account of former Municipal Treasurer,
the said amount of P11,009.64 was deducted from the
accountability of Feliciano Agbanlog.
However, the absence of designated limits in the court's
discretion to fix length of imprisonment constitutes an
undue delegation of legislative power.
As regards the disbursement voucher billed as a cash
advance for various expenses in the amount of
P12,504.49, this voucher was disallowed by the auditors
Section 32 of Republic Act No. 4670 provides for an because there was no appropriation for this
indeterminable period of imprisonment, with neither a disbursement. It is indicated in the voucher that the giving
minimum nor a maximum duration having been set by the out of this money was in the nature of a cash advance.
legislative authority. The courts are thus given wide The purpose for which the cash advance was given out
latitude of discretion to fix the term of imprisonment, was, however, not clearly indicated.
without even the benefit of any sufficient standard, such
that the duration thereof may range from one minute to
the life span of the accused. Irremissibly, this cannot be
Petitioner admits the shortage of the accountable funds
allowed. It vests in the courts a power and a duty
charged by the prosecution but claims that the
essentially legislative in nature and which, as applied to
prosecution failed to show that the shortage accrued
this case, does violence to the rules on separation of
during his short stint as acting treasurer. The Court found
powers as well as the non-delegability of legislative
petitioner guilty beyond reasonable doubt of Malversation
powers. On the foregoing considerations, the penalty of
of Public Funds and sentencing him to suffer "the
imprisonment provided in Section 32 thereof should be
indeterminate penalty of, from 11 years and 1 day of
declared unconstitutional.
Prision Mayor, as minimum to 16 years, 5 months and 11
days of Reclusion Temporal, as maximum.

Petitioner questions as oppressive and unconstitutional


the penalty imposed on him.

ISSUE: WON the penalty was excessive?


RULING: No. ISSUE: Is DeathPenalty is cruel and unusual
punishment?
Petitioner argues that considering the value of the peso in
1932 when the Revised Penal Code was enacted and the RULING: No.
value of the peso today, the penalty for malversation of
P21,000 should only be an imprisonment of one or two
years. The penalty is neither cruel, unjust nor excessive. In the
US case of Kemmler, it was held that punishments are
cruel when they involve torture or a lingering death. It
Assuming arguendo that inflation has in effect made more implies there something inhuman, barbarous, something
severe the penalty for malversing P21,000, the remedy more than the extinguishment of life. It is degrading if it
cannot come from this Court but from Congress. The involves public humiliation. The severity is not sufficient,
Court can intervene and strike down a penalty as cruel, but must be disproportionate to the crime committed.
degrading or inhuman only when it has become so Excessiveness is measured by
flagrantly oppressive and so wholly disproportionate to
the nature of the offense as to shock the moral senses.
7) seriousness of the crime, 2) policy of the
legislative,
Considering that malversation of public funds by a public
officer is a betrayal of the public trust, We are not
prepared to say that the penalty imposed on petitioner is (c) perversity of the accused.
so disproportionate to the crime committed as to shock
the moral sense.
The issue in Furman vs. Georgia is not so much the death
penalty itself, but the arbitrariness pervading the
C. CRUEL, DEGRADING AND INHUMAN PUNISHMENTS
procedures by which the death penalty was imposed by
the jury. It was nullified because the discretion in which
the statute vested in trial judges and sentencing juries
PEOPLE vs. ECHEGARAY was uncontrolled and without any parameters, guidelines,
267 SCRA 682 (1997)
or standards.

FACTS:
With regard to the case of Coker vs. Georgia, the SC held
Accused-apellant Leo Echegaray was charged and
that this case has no bearing on Philippine experience
convicted for the crime of raping his ten-year old
and culture. Such a premise is in fact an ennobling of the
daughter. The crime having been committed in April,
biblical notion of retributive justice of "an eye for an eye,
1994, during which time RA 7659, commonly known as
a tooth for a tooth". But, the forfeiture of life simply
the Death Penalty Law, was already in effect, accused-
because life was taken, never was a defining essence of
appellant was inevitably meted out the supreme penalty
the death penalty in the context of our legal history and
of death.
cultural experience; rather, the death penalty is imposed
in heinous crimes because the perpetrators thereof have
committed unforgivably execrable acts that have so
In appealing the conviction, it raised the constitutionality deeply dehumanized a person or criminal acts with
of the Death Penalty Law as being severe and excessive, severely destructive effects, and because they have so
cruel and unusual in violation of the constitution. He caused irreparable and substantial injury to both their
invokes the ruling in Furman vs. Georgia wherein the US victim and the society and a repetition of their acts would
Supreme Court categorically ruled that death penalty is pose actual threat to the safety of individuals and the
cruel and degrading. He also argues that death is an survival of government, they must be permanently
excessive and cruel punishment for a crime of rape prevented from doing so.
because there is no taking of life in rape. He invokes the
ruling in Coker vs. Georgia which said that while rape
deserves serious punishment, it should not involve the
RA 7659 already sufficiently defined what are heinous
taking of human life. In rape, life is not over for the victim.
crimes – crimes punished with death are those that are
Death penalty should only be imposed where the crime
grievous, odious, and hateful by reason of inherent
was murder.
viciousness, atrocity and perversity, those that are
repugnant and outrageous to common standards of
norms and decency and morality in a just, civilized and
ordered society. They also include crimes which are
despicable because life is callously taken, or the victim is ISSUE: WON said punishment is cruel and inhuman?
treated as an animal or dehumanized.

RULING: No.
PEOPLE vs. TONGKO
290 SCRA 595 (1998)

In People v. de la Cruz, the Court held that the prohibition


Accused Roberto Tongko was found guilty of estafa under of cruel and unusual punishments is generally aimed at
Article 315(2)(d) of the Revised Penal Code. He was the form or character of the punishment rather than its
sentenced to suffer 27 years of reclusion perpetua and to severity in respect of duration or amount. In People v.
indemnify Carmelita V. Santos by way of actual damages Estoista, the Court further held that it takes more than
in the sum of P100,000. merely being harsh, excessive, out of proportion, or
severe for a penalty to be obnoxious to the Constitution.
The fact that the punishment authorized by the statute is
Accordingly, the accused opened savings and current severe does not make it cruel and unusual. Expressed in
accounts with Amanah Bank. In the morning of August 20, other terms, it has been held that to come under the ban,
1993, Marites Bo-ot brought the accused to the office of the punishment must be "flagrantly and plainly
Carmelita V. Santos to borrow money. The accused oppressive," "Wholly disproportionate to the nature of the
asked for P50,000 to be paid not later than December offense as to shock the moral sense of the community."
1993 and assured Santos that his receivable would come
in by November 1993. He persuaded Santos to give the
loan by issuing 5 checks, each in the sum of P10,000, The legislature was not thoughtless in imposing severe
postdated December 20, 1993 and by signing a penalties for violation of par. 2(d) of Article 315 of the
promissory note. Revised Penal Code. The history of the law will show that
the severe penalties were intended to stop the upsurge of
swindling by issuance of bouncing checks.
The promissory note was co-signed by Bo-ot. In the
afternoon of the same date, the accused returned to
Santos and borrowed an additional P50,000. Again, he ECHEGARAY vs. SECRETARY
issued five (5) checks, each worth P10,000 postdated 297 SCRA 754 (1998)
December 20, 1993. He also signed a promissory note
together with Bo-ot. On September 14, 1993, Amanah FACTS:
Bank closed accused's current account for lack of funds.
On October 1993, accused himself requested for the The Court affirmed the conviction of petitioner Leo
closing of his savings account. Echegaray for the crime of rape of the 10 year-old
daughter of his common-law spouse and the imposition
upon him of the death penalty for the said crime. He
Estafa has the following elements: (1) postdating or assailed the constitutionality of RA 7659 (the death
issuance of a check in payment of an obligation penalty law) and the imposition of the death penalty for
contracted at the time the check was issued; the crime of rape.

510) lack of sufficiency of funds to cover the check; In the meantime, Congress had seen it fit to change the
and mode of execution of the death penalty from electrocution
to lethal injection, and passed RA.

511) damage to the payee thereof.


8177, AN ACT DESIGNATING DEATH BY LETHAL
INJECTION AS THE METHOD OF CARRYING OUT
CAPITAL PUNISHMENT, AMENDING FOR THE
Accused contends that the penalty of twenty seven (27) PURPOSE ARTICLE 81 OF THE REVISED PENAL
years of reclusion perpetua is too harsh and out of CODE, AS AMENDED BY SECTION 24 OF REPUBLIC
proportion to the crime he committed. He submits that his ACT NO. 7659.
sentence violates section 19(1) Article

Echegaray filed a Petition for Prohibition, Injunction


3. of the Constitution which prohibits the infliction of and/or Temporary Restraining Order to enjoin
cruel, degrading or inhuman punishment. respondents Secretary of Justice and Director of the
Bureau of Prisons from carrying out the execution by oft-cited case of Harden v. Director of Prisons, this Court
lethal injection of petitioner under R.A. 8177 and its held that punishments are cruel when they involve torture
implementing rules as these are unconstitutional and void or a lingering death; but the punishment of death is not
for being: (a) cruel, degrading and inhuman punishment cruel, within the meaning of that word as used in the
per se as well as by reason of its being (b) arbitrary, constitution. It implies there something inhuman and
unreasonable and a violation of due process, barbarous, something more than the mere
extinguishment of life.

(c) a violation of the Philippines' obligations under


international covenants, (d) an undue delegation of Petitioner further contends that the infliction of "wanton
legislative power by Congress, (e) an unlawful exercise pain" in case of possible complications in the intravenous
by respondent Secretary of the power to legislate, and injection, considering that respondent Director is an
untrained in administering the lethal injection. Such
supposition is highly unsubstantiated. Any infliction of
(f) an unlawful delegation of delegated powers by pain in lethal injection is merely incidental in carrying out
the Secretary of Justice to respondent Director. the execution of death penalty and does not fall within the
constitutional proscription against cruel, degrading and
inhuman punishment.
The Office of Solicitor General stated that (1) this Court
has already upheld the constitutionality of the Death
LIM vs. PEOPLE
Penalty Law, and has repeatedly declared that the death
390 SCRA 194 (2002)
penalty is not cruel, unjust, excessive or unusual FACTS:
punishment; (2) execution by lethal injection, as
authorized under R.A. 8177, is constitutional, lethal Spouses Lim issued to Wilson Cham two postdated
injection being the most modern, more humane, more checks in the amount of P365,750 and P429,000. The first
economical, safer and easier to apply than electrocution check was dishonored upon presentment for having been
or the gas chamber; (3) the International Covenant on drawn against insufficient funds while the other check was
Civil and Political Rights does not expressly or impliedly not presented for payment upon request of Spouses Lim
prohibit the imposition of the death penalty; (4) R.A.8177 who promised to replace the dishonored check. When
properly delegated legislative power to respondent they reneged on their promise to cover the amount of the
Director. first check, Cham filed a complaint affidavit before the
Office of the City Prosecutor of Quezon City charging
them with the crime of estafa under Art. 315, par. 2 (d) of
ISSUE: WON said punishment is cruel and inhuman? the Revised Penal Code, as amended by P.D. 818. A
warrant of arrest was subsequently issued.

RULING: No.
Spouses Lim filed an “Urgent Motion to Quash
Information and Warrant of Arrest” which was denied by
the trial court. Their motion for bail was also denied. They
In lethal injection, the condemned inmate is strapped on
filed a petition for certiorari imputing grave abuse of
a hospital gurney and wheeled into the execution room. A
discretion, arguing that P.D. 818 violated the
trained technician inserts a needle into a vein in the
constitutional provisions on due process, bail and
inmate's arm and begins an intravenous flow of saline
imposition of cruel, degrading or inhuman punishment.
solution. At the warden's signal, a lethal combination of
drugs is injected into the intravenous line. The deadly
concoction typically includes three drugs: (1) a nonlethal
dose of sodium thiopenthotal, a sleep inducing They contend that, inasmuch as the amount of the subject
barbiturate; (2) lethal doses of pancuronium bromide, a check is P365,750, they can be penalized with reclusion
drug that paralyzes the muscles; and (3) potassium perpetua or 30 years of imprisonment. This penalty is too
chloride, which stops the heart within seconds. The first severe and disproportionate to the crime they committed
two drugs are commonly used during surgery to put the and infringes on the express mandate of Article III,
patient to sleep and relax muscles; the third is used in Section 19 of the Constitution which prohibits the infliction
heart bypass surgery. of cruel, degrading and inhuman punishment. They argue
that while PD 818 increased the imposable penalties for
estafa committed under Article 315, par. 2 (d), RPC, it did
not increase the amounts corresponding to the said new
It is well-settled in jurisprudence that the death penalty per
penalties.
se is not a cruel, degrading or inhuman punishment. In the
RULING: No.

ISSUE: WON said punishment is cruel and inhuman?

It is contended that RA 4670 is unconstitutional on the


ground that the imposable but indefinite penalty of
RULING: No. imprisonment provided therein constitutes a cruel and
unusual punishment, in defiance of the express mandate
of the Constitution. This contention is inaccurate and
P.D. 818, a decree increasing the penalty for estafa by should be rejected.
means of bouncing checks, is constitutional. The increase
in the penalty, far from being cruel and degrading, was
motivated by a laudable purpose, namely, to effectuate A punishment authorized by statute is not cruel or unusual
the repression of an evil that undermines the country’s or disproportionate to the nature of the offense unless it is
commercial and economic growth, and to serve as a a barbarous one unknown to the law or so wholly
necessary precaution to deter people from issuing disproportionate to the nature of the offense as to shock
bouncing checks. The fact that PD No. 818 did not the moral sense of the community.
increase the amounts corresponding to the new penalties
only proves that the amount is immaterial and
inconsequential. What the law sought to avert was the
proliferation of estafa cases committed by means of However, the absence of designated limits in the court's
bouncing checks. Taking into account the salutary discretion to fix length of imprisonment constitutes an
purpose for which said law was decreed, the court undue delegation of legislative power.
concluded that PD 818 does not violate Section 19 of
Article III of the Constitution.
Section 32 of Republic Act No. 4670 provides for an
indeterminable period of imprisonment, with neither a
D. INDEFINITE IMPRISONMENT minimum nor a maximum duration having been set by the
legislative authority. The courts are thus given wide
latitude of discretion to fix the term of imprisonment,
PEOPLE vs. DACUYCUY 173 SCRA 901 (1989)
without even the benefit of any sufficient standard, such
that the duration thereof may range from one minute to
the life span of the accused. Irremissibly, this cannot be
FACTS: allowed. It vests in the courts a power and a duty
essentially legislative in nature and which, as applied to
this case, does violence to the rules on separation of
Private respondents Celestino Matondo, Segundino powers as well as the non-delegability of legislative
Caval and Cirilo Zanoria, public school officials of Leyte, powers. On the foregoing considerations, the penalty of
were charged before the municipal court for violation of imprisonment provided in Section 32 thereof should be
RA 4670. At the arraignment, they pleaded not guilty to declared unconstitutional.
the charge. Thereafter, they orally moved to quash the
Recall the facts. Section 32 of RA 4670 provides for an
complaint for lack of jurisdiction over the offense allegedly
indeterminable period of imprisonment, with neither a
due to the correctional nature of the penalty of
minimum nor a maximum duration having been set by the
imprisonment prescribed for the offense; but it was
legislative authority. The courts are thus given a wide
denied.
latitude of discretion to fix the term of imprisonment
without even the benefit of any sufficient standard, such
that the duration thereof may range, in the words of
So they filed a motion for reconsideration of the denial respondent judge, from one minute to the lifespan f the
order and alleged that Section 32 of RA 4670 is accused. It cannot be allowed.
unconstitutional for: (1) It imposes a cruel and unusual
punishment, the term of imprisonment being unfixed and It vests in the courts a power and duty essentially
may run to reclusion perpetua; and (2) It also constitutes legislative in nature and which, as applied in this case,
an undue delegation of legislative power, the duration of does violence to the rules on separation of powers as well
the penalty of imprisonment being solely left to the as the non-delegability of legislative powers. This time,
discretion of the court. the presumption of constitutionality has to yield. By virtue
of separability clause in Section 34 of RA 4670, the
penalty of imprisonment provided in Section 32 should be
declared unconstitutional.
ISSUE: WON the indefinite penalty of imprisonment
constitutes a cruel and unusual punishment?
THE PROTECTION AGAINST DOUBLE JEOPARDY RULING: No.

A. TWO SITUATIONS COMTEPLATED


The basic premise of the petitioner's position is that the
constitutional protection against double jeopardy is
protection against a second or later jeopardy of conviction
PEOPLE vs. RELOVA
for the same offense. The petitioner stresses that the first
148 SCRA 292 (1987)
information filed before the City Court of Batangas City
was one for unlawful or unauthorized installation of
FACTS: electrical wiring and devices, acts which were in violation
of an ordinance of the City Government of Batangas. Only
On February 1975, members of the Batangas City Police two elements are needed to constitute an offense under
together with personnel of the Batangas Electric Light this City Ordinance: (1) that there was such an
System, equipped with a search warrant issued by a city installation; and
judge of Batangas City, searched and examined the
premises of the Opulencia Carpena Ice Plant and Cold
Storage owned and operated by the private respondent
Manuel Opulencia. The police discovered that electric (2) no authority therefor had been obtained from the
wiring, devices and contraptions had been installed, Superintendent of the Batangas City Electrical System.
without the necessary authority from the city government,
and architecturally concealed inside the walls of the
building owned by the private respondent. These electric The offense under the City Ordinance is the installing of
devices and contraptions were, in the allegation of the electric wiring and devices without authority from the
petitioner designed purposely to lower or decrease the proper officials of the city government. To constitute an
readings of electric current consumption in the electric offense under the city ordinance, it is not essential to
meter of the said electric [ice and cold storage] plant. establish any mens rea or intent to appropriate and steal
on the part of offender. In contrast, the offense of theft
under Article 308 of RPC has quite different essential
On November 1975, an Assistant City Fiscal of Batangas elements. The petitioner alleged that theft of electricity
City filed before the City Court of Batangas City an can be effected even without illegal or unauthorized
information against Manuel Opulencia for violation of installations of any kind.
Ordinance No. 1, Series of 1974, Batangas City. A The petitioner concludes that the unauthorized installation
violation of this ordinance was, under its terms, punished by the ordinance of Batangas City is not the
punishable by a fine "ranging from P5 to P50 or same as theft of electricity under RPC; that the second
imprisonment, which shall not exceed thirty (30) days, or offense is not an attempt to commit the first or a frustration
both, at the discretion of the court." thereof and that the second offense is not necessarily
included in the offense charged in the first information.

14 days later, the Acting City Fiscal of Batangas City filed


before the Court of First Instance of Batangas, Branch II The above arguments are correct. This is clear both from
another information against Manuel Opulencia for theft of express terms of constitutional provision involved - which
electric power under Article 308 in relation to Article 309, reads as follows: "No person shall be twice put in jeopardy
paragraph (1), of the Revised Penal Code. That without of punishment for the same offense. If an act is punished
knowledge and consent of the Batangas Electric Light by a law and an ordinance, conviction or acquittal under
System, accused feloniously take, steal and appropriate either shall constitute a bar to another prosecution for the
electric current valued of P41,062. same act."

Before he could be arraigned thereon, Manuel Opulencia Our Bill of Rights deals with 2 kinds of double jeopardy.
filed a Motion to Quash alleging that he had been The first sentence of clause 20, section 1, Article III,
previously acquitted of offense charged in second ordains that "no person shall be twice put in jeopardy of
information and that the filing thereof was violative of his punishment for the same offense." The second sentence
constitutional right against double jeopardy. of said clause provides that "if an act is punishable by a
law and an ordinance, conviction or acquittal under either
shall constitute a bar to another prosecution for the same
ISSUE: WON there was double jeopardy? act."
Thus, the first sentence prohibits double jeopardy of PEOPLE vs. CITY OF COURT
punishment for the same offense, whereas the second 154 SCRA 195 (1987)
contemplates double jeopardy of punishment for the
same act. Under the first sentence, one may be twice put FACTS:
in jeopardy of punishment of the same act, provided that
he is charged with different offenses, or the offense Agapito Gonzales and Roberto Pangilinan was accused
charged in one case is not included in, or does not of violating Section 7 of RA 3060 (An Act Creating the
include, the crime charged in the other case. The second Board of Censors for Motion Pictures) in relation to Article
sentence applies, even if the offenses charged are not the 201 (Immoral doctrines , obscene publications and
same, owing to the fact that one constitutes a violation of exhibitions and indecent shows) of the RPC. On April 07,
an ordinance and the other a violation of a statute. If the 1972, two information were filed against the accused. The
two charges are based on one and the same act first one, filed for violation of RA 3060, alleged that the
conviction or acquittal under either the law or the accused, without having previously submitted to the
ordinance shall bar a prosecution under the other. Board of censors for Motion Pictures for preview and
Incidentally, such conviction or acquittal is not examination, exhibited a motion film in a public place.
indispensable to sustain the plea of double jeopardy of
punishment for the same offense. So long as jeopardy
has attached under one of the information charging said The second one, filed for violation of Article 201, alleged
offense, the defense may be availed of in the other case that the accused exhibited motion pictures “depicting and
involving the same offense, even if there has been neither showing scenes of totally naked female and male persons
conviction nor acquittal in either case. with exposed private parts doing the sex act in various
lewd and obvious positions, among other similarly and
equally obscene and morally offensive scenes, in a place
In the instant case, the relevant acts took place within the open to public view, to wit: at Rm 309, DeLeon Bldg, Raon
same time frame: from November 1974 to February 1975. St. corner Rizal Avenue.”
During this period, the accused Manuel Opulencia
installed or permitted the installation of electrical wiring
and devices in his ice plant without obtaining the Accused Gonzales moved to quash the information in the
necessary permit or authorization from the municipal criminal case for ground of double jeopardy as the case
authorities. The accused conceded that he effected or pending against him for violation of RA 3060, allegedly
permitted such unauthorized installation for the very contains the same allegations in the criminal case.
purpose of reducing his electric power bill. This corrupt Respondent City Court (City Court of Manila, Branch 6)
intent was thus present from the very moment that such dismissed the criminal case on the basis that the
unauthorized installation began. The immediate physical allegations in the two information are identical and the
effect of the unauthorized installation was the inward flow plea entered in one case by the accused herein can be
of electric current into Opulencia's ice plant without the reasonably seen as exposing him to double jeopardy in
corresponding recording thereof in his electric meter. In the other case.Petitioner contends that the accused could
other words, the "taking" of electric current was integral not invoke the constitutional guarantee against double
with the unauthorized installation of electric wiring and jeopardy, when there had been no conviction, acquittal,
devices. dismissal or termination of criminal proceedings in
another case for the same offense.

While the rule against double jeopardy prohibits


prosecution for the same offense, it seems elementary ISSUE: WON there was double jeopardy?
that an accused should be shielded against being
prosecuted for several offenses made out from a single
act. Otherwise, an unlawful act or omission may give use
to several prosecutions depending upon the ability of the
prosecuting officer to imagine or concoct as many
offenses as can be justified by said act or omission by
simply adding or subtracting essential elements. It
remains to point out that the dismissal by the Batangas
City Court of the information for violation of the Batangas
City Ordinance upon the ground that such offense had
already prescribed, amounts to an acquittal of the
accused of that offense.
RULING: No. B. RULES OF COURT PROVISIONS

It is a settled rule that to raise the defense of double MELO vs. PEOPLE
jeopardy, three requisites must be present: (1) a first FACTS:
jeopardy must have attached prior to the second; Petitioner Conrado Melo was charged with frustrated
homicide, for having allegedly inflicted upon Benjamin
Obillo, with a kitchen knife and with intent to kill, several
(2) the first jeopardy must have been validly serious wounds on different parts of the body, requiring
terminated; and (3) the second jeopardy must be for the medical attendance for a period of more than 30 days.
same offense, or the second offense includes or is The accused pleaded not guilty to the offense charged.
necessarily included in the offense charged in the first However, Obillo died and so an amended information was
information, or is an attempt to commit the same or a filed charging the accused with consummated homicide.
frustration thereof. All these requisites are absent. The accused filed a motion to quash the amended
information alleging double jeopardy but it was denied by
the respondent court; hence, the instant petition for
The 2 informations with which the accused was charged, prohibition to enjoin the respondent court from further
do not make out only one offense, contrary to private entertaining the amended information.
respondent's allegations. The offense defined in section 7
of Rep. Act No. 3060 punishing the exhibition of motion
pictures not duly passed by the Board of Censors for ISSUE: WON there is double jeopardy?
Motion Pictures does not include or is not included in the
offense defined in Article 201 (3) of the Revised Penal
Code punishing the exhibition of indecent and immoral RULING: No.
motion pictures.

Rule 106, section 13, 2d paragraph, provides:


The two (2) offenses do not constitute a jeopardy to each
other. A scrutiny of the two (2) laws involved would show
that the two (2) offenses are different and distinct from
"If it appears at any time before judgment that a mistake
each other. It is evident that the elements of the two (2)
has been made in charging the proper offense, the court
offenses are different. The gravamen of the offense
may dismiss the original complaint or information and
defined in Rep. Act No. 3060 is the public exhibition of any
order the filing of a new one charging the proper offense,
motion picture which has not been previously passed by
provided the defendant would not be placed thereby in
the Board of Censors for Motion Pictures. The motion
double jeopardy, and may also require the witnesses to
picture may not be indecent or immoral but if it has not
give bail for their appearance at the trial."
been previously approved by the Board, its public
showing constitutes a criminal offense. On the other hand,
the offense punished in Article 201 (3) of the Revised
Penal Code is the public showing of indecent or immoral Under this provision, it was proper for the court to dismiss
plays, scenes, acts, or shows, not just motion pictures. the first information and order the filing of a new one for
reason that the proper offense was not charged in the
former and the latter did not place the accused in a
second jeopardy for the same or identical offense.
The crime punished in Rep. Act No. 3060 is a malum
prohibitum in which criminal intent need not be proved
because it is presumed, while the offense punished in
Article 201 (3) of the Revised Penal Code is malum in se, The rule of "double jeopardy" meant that when a person
in which criminal intent is an indispensable ingredient. is charged with an offense and the case is terminated
Considering these differences in elements and nature, either by acquittal or conviction or in any other manner
there is no Identity of the offenses here involved for which without the consent of the accused, the latter cannot
legal jeopardy in one may be invoked in the other. again be charged with the same or identical offense.
Evidence required to prove one offense is not the same There is identity between the two offenses when the
evidence required to prove the other. The defense of evidence to support a conviction for one offense would be
double jeopardy cannot prosper. sufficient to warrant a conviction for the other. This so-
called "same-evidence test" which was found to be vague
and deficient, was restated by the Rules of Court in a
clearer and more accurate form.
RULING:

Under said Rules there is identity between two offenses


not only when the second offense is exactly the same as
the first, but also when the second offense is an attempt In Melo vs. People, this Court held that "where after the
to commit the first or a frustration thereof, or when it first prosecution a new fact supervenes for which the
necessarily includes or is necessarily included in the defendant is responsible, which changes the character of
offense charged in the first information. In this connection, the offense and, together with the facts existing at a time,
an offense may be said to necessarily include another constitutes a new and distinct offense, the accused
when some of the essential ingredients of the former as cannot be said to be in second jeopardy if indicted for the
alleged in the information constitute the latter. And vice- second offense."
versa, an offense may be said to be necessarily included
in another when all the ingredients of the former constitute
a part of the elements constituting the latter (Rule 116, However, the trial court held that the doctrine of Melo vs.
sec. 5.) People does not apply in the case at bar in view of this
Court's ruling in People vs. Buan, that Article 365 of the
Penal Code punishes the negligent state of mind and not
In other words, one who has been charged with an the resulting injury. The trial court concluded that once
offense cannot be again charged with the same or prosecuted for and convicted of negligence, the accused
identical offense though the latter be lesser or greater cannot again be prosecuted for the same negligence
than the former. although for a different resulting injury.

This rule of identity does not apply, however, when the In the case at bar, the incident occurred on October 17,
second offense was not in existence at the time of the first 1971. The following day, October 18, an information for
prosecution, for the simple reason that in such case there serious physical injuries thru reckless imprudence was
is no possibility for the accused, during the first filed against private respondent driver of the truck. On the
prosecution, to be convicted for an offense that was then same day, the victim Diolito de la Cruz died.
inexistent. Thus, where the accused was charged with
physical injuries and after conviction the injured person
dies, the charge for homicide against the same accused Well-settled is the rule that one who has been charged
does not put him twice in jeopardy. In this case, there was with an offense cannot be charge again with the same or
no double jeopardy because the death was a supervening identical offense though the latter be lesser or greater
event. than the former. However, as held in the case of Melo vs.
People, supra, the rule of identity does not apply when the
second offense was not in existence at the time of the first
PEOPLE vs. CITY OF COURT prosecution, for the reason that in such case there is no
121 SCRA 637 (1983) possibility for the accused, during the first prosecution, to
be convicted for an offense that was then inexistent.
FACTS:
"Thus, where the accused was charged with physical
This case involves the dismissing of information for injuries and after conviction, the injured person dies, the
homicide thru reckless imprudence filed against private charge for homicide against the same accused does not
respondent, Francisco Gapay y Mallares, on the ground put him twice in jeopardy." Stated differently where after
of double jeopardy. Respondent court held that the private the first prosecution a new fact supervenes for which the
respondent having been previously tried and convicted of defendant is responsible, which changes the character of
serious physical injuries thru reckless imprudence for the the offense and, together with the facts existing at the
resulting death of the victim would place the accused in time, constitutes a new and distinct offense, the accused
double jeopardy. cannot be said to be in second jeopardy if indicted for the
new offense.

As stated above, the victim Diolito dela Cruz died on the


ISSUE: WON a person who has been prosecuted for
day the information was filed, and the accused was
serious physical injuries thru reckless imprudence and
arraigned two (2) days after, or on October 20, 1972.
convicted thereof may be prosecuted subsequently for
When the information for homicide thru reckless
homicide thru reckless imprudence if the offended party
imprudence was, therefore, filed on October 24, 1972, the
dies as a result of the same injuries he had suffered?
accused-private respondent was already in jeopardy.
ACCORDINGLY, the order of dismissal of the lower court
is affirmed.
PEOPLE vs. YORAC IVLER vs. MODESTO – SAN PEDRO
635 SCRA 191 (2010)

FACTS:
FACTS:
Rodrigo Yorac was prosecuted for frustrated murder
arising allegedly from having assaulted with a piece of Following a vehicular collision in August 2004, Jason Ivler
wood the offended party (Lam Hock), for which he had was charged before the MeTC, with two separate
been previously tried and sentenced for slight physical offenses: (1) Reckless Imprudence Resulting in Slight
injuries, his plea being one of guilt. He started serving his Physical Injuries (Criminal Case No. 82367) for injuries
sentence forthwith. The later information for frustrated sustained by Evangeline L. Ponce and (2) Reckless
murder was based on a second medical certificate after Imprudence Resulting in Homicide and Damage to
the lapse of one week from the former previously given by Property (Criminal Case No. 82366) for the death of
the same physician who, apparently, was much more Ponce’s husband Nestor C. Ponce and damage to the
thorough the second time, to the effect that the victim did spouses Ponce’s vehicle.
suffer a greater injury than was at first ascertained.

On 7 September 2004, Ivler pleaded guilty to the charge


A motion to quash was filed by the accused on the ground and was meted out the penalty of public censure. Invoking
that, having been previously convicted of slight physical this conviction, Ivler moved to quash the Information for
injuries and having already served the penalty imposed placing him in jeopardy of second punishment for the
on him for the very same offense, the prosecution for same offense of reckless imprudence. However, it was
frustrated murder arising out of the same act committed denied.
against the same offended party, the crime of slight
physical injuries necessarily being included in that of
frustrated murder, he would be placed in second jeopardy Ivler submits that the two cases concern the same offense
if indicted for new offense. of reckless imprudence. He argues that his constitutional
right not to be placed twice in jeopardy of punishment for
the same offense bars his prosecution in Criminal Case
ISSUE: WON the defendant, who had already been No. 82366, having been previously convicted in Criminal
convicted of slight physical injuries and had served Case No. 82367 for the same offense of reckless
sentence therefor, may be prosecuted anew for frustrated imprudence charged in Criminal Case No. 82366. He
murder for the same act committed against the same submits that the multiple consequences of such crime are
person? material only to determine his penalty.

RULING: No. There was double jeopardy. A defendant in Respondent Ponce calls the Court’s attention to
a criminal case should therefore be adjudged either guilty jurisprudence holding that light offenses (e.g. slight
or not guilty and thereafter left alone in peace, in the latter physical injuries) cannot be complexed under Article 48 of
case the State being precluded from taking an appeal. If the Revised Penal Code with grave or less grave felonies
the X-ray examination discloses the existence of a (e.g. homicide). Hence, the prosecution was obliged to
fracture on January 17, 1957, that fracture must have separate the charge in Criminal Case No. 82366 for the
existed when the first examination was made on slight physical injuries from Criminal Case No. 82367 for
December 10, 1956. There is no new or supervening fact the homicide and damage to property.
that could be said to have developed or arisen since the
filing of original action. The wound causing the delay in
healing was already in existence at the time of the first ISSUE: Is there double jeopardy?
examination, but said delay was caused by the very
superficial examination then made. No supervening fact
had occurred which justifies the application of the rule in RULING: Prior Conviction or Acquittal of Reckless
Melo vs. People, for which reason we are constrained to
Imprudence Bars Subsequent Prosecution for the Same
apply the general rule of double jeopardy.
Quasi-Offense

The doctrine that reckless imprudence under Article 365


is a single quasi-offense by itself and not merely a means
to commit other crimes such that conviction or acquittal of
such quasi-offense bars subsequent prosecution for the
same quasi-offense, regardless of its various resulting
acts, undergirded this Court’s unbroken chain of YSIDORO vs. LEONARDO–DECASTRO
jurisprudence on double jeopardy as applied to Article 665 SCRA 89 (2012)
365. These cases uniformly barred the second
prosecutions as constitutionally impermissible under the FACTS:
Double Jeopardy Clause.
Ysidoro, as Municipal Mayor of Leyte, Leyte, was charged
before the Sandiganbayan for failure to give to Nierna S.
For the essence of the quasi offense of criminal Doller, Municipal Social Welfare and Development Officer
negligence under article 365 of the Revised Penal Code (MSWDO) of Leyte without legal basis her RATA for the
lies in the execution of an imprudent or negligent act that, months of August, September, October, November and
if intentionally done, would be punishable as a felony. The December, all in the year 2001 amounting to P22,125 and
law penalizes thus the negligent or careless act, not the her Productivity Pay in the year 2000 amounting to
result thereof. The gravity of the consequence is only P2000. Ysidoro filed an omnibus motion to quash the
taken into account to determine the penalty, it does not information and for judicial determination of probable
qualify the substance of the offense. And, as the careless cause but it was denied by the Sandiganbayan.
act is single, whether the injurious result should affect one
person or several persons, the offense (criminal
negligence) remains one and the same, and cannot be The Sandiganbayan preventively suspended Ysidoro for
split into different crimes and prosecutions. ninety (90) days in accordance with Section 13 of R.A. No.
3019, which states that “Any incumbent public officer
against whom any criminal prosecution under a valid
Reckless Imprudence is a Single Crime, its information under this Act or under Title 7, Book II of the
Consequences on Persons and Property are Material Revised Penal Code or for any offense involving fraud
Only to Determine the Penalty. The two charges against upon government or public funds or property whether as
Ivler, arising from the same facts, were prosecuted under a simple or as complex offense and in whatever stage of
the same provision of RPC, as amended, namely, Article execution and mode of participation, is pending in court,
365 defining and penalizing quasi-offenses. shall be suspended from office.”

The proposition (inferred from Art. 3 of the Revised Penal ISSUE: Is there double jeopardy?
Code) that “reckless imprudence” is not a crime in itself
but simply a way of committing it and merely determines
a lower degree of criminal liability is too broad to deserve RULING: No.
unqualified assent.

The Constitution has expressly adopted the double


Article 365 of RPC fixes the penalty for reckless jeopardy policy and thus bars multiple criminal trials,
imprudence at arresto mayor maximum, to prision thereby conclusively presuming that a second trial would
correccional [medium], if the willful act would constitute a be unfair if the innocence of the accused has been
grave felony, notwithstanding that the penalty for the latter confirmed by a previous final judgment. Further
could range all the way from prision mayor to death, prosecution via an appeal from a judgment of acquittal is
according to the case. It can be seen that the actual likewise barred because the government has already
penalty for criminal negligence bears no relation to the been afforded a complete opportunity to prove the
individual willful crime, but is set in relation to a whole criminal defendant’s culpability.
class, or series, of crime.

However, the rule against double jeopardy cannot be


properly invoked in a Rule 65 petition, predicated on two
(2) exceptional grounds, namely: in a judgment of
acquittal rendered with grave abuse of discretion by the
court; and where the prosecution had been deprived of
due process.

There was no double jeopardy because it was two


different offenses.
BAUTISTA vs. CUNETA-PANGILINAN RULING: No.
684SCRA521 (2012)
There is no liberl because Sharon was not actually named
in the article. It was in fact a blind item.
FACTS:

Two informations were filed against against Pete G.


Ampoloquio, Jr. (Ampoloquio), and petitioners Bautista Under Section 23,29 Rule 119 of the Rules of Court on
and Alcantara, for the crime of libel, committed by Demurrer to Evidence, after the prosecution terminates
publishing defamatory articles against respondent Sharon the presentation of evidence and rests its case, the trial
Cuneta-Pangilinan in the tabloid Bandera. It says court may dismiss the case on the ground of insufficiency
of evidence upon the filing of a Demurrer to Evidence by
the accused with or without leave of court. If the accused
MAGTIGIL KA, SHARON! Sharon Cuneta, the mega-taba files a Demurrer to Evidence with prior leave of court and
singer-actress, I’d like to believe, is really brain-dead. the same is denied, he may adduce evidence in his
Magsalita ka, Cuneta, at sabihin mong hindi ito totoo. Dios defense. However, if the Demurrer to Evidence is filed by
mio perdon, what she gets to see are those purportedly the accused without prior leave of court and the same is
biting commentaries about her katabaan and kaplastikan denied, he waives his right to present evidence and
but she has simply refused to acknowledge the good submits the case for judgment on the basis of the
reviews we’ve done on her. evidence for the prosecution.

Respondent’s undated Complaint-Affidavit alleged that Corollarily, after the prosecution rests its case, and the
Bautista and Alcantara were Editor and Associate Editor, accused files a Demurrer to Evidence, the trial court is
respectively, of the publication Bandera, and their co- required to evaluate whether the evidence presented by
accused, Ampoloquio, was the author of the alleged the prosecution is sufficient enough to warrant the
libelous articles which were published therein, and subject conviction of the accused beyond reasonable doubt. If the
of the two informations. trial court finds that the prosecution evidence is not
sufficient and grants the accused's Demurrer to Evidence,
the ruling is an adjudication on the merits of the case
which is tantamount to an acquittal and may no longer be
Petitioners allege that the Order of the RTC, dated April appealed. Any further prosecution of the accused after an
25, 2008, granting the Demurrer to Evidence was acquittal would, thus, violate the constitutional
tantamount to an acquittal. As such, the prosecution can proscription on double jeopardy.
no longer interpose an appeal to the CA, as it would place
them in double jeopardy. Petitioners contend that
respondent's petition for certiorari with the CA should not
have prospered, because the allegations therein, in effect, Nevertheless, petitioners could no longer be held liable in
assailed the trial court's judgment, not its jurisdiction. In view of the procedural infirmity that the petition for
other words, petitioners posit that the said Order was in certiorari was not undertaken by the OSG, but instead by
the nature of an error of judgment rendered, which was respondent in her personal capacity. Although the
not correctible by a petition for certiorari with the CA. conclusion of the trial court may be wrong, to reverse and
set aside the Order granting the demurrer to evidence
would violate petitioners’ constitutionally-enshrined right
against double jeopardy. Had it not been for this
Respondent counters that petitioners failed to show procedural defect, the Court could have seriously
special and important reasons to justify their invocation of considered the arguments advanced by the respondent in
the Court's power to review under Rule 45 of the Rules of seeking the reversal of the Order of the RTC.
Court. She avers that the acquittal of petitioners does not
preclude their further prosecution if the judgment The granting of a demurrer to evidence should, therefore,
acquitting them is void for lack of jurisdiction. Further, she be exercised with caution, taking into consideration not
points out that contrary to petitioners’ contention, the only the rights of the accused, but also the right of the
principle of double jeopardy does not attach in cases private offended party to be vindicated of the wrongdoing
where the court's judgment acquitting the accused or done against him, for if it is granted, the accused is
dismissing the case is void. acquitted and the private complainant is generally left with
no more remedy. In such instances, although the decision
ISSUE: Is there double jeopardy?
of the court may be wrong, the accused can invoke his
right against double jeopardy.
THE PRIVILEGE OF THE WRIT OF HABEAS CORPUS IN RE GONZALES

A. FUNCTIONS OF THE WRIT FACTS:

In line with their participation in the “Oakwood Mutiny” that


led to Pres.Gloria Macapagal Arroyo’s issuance of
VILLACENCIO v. LUKBAN Proclamation No. 427 declaring the country to be under a
"state of rebellion” and General Order No. 4 directing the
FACTS: AFP and the PNP to carry out all reasonable measures,
giving due regard to constitutional rights, to suppress and
Justo Lukban, who was then the Mayor of the City of quell the "rebellion.", petitioners were taken into custody
Manila, ordered the deportation of 170 prostitutes to by their Service Commander. Gonzales and Mesa were
Davao. His reason for doing so was to preserve the not charged before a court martial with violation of the
morals of the people of Manila. He claimed that the Articles of War. They were, however, among the soldiers
prostitutes were sent to Davao, purportedly, to work for an charged before Branch 61 of the Regional Trial Court
haciendero Feliciano Ynigo. The prostitutes were (RTC) of Makati City, with the crime of Coup D’ etat as
confined in houses from October 16 to 18 of that year defined under Article 134-A of the Revised Penal Code.
before being boarded, at the dead of night, in two boats They were consequently detained in Fort Bonifacio under
bound for Davao. The women were under the assumption the custody of the Philippine Marines. A petition for bail
that they were being transported to another police station was filed by the accused soldiers which the RTC
while Ynigo, the haciendero from Davao, had no idea that subsequently granted. Despite of the order and the
the women being sent to work for him were actually service thereof, petitioner was not released. As a
prostitutes. response, the People of the Philippines moved for partial
reconsideration of the order granting bail. With the denial
The families of the prostitutes came forward to file
of the Motion for Partial Reconsideration, the People filed
charges against Lukban, Anton Hohmann, the Chief of with the Court of Appeals on 4 February 2005 a special
Police, and Francisco Sales, the Governor of Davao. civil action for certiorari under Rule 65 of the Rules of
They prayed for a writ of habeas corpus to be issued Court with urgent prayer for Temporary Restraining Order
against the respondents to compel them to bring back the (TRO) and/or Writ of Preliminary Injunction. Moreover,
170 women who were deported to Mindanao against their since Gonzales and Mesa continued to be in detention, a
will. Petition for Habeas Corpus was filed by petitioner Pulido
During the trial, it came out that, indeed, the women were on their behalf. In response, Respondents prayed that the
deported without their consent. In effect, Lukban forcibly Petition for Habeas Corpus be dismissed primarily on two
assigned them a new domicile. Most of all, there was no grounds: (1) the continued detention of Gonzales and
law or order authorizing Lukban's deportation of the 170 Mesa is justified because of the pendency of the Petition
prostitutes. for Certiorari questioning the order dated 8 July2004 of
the RTC granting bail to Gonzales and Mesa before the
ISSUE: 7th Division of the Court of Appeals and (2) petitioner is
guilty of forum shopping because of his failure to state in
W/N Habeas Corpus that the prostitutes filed will prosper.
the petition that the order granting bail has been elevated
RULING: to the Court of Appeals and pending before its 7th
Division. Thus, we have this case.
The writ of habeas corpus was devised and exists as a
speedy and effectual remedy to relieve persons from ISSUE:
unlawful restraint, and as the best and only sufficient
W/N the petition for habeas corpus was proper despite of
defense of personal freedom.
the pending special civil action for certiorari before the
That the writ of habeas corpus was properly granted, and Court of Appeals 7th Division.
that the Mayor of the city of Manila who was primarily
RULING:
responsible for the deportation, is in contempt of court for
his failure to comply with the order of the court. When the release of the persons in whose behalf the
application for a Writ of Habeas Corpus was filed is
effected, the Petition for the issuance of the writ becomes
moot and academic. With the release of both Mesa and
Gonzales, the Petition for Habeas Corpus has, indeed,
been rendered moot. Courts of justice constituted to pass
upon substantial rights will not consider questions where
no actual interests are involved. Thus, the well-settled rule
that courts will not determine a moot question. Where the contrary, respondents, being Eufemia’s adopted children,
issues have become moot and academic, there ceases to are taking care of her.
be any justiciable controversy, thus rendering the
FLETCHER v. DIRECTOR OF BUREAU
resolution of the same of no practical value. This Court
will therefore abstain from expressing its opinion in a case
where no legal relief is needed or called for. FACTS:

Petitioner Martin Gibbs Fletcher seeks his release from


prison in this petition for the issuance of the writ of habeas
VELUZ v. VILLANUEVA
corpus. He claims that his prison sentence of 12 to 17
years was commuted by then President Fidel V. Ramos
FACTS: to nine to 12 years. Since he had already served 14 years,
three months and 12 days, including his good conduct
Eufemia E. Rodriguez, a 94-year old widow, allegedly allowance, his continued imprisonment is illegal.
suffering from a poor state of mental health and
deteriorating cognitive abilities, was living with petitioner, ISSUE:
Edgardo Veluz, her nephew, since 2000.
W/N petition for habeas corpus is granted
On January 11, 2005, respondents, legally adopted
children of Eufemia, took their mother from Veluz’ house. RULING:
Veluz made repeated demands for the return of Eufemia We note the issuance of a warrant for petitioners arrest
but these proved futile. Claiming that respondents were on March 8, 1996, the date he was first set for arraignment
restraining Eufemia of her liberty, he filed a petition for in Criminal Case No. 94-6988. Pursuant to Section 4,
habeas corpus in the Court of Appeals. However, CA Rule 102 of the Rules of Court, the writ cannot be issued
ruled that Veluz failed (1) to present any convincing proof and petitioner cannot be discharged since he has been
that respondents were unlawfully restraining their mother charged with another criminal offense. His continued
of her liberty, and (2) to establish his legal right to the detention is without doubt warranted under the
custody of Eufemia as he was not her legal guardian. circumstances.
Hence, this case was filed. Petitioner claimed that, in Petitioner asserts that his sentence in Criminal Case No.
determining if a writ of habeas corpus should issue, a 95-995 was commuted by then President Ramos.
court should limit itself to determining whether or not a However, he presented no proof of such commutation.
person is unlawfully being deprived of liberty. There is no Other than indorsements by the Chief Justice, Public
need to consider legal custody or custodial rights. Attorney’s Office and Undersecretary of the Department
ISSUE: of Justice, no document purporting to be the commutation
of his sentence by then President Ramos was attached in
W/N the issue on legal custody is insignificant for a his petition and in his subsequent missives to this Court.
petition for habeas corpus. His barren claim of commutation therefore deserves scant
consideration, lest we be accused of usurping the
RULING: Presidents sole prerogative to commute petitioner’s
The writ of habeas corpus extends to all cases of illegal sentence in Criminal Case No. 95-995.
confinement or detention by which any person is deprived
of his liberty or by which the rightful custody of a person
is being withheld from the one entitled thereto. Thus, it Ampatuan v. Macaraig
contemplates two instances: (1) deprivation of a person’s (G.R. No. 182497 June 29, 2010)
liberty either through illegal confinement or through
detention and (2) withholding of the custody of any person
from someone entitled to such custody. FACTS: A press briefing was then conducted where it
was announced that PO1 Ampatuan was arrested for the
In deciding a petition for habeas corpus, a court must first killing of two Commission on Elections (COMELEC)
inquire into whether a person is being restrained of his Officials. He was then detained at the Police Jail in United
liberty. Where such restraint exists, only then will inquiry Nations Avenue, Manila. Thereafter, PO1 Ampatuan was
into the cause of detention shall proceed. Only if the brought to inquest Prosecutor Renato Gonzaga of the
alleged cause is thereafter found to be unlawful will the Office of the City Prosecutor of Manila due to the alleged
writ be granted. Indeed, while habeas corpus is a writ of murder of Atty. Alioden D. Dalaig, head of the Law
right, it will not issue as a matter of course or as a mere Department of the COMELEC. On 20 April 2008, PO1
perfunctory operation on the filing of the petition. Ampatuan was turned-over to the Regional Headquarters
Support Group in Camp Bagong Diwa, Taguig City.
Herein, there is no proof that Eufemia is being detained
and restrained of her liberty by respondents. On the Petitioner, who is the wife of PO1 Ampatuan, continues
that on 21 April 2008, Chief Inquest Prosecutor Nelson
Salva ordered the release for further investigation of PO1 measure to assure the PNP authorities that the police
Ampatuan. The Order was approved by the City officers concerned are always accounted for.
Prosecutor of Manila. But Police Senior Superintendent
Co Yee Co, Jr., and Police Chief Inspector Agapito Since the basis of PO1 Ampatuan’s restrictive custody is
Quimson refused to release PO1 Ampatuan. the administrative case filed against him, his remedy is
within such administrative process.
This prompted Petitioner to file the petition for writ of
habeas corpus in the RTC of Manila, Branch 37.
B. FUNCTIONS OF THE WRIT OF HABEAS CORPUS AS A
On 24 April 2008, finding the petition to be sufficient in POST CONVICTION REMEDY
form and substance, respondent Judge Virgilio V.
Macaraig ordered the issuance of a writ of habeas corpus
commanding therein respondents to produce the body of
Lamen v. Director
PO1 Ampatuan and directing said respondents to show (G.R. No. 117078 February 22, 1995)
cause why they are withholding or restraining the liberty
of PO1 Ampatuan.
FACTS: The petitioners filed the instant petition to secure
Respondents assert that PO1 Ampatuan is under
the release of Willy Bagawe y Pagalla, who was earlier
restrictive custody since he is facing an administrative
convicted of the violation of Section 4, Article II of R.A. No.
case for grave misconduct, claiming that habeas corpus
6425, as amended, for selling and delivering marijuana
will not lie for a PNP personnel under restrictive custody.
and sentenced to suffer the penalty of life imprisonment
They claim that this is authorized under Section 52, Par.
and to pay a fine of P20,000.00. The prohibited drugs
4 of R.A. 8551 authorizing the Chief of PNP to place the
involved consist of three matchboxes containing
PNP personnel under restrictive custody during the
marijuana flowering tops with a total weight of 3.5 grams,
pendency of administrative case for grave misconduct.
one matchbox with a marijuana cigarette butt, and one
ISSUE: Whether or not PO1 Ampatuan should be brown paper bag containing 10 grams of marijuana
released and a writ of habeas corpus is proper. flowering tops.

RULING: No, PO1 Ampatuan should not be released and Willy Bagawe y Pagalla is presently serving his sentence
a writ of habeas corpus should not be given due course. in the New Bilibid Prisons in Muntinglupa, Metro Manila.
He has been under incarceration since his arrest on 14
In the instant case, PO1 Ampatuan is also facing April 1987.
administrative charges for Grave Misconduct. A petition
for habeas corpus will be given due course only if it shows The petitioners contend that since the gross quantity of
that petitioner is being detained or restrained of his liberty the marijuana involved in this case is only 13.6 grams,
unlawfully, but a restrictive custody and monitoring of then in the light of People vs. Simon, the maximum term
movements or whereabouts of police officers under imposed on Bagawe should be reduced to two (2) years,
investigation by their superiors is not a form of illegal four (4) months and one (1) day of prision correccional;
detention or restraint of liberty. and since he has already served more than six years, he
should be released from imprisonment.
In this case, PO1 Ampatuan has been placed under
Restrictive Custody. Republic Act No. 6975 (also known In its comment, the Office of the Solicitor General agrees
as the Department of Interior and Local Government Act with the petitioners that Simon should apply but disagrees
of 1990), as amended by Republic Act No. 8551 (also with their computation of the maximum penalty. It
known as the Philippine National Police Reform and recommends that, following the penalties imposed by us
Reorganization Act of 1998), clearly provides that in Simon and in People vs. Saycon,7 the proper penalty
members of the police force are subject to the should be six (6) months of arresto mayor as minimum to
administrative disciplinary machinery of the PNP. six (6) years of prision correccional as maximum, without
fine.
Given that PO1 Ampatuan has been placed under
restrictive custody, such constitutes a valid argument for ISSUE: Whether or not Willy Bagawe y Pagalla may use
his continued detention. This Court has held that a the Writ of Habeas Corpus as a post-conviction remedy.
restrictive custody and monitoring of movements or
RULING: Article 22 of the Revised Penal Code operates
whereabouts of police officers under investigation by their
to benefit Willy Bagawe since R.A. No. 7659, is favorable
superiors is not a form of illegal detention or restraint of
to him and since he is not a habitual criminal.
liberty.
Following Simon, the penalty which could have been
Restrictive custody is, at best, nominal restraint which is
imposed on Willy Bagawe under R.A. No. 6425, as further
beyond the ambit of habeas corpus. It is neither actual nor
amended by R.A. No. 7659, would have been prision
effective restraint that would call for the grant of the
correccional and, after applying the Indeterminate
remedy prayed for. It is a permissible precautionary
Sentence Law, he would have been sentenced to an
indeterminate penalty ranging from six (6) months of been fatal had it not been for the timely medical
arresto mayor, as minimum, to four (4) years and two (2) assistance given to them.
months of prision correccional as maximum.
On August 23, soon after noontime, the President of the
Since the decision in G.R. Nos. 88515-16 had long Philippines announced the issuance of Proclamation No.
become final and Willy Bagawe is in fact serving his 889 which is a declaration of a state of lawlessness. The
sentence, we cannot alter or modify the penalty therein said proclamation suspended the privilege of the writ of
imposed. Nevertheless, the writ of habeas corpus comes habeas corpus, for persons presently detained, as well
to his rescue since he has undergone imprisonment for a as others who may be hereafter similarly detained for the
period more than the maximum imprisonment which could crimes of insurrection or rebellion, and all other crimes
have been properly imposed on him taking into account and offenses committed by them in furtherance or on the
the favorable statute, R.A. No. 7659. In the 1932 case of occasion thereof, or incident thereto, or in connection
Directo vs. Director of Prisons,8 we ruled: therewith.
In view of the foregoing considerations
we are of the opinion and hold: (1) that Petitions for writ of habeas corpus were filed by the
Article 22 of the Revised Penal Code petitioners for having been arrested without a warrant and
which makes penal provisions retroactive then detained upon the authority of the proclamation. The
so far as they favor the accused, petition also seeks to assail the validity of the
provided he is not a habitual criminal, proclamation
does not authorize a court whose
sentence has become final and
executory to make a substantial ISSUE: Whether or not Proclamation No. 889 which
amendment, and any amendment made suspended the privilege of the writ of habeas corpus is
in such sentence, though it be to give constitutional
effect to a penal provision favorable to RULING: Yes, it is constitutional. The petition assailing its
the accused, would be null and void for validity is denied.
lack of jurisdiction; and (2) that the only
means of giving retroactive effect to a As heretofore adverted to, for the valid suspension of the
penal provision favorable to the accused privilege of the writ: (a) there must be "invasion,
when the trial judge has lost jurisdiction insurrection or rebellion" or — pursuant to paragraph (2),
over the case, is the writ of habeas section 10 of Art. VII of the Constitution — "imminent
corpus. danger thereof"; and (b) public safety must require the
aforementioned suspension. The President declared in
IN VIEW OF THE FOREGOING, the instant petition is Proclamation No. 889, as amended, that both conditions
GRANTED and WILLY BAGAWE y PAGALLA, accused- are present.
appellant in G.R. Nos. 88515-16, is hereby ordered
RELEASED from detention, unless for any other lawful As regards the first condition, our jurisprudence attests
cause his further confinement is warranted. abundantly to the Communist activities in the Philippines.
The magnitude of the rebellion has a bearing on the
SO ORDERED.
second condition essential to the validity of the
suspension of the privilege — namely, that the
C. SUSPENSION OF THE PRIVILEGE suspension be required by public safety.

In case of invasion, insurrection or rebellion or imminent


Lansang v. Garcia danger thereof, the President has, under the Constitution,
(G.R. No. L-33964 December 11, 1971)
three (3) courses of action open to him, namely: (a) to call
out the armed forces; (b) to suspend the privilege of the
writ of habeas corpus; and (c) to place the Philippines or
FACTS: In the evening of August 21, 1971, at about 9
any part thereof under martial law. He had, already, called
p.m., while the Liberal Party of the Philippines was holding
out the armed forces, which measure, however, proved
a public meeting at Plaza Miranda, Manila, for the
inadequate to attain the desired result. Of the two (2)
presentation of its candidates in the general elections
other alternatives, the
scheduled for November 8, 1971, two (2) hand grenades
suspension of the privilege is the least harsh.
were thrown, one after the other, at the platform where
said candidates and other persons were. As a
In view of the foregoing, it does not appear that the
consequence, eight (8) persons were killed and many
President has acted arbitrary in issuing Proclamation No.
more injured, including practically all of the
889, as amended, nor that the same is unconstitutional.
aforementioned candidates, some of whom sustained
The court has satisfied itself that there was a massive
extensive, as well as serious, injuries which could have
systematic Communist oriented movement to overthrow
the government through the use of force, thus it has RULING: Yes, the CA committed grave abuse of
decided to uphold the suspension of the privilege of the discretion in denying petitioner’s motion to appeal as a
Writ of Habeas Corpus. pauper litigant.

The Bill of Rights decrees that “Free access to the courts


and quasi-judicial bodies and adequate legal assistance
AFFIRMATIVE RIGHTS shall not be denied to any person by reason of poverty”

A perusal of the records shows that petitioner has


A. FREE ACCESS TO THE COURTS complied with all the evidentiary requirements for
prosecuting a motion to appear in court as a pauper. He
has executed an affidavit attesting to the fact that he and
his immediate family do not earn a gross income of more
Martinez v. People than P3,000.00 a month, and that their only real property,
(G.R. No. 132852 May 31, 2000) a hut, cannot be worth more than P10,000.00. He has
also submitted a joint affidavit executed by Florencia L.
Ongtico and Helen Maur, both residents of Butuan City,
FACTS: Petitioner was accused of homicide in a criminal who generally attested to the same allegations contained
case. During the hearing, petitioner represented by Atty. in petitioner's own affidavit. Based on this evidence, the
Chavez of PAO objected to petitioner’s motion to be Court finds that petitioner is qualified to litigate as an
allowed to litigate as pauper and moved instead to strike indigent.
the entire testimony of the first witness for the
prosecution. On July 1994 the trial court issued an order The Court ordered the CA to allow petitioner to litigate as
overruling the objection. On 8 August 1994 the court pauper and to return to him the amount of P420.00
denied the motion for reconsideration. This prompted representing the docket fees he paid.
petitioner to go to the Court of Appeals by way of a petition
for certiorari alleging that the trial court acted with grave
abuse of discretion amounting to lack of jurisdiction when B. PROTECTION AND ENFORCEMENT OF
it issued the assailed orders. CONSTITUTIONAL RIGHTS

(WRIT OF AMPARO)
On 23 August 1994 petitioner filed before the Court of
Appeals a Motion to Litigate as Pauper attaching thereto
supporting affidavits executed by petitioner himself and by Tapuz v. Del Rosario
two (2) ostensibly disinterested persons attesting to (G.R. No. 182484 June 17, 2008)
petitioner's eligibility to avail himself of this privilege. The
CA denied the motion and directed petitioner to remit
docketing fees in the amount of P420.00 within 5 days FACTS: The private respondents filed with the MCTC a
from notice. The petitioner filed a Motion for complaint for forcible entry and damages with the
Reconsideration of the order denying his motion to litigate issuance of a writ of preliminary mandatory injunction
as a pauper but the same was denied. Petitioner then filed against petitioners. The private respondents alleged in
a Manifestation on 28 October 1997 wherein he stated their complaint that they are the registered owners of the
through counsel that he was transmitting the docket fees disputed land and they were the disputed land’s prior
required of his client "under protest" and that the money possessors when the petitioners – armed with bolos
remitted was advanced by his counsel, Atty. Jesus G. carrying firearms and together with unidentified persons
Chavez himself. The transmittal of the amount was numbering 120 – entered the disputed land by force and
evidenced by two (2) postal money orders attached to the intimidation, without the private respondent’s permission
Motion to Litigate as Pauper. and against the objections of the private respondents
security men, and built thereon a nipa and bamboo
In the assailed Resolution, the CA dismissed the petition, structure.
citing petitioner’s failure to pay the required docket fee.
Petitioner moved for reconsideration citing his compliance In their answer, the petitioners denied the allegations of
with the docket fee requirements as alleged in his the complaint. They essentially claimed that: (1) they are
Manifestation, but the CA in the assailed resolution the actual and prior possessors of the disputed land; (2)
denied this latest motion on the ground that the amount on the contrary, the private respondents are the intruders;
remitted by petitioner as docket fee was short of P150.00 and (3) the private respondents' certificate of title to the
ISSUE: Whether or not the Court of Appeals gravely disputed property is spurious. They asked for the
abused its discretion in denying petitioner's motion to dismissal of the complaint and interposed a counterclaim
appeal as a pauper litigant. for damages.
It was against this factual backdrop that the petitioners Canlas vs. Napico Homeowners’ Association
filed the present petition last 29 April 2008. The petition G.R. No. 182795, June 5, 2008
contains and prays for three remedies, namely: a petition
for certiorari under Rule 65 of the Revised Rules of Court; FACTS: The petitioners sought for the issuance of a Writ
the issuance of a writ of habeas data under the Rule on Amporo claiming that they were deprived of their life and
the Writ of Habeas Data; and finally, the issuance of the liberty to shelter as result of various activities of the
writ of amparo under the Rule on the Writ of Amparo. respondents; and that, as illegal settlers in a certain parcel
of land situates in Barangay Manggahan, Pasig City, their
ISSUE: Whether or not the petitioners may be granted the dwelling/houses is about to be demolished.
remedy of writ of habeas data and writ of amparo
ISSUE: Whether or not the writ of amparo can be issued.
RULING: We find the petitions for certiorari and issuance
RULING: The threatened demolition of a dwelling by
of a writ of habeas data fatally defective, both in
virtue of a final judgment of the court, which in this case
substance and in form. The petition for the issuance of the
was affirmed with finality by this Court is not included
writ of amparo, on the other hand, is fatally defective with
among the enumeration of rights as stated in the above-
respect to content and substance.
quoted Section 1 for which the remedy of a writ
of amparo is made available. Their claim to their dwelling,
The Writ of Amparo assuming they still have any despite the final and
The writ of amparo was originally conceived as a executory judgment adverse to them, does not constitute
right to life, liberty and security. There is, therefore, no
response to the extraordinary rise in the number of killings
and enforced disappearances, and to the perceived lack legal basis for the issuance of the writ of amparo.
of available and effective remedies to address these
extraordinary concerns. It is intended to address
violations of or threats to the rights to life, liberty or Secretary of Defense vs. Manalo
security, as an extraordinary and independent remedy G.R. No. 180906, October 7, 2008
beyond those available under the prevailing Rules, or as
a remedy supplemental to these Rules. What it is not, is FACTS: Raymund and Renalo Manalo were suspected
a writ to protect concerns that are purely property or sympathizers of the NPA and therefore was abducted by
commercial. Neither is it a writ that we shall issue on armed men belonging to the CAFGU and forced
amorphous and uncertain grounds. disappearance. They were tortured and could have been
subjected to more if they were not able to escape.
Where, as in this case, there is an ongoing civil process Subsequently, the filed a Petition for petition for
dealing directly with the possessory dispute and the Prohibition, Injunction, and Temporary Restraining Order
reported acts of violence and harassment, we see no to stop the military officers and agents from depriving
point in separately and directly intervening through a writ them of their right to liberty and other rights. The Rule on
of amparo in the absence of any clear prima facie showing the Writ of Amparo then took effect.
that the right to life, liberty or security - the personal
concern that the writ is intended to protect - is immediately ISSUE: Whether or not the respondents have the right to
in danger or threatened, or that the danger or threat is the privilege of the Writ of Amparo even if their enforced
continuing. disappearance had already passed as the have escaped
from captivity.
The Writ of Habeas Data
RULING: The Supreme Court ruled that there is a
Specifically, we see no concrete allegations of unjustified
continuing violation of the Manalo’s right to security. The
or unlawful violation of the right to privacy related to the
Writ of Amparo is the most potent remedy available to any
right to life, liberty or security. The petition likewise has
person whose right to life, liberty, and security has been
not alleged, much less demonstrated, any need for
violated or is threatened with violation by an unlawful act
information under the control of police authorities other
or omission by public officials or employees and by private
than those it has already set forth as integral annexes.
individuals or entities. Understandably, since their
The necessity or justification for the issuance of the writ,
escape, the Manalos have been under concealment and
based on the insufficiency of previous efforts made to
protection by private citizens because of the threat to their
secure information, has not also been shown. In sum, the
life, liberty, and security. The circumstances of their
prayer for the issuance of a writ of habeas data is nothing
abduction, detention, torture, and escape reasonably
more than the "fishing expedition" that this Court - in the
support a conclusion that there is an apparent threat that
course of drafting the Rule on habeas data - had in mind
they will again be abducted, tortured, and this time, even
in defining what the purpose of a writ of habeas data is
executed. These constitute threats to their liberty,
not. In these lights, the outright denial of the petition for
security, and life, actionable through a petition for a writ of
the issuance of the writ of habeas data is fully in order.
amparo.”
Reyes vs. Court of Appeals Manila Electric vs. Lim
G.R. No. 182161, December 3, 2009 G.R. No. 184769, October 5, 2010

FACTS: Reverend Father Robert P. Reyes was among FACTS: Rosario Lim, a administrative clerk at
those arrested in the Manila Peninsula Hotel siege on MERALCO, was the subject of an anonymous letter
November 30, 2007 for the crime of rebellion. This was denouncing her, which was posted and spread in their
then dismissed by the RTC for the lack of probable cause. office. Shed then reported to the local police but was
The petitioner invokes the Writ of Amparo which is directed to transfer to another branch in light of these
anchored on the ground that respondents violated accusations. She asked for the deferment of the transfer
petitioner’s constitutional right to travel. Petitioner argues
however no response was received. Hence, she filed a
that the DOJ Secretary has no power to issue a Hold
petition for the issuance of the writ of habeas data against
Departure Order (HDO) and the subject HDO No. 45 has
no legal basis since the criminal case against him has the petitioners MERALCO alleging that the failure and
already been dismissed. refusal to provide her with details and information about
the report concerning her security and safety amounted
to a violation of her to privacy in life, liberty and security,
ISSUE: Whether or not the restriction on the petitioner’s correctible by habeas data. She requested for full
right to travel was unlawful. disclosure of report.
RULING: The restriction on petitioner’s right to travel as ISSUE: Whether or not the writ of habeas data can be
a consequence of the pendency of the criminal case filed issued in favor of Lim.
against him was not unlawful. Petitioner has also failed to
establish that his right to travel was impaired in the RULING: The court ruled no. The case of the petitioner
manner and to the extent that it amounted to a serious Lim does not fall within the proving of a writ of habeas
violation of his right to life, liberty and security, for which data – that a writ of habeas data is a remedy available to
there exists no readily available legal recourse or remedy. any person whose right to privacy in life, liberty, or
security is violated or threatened by an unlawful act or
omission of a private individual or entity engaged in the
gathering, collecting, or storing of data or information
regarding the person...” against abuse in this age of
So vs. Tacla information technology. There is no showing from the
G.R. No. 190473, October 19,2010
facts that MERALCO committed any unjustifiable or
unlawful violation of respondent’s right to privacy vis-à-vis
FACTS: David So filed a petition for the writs of habeas the right to life, liberty and security.
corpus and amparo on behalf of his daughter who was
accused of qualified theft in the criminal case pending
before Judge Tacla. He alleged that his daughter was (WRIT OF HABEAS DATA)
under a life-threatening situation while confined in a
government hospital to ascertain the actual psychological
Gamboa vs. Chan
state of the latter. During pendency of the case, Tacla G.R. No. 193636, July 24, 2012
dismissed the criminal case for qualified theft against
Maria.
FACTS: Gamboa, a Mayor in Ilocos Norte, alleged that
ISSUE: Whether or not the petition for habeas corpus and after a series of surveillance operations conducted by the
amparo should be dismissed for having been rendered PNP-Ilocos Norte against her, she was said to be
moot and academic. maintaining a private army group from a confidential
RULING: The petition for habeas corpus and amparo has report and that through Thru local TV news and print
to be dismissed for being moot and academic anent the media, her name has been tagged as one of those
dismissal in the qualified theft case. Also, the most basic politicians alleged to be maintaining the said group.
criterion for the issuance of the writ, therefore, is that the Gamboa then questioned such inclusion stating that such
individual seeking such relief is illegally deprived of his is a form of harassment a violation of her right to privacy
freedom of movement or place under some form of illegal towards these confidential reports by filing a writ of
restraint. If an individual’s liberty is restrainted via some habeas data requiring the government to release these
legal process, the writ of habeas corpus is unavailing. reports to ascertain.
Absent unlawful restraint and threat or violation of life, ISSUE: Whether or not the writ of habeas data is the
liberty or security, the petitions could still have not proper remedy.
prospered.
RULING: The court ruled that the remedy of writ of
habeas data is not proper in this case. The writ of habeas
data is an independent and summary remedy designed to
protect the image, privacy, honor, information, and allegation as to what particular acts or omission of
freedom of information of an individual, and to provide a respondents violated or threatened petitioner’s right to
forum to enforce one’s right to the truth and to life, liberty and security. The CA also correctly held that
informational privacy. In this case, the petition was not petitioner failed to present substantial evidence that his
granted because such information holds legitimate public right to life, liberty and security were violated, or how his
interest in dismantling these private armies. PNP released right to privacy was threatened by respondents. He did
information to the Commission without prior not specify the particular documents to be secured, their
communication to Gamboa and without affording her the location or what particular government office had custody
opportunity to refute the same cannot be interpreted as a thereof, and who has possession or control of the same.
violation or threat to her right to privacy since that act is
an inherent and crucial component of intelligence The Court has ruled that in view of the recognition of the
gathering and investigation. It must be emphasized that in evidentiary difficulties attendant to the filing of a petition
order for the privilege of the writ to be granted, there must for the privilege of the writs of amparo and habeas data,
exist a nexus between the right to privacy on the one not only direct evidence, but circumstantial evidence,
hand, and the right to life, liberty or security on the other. indicia, and presumptions may be considered, so long as
they lead to conclusions consistent with the admissible
evidence adduced.

With the foregoing in mind, the Court still finds that the CA
Saez v. Arroyo
did not commit a reversible error in declaring that no
(G.R. No. 183533 September 25, 2012)
substantial evidence exist to compel the grant of the
reliefs prayed for by the petitioner. The Court took a
FACTS: On March 6, 2008, the petitioner filed with the second look on the evidence on record and finds no
Court a petition to be granted the privilege of the writs of reason to reconsider the denial of the issuance of the writs
amparo and habeas data with prayers for temporary prayed for.
protection order, inspection of place and production of
documents.5 In the petition, he expressed his fear of
being abducted and killed; hence, he sought that he be
placed in a sanctuary appointed by the Court. He likewise
prayed for the military to cease from further conducting
surveillance and monitoring of his activities and for his
name to be excluded from the order of battle and other
government records connecting him to the Communist
Party of the Philippines (CPP).

The CA conducted hearings with an intent to clarify what


actually transpired and to determine specific acts which
threatened the petitioner’s right to life, liberty or security.
During the hearings, the petitioner narrated that starting
April 16, 2007, he noticed that he was always being
followed by a certain "Joel," a former colleague at Bayan
Muna. "Joel" pretended peddling pandesal in the vicinity
of the petitioner’s store. Three days before the petitioner
was apprehended, "Joel" approached and informed him
of his marital status and current job as a baker in Calapan,
Mindoro Oriental. "Joel" inquired if the petitioner was still
involved with ANAKPAWIS. When asked by the CA
justices during the hearing if the petitioner had gone home
to Calapan after having filed the petition, he answered in
the negative explaining that he was afraid of Pvt. Osio
who was always at the pier.

ISSUE: Whether or not the petitioner may be granted the


remedy of writ of habeas data and writ of amparo

RULING: The court denied the petition.

A careful perusal of the subject petition shows that the CA


correctly found that the petition was bereft of any

You might also like